Download as pdf or txt
Download as pdf or txt
You are on page 1of 97

MATHEMATICS 9th Science Group

7/18/2020
Chapter 1.
MATRICES AND DETERMINANTS

A project of: www.notespk.com


Contact or Suggest Us: info@notespk.com
Contents
Introduction: ......................................................................... 1
Exercise 1.1 ........................................................................... 1
Exercise 1.2 ....................................................................3
Exercise 1.3 ........................................................................... 5
Exercise1.4 .......................................................................... 11
Exercise 1.5 ......................................................................... 14
Exercise 1.6 ......................................................................... 18
Class 9th Chapter 1 www.notes.pk.com

Introduction: Exercise 1.1


The matrices and determinants are used in the field Question.1.Find the order of the following
of Mathematics, Physics, Statistics, Electronics and matrices.
other branches of science. The Matrices have 𝟐 𝟑
(i). 𝑨 = [ ]
played a very important role in this age of computer −𝟓 𝟔
science. The idea of matrices was given by the Solution.
𝑶𝒓𝒅𝒆𝒓 𝒐𝒇 𝑨 = 𝑶(𝑨) =2-by-2 𝒐𝒓 𝟐 × 𝟐
Arthur Cayley, an English Mathematician of 19th 𝟐 𝟎
century, who first developed, (ii). 𝑩 = [ ]
𝟑 𝟓
𝑇ℎ𝑒𝑜𝑟𝑦 𝑜𝑓 𝑀𝑎𝑡𝑟𝑖𝑐𝑒𝑠′′ ′′in 1858. Solution.
Matrix: 𝑶𝒓𝒅𝒆𝒓 𝒐𝒇 𝑩 = 𝑶(𝑩) =2-by-2𝒐𝒓 𝟐 × 𝟐
“An arrangement of different elements in (iii). 𝑪 = [𝟐 𝟒]
Solution.
the rows and columns, within square brackets is
𝑶𝒓𝒅𝒆𝒓 𝒐𝒇 𝑪 = 𝑶(𝑪) = 𝟏-by-2 𝒐𝒓 𝟏 × 𝟐
called Matrix”. 𝟒
𝑎11 𝑎12
e.g 𝐴 = [𝑎 ]. (iv). 𝑫 = [𝟎]
21 𝑎22
𝟔
The real numbers used in the formation of the Solution.
matrix are called entries or elements of the matrix. 𝑶𝒓𝒅𝒆𝒓 𝒐𝒇 𝑫 = 𝑶(𝑫) =3-by-1𝒐𝒓 𝟑 × 𝟏
The matrices are denoted by the capital letters 𝒂 𝒅
𝐴, 𝐵, 𝐶, 𝐷, … , 𝑀, 𝑁 𝑒𝑡𝑐. of the English alphabets. (v). 𝑬 = [𝒃 𝒆 ]
Rows and Columns of a Matrix: 𝒄 𝒇
𝑎11 𝑎12 Solution.
In a matrix𝐴 = [𝑎 ], the entries presented 𝑶𝒓𝒅𝒆𝒓 𝒐𝒇 𝑬 = 𝑶(𝑬) =3-by-2 𝒐𝒓 𝟑 × 𝟐
21 𝑎22
in the horizontal way are called rows. (vi). 𝑭 = [𝟐]
𝑎11 𝑎12 Solution.
In a matrix𝐴 = [𝑎 ], the entries presented 𝑶𝒓𝒅𝒆𝒓 𝒐𝒇 𝑭 = 𝑶(𝑭) =1-by-1𝒐𝒓 𝟏 × 𝟏
21 𝑎22
in the vertical way are called columns. 2 3 0
(vii). 𝐺 = [1 2 3]
Order of a Matrix:
2 4 5
Order of Matrix tells us about no of rows and Solution.
columns. 𝑶𝒓𝒅𝒆𝒓 𝒐𝒇 𝑮 = 𝑶(𝑮) =3-by-3𝒐𝒓 𝟑 × 𝟑
𝑶𝒓𝒅𝒆𝒓 𝒐𝒇 𝒂 𝒎𝒂𝒕𝒓𝒊𝒙 = 𝒏𝒐. 𝒐𝒇 𝒓𝒐𝒘𝒔 × 𝒏𝒐. 𝒐𝒇 𝒄𝒐𝒍𝒖𝒎𝒏𝒔. 2 3 4
(viii). 𝐻 = [ ]
If a matrix 𝑨 has m rows and n column then its 1 0 6
order is Solution.
𝑶(𝑨) = 𝒎 × 𝒏 𝒐𝒓 𝒎 − 𝒃𝒚 − 𝒏. 𝑶𝒓𝒅𝒆𝒓 𝒐𝒇 𝑯 = 𝑶(𝑯) =2-by-3 𝒐𝒓 𝟐 × 𝟑
Question.2. which of the following matrices
For example
are equal?
1 2 3
𝐴 = [3], 𝐵 = [3 5]
𝐴 = [9 7 6] ℎ𝑎𝑠 𝑜𝑟𝑑𝑒𝑟 3 − 𝑏𝑦 − 3 𝑜𝑟 3 × 3.
𝐶 = [5 − 2], 𝐷 = [5 3]
4 6 8 4 0 2
Equal matrices: 𝐸=[ ], 𝐹 =[ ],
6 2 6
“Two matrices are said to be equal if 3−1 4 0
𝐺=[ ] 𝐻=[ ],
i. The order of matrix 𝑨 =The order of 3+3 6 2
2+2 2−2
Matrix B 𝐼 = [3 3 + 2] , 𝐽=[ ]
2+4 2+0
ii. Their corresponding elements are equal. Solution.
Thus From above matrices
𝑨 = 𝑩. " 𝐴=𝐶
Example: 𝐸=𝐻=𝐽
1 2 1 1+1 𝐹=𝐺
𝐴=[ ] 𝑎𝑛𝑑 𝐵 = [ ]
−5 7 −5 5 + 2 Question.3. Find the values of 𝒂, 𝒃, 𝒄 𝒂𝒏𝒅 𝒅
𝑎𝑟𝑒 𝑒𝑞𝑢𝑎𝑙 𝑚𝑎𝑡𝑟𝑖𝑐𝑒𝑠. which satisfy the matrix equation.
𝒂 + 𝒄 𝒂 + 𝟐𝒃 𝟎 −𝟕
[ ]=[ ]
𝒄 − 𝟏 𝟒𝒅 − 𝟔 𝟑 𝟐𝒅

1|Page
Class 9th Chapter 1 www.notes.pk.com
Solution. Null or Zero Matrix:
Given “A matrix whose each element is zero, is called a
𝑎 + 𝑐 𝑎 + 2𝑏 0 −7 null or zero matrix. It is denoted by 𝑂.”
[ ]=[ ]
𝑐 − 1 4𝑑 − 6 3 2𝑑 Examples:
By the definition of equal matrices, we have
0 0 0
𝑎 + 𝑐 = 0 → (𝑖) , 𝑎 + 2𝑏 = −7 → (𝑖𝑖) , [0] , [0 0] , [0 0 0] are null matrices.
𝑐 − 1 = 3 → (𝑖𝑖𝑖) , 4𝑑 − 6 = 2𝑑 → (𝑖𝑣)
0 0 0
From (iii), we have Transpose of a Matrix:
𝑐 =3+1=4 “A matrix obtained by changing the rows
From (iv), we have into columns or columns into rows of a matrix
4𝑑 − 6 = 2𝑑
is called transpose of that matrix. If A is a
4𝑑 − 2𝑑 = 6
matrix, then its transpose matrix is denoted by
2𝑑 = 6
6 𝑨𝒕 . "
𝑑= Example:
2 1 2 3 1 9 4
𝑑=3
If 𝐴 = [9 7 6] 𝑡ℎ𝑒𝑛 𝐴𝑡 = [2 7 6]
Using value of 𝑐 = 4 𝑖𝑛 (𝑖), 𝑤𝑒 ℎ𝑎𝑣𝑒
4 6 8 3 6 8
𝑎+4=0 1 1
𝑎 = −4 1 3 2 𝑡
If 𝐵 = [ ] 𝑡ℎ𝑒𝑛 𝐵 = [ 3 9]
Using value of 𝑎 = −4 𝑖𝑛 (𝑖𝑖), 𝑤𝑒 ℎ𝑎𝑣𝑒 1 9 4
2 4
−4 + 2𝑏 = −7 If a matrix B is of order 2-by-3 then order its
2𝑏 = −7 + 4 transpose matrix 𝐵 𝑡 is 3-by-2.
2𝑏 = −3 Negative of a Matrix:
3 “Let A be a matrix. Then its negative, −𝐴 is
𝑏=−
2 obtained by changing the signs of all the entries
3
Hence 𝑎 = −4 , 𝑏 = − 2 , 𝑐 = 4 𝑎𝑛𝑑 𝑑 = 3. of A.”
Types of Matrices: Example:
Row matrix: 1 −2 −1 2
If 𝐴 = [ ] , 𝑡ℎ𝑒𝑛 − 𝐴 = [ ].
“A matrix having single row is called Row 3 4 −3 −4
Symmetric matrix:
Matrix.”
Example: “Let Α be the square matrix, if 𝐴𝑡 = 𝐴 then Α is
𝑀 = [1 2 3] 𝑖𝑠 𝑎 𝑟𝑜𝑤 𝑚𝑎𝑡𝑟𝑖𝑥 𝑜𝑓 𝑜𝑟𝑑𝑒𝑟 1 − 𝑏𝑦 − 3. called symmetric matrix.”
Column matrix: Example:
A matrix having single column is called column 1 2 3
Matrix. 𝐴 = [ 2 −1 4] 𝑖𝑠 𝑎 𝑠𝑞𝑢𝑎𝑟𝑒 𝑚𝑎𝑡𝑟𝑖𝑥 𝑡ℎ𝑒𝑛
Example: 3 4 0
1 2 3
3
𝐴𝑡 = [ 2 −1 4] = 𝐴.
𝑀 = [6] 𝑖𝑠 𝑎 𝑐𝑜𝑙𝑢𝑚𝑛 𝑚𝑎𝑡𝑟𝑖𝑥 𝑜𝑓 𝑜𝑟𝑑𝑒𝑟 3 − 𝑏𝑦 − 1.
2
3 4 0
Rectangular matrix: 𝑇ℎ𝑢𝑠 𝐴 𝑖𝑠 𝑠𝑦𝑚𝑚𝑡𝑒𝑟𝑖𝑐 𝑚𝑎𝑡𝑟𝑖𝑥.
Skew-symmetric matrix:
A matrix in which number of rows is not equal to
“Let Α be the square matrix, if 𝑨𝒕 = −𝑨 then
number of columns is called rectangular Matrix.
Α is called skew symmetric matrix.”
Example:
𝑎 𝑑 Example:
2 3 4 0 2 3
[𝑏 𝑒 ] and [ ] are rectangular matrices.
1 0 6 𝐴 = [− 2 0 1] 𝑖𝑠 𝑎 𝑠𝑞𝑢𝑎𝑟𝑒 𝑚𝑎𝑡𝑟𝑖𝑥 𝑡ℎ𝑒𝑛
𝑐 𝑓
− 3 −1 0
0 −2 −3 0 2 3
Square matrix: 𝐴𝑡 = [ 2 0 −1] = − [− 2 0 1]
“A matrix in which number of rows is equal to 3 1 0 − 3 −1 0
the number of columns then matrix is called square = −𝐴.
matrix.” 𝑇ℎ𝑢𝑠 𝐴 𝑖𝑠 𝑎 𝑠𝑘𝑒𝑤 − 𝑠𝑦𝑚𝑚𝑡𝑒𝑟𝑖𝑐 𝑚𝑎𝑡𝑟𝑖𝑥.
Example: Diagonal matrix:
1 2 3 “A square matrix A is called a diagonal matrix if
𝐴 = [9 7 6] ℎ𝑎𝑠 𝑜𝑟𝑑𝑒𝑟 3 − 𝑏𝑦 − 3. at least any one of the entries of its diagonal is not
4 6 8
2|Page
Class 9th Chapter 1 www.notes.pk.com
zero and non-diagonal entries are zero.” Solution.
Example: (𝑎). Square Matrices: (iii). (iv). (viii).
1 0 0 (b). Rectangular Matrices: (i). (ii). (v).
2 0 1 0
𝐴 = [ 0 2 0] , 𝐵 = [ ] 𝑎𝑛𝑑 𝐶 = [ ] (c). Row Matrices: (vi).
0 3 0 0
0 0 3 (d). (ii). (vii).
𝑎𝑟𝑒 𝑐𝑎𝑙𝑙𝑒𝑑 𝑑𝑖𝑎𝑔𝑜𝑛𝑎𝑙 𝑚𝑎𝑡𝑟𝑖𝑐𝑒𝑠. (e). (iv).
Scalar Matrix: (f). (ix).
“A diagonal matrix having same elements in Question.3. From the following matrices,
principle diagonal except 1 or 0 is called scalar identify Diagonal matrices, Scalar matrices
matrix.” and Unit (identity) matrices.
Example: 𝟒 𝟎 𝟐 𝟎 𝟏 𝟎
𝑨=[ ], 𝑩=[ ], 𝑪 = [ ]
𝑘 0 0 𝟎 𝟒 𝟎 −𝟏 𝟎 𝟏
𝐴 = [ 0 𝑘 0] 𝑎𝑛𝑑 𝐵 𝟑 𝟎 𝟓−𝟑 𝟎
𝑫=[ ] , 𝑬=[ ]
0 0 𝑘 𝟎 𝟎 𝟎 𝟏+𝟏
3 0 Solution.
=[ ] 𝑎𝑟𝑒 𝑆𝑐𝑎𝑙𝑎𝑟 𝑚𝑎𝑡𝑟𝑖𝑐𝑒𝑠.
0 3 Diagonal Matrices: 𝐴 , 𝐵 , 𝐶 , 𝐷 , 𝐸.
Unit Matrix or Identity Matrix: Scalar Matrices: 𝐴, 𝐶, 𝐸.
A diagonal matrix is called identity matrix if all Unit Matrices: 𝐶.
diagonal entries are 1. It is denoted by 𝐼. Question.4. Find the negative of matrices A, B,
1 0 0 C, D and E when:
𝐼 = [ 0 1 0] 𝑎𝑛𝑑 𝐼 𝟏
0 0 1 (i). 𝑨 = [ 𝟎 ]
1 0
=[ ] 𝑎𝑟𝑒 𝑖𝑑𝑒𝑛𝑡𝑖𝑡𝑦 𝑚𝑎𝑡𝑟𝑖𝑐𝑒𝑠. −𝟏
0 1
Solution.
−1
Exercise 1.2 −𝐴 = [ 0 ]
Question.1. From the following matrices, identify 1
(ii). 𝐵 = [5 1 −6]
unit matrices, row matrices, column matrices and
Solution.
null matrices.
−𝐵 = [−5 −1 6]
𝟒
𝟎 𝟎 2 3
𝑨=[ ], 𝑩 = [𝟐 𝟑 𝟒], 𝑪 = [ 𝟎] (iii). 𝐶 = [ ]
𝟎 𝟎 0 5
𝟔 Solution.
𝟓
𝟏 𝟎 −2 −3
𝑫=[ ], 𝑬 = [𝟎], 𝑭 = [ 𝟔] −𝐶 = [ ]
𝟎 𝟏 0 −5
𝟕 2 3
Solution. (iv). 𝐷 = [ ]
−4 5
Identity Matrices: 𝐷 Solution.
Row Matrices: 𝐵 𝑎𝑛𝑑 𝐸. −2 −3
−𝐷 = [ ]
Column Matrices: 𝐶 , 𝐸 𝑎𝑛𝑑 𝐹. 4 −5
Null Matrices: 𝐴 𝑎𝑛𝑑 𝐸. 1 2
(v). 𝐸 = [ ]
Question.2. From the following matrices, identify 3 4
Solution.
(a) Square matrices, (b) Rectangular matrices, (c) −1 −2
Row matrices, (d) Column matrices, (e) Identity −𝐸 = [ ]
−3 −4
Matrices, (f) Null matrices. Question.5. Find the transpose of the following
(i). matrices:
𝟑 𝟎 𝟏 𝟐
−𝟖 𝟐 𝟕
[ ] (𝒊𝒊). [ 𝟎] (𝒊𝒊𝒊). [ 𝟔 −𝟒] (𝒊𝒗). [ 𝟏(i). 𝟎𝑨] =(𝒗).
[ 𝟏[]𝟑 𝟒]
𝟏𝟐 𝟎 𝟒 𝟑 −𝟐 𝟎 𝟏
𝟏 −𝟐 𝟓 𝟔
(vi). [𝟑 𝟏𝟎 −𝟏] (vii). Solution.
𝟏 𝟏 𝟐 𝟑 𝟎 𝟎 𝑨𝒕 = [𝟎 𝟏 −𝟏]
[ 𝟎] (𝒗𝒊𝒊𝒊). [−𝟏 𝟐 𝟎] (𝒊𝒙). [ 𝟎 𝟎] 𝟏 𝟐
𝟎 𝟎 𝟎 𝟏 𝟎 𝟎 (ii). 𝑪 = [ 𝟐 −𝟏]
𝟑 𝟎
3|Page
Class 9th Chapter 1 www.notes.pk.com
Solution. Addition of matrices:
𝟏 𝟐 𝟑 “Let A and B be any two matrices of same order
𝑪𝒕 = [ ]
𝟐 −𝟏 𝟎 then A and B are comfortable for addition.”
𝟐 𝟑 Addition of A and B, Written as 𝐴 + 𝐵 is obtained
(iii). 𝑫 = [ ]
𝟎 𝟓 by adding the entries of the matrix A to the
Solution. corresponding entries of
𝟐 𝟎
𝑫𝒕 = [ ] the matrix B.”
𝟑 𝟓 Example:
𝟐 𝟑 2 3 0
(iv). 𝑬 = [ ] 𝐿𝑒𝑡 𝐴 = [ ] 𝑎𝑛𝑑 𝐵
−𝟒 𝟓 1 0 6
Solution. −2 3 4
=[ ] 𝑎𝑟𝑒 𝑐𝑜𝑚𝑓𝑜𝑟𝑡𝑎𝑏𝑙𝑒 𝑓𝑜𝑟 𝑎𝑑𝑑𝑖𝑡𝑖𝑜𝑛.
𝟐 −𝟒 1 2 3
𝑬𝒕 = [ ] 2−2 3+3 0+4 0 6 4
𝟑 𝟓 𝐴+𝐵 = [ ]=[ ]
𝟏 𝟐 1+1 0+2 6+3 2 2 9
(v). 𝑭 = [ ] Subtraction of matrices:
𝟑 𝟒
Let A and B be any two matrices of same order then
Solution.
A and B are comfortable for Subtraction.
𝟏 𝟑
𝑭𝒕 = [
] Subtraction of A and B, Written as 𝐴 − 𝐵 is
𝟐 𝟒
𝟏 𝟐 obtained by subtracting the entries of the matrix A
Question.6. Verify that if 𝑨 = [ ] ,𝑩 = to the corresponding entries of the matrix B.
𝟎 𝟏
𝟏 𝟏 Example:
[ ] , 𝒕𝒉𝒆𝒏 2 3 0 −2 3 4
𝟐 𝟎 𝐿𝑒𝑡 𝐴 = [ ] 𝑎𝑛𝑑 𝐵 = [ ]
(i). (𝑨𝒕 )𝒕 = 𝑨 1 0 6 1 2 3
𝑎𝑟𝑒 𝑐𝑜𝑚𝑓𝑜𝑟𝑡𝑎𝑏𝑙𝑒 𝑓𝑜𝑟 Subtraction.
Solution. 2+2 3−3 0−4 4 0 −4
𝐴−𝐵 =[ ]=[ ]
Given 1−1 0−2 6−3 0 −2 3
𝟏 𝟐 Multiplication of a Matrix by a Real Number:
𝑨=[ ] Let A be any matrix and the real number 𝑘 be a
𝟎 𝟏
𝟏 𝟎 scalar. Then the scalar multiplication of matrix A
𝑨𝒕 = [ ]
𝟐 𝟏 with 𝑘 is obtained by multiplying each entry of
(𝑨𝒕 )𝒕 = [ 𝟏 𝟐] = 𝑨 matrix A with 𝑘. It is denoted by 𝑘𝐴.
𝟎 𝟏 Example:
(𝑨𝒕 )𝒕 = 𝑨 2 3 0 2𝑘 3𝑘 0
𝐿𝑒𝑡 𝐴 = [ ] 𝑡ℎ𝑒𝑛 𝑘𝐴 = [ ]
Hence Proved. 1 0 6 1𝑘 0 6𝑘
Commutative Law for Addition.
(ii). (𝑩𝒕 )𝒕 = 𝑩
If A and B are two matrices of the same order, Then
Solution. 𝐴 + 𝐵 = 𝐵 + 𝐴 𝑖𝑠 called commutative law under
Given addition.
𝟏 𝟏 𝐴+𝐵 =𝐵+𝐴
𝑩=[ ]
𝟐 𝟎 Associative Law for Addition:
𝟏 𝟐
𝑩𝒕 = [ ] If A, B and C are three matrices of the same order,
𝟏 𝟎 Then
(𝑩𝒕 )𝒕 = [ 𝟏 𝟏] = 𝑩 (𝐴 + 𝐵) + 𝐶 = 𝐴 + (𝐵 + 𝐶) 𝑖𝑠 Called Associative
𝟐 𝟎
(𝑩𝒕 )𝒕 = 𝑩 law under addition.
(𝐴 + 𝐵) + 𝐶 = 𝐴 + (𝐵 + 𝐶)
Hence Proved Additive Identity of a Matrix:
If A and B are two matrices of same order and
𝐴+𝐵 =𝐴=𝐵+𝐴
Then matrix B is called additive identity of matrix A.
For any matrix A and zero matrix of same order, O
is called additive identity of A as
𝐴 + 𝑂 = 𝐴 = 𝑂 + 𝐴.
Additive Inverse of a Matrix:
If A and B are two matrices of same order and 𝐴 +
𝐵 =𝑂 =𝐵+𝐴
Then matrix B is called additive inverse of matrix A.
“Additive inverse of any matrix A is obtained by
changing to negative of the symbols (entries) of
each non zero entry of A.”

4|Page
Class 9th Chapter 1 www.notes.pk.com
1 1 −1 2 1 1
Exercise 1.3 𝐴+[
1 1
]=[
−2 1
]+[
1 1
]
Question.1. which of the following matrices are
comfortable for addition? −1 + 1 2 + 1
=[ ]
𝟏 𝟎 −2 + 1 1 + 1
𝟐 𝟏 𝟑 0 3
𝑨=[ ], 𝑩 = [ ], 𝑪 = [ 𝟐 −𝟏] =[ ]
−𝟏 𝟑 𝟏 −1 2
𝟏 −𝟐
𝟐+𝟏 −𝟏 𝟎 Answer.
𝑫=[ ], 𝑬=[ ], −𝟐
𝟑 𝟏 𝟐 (ii). 𝑩 + [ ]
𝟑 𝟐 𝟑
𝑭 = [𝟏+𝟏 −𝟒 ] Solution.
𝟑+𝟐 𝟐+𝟏 −2 1 −2
𝐵+[ ]= [ ]+[ ]
Solution. 3 −1 3
Since order of 𝐴 𝑎𝑛𝑑 𝐸 𝑎𝑟𝑒 same so they are 1−2
=[ ]
−1 + 3
comfortable for addition. −1
Also order of 𝐵 𝑎𝑛𝑑 𝐷 𝑎𝑟𝑒 same so they are =[ ]
2
comfortable for addition. Answer.
Also order of 𝐶 𝑎𝑛𝑑 𝐹 𝑎𝑟𝑒 same so they are (iii). 𝑪 + [−𝟐 𝟏 𝟑]
comfortable for addition. Solution.
Question.2. Find the additive inverse of the following 𝐶 + [−2 1 3] = [1 −1 2] + [−2 1 3]
matrices: = [1−2 −1 + 1 2 + 3]
𝟐 𝟒 = [−1 0 5]
(i). 𝑨 = [ ]
−𝟐 𝟏 Answer.
Solution. 𝟎 𝟏 𝟎
2 4 (iv). 𝑫 + [ ]
𝐴𝑑𝑑𝑖𝑡𝑖𝑣𝑒 𝑖𝑛𝑣𝑒𝑟𝑠𝑒 𝑜𝑓 𝐴 = −𝐴 = [ ] 𝟐 𝟎 𝟏
−2 1 Solution.
𝟏 𝟎 −𝟏 0 1 0 1 2 3 0 1 0
(ii). 𝑩 = [ 𝟐 −𝟏 𝟑 ] 𝐷+[ ]=[ ]+[ ]
2 0 1 −1 0 2 2 0 1
𝟑 −𝟐 𝟏 1+0 2+1 3+0
Solution. =[ ]
−1 + 2 0 + 0 2 + 1
−1 0 1 1 3 3
=[ ]
𝐴𝑑𝑑𝑖𝑡𝑖𝑣𝑒 𝑖𝑛𝑣𝑒𝑟𝑠𝑒 𝑜𝑓 𝐵 = −𝐵 = [− 2 1 −3] 1 0 3
− 3 2 −1 Answer.
𝟒 (v). 𝟐𝑨
(iii). 𝑪 = [ ]
−𝟐 Solution.
Solution. −1 2
−4 2𝐴 = 2 [ ]
𝐴𝑑𝑑𝑖𝑡𝑖𝑣𝑒 𝑖𝑛𝑣𝑒𝑟𝑠𝑒 𝑜𝑓 𝐶 = −𝐶 = [ ] 2 1
2 −2 4
𝟏 𝟎 =[ ]
4 2
(iv). 𝑫 = [−𝟑 −𝟐] Answer.
𝟐 𝟏 (vi). (−𝟏)𝑩
Solution.
Solution.
−1 0
1
𝐴𝑑𝑑𝑖𝑡𝑖𝑣𝑒 𝑖𝑛𝑣𝑒𝑟𝑠𝑒 𝑜𝑓 𝐷 = −𝐷 = [ 3 2] (−1)𝐵 = (−1) [ ]
−1
−2 −1 −1
𝟏 𝟎 =[ ]
(v). 𝑬 = [ ] 1
𝟎 𝟏 Answer.
Solution.
−1 0 (vii). (−𝟐)𝑪
𝐴𝑑𝑑𝑖𝑡𝑖𝑣𝑒 𝑖𝑛𝑣𝑒𝑟𝑠𝑒 𝑜𝑓 𝐸 = −𝐸 = [ ] Solution.
0 −1
𝟏 (−2)𝐶 = (−2)[1 −1 2]
(vi). 𝑭 = [ √𝟑 ] = [−1 2 −4]
−𝟏 √𝟐
Solution. Answer.
(viii). 𝟑𝑫
𝐴𝑑𝑑𝑖𝑡𝑖𝑣𝑒 𝑖𝑛𝑣𝑒𝑟𝑠𝑒 𝑜𝑓 𝐹 = −𝐹 = [−√3 −1 ] Solution.
1 −√2
−𝟏 𝟐 𝟏 1 2 3
Question.3.If 𝑨 = [ ],𝑩 = [ ],𝑪 = 3𝐷 = 3 [ ]
−𝟐 𝟏 −𝟏 −1 0 2
3 6 9
[𝟏 −𝟏 𝟐], 𝑫 = [ 𝟏 𝟐 𝟑] =[
−3 0 6
]
−𝟏 𝟎 𝟐
Then find, Answer.
𝟏 𝟏 (ix). 𝟑𝑪
(i). 𝑨 + [ ]
𝟏 𝟏 Solution.
Solution. 3𝐶 = 3[1 −1 2]

5|Page
Class 9th Chapter 1 www.notes.pk.com
= [3 −6 6] 1 2 3 1 0 −2
Answer. [2 3 1] + [−2 −1 0 ]
3 1 2 0 2 −1
1+1 2+0 3−2
= [2 − 1 3 − 1 1 + 0]
Question.4. perform the indicated operations and
3+0 1+2 2−1
simplify the following 2 2 1
1 0 0 2 1 1 = [ 1 2 1]
(i). ([ ]+[ ]) + [ ]
0 1 3 0 0 1 3 3 1
Solution. Answer.
1 0 0 2 1 1 𝟏 𝟐 𝟏 𝟐 𝟏 𝟏
([ ]+[ ]) + [ ] (vi). ([ ]+[ ]) + [ ]
0 1 3 0 0 1 𝟎 𝟏 𝟎 𝟏 𝟏 𝟏
1+0 0+2 1 1
= ([ ]) + [ ] Solution.
0+ 3 1+0 0 1 1 2 1 2 1 1
1 2 1 1 ([ ]+[ ]) + [ ]
= [ ]+[ ] 0 1 0 1 1 1
3 1 0 1 1+1 2+2 1 1
1+1 2+1 = ([ ]) + [ ]
= [ ] 0+0 1+1 1 1
3+0 1+1 2 4 1 1
2 3 =[ ]+[ ]
= [ ] 0 2 1 1
3 2 2+1 4+1
Answer. = [ ]
0+1 2+1
𝟏 𝟎 𝟎 𝟐 𝟏 𝟏 3 5
(ii). [ ] + ([ ]−[ ]) = [ ]
𝟎 𝟏 𝟑 𝟎 𝟏 𝟎 1 3
Solution. Answer.
1 0 0 2 1 1 𝟏 𝟐 𝟑
[ ] + ([ ]−[ ])
0 1 3 0 1 0 Question5.For the matrices 𝑨 = [ 𝟐 𝟑 𝟏] , 𝑩 =
1 0 0−1 2−1
=[ ] + ([ ]) 𝟏 −𝟏 𝟎
0 1 3−1 0−1 𝟏 −𝟏 𝟏
1 0 −1 1
=[ ]+[ ] [ 𝟐 −𝟐 𝟐] ,
0 1 2 −1
1−1 0+1 𝟑 𝟏 𝟑
=[ ] −𝟏 𝟎 𝟎
0+2 1−1
0 1 𝑪 = [ 𝟎 −𝟐 𝟑] , Verify the following rules:
=[ ] 𝟏 𝟏 𝟐
2 0
Answer. (i). 𝑨 + 𝑪 = 𝑪 + 𝑨
(iii). [𝟐 𝟑 𝟏] + ([𝟏 𝟎 𝟐] − [𝟐 𝟐 𝟐]) Solution.
Solution. 𝐿. 𝐻. 𝑆 = 𝐴 + 𝐶
[2 3 1] + ([1 0 2] − [2 2 2]) 1 2 3 −1 0 0
𝐿. 𝐻. 𝑆 = [ 2 3 1] + [ 0 −2 3]
= [2 3 1]
+ ([1 − 2 0 − 2 2 − 2]) 1 −1 0 1 1 2
1−1 2+0 3+0
= [2 3 1] + [−1 −2 0] 𝐿. 𝐻. 𝑆 = [ 2 + 0 3 − 2 1 + 3]
= [2 − 1 3 − 2 1 + 0] 1 + 1 −1 + 1 0 + 2
= [1 1 1] 0 2 3
Answer. 𝐿. 𝐻. 𝑆 = [ 2 1 4]
𝟏 𝟐 𝟑 𝟏 𝟏 𝟏 2 −0 2
(iv). [−𝟏 −𝟏 −𝟏] + [ 𝟐 𝟐 𝟐] 𝑅. 𝐻. 𝑆 = 𝐶 + 𝐴
𝟎 𝟏 𝟐 𝟑 𝟑 𝟑 −1 0 0 1 2 3
Solution. 𝑅. 𝐻. 𝑆 = [ 0 −2 3] + [ 2 3 1]
1 2 3 1 1 1 1 1 2 1 −1 0
[−1 −1 −1] + [ 2 2 2] −1 + 1 0 + 2 0 + 3
0 1 2 3 3 3 𝑅. 𝐻. 𝑆 = [ 0 + 2 −2 + 3 3 + 1]
1+1 2+1 3+1 1+1 1−1 2+0
= [−1 + 2 −1 + 2 −1 + 2] 0 2 3
0+3 1+3 2+3 𝐿. 𝐻. 𝑆 = [ 2 1 4]
2 3 4 2 −0 2
= [ 1 1 1] Hence Proved 𝐿. 𝐻. 𝑆 = 𝑅. 𝐻. 𝑆.
3 4 5 (ii). 𝑨 + 𝑩 = 𝑩 + 𝑨
Answer. Solution.
𝟏 𝟐 𝟑 𝟏 𝟎 −𝟐 𝐿. 𝐻. 𝑆 = 𝐴 + 𝐵
(vi). [ 𝟐 𝟑 𝟏] + [−𝟐 −𝟏 𝟎 ] 1 2 3 1 −1 1
𝟑 𝟏 𝟐 𝟎 𝟐 −𝟏 𝐿. 𝐻. 𝑆 = [ 2 3 1] + [ 2 −2 2]
Solution. 1 −1 0 3 1 3

6|Page
Class 9th Chapter 1 www.notes.pk.com
1+1 2−1 3+1 1 2 3 1 −1 1
𝐿. 𝐻. 𝑆 = [ 2 + 2 3 − 2 1 + 2] 𝑅. 𝐻. 𝑆 = 2 [ 2 3 1] + [ 2 −2 2]
1+3 −1 + 1 0 + 3 1 −1 0 3 1 3
2 1 4 2 4 6 1 −1 1
𝐿. 𝐻. 𝑆 = [4 1 3] 𝑅. 𝐻. 𝑆 = [ 4 6 2] + [ 2 −2 2]
4 0 3 2 −2 0 3 1 3
𝑅. 𝐻. 𝑆 = 𝐵 + 𝐴 2+1 4−1 6+1
1 −1 1 1 2 3 𝑅. 𝐻. 𝑆 = [ 4 + 2 6 − 2 2 + 2]
𝑅. 𝐻. 𝑆 = [ 2 −2 2] + [ 2 3 1] 2 + 3 −2 + 1 0 + 3
3 1 3 1 −1 0 3 3 7
1+1 −1 + 2 1 + 3 𝑅. 𝐻. 𝑆 = [6 4 4]
𝑅. 𝐻. 𝑆 = [ 2 + 2 −2 + 3 2 + 1] 5 −1 3
3+1 1−1 3+0 Hence Proved 𝐿. 𝐻. 𝑆 = 𝑅. 𝐻. 𝑆.
2 1 4 (v). (𝑪 − 𝑩) + 𝑨 = 𝑪 + (𝑨 − 𝑩)
𝐿. 𝐻. 𝑆 = [4 1 3] Solution.
4 0 3 𝐿. 𝐻. 𝑆 = (𝐶 − 𝐵) + 𝐴
Hence Proved 𝐿. 𝐻. 𝑆 = 𝑅. 𝐻. 𝑆. −1 0 0 1 −1 1
(iii). 𝑩 + 𝑪 = 𝑪 + 𝑩 𝐿. 𝐻. 𝑆 = ([ 0 −2 3] − [ 2 −2 2])
Solution. 1 1 2 3 1 3
𝑳. 𝐻. 𝑆 = 𝐵 + 𝐶 1 2 3
1 −1 1 −1 0 0 + [ 2 3 1]
𝐿. 𝐻. 𝑆 = [ 2 −2 2] + [ 0 −2 3] 1 −1 0
3 1 3 1 1 2 −1 − 1 0 + 1 0 − 1
1 − 1 −1 + 0 1 + 0 𝐿. 𝐻. 𝑆 = ([ 0 − 2 −2 + 2 3 − 2])
𝐿. 𝐻. 𝑆 = [ 2 + 0 −2 − 2 2 + 3] 1−3 1−1 2−3
3+1 1+1 3+2 1 2 3
0 −1 1 + [ 2 3 1]
𝐿. 𝐻. 𝑆 = [2 −4 5] 1 −1 0
4 2 5 −2 1 −1 1 2 3
𝑅. 𝐻. 𝑆 = 𝐶 + 𝐵 𝐿. 𝐻. 𝑆 = [ −2 0 1 ] + [ 2 3 1]
−1 0 0 1 −1 1 −2 0 −1 1 −1 0
𝑅. 𝐻. 𝑆 = [ 0 −2 3] + [ 2 −2 2] −2 + 1 1 + 2 −1 + 3
1 1 2 3 1 3 𝐿. 𝐻. 𝑆 = [−2 + 2 0 + 3 1 + 1 ]
−1 + 1 0 − 1 0 + 1 −2 + 1 0 − 1 −1 + 0
𝑅. 𝐻. 𝑆 = [ 0 + 2 −2 − 2 3 + 2] −1 3 2
1+3 1+1 2+3 𝐿. 𝐻. 𝑆 = [ 0 3 2]
0 −1 1 −1 −1 −1
𝑅. 𝐻. 𝑆 = [2 −4 5] 𝑅. 𝐻. 𝑆 = 𝐶 + (𝐴 − 𝐵)
4 2 5 −1 0 0
Hence Proved 𝐿. 𝐻. 𝑆 = 𝑅. 𝐻. 𝑆. 𝑅. 𝐻. 𝑆 = [ 0 −2 3]
(iv). 𝑨 + (𝑩 + 𝑨) = 𝟐𝑨 + 𝑩 1 1 2
1 2 3 1 −1 1
Solution. + ([ 2 3 1] − [ 2 −2 2])
𝐿. 𝐻. 𝑆 = 𝐴 + (𝐵 + 𝐴) 1 −1 0 3 1 3
1 2 3 −1 0 0
𝐿. 𝐻. 𝑆 = [ 2 3 1] 𝑅. 𝐻. 𝑆 = [ 0 −2 3]
1 −1 0 1 1 2
1 −1 1 1 2 3 1−1 2+1 3−1
+ ([ 2 −2 2] + [ 2 3 1]) + ([ 2 − 2 3 + 2 1 − 2])
3 1 3 1 −1 0 1 − 3 −1 − 1 0 − 3
1 2 3 1 + 1 −1 + 2 1 + 3 −1 0 0 0 3 2
𝐿. 𝐻. 𝑆 = [ 2 3 1] + ([ 2 + 2 −2 + 3 2 + 1]) 𝑅. 𝐻. 𝑆 = [ 0 −2 3] + [ 0 5 −1]
1 −1 0 3+1 1−1 3+0 1 1 2 −2 −2 −3
1 2 3 2 1 4 −1 + 0 0 + 3 0 + 2
𝐿. 𝐻. 𝑆 = [ 2 3 1] + [4 1 3] 𝑅. 𝐻. 𝑆 = [ 0 + 0 −2 + 5 3 − 1]
1 −1 0 4 0 3 1−2 1−2 2−3
1+2 2+1 3+4 −1 3 2
𝐿. 𝐻. 𝑆 = [2 + 4 3 + 1 1 + 3] 𝑅. 𝐻. 𝑆 = [ 0 3 2]
1 + 4 −1 + 0 0 + 3 −1 −1 −1
3 3 7 Hence Proved 𝐿. 𝐻. 𝑆 = 𝑅. 𝐻. 𝑆.
𝐿. 𝐻. 𝑆 = [6 4 4] (vi). 𝟐𝑨 + 𝑩 = 𝑨 + (𝑨 + 𝑩)
5 −1 3 Solution.
𝑅. 𝐻. 𝑆 = 2𝐴 + 𝐵
𝐿. 𝐻. 𝑆 = 2𝐴 + 𝐵

7|Page
Class 9th Chapter 1 www.notes.pk.com
1 2 3 1 −1 1 −1 0 0 1 −1 1
𝐿. 𝐻. 𝑆 = 2 [ 2 3 1] + [ 2 −2 2] 𝑅. 𝐻. 𝑆 = ([ 0 −2 3] − [ 2 −2 2])
1 −1 0 3 1 3 1 1 2 3 1 3
2 4 6 1 −1 1 1 2 3
𝐿. 𝐻. 𝑆 = [ 4 6 2] + [ 2 −2 2] − [ 2 3 1]
2 −2 0 3 1 3 1 −1 0
2+1 4−1 6+1 −1 − 1 0 + 1 0 − 1
𝐿. 𝐻. 𝑆 = [ 4 + 2 6 − 2 2 + 2] 𝑅. 𝐻. 𝑆 = ([ 0 − 2 −2 + 2 3 − 2])
2 + 3 −2 + 1 0 + 3 1−3 1−1 2−3
3 3 7 1 2 3
𝐿. 𝐻. 𝑆 = [6 4 4] − [ 2 3 1]
5 −1 3 1 −1 0
−2 1 −1 1 2 3
𝑅. 𝐻. 𝑆 = 𝐴 + (𝐴 + 𝐵) 𝑅. 𝐻. 𝑆 = [−2 0 1 ] − [ 2 3 1]
1 2 3 −2 0 −1 1 −1 0
𝐿. 𝐻. 𝑆 = [ 2 3 1] −2 − 1 1 − 2 −1 − 3
1 −1 0 𝑅. 𝐻. 𝑆 = [−2 − 2 0 − 3 1 − 1 ]
1 2 3 1 −1 1 −2 − 1 0 + 1 −1 − 1
+ ([ 2 3 1] + [ 2 −2 2]) −3 −1 −4
1 −1 0 3 1 3 𝑅. 𝐻. 𝑆 = [−4 −3 0 ]
1 2 3 −3 1 −1
𝑅. 𝐻. 𝑆 = [ 2 3 1] Hence Proved 𝐿. 𝐻. 𝑆 = 𝑅. 𝐻. 𝑆.
1 −1 0 (viii). (𝑨 + 𝑩) + 𝑪 = 𝑨 + (𝑩 + 𝑪)
1+1 2−1 3+1 Solution.
+ ([ 2 + 2 3 − 2 1 + 2]) 𝐿. 𝐻. 𝑆 = (𝐴 + 𝐵) + 𝐶
1! + 3 −1 + 1 0 + 3 1 2 3 1 −1 1
1 2 3 2 1 4 𝐿. 𝐻. 𝑆 = ([ 2 3 1] + [ 2 −2 2])
𝑅. 𝐻. 𝑆 = [ 2 3 1] + [4 1 3] 1 −1 0 3 1 3
1 −1 0 4 0 3 −1 0 0
1+2 2+1 3+4 + [ 0 −2 3]
𝑅. 𝐻. 𝑆 = [2 + 4 3 + 1 1 + 3] 1 1 2
1 + 4 −1 + 0 0 + 3 1+1 2−1 3+1 −1 0 0
3 3 7 𝐿. 𝐻. 𝑆 = [ 2 + 2 3 − 2 1 + 2] + [ 0 −2 3]
𝑅. 𝐻. 𝑆 = [6 4 4] 1 + 3 −1 + 1 0 + 3 1 1 2
5 −1 3 2 1 4 −1 0 0
Hence Proved 𝐿. 𝐻. 𝑆 = 𝑅. 𝐻. 𝑆. 𝐿. 𝐻. 𝑆 = [4 1 3] + [ 0 −2 3]
(vii). (𝑪 − 𝑩) − 𝑨 = (𝑪 − 𝑨) − 𝑩 4 0 3 1 1 2
Solution. 2−1 1+0 4+0
𝐿. 𝐻. 𝑆 = (𝐶 − 𝐵) − 𝐴 𝐿. 𝐻. 𝑆 = [4 + 0 1 − 2 3 + 6]
−1 0 0 1 −1 1 4+1 0+1 3+2
𝐿. 𝐻. 𝑆 = ([ 0 −2 3] − [ 2 −2 2]) 1 1 4
1 1 2 3 1 3 𝐿. 𝐻. 𝑆 = [4 −1 9]
1 2 3 5 1 5
− [ 2 3 1] 𝑅. 𝐻. 𝑆 = 𝐴 + (𝐵 + 𝐶)
1 −1 0 1 2 3
−1 − 1 0 + 1 0 − 1 𝑅. 𝐻. 𝑆 = [ 2 3 1]
𝐿. 𝐻. 𝑆 = ([ 0 − 2 −2 + 2 3 − 2]) 1 −1 0
1−3 1−1 2−3 1 −1 1 −1 0 0
1 2 3 + ([ 2 −2 2] + [ 0 −2 3])
− [ 2 3 1] 3 1 3 1 1 2
1 −1 0 1 2 3 1 − 1 −1 + 0 1 + 0
−2 1 −1 1 2 3 𝑅. 𝐻. 𝑆 = [ 2 3 1] + ([ 2 + 0 −2 − 2 2 + 3])
𝐿. 𝐻. 𝑆 = [ −2 0 1 ] − [ 2 3 1] 1 −1 0 3+1 1+1 3+2
−2 0 −1 1 −1 0 1 2 3 0 −1 1
−2 − 1 1 − 2 −1 − 3 𝑅. 𝐻. 𝑆 = [ 2 3 1] + [2 −4 5]
𝐿. 𝐻. 𝑆 = [−2 − 2 0 − 3 1 − 1 ] 1 −1 0 4 2 5
−2 − 1 0 + 1 −1 − 0 1+0 2−1 3+1
−3 −1 −4 𝑅. 𝐻. 𝑆 = [ 2 + 2 3 − 4 1 + 5]
𝐿. 𝐻. 𝑆 = [−4 −3 0 ] 1 + 4 −1 + 2 0 + 5
−3 1 −1 1 1 4
𝑅. 𝐻. 𝑆 = (𝐶 − 𝐴) − 𝐵 𝑅. 𝐻. 𝑆 = [4 −1 9]
5 1 5
Hence Proved 𝐿. 𝐻. 𝑆 = 𝑅. 𝐻. 𝑆.
(ix). 𝑨 + (𝑩 − 𝑪) = (𝑨 − 𝑪) + 𝑩

8|Page
Class 9th Chapter 1 www.notes.pk.com
Solution. 2 1 4
𝐿. 𝐻. 𝑆 = 𝐴 + (𝐵 − 𝐶) 𝑅. 𝐻. 𝑆 = 2 [4 1 3]
1 2 3 4 0 3
𝐿. 𝐻. 𝑆 = [ 2 3 1] 4 2 8
1 −1 0 𝑅. 𝐻. 𝑆 = [8 4 6]
1 −1 1 −1 0 0 7 0 6
+ ([ 2 −2 2] − [ 0 −2 3]) Hence Proved 𝐿. 𝐻. 𝑆 = 𝑅. 𝐻. 𝑆.
3 1 3 1 1 2 𝟏 −𝟐
Question.6. If 𝑨 = [ ] 𝒂𝒏𝒅 𝑩 =
1 2 3 1 + 1 −1 − 0 1 − 0 𝟑 𝟒
𝐿. 𝐻. 𝑆 = [ 2 3 1] + ([ 2 − 0 −2 + 2 2 − 3]) 0 7
[ ] , 𝑓𝑖𝑛𝑑
1 −1 0 3−1 1−1 3−2 −3 8
1 2 3 2 −1 1 (i). 3𝐴 − 2𝐵
𝐿. 𝐻. 𝑆 = [ 2 3 1] + [2 0 −1] Solution.
1 −1 0 2 0 1 1 −2 0 7
3𝐴 − 2𝐵 = 3 [ ]− 2[ ]
1+2 2−1 3+1 3 4 −3 8
𝐿. 𝐻. 𝑆 = [ 2 + 2 3 + 0 1 − 1] 3 −6 0 14
3𝐴 − 2𝐵 = [ ]−[ ]
1 + 2 −1 + 0 0 + 1 9 12 −6 16
3 1 4 3 − 0 −6 − 14
3𝐴 − 2𝐵 = [ ]
𝐿. 𝐻. 𝑆 = [4 3 0] 9 + 6 12 − 16
3 −20
3 −1 1 3𝐴 − 2𝐵 = [ ]
𝑅. 𝐻. 𝑆 = (𝐴 − 𝐶) + 𝐵 15 −4
1 2 3 −1 0 0 Answer.
𝑅. 𝐻. 𝑆 = ([ 2 3 1] − [ 0 −2 3]) (ii). 𝟐𝑨𝒕 − 𝟑𝑩𝒕
1 −1 0 1 1 2 Solution.
1 −1 1 1 −2 𝑡 0 7𝑡
+ [ 2 −2 2] 2𝐴𝑡 − 3𝐵𝑡 = 2 [ ] − 3[ ]
3 4 −3 8
3 1 3 1 3 0 −3
1+1 2−0 3−0 1 −1 1 = 2[ ]− 3[ ]
−2 4 7 8
𝑅. 𝐻. 𝑆 = ([ 2 − 0 3 + 2 1 − 3]) + [ 2 −2 2] 2 6 0 −9
=[ ]−[ ]
1 − 1 −1 − 1 0 − 2 3 1 3 −4 8 21 24
2 2 3 1 −1 1 2−0 6+9
=[ ]
𝑅. 𝐻. 𝑆 = [ 2 5 −2] + [ 2 −2 2] −4 − 21 8 − 24
0 −2 −2 3 1 3 2 15
=[ ]
2+1 2−1 3+1 −25 −16
𝑅. 𝐻. 𝑆 = [ 2 + 2 5 − 2 −2 + 2] Answer.
0 + 3 −2 + 1 −2 + 3 𝟐 𝟒 𝟏 𝒃
Question.7. If 𝟐 [ ] + 𝟑[ ]=
3 1 4 −𝟑 𝒂 𝟖 −𝟒
𝑅. 𝐻. 𝑆 = [4 3 0] 𝟕 𝟏𝟎
[ ] , 𝒇𝒊𝒏𝒅
3 −1 1 𝟏𝟖 𝟏
Hence Proved. 𝑳. 𝑯. 𝑺 = 𝑹. 𝑯. 𝑺. Solution.
(x). 𝟐𝑨 + 𝟐𝑩 = 𝟐(𝑨 + 𝑩) Given that
2 4 1 𝑏 7 10
Solution. 2[ ] + 3[ ]= [ ]
−3 𝑎 8 −4 18 1
𝐿. 𝐻. 𝑆 = 2𝐴 + 2𝐵 4 8 3 3𝑏 7 10
1 2 3 1 −1 1 [ ]+[ ]= [ ]
−6 2𝑎 24 −12 18 1
𝐿. 𝐻. 𝑆 = 2 [ 2 3 1] + 2 [ 2 −2 2] 4+3 8 + 3𝑏 7 10
[ ]= [ ]
1 −1 0 3 1 3 −6 + 24 2𝑎 − 12 18 1
2 4 6 2 −2 2 7 8 + 3𝑏 7 10
𝐿. 𝐻. 𝑆 = [ 4 6 2] + [ 4 −4 4] [ ]= [ ]
18 2𝑎 − 12 18 1
2 −2 0 6 2 6 By the definition of the equal matrix, we have
2+2 4−2 6+2 8 + 3𝑏 = 10 , 2𝑎 − 12 = 1
𝐿. 𝐻. 𝑆 = [ 4 + 4 6 − 4 2 + 4] 3𝑏 = 10 − 8 , 2𝑎 = 1 + 12
2 + 6 −2 + 2 0 + 6 2 13
4 2 8 𝑏= , 𝑎=
𝐿. 𝐻. 𝑆 = [8 4 6] 3 2
13 2
7 0 6 Hence 𝑎 = 2 𝑎𝑛𝑑 𝑏 = 3.
𝑅. 𝐻. 𝑆 = 2(𝐴 + 𝐵) 𝟏 𝟐
Question.8. If 𝑨 = [ ],𝑩 =
1 2 3 1 −1 1 𝟎 𝟏
𝐿. 𝐻. 𝑆 = 2 ([ 2 3 1] + [ 2 −2 2]) 𝟏 𝟏
[ ] , 𝒕𝒉𝒆𝒏 𝒗𝒆𝒓𝒊𝒇𝒚 𝒕𝒉𝒂𝒕
1 −1 0 3 1 3 𝟐 𝟎
1+1 2−1 3+1 (i). (𝑨 + 𝑩)𝒕 = 𝑨𝒕 + 𝑩𝒕
𝑅. 𝐻. 𝑆 = 2 ([ 2 + 2 3 − 2 1 + 2]) Solution.
1! + 3 −1 + 1 0 + 3 𝐿. 𝐻. 𝑆 = (𝐴 + 𝐵)𝑡
1 2 1 1 𝑡
𝐿. 𝐻. 𝑆 = ([ ]+[ ])
0 1 2 0

9|Page
Class 9th Chapter 1 www.notes.pk.com
1+1 2+1 𝑡 0 2
𝐿. 𝐻. 𝑆 = ([ ]) 𝐴 − 𝐴𝑡 = [ ] − − − (𝑖)
0+2 1+0 −2 0
𝑡 Now
2 3
𝐿. 𝐻. 𝑆 = [ ] 0 2𝑡
2 1 (𝐴 − 𝐴𝑡 )𝑡 = [ ]
2 2 −2 0
𝐿. 𝐻. 𝑆 = [ ] 0 −2
3 1 (𝐴 − 𝐴𝑡 )𝑡 = [ ]
𝑅. 𝐻. 𝑆 = 𝐴 + 𝐵𝑡
𝑡
2 0
1 2𝑡 1 1𝑡 0 2
𝑅. 𝐻. 𝑆 = [ ] +[ ] (𝐴 − 𝐴𝑡 )𝑡 = − [ ]
0 1 2 0 −2 0
1 0 1 2 Using equation (i), we have
𝑅. 𝐻. 𝑆 = [ ]+[ ] (𝐴 − 𝐴𝑡 )𝑡 = −(𝐴 − 𝐴𝑡 )
2 1 1 0
1+1 0+2 𝑡
Hence 𝐴 − 𝐴 𝑖𝑠 𝑆𝑘𝑒𝑤 − 𝑠𝑦𝑚𝑚𝑒𝑡𝑟𝑖𝑐.
𝑅. 𝐻. 𝑆 = [ ]
2+1 1+0 (iii). 𝑩 + 𝑩𝒕 𝒊𝒔 𝒔𝒚𝒎𝒎𝒆𝒕𝒓𝒊𝒄.
2 2
𝑅. 𝐻. 𝑆 = [ ] Solution.
3 1
Hence Proved. 𝐿. 𝐻. 𝑆 = 𝑅. 𝐻. 𝑆. 1 1 1 1𝑡
𝐵 + 𝐵𝑡 = [ ]+[ ]
(ii). (𝑨 − 𝑩)𝒕 = 𝑨𝒕 − 𝑩𝒕 2 0 2 0
1 1 1 2
Solution. 𝐵 + 𝐵𝑡 = [ ]+[ ]
2 0 1 0
𝐿. 𝐻. 𝑆 = (𝐴 − 𝐵)𝑡 1+1 1+2
𝐵 + 𝐵𝑡 = [ ]
1 2 1 1 𝑡 2+1 0+0
𝐿. 𝐻. 𝑆 = ([ ]−[ ]) 2 3
0 1 2 0 𝐵 + 𝐵𝑡 = [ ] − − − (𝑖)
1−1 2−1 𝑡 3 0
𝐿. 𝐻. 𝑆 = ([ ]) Now
0−2 1−0 𝑡
0 1𝑡 (𝐵 + 𝐵𝑡 )𝑡 = [2 3]
𝐿. 𝐻. 𝑆 = [ ] 3 0
−2 1
𝐿. 𝐻. 𝑆 = [
0 −2
] (𝐵 + 𝐵𝑡 )𝑡 = [2 3]
1 1 3 0
𝑡 𝑡
𝑅. 𝐻. 𝑆 = 𝐴 − 𝐵 Using equation (i), we have
1 2𝑡 1 1𝑡 (𝐵 + 𝐵𝑡 )𝑡 = 𝐵 + 𝐵𝑡
𝑅. 𝐻. 𝑆 = [ ] −[ ] 𝑡
Hence 𝐵 + 𝐵 𝑖𝑠 𝑠𝑦𝑚𝑚𝑒𝑡𝑟𝑖𝑐.
0 1 2 0
1 0 1 2 (iii). 𝑩 − 𝑩𝒕 𝒊𝒔 𝑺𝒌𝒆𝒘 − 𝒔𝒚𝒎𝒎𝒆𝒕𝒓𝒊𝒄.
𝑅. 𝐻. 𝑆 = [ ]−[ ]
2 1 1 0 Solution.
1−1 0−2
𝑅. 𝐻. 𝑆 = [ ] 1 1 1 1𝑡
2−1 1−0 𝐵 − 𝐵𝑡 = [ ]−[ ]
0 −2 2 0 2 0
𝑅. 𝐻. 𝑆 = [ ] 1 1 1 2
1 1 𝐵 − 𝐵𝑡 = [ ]−[ ]
Hence Proved. 𝐿. 𝐻. 𝑆 = 𝑅. 𝐻. 𝑆. 2 0 1 0
1−1 1−2
(iii). 𝑨 + 𝑨𝒕 𝒊𝒔 𝒔𝒚𝒎𝒎𝒆𝒕𝒓𝒊𝒄. 𝐵 − 𝐵𝑡 = [ ]
2−1 0−0
Solution. 0 −1
𝐵 − 𝐵𝑡 = [ ] − − − (𝑖)
1 2 1 2𝑡 1 0
𝐴 + 𝐴𝑡 = [ ]+[ ]
0 1 0 1 Now
1 2 1 0 0 −1 𝑡
𝐴 + 𝐴𝑡 = [ ]+[ ] (𝐵 − 𝐵𝑡 )𝑡 = [ ]
0 1 2 1 1 0
1+1 2+0
𝐴 + 𝐴𝑡 = [
0+2 1+1
] (𝐵 − 𝐵𝑡 )𝑡 = [ 0 1]
−1 0
2 2
𝐴 + 𝐴𝑡 = [
2 2
] − − − (𝑖) (𝐵 − 𝐵𝑡 )𝑡 = − [0 −1]
1 0
Now
𝑡
(𝐴 + 𝐴𝑡 )𝑡 = [2 2] Using equation (i), we have
2 2 (𝐵 − 𝐵𝑡 )𝑡 = −(𝐵 − 𝐵𝑡 )
(𝐴 + 𝐴 ) = [2 2]
𝑡 𝑡
2 2
Using equation (i), we have
(𝐴 + 𝐴𝑡 )𝑡 = 𝐴 + 𝐴𝑡
𝑡
Hence 𝐴 + 𝐴 𝑖𝑠 𝑠𝑦𝑚𝑚𝑒𝑡𝑟𝑖𝑐.
(iv). 𝑨 − 𝑨𝒕 𝒊𝒔 𝑺𝒌𝒆𝒘 − 𝒔𝒚𝒎𝒎𝒆𝒕𝒓𝒊𝒄.
Solution.
1 2 1 2𝑡
𝐴 − 𝐴𝑡 = [ ]−[ ]
0 1 0 1
1 2 1 0
𝐴 − 𝐴𝑡 = [ ]−[ ]
0 1 2 1
1−1 2−0
𝐴 − 𝐴𝑡 = [ ]
0−2 1−1

10 | P a g e
Class 9th Chapter 1 www.notes.pk.com
Multiplication of Matrices: Sol:
Two matrices A and B are conformable for 6 3 0
𝐵𝐴 = [ ] [ ]
multiplication if 5 −1 2
Since
No of col of 𝐴 =No. Of Rows of 𝐵
No of col of 𝐴 = 1 ≠ 2 =No. Of Rows of 𝐵
Multiplication is not possible.
Exercise1.4 Q#3) Find the following products.
Q#1) Which of the following product matrices is 𝟒
(i). [𝟏 𝟐] [ ]
conformable for multiplication?. 𝟎
𝟏 −𝟏 −𝟐 𝟒
(i). [ ][ ] Sol: [𝟏 𝟐] [ ]
𝟎 𝟐 𝟑 𝟎
Sol: = [(1)(4) + (2)(0)]
Conformable for multiplication because = [4 + 0]
No of col of 1st Matrix= 2 =No. Of Rows of 2nd = [4]
𝟓
Matrix (ii). [𝟏 𝟐] [ ]
𝟏 −𝟏 𝟐 −𝟏 −𝟒
(ii). [ ][ ] 𝟓
𝟏 𝟎 𝟏 𝟑 Sol: [𝟏 𝟐] [ ]
Sol: −𝟒
= [(1)(5) + (2)(−4)]
Conformable for multiplication because = [5 − 8]
No of col of 1st Matrix= 2 =No. Of Rows of 2nd = [−3]
Matrix 𝟒
𝟏 𝟎 𝟏 (iii). [−𝟑 𝟎] [ ]
(iii). [ ] [ ] 𝟎
−𝟏 −𝟏 𝟐 𝟒
Sol: [−𝟑 𝟎] [ ]
Sol: 𝟎
Not conformable for multiplication because = [(−3)(4) + (0)(0)]
No of col of 1st Matrix= 1 ≠ 2 =No. Of Rows of = [−12 + 0]
2nd Matrix = [−12]
𝟏 𝟐 𝟒
𝟏 𝟎 −𝟏 (iv). [𝟔 𝟎] [ ]
(iv). [ 𝟎 −𝟏] [ ] 𝟎
𝟎 𝟏 𝟐 4
−𝟏 −𝟐 [
Sol: 6 0 [ ] ]
Sol: 0
= [(6)(4) + (0)(0)]
Conformable for multiplication because
= [24 + 0]
No of col of 1st Matrix= 2 =No. Of Rows of 2nd = [24]
Matrix 𝟏 𝟐
𝟏 −𝟏 𝟒 𝟓
𝟑 𝟐 𝟏 (v). [−𝟑 𝟎 ] [ ]
(v). [ ][ 𝟎 𝟐] 𝟎 −𝟒
𝟎 𝟏 −𝟏 𝟔 −𝟏
−𝟐 𝟑 1 2
Sol: 4 5
Sol: [−3 0 ] [ ]
Conformable for multiplication because 0 −4
6 −1
No of col of 1st Matrix= 3 =No. Of Rows of 2nd (1)(4) + (2)(0) (1)(5) + (2)(−4)
Matrix = [(−3)(4) + (0)(0) (−3)(5) + (0)(−4)]
𝟑 𝟎 𝟔 (6)(4) + (−1)(0) (6)(5) + (−1)(−4)
Q#2) If 𝑨 = [ ], 𝑩 = [ ]
−𝟏 𝟐 𝟓 4+0 5−8
Find (i). 𝑨𝑩 = [−12 + 0 −15 + 0]
(ii). 𝑩𝑨 (if possible) 24 + 0 30 + 4
(i). 𝑨𝑩 4 −3
Sol: = [−12 −15]
3 0 6 24 34
𝐴𝐵 = [ ][ ]
−1 2 5
(3)(6) + (0)(5)
=[ ]
(−1)(6) + (2)(5)
18 + 0 Q#4) Multiply the following matrices.
=[ ]
−6 + 10 𝟐 𝟑
18 𝟐 −𝟏
=[ ] (a). [𝟏 𝟏 ] [ ]
4 𝟑 𝟎
(ii). 𝑩𝑨 (if possible) 𝟎 −𝟐

11 | P a g e
Class 9th Chapter 1 www.notes.pk.com
𝟐 𝟑 (−1)(0) + (2)(0) (−1)(0) + (2)(0)
𝟐 −𝟏 =[ ]
Sol: [𝟏 𝟏 ] [ ] (1)(0) + (1)(0) (1)(0) + (1)(0)
𝟑 𝟎
𝟎 −𝟐 0+0 0+0
(2)(2) + (3)(3) (2)(−1) + (3)(0) =[ ]
0+0 0+0
= [ (1)(2) + (1)(3) (1)(−1) + (1)(0) ] 0 0
=[ ]
(0)(2) + (−2)(3) (0)(−1) + (−2)(0) 0 0
−𝟏 𝟑 𝟏 𝟐
4 + 9 −2 + 0 Q#5) Let𝑨 = [ ], 𝑩 = [ ] and
𝟐 𝟎 −𝟑 −𝟓
= [2 + 3 −1 + 0] 𝟐 𝟏
0−6 0+0 𝑪=[ ], Verify that
𝟏 𝟑
13 −2 (i). 𝑨𝑩 = 𝑩𝑨
= [ 5 −1] Sol: 𝑳. 𝑯. 𝑺 = 𝑨𝑩
−6 0 −1 3 1 2
𝟏 𝟐 =[ ][ ]
𝟏 𝟐 𝟑 2 0 −3 −5
(b). [ ] [ 𝟑 𝟒] (−1)(1) + (3)(−3) (−1)(2) + (3)(−5)
𝟒 𝟓 𝟔 =[ ]
−𝟏 𝟏 (2)(1) + (0)(−3) (2)(2) + (0)(−5)
1 2
1 2 3 −1 − 9 −2 − 15
Sol: [ ] [ 3 4] =[ ]
4 5 6 2+0 4+0
−1 1 −10 −17
=[
(1)(1) + (2)(3) + (3)(−1) (1)(2) + (2)(4) + (3)(1)
]
=[ ] →(1)
(4)(1) + (5)(3) + (6)(−1) (4)(2) + (5)(4) + (6)(1) 2 4
1+6−3 2+8+3 : 𝑅. 𝐻. 𝑆 = 𝐵𝐴
=[ ] 1 2 −1 3
4 + 15 − 6 8 + 20 + 6 =[ ][ ]
4 13 −3 −5 2 0
=[ ] (1)(−1) + (2)(2) (1)(3) + (2)(0)
13 34 =[ ]
𝟏 𝟐 (−3)(−1) + (−5)(2) (−3)(3) + (−5)(0)
𝟏 𝟐 𝟑 −1 + 4 3 + 0
(c). [ 𝟑 𝟒] [ ] =[ ]
𝟒 𝟓 𝟔 3 − 10 −6 + 0
−𝟏 𝟏
1 2 3 3
1 2 3 =[ ] →(2)
Sol: [ 3 4] [ ] −7 −6
4 5 6 From (1) and (2), we have
−1 1
(1)(1) + (2)(4) (1)(2) + (2)(5) (1)(3) + (2)(6) 𝐴𝐵 ≠ 𝐵𝐴
= [ (3)(1) + (4)(4) (3)(2) + (4)(5) (3)(3) + (4)(6) ] (ii). 𝑨(𝑩𝑪) = (𝑨𝑩)𝑪
(−1)(1) + (1)(4) (−1)(2) + (1)(5) (−1)(3) + (1)(6) Sol:
1 + 8 2 + 10 3 + 12 𝐿. 𝐻. 𝑆 = 𝐴(𝐵𝐶)
= [ 3 + 16 6 + 20 9 + 24 ] −1 3 1 2 2 1
=[ ] ([ ][ ])
−1 + 4 −2 + 5 −3 + 6 2 0 −3 −5 1 3
9 12 15 =[
−1 3
] ([
(1)(2) + (2)(1) (1)(1) + (2)(3)
])
= [19 26 33] 2 0 (−3)(2) + (−5)(1) (−3)(1) + (−5)(3)
3 3 3 −1 3 2 + 2 1+6
=[ ][ ]
2 0 −6 − 5 −3 − 15
𝟓 −1 3 4 7
𝟖 𝟓 𝟐
− 𝟐] =[ ][ ]
(d). [ ][ 2 0 −11 −18
𝟔 𝟒 −𝟒
𝟒 (−1)(4) + (3)(−11) (−1)(7) + (3)(−18)
5 =[ ]
8 5 2 − (2)(4) + (0)(−11) (2)(7) + (0)(−18)
Sol: [ ][ 2] −4 − 33 −7 − 54
6 4 −4 4 =[ ]
= 8+0 14 + 0
−37 −61
5 =[ ] →(1)
(8)(2) + (5)(−4) (8) (− ) + (5)(4) 8 14
[ 2 ] 𝑅. 𝐻. 𝑆 = (𝐴𝐵)𝐶
5 −1 3 1 2 2 1
(6)(2) + (4)(−4) (6) (− ) + (4)(4) = ([ ][ ]) [ ]
2 2 0 −3 −5 1 3
16 − 20 −20 + 20 (−1)(1) + (3)(−3) (−1)(2) + (3)(−5) 2 1
=[ ] = ([ ]) [ ]
(2)(1) + (0)(−3) (2)(2) + (0)(−5) 1 3
12 − 16 −15 + 16
−4 0 −1 − 9 −2 − 15 2 1
=[ ] =[ ][ ]
−4 1 2+0 4+0 1 3
−𝟏 𝟐 𝟎 𝟎 −10 −17 2 1
(e). [ ][ ] =[ ][ ]
𝟏 𝟑 𝟎 𝟎 2 4 1 3
−1 2 0 0 (−10)(2) + (−17)(1) (−10)(1) + (−17)(3)
Sol: [ ][ ] =[ ]
1 3 0 0 (2)(2) + (4)(1) (2)(1) + (4)(3)

12 | P a g e
Class 9th Chapter 1 www.notes.pk.com
−20 − 17 −10 − 51 (−1)(1) + (3)(−3) (−1)(2) + (3)(−5)
=[ ] =[ ]
4+4 2 + 12 (2)(1) + (0)(−3) (2)(2) + (0)(−5)
−37 −61 (−1)(2) + (3)(1) (−1)(1) + (3)(3)
=[ ]…(2) −[ ]
8 14 (2)(2) + (0)(1) (2)(1) + (0)(3)
From (1) and (2), we have −1 − 9 −2 − 15 −2 + 3 −1 + 9
𝐴(𝐵𝐶) = (𝐴𝐵)𝐶 =[ ]−[ ]
2+0 4+0 4+0 2+0
(iii). 𝑨(𝑩 + 𝑪) = 𝑨𝑩 + 𝑨𝑪 −10 −17 1 8
=[ ]−[ ]
Sol: 2 4 4 2
𝐿. 𝐻. 𝑆 = 𝐴(𝐵𝐶) −10 − 1 −17 − 8
=[ ]
−1 3 1 2 2 1 2−4 4−2
=[ ] ([ ]+[ ]) −11 −25
2 0 −3 −5 1 3 =[ ] →(2)
−1 3 1 + 2 2+1 −2 2
=[ ][ ] From (1) and (2), we have
2 0 −3 + 1 −5 + 3
−1 3 3 3 𝐴(𝐵 − 𝐶) = 𝐴𝐵 − 𝐴𝐶
=[ ][ ]
2 0 −2 −2 Q#6) For the matrices𝐴 = [−1 3], 𝐵 = [ 1 2
]
(−1)(3) + (3)(−2) (−1)(3) + (3)(−2) 2 0 −3 −5
=[ ] −2 6
(2)(3) + (0)(−2) (2)(3) + (0)(−2) and 𝐶 = [ ], verify that
−3 − 6 −3 − 6 3 −9
=[ ] (i). (𝑨𝑩)𝒕 = 𝑩𝒕 𝑨𝒕
6+0 6+0
−9 −9 Sol: : 𝑳. 𝑯. 𝑺 = (𝑨𝑩)𝒕
=[ ] →(1)
6 6 First we find 𝐴𝐵
𝑅. 𝐻. 𝑆 = 𝐴𝐵 + 𝐵𝐶 −1 3 1 2
−1 3 1 2 −1 3 2 1 𝐴𝐵 = [ ][ ]
= ([ ][ ]) + ([ ][ ]) 2 0 −3 −5
2 0 −3 −5 2 0 1 3 (−1)(1) + (3)(−3) (−1)(2) + (3)(−5)
(−1)(1) + (3)(−3) (−1)(2) + (3)(−5) =[ ]
=[ ] (2)(1) + (0)(−3) (2)(2) + (0)(−5)
(2)(1) + (0)(−3) (2)(2) + (0)(−5) −1 − 9 −2 − 15
(−1)(2) + (3)(1) (−1)(1) + (3)(3) =[ ]
2+0 4+0
+[ ] −10 −17
(2)(2) + (0)(1) (2)(1) + (0)(3) AB = [ ]
−1 − 9 −2 − 15 −2 + 3 −1 + 9 2 4
=[ ]+[ ] Taking transpose on both side
2+0 4+0 4+0 2+0 𝑡
−10 −17 1 8
=[ ]+[ ] (AB)t = [−10 −17]
2 4 4 2 2 4
−10 + 1 −17 + 8 t −10 2
=[ ] (AB) = [ ]…(1)
2+4 4+2 −17 4
−9 −9 : 𝑅. 𝐻. 𝑆 = 𝐵 𝐴 𝑡 𝑡
=[ ] →(2)
6 6 1 2 𝑡 −1 3 𝑡
From (1) and (2), we have =[ ] [ ]
−3 −5 2 0
𝐴(𝐵 + 𝐶) = 𝐴𝐵 + 𝐴𝐶 1 −3 −1 2
=[ ][ ]
(iv). 𝑨(𝑩 − 𝑪) = 𝑨𝑩 − 𝑨𝑪 2 −5 3 0
Sol: (1)(−1) + (−3)(3) (1)(2) + (−3)(0)
=[ ]
𝐿. 𝐻. 𝑆 = 𝐴(𝐵𝐶) (2)(−1) + (−5)(3) (2)(2) + (−5)(0)
−1 3 1 2 2 1 −1 − 9 2 + 0
=[ ] ([ ]−[ ]) =[ ]
2 0 −3 −5 1 3 −2 − 15 4 + 0
−1 3 1 − 2 2−1 −10 2
=[ ][ ] 𝐵 𝑡 𝐴𝑡 = [ ] →(2)
2 0 −3 − 1 −5 − 3 −17 4
−1 3 −1 1 From (1) and (2), we have
=[ ][ ]
2 0 −4 −8 (𝐴𝐵)𝑡 = 𝐵 𝑡 𝐴𝑡
(−1)(−1) + (3)(−4) (−1)(1) + (3)(−8) (ii). (𝑩𝑪)𝒕 = 𝑪𝒕 𝑩𝒕
=[ ]
(2)(−1) + (0)(−4) (2)(1) + (0)(−8) Sol: 𝑳. 𝑯. 𝑺 = (𝑩𝑪)𝒕
1 − 12 −1 − 24 First we find 𝐵𝐶
=[ ]
−2 + 0 2+0 1 2 −2 6
−11 −25 𝐵𝐶 = [ ][ ]
=[ ]…(1) −3 −5 3 −9
−2 2 (1)(−2) + (2)(3) (1)(6) + (2)(−9)
𝑅. 𝐻. 𝑆 = 𝐴𝐵 − 𝐵𝐶 =[ ]
−1 3 1 2 −1 3 2 1 (−3)(−2) + (−5)(3) (−3)(6) + (−5)(−9)
= ([ ][ ]) − ([ ][ ]) −2 + 6 6 − 18
2 0 −3 −5 2 0 1 3 =[ ]
6 − 15 −18 + 45
4 −12
=[ ] →(2)
−9 27
Taking transpose on both side

13 | P a g e
Class 9th Chapter 1 www.notes.pk.com
𝑡 0 −4
(AB)t = [ 4 −12] 𝐴𝑑𝑗𝐴 = [
−2 −1
]
−9 27
(AB) = [ 4
t −9
] →(1)
−12 27 Exercise 1.5
: 𝑅. 𝐻. 𝑆 = 𝐶 𝑡 𝐵 𝑡
Q#1) Find the determinant of the following
−2 6 𝑡 1 2 𝑡
=[ ] [ ] matrices.
3 −9 −3 −5 −𝟏 𝟏
−2 3 1 −3 (i). 𝑨 = [ ]
=[ ][ ] 𝟐 𝟎
6 −9 2 −5
(−2)(1) + (3)(2) (−2)(−3) + (3)(−5) Sol:
=[ ]
(6)(1) + (−9)(2) (6)(−3) + (−9)(−5) |𝐴| = |−1 1|
−2 + 6 6 − 15 2 0
=[ ] = (−1)(0) − (1)(2)
6 − 18 −18 + 45
4 −9 = 0 − 2 = −2
𝐶 𝑡 𝐵𝑡 = [ ] →(2) 𝟏 𝟑
−12 27 (ii). 𝑩 = [ ]
From (1) and (2), we have 𝟐 −𝟐
(𝐵𝐶)𝑡 = 𝐶 𝑡 𝐵 𝑡 Sol:
Determinant of 2×2 matrix: |𝐵| = |1 3 |
𝑎 𝑏 2 −2
Let 𝐴=[ ] be 2×2 square matrix, the = (1)(−2) − (3)(2)
𝑐 𝑑
determinant of 𝐴 is denoted by |𝐴| or detA = −2 − 6 = −8
𝟑 𝟐
And given as (iii). 𝑪 = [ ]
𝟑 𝟐
|𝐴| = |𝑎 𝑏 | Sol:
𝑐 𝑑
= (𝑎)(𝑑) − (𝑏)(𝑐) |𝐶| = |3 2|
3 2
= 𝑎𝑑 − 𝑏𝑐 = (3)(2) − (3)(2)
−𝟏 𝟏
For example, 𝑨 = [ ] =6−6=0
𝟐 𝟎 𝟑 𝟐
|𝐴| = |−1 1| (iv). 𝑫 = [
𝟏 𝟒
]
2 0
= (−1)(0) − (1)(2) Sol:
= 0 − 2 = −2 |𝐷| = |3 2|
1 4
Singular and Non-singular matrices: = (3)(4) − (2)(1)
Singular matrix: = 12 − 2 = 10
A square matrix 𝐴 is called Singular matrix if its Q#2)
determinant is zero i.e.|𝐴| = 0 Find which of the following matrices are singular
3 2
For example, 𝐴 = [ ] or non-singular?
3 2 𝟑 𝟔
|𝐴| = |3 2| (i). 𝑨 = [
𝟐 𝟒
]
3 2 Sol:
= (3)(2) − (3)(2)
=6−6=0 |𝐴| = |3 6|
2 4
Non-Singular matrix: = (3)(4) − (6)(2)
A square matrix 𝐴 is called Non-Singular matrix if = 12 − 12 = 0
its determinant is not zero i.e.|𝐴| ≠ 0 Hence, matrix 𝐴 is singular matrix.
−1 1 4 1
For example, 𝐴 = [ ] (ii). 𝐵 = [ ]
2 0 3 2
|𝐴| = |−1 1| Sol:
2 0
= (−1)(0) − (1)(2) |𝐵| = |4 1|
3 2
= 0 − 2 = −2 ≠ 0 = (4)(2) − (1)(3)
Adjoint of Matrix A: =8−3=5
𝑎 𝑏 Which is not zero and hence, matrix 𝐴 is Non-
“Adjoint of a square matrix 𝐴=[ ] is obtained
𝑐 𝑑 singular matrix.
by interchanging the diagonal entries and
𝟕 −𝟗
changing the sign of other entries.” (iii). 𝑪 = [ ]
𝟑 𝟓
−1 4
For example, 𝐴 = [ ] Sol:
2 0

14 | P a g e
Class 9th Chapter 1 www.notes.pk.com
|𝐶| = |7 −9| −𝟐 𝟔
(iii). 𝑪 = [ ]
3 5 𝟑 −𝟗
= (7)(5) − (−9)(3) Sol: First we find the determinant of 𝐶 as
= 35 + 27 = 62 |𝐶| = |−2 6 |
Which is not zero and hence, matrix 𝐴 is Non- 3 −9
singular matrix. = (−2)(−9) − (3)(6)
𝟓 −𝟏𝟎 = 18 − 18 = 0
(iv). 𝑫 = [ ] Which is zero and hence, matrix 𝐶 is singular
−𝟐 𝟒
Sol: matrix and 𝐶 −1 does not exist.
|𝐷| = | 5 −10|
𝟏 𝟑
−2 4 (iv). 𝑫 = [ 𝟐 𝟒 ]
= (5)(4) − (−10)(−2) 𝟏 𝟐
Sol: First we find the determinant of 𝐷 as
= 20 − 20 = 0
1 3
Hence, matrix 𝐴 is singular matrix.
|𝐷| = |2 4|
Q#3) Find the multiplicative inverse (if exists) of
1 2
each: 1 3 3
−𝟏 𝟑 = ( ) (2) − ( ) (1) = 1 −
(i). 𝑨 = [ ] 2 4 4
𝟐 𝟎 4−3 1
Sol: First we find the determinant of 𝐴 as = =
4 4
|𝐴| = |−1 3| Which is not zero and hence, matrix 𝐷 is Non-
2 0
= (−1)(0) − (3)(2) singular matrix and 𝐷−1 exist.
3
= 0 − 6 = −6 2 −4
Which is not zero and hence, matrix 𝐴 is Non- Now, 𝐴𝑑𝑗𝐷 = [ 1 ]
singular matrix and 𝐴−1 exist. −1 2
0 −3 As
Now, 𝐴𝑑𝑗𝐴 = [ ]
−2 −1 1
As 𝐷−1 = 𝐴𝑑𝑗𝐷
|D|
1
𝐴−1 = 𝐴𝑑𝑗𝐴 Putting values
|𝐴| 3 3
Putting values 1 2 − 4 2 −
𝐷−1 = [ 4] = [ 4]
0 −3 1 1 1 1
𝐴−1 =
1
[
0 −3
] = [−6 −6] 4 −1 2 −1
2
−6 −2 −1 −2 −1 −1 8 −3
𝐷 =[ ]
−6 −6 −4 2
1 𝟏 𝟐 𝟑 −𝟏
0 Q#4) If 𝑨 = [ ] and 𝑩 = [ ], then
−1 2] 𝟒 𝟔 𝟐 −𝟐
𝐴 =[ (i). 𝑨(𝑨𝒅𝒋𝑨) = (𝑨𝒅𝒋𝑨)𝑨 = (𝒅𝒆𝒕𝑨)𝑰
1 1
Sol: First we find the determinant of 𝐴 as
3 6
(ii). 𝑩 = [
𝟏 𝟐
] |𝐴| = |1 2|
−𝟑 −𝟓 4 6
Sol: First we find the determinant of 𝑩 as = (1)(6) − (2)(4)
= 6 − 8 = −2
|𝐵| = | 1 2
| 6 −2
−3 −5 Now, 𝐴𝑑𝑗𝐴 = [ ]
= (1)(−5) − (2)(−3) −4 1
1 2 6 −2
= −5 + 6 = 1 Let 𝐴(𝐴𝑑𝑗𝐴) = [ ][ ]
4 6 −4 1
Which is not zero and hence, matrix 𝐵 is Non- (1)(6) + (2)(−4) (1)(−2) + (2)(1)
singular matrix and 𝐵 −1 exist. =[ ]
(4)(6) + (6)(−4) (4)(−2) + (6)(1)
−5 −2 6−8 −2 + 2
Now, 𝐴𝑑𝑗𝐵 = [ ] =[ ]
3 1 24 − 24 −8 + 6
As −2 0
1 𝐴(𝐴𝑑𝑗𝐴) = [ ]…(1)
0 −2
𝐵 −1 = 𝐴𝑑𝑗𝐵 6 −2 1 2
|𝐵| And (𝐴𝑑𝑗𝐴)𝐴 = [ ][ ]
Putting values −4 1 4 6
(6)(1) + (−2)(4) (6)(2) + (−2)(6)
1 −5 −2 −5 −2 =[ ]
𝐵 −1 = [ ]=[ ] (−4)(1) + (1)(4) (−4)(2) + (1)(6)
1 3 1 3 1

15 | P a g e
Class 9th Chapter 1 www.notes.pk.com
6 − 8 12 − 12 1 2 −3 2
=[ ] [ ][ ]
−4 + 4 −8 + 6 2 3 2 −1
(𝐴𝑑𝑗𝐴)A = [−2 0 ]…(2) =[
(1)(−3) + (2)(2) (1)(2) + (2)(−1)
]
0 −2 (2)(−3) + (3)(2) (2)(2) + (3)(−1)
1 0 −2 0 −3 + 4 2 − 2
Also, (𝑑𝑒𝑡𝐴)𝐼 = −2 [ ]=[ ]…(3) =[ ]
0 1 0 −2 −6 + 6 4 − 3
From Eq(1), (2) and (3), we have 1 0
𝐴(𝐴𝑑𝑗𝐴) = (𝐴𝑑𝑗𝐴)𝐴 = (𝑑𝑒𝑡𝐴)𝐼 =[ ]
0 1
(ii). 𝑩𝑩−𝟏 = 𝑩−𝟏 𝑩 = 𝑰 Yes, the given matrices are multiplicative inverse
Sol: First we find the determinant of of each other.
𝟒 𝟎 −𝟒 −𝟐
|𝐵| = |3 −1| Q#6) If 𝑨 = [ ], 𝑩 = [ ] and 𝑫 =
2 −2 −𝟏 𝟐 𝟏 −𝟏
= (3)(−2) − (−1)(2) 𝟑 𝟏
[ ], then verify that
= −6 + 2 = −4 −𝟐 𝟐
−2 1 (i). (𝑨𝑩)−𝟏 = 𝑩−𝟏 𝑨−𝟏
Now, 𝐴𝑑𝑗𝐵 = [ ] Sol: L.H.S=(𝑨𝑩)−𝟏
−2 3
As First we find
1 4 0 −4 −2
𝐵 −1 = 𝐴𝑑𝑗𝐵 𝐴𝐵 = [ ][ ]
|𝐵| −1 2 1 −1
(4)(−4) + (0)(1) (4)(−2) + (0)(−1)
Putting values =[ ]
1 −2 1 (−1)(−4) + (2)(1) (−1)(−2) + (2)(−1)
𝐵 −1 = [ ] =[
−16 + 0 −8 + 0
]
−4 −2 3 4+2 2−2
1 3 −1 −2 1
Let 𝐵𝐵 −1 = −4 [ ][ ] =[
−16 −8
]
2 −2 −2 3 6 0
1 (3)(−2) + (−1)(−2) (3)(1) + (−1)(3)
= [ ] Now, we find the its determinant
−4 (2)(−2) + (−2)(−2) (2)(1) + (−2)(3)
1 −6 + 2 3 − 3 |𝐴𝐵| = |−16 −8|
= [ ] 6 0
−4 −4 + 4 2 − 6 = (−16)(0) − (−8)(6)
1 −4 0 1 0
= −4 [ ]=[ ] = 𝐼 →(1) = 0 − (−48) = 48
0 −4 0 1 Which is not zero and hence, matrix 𝐴𝐵 is Non-
1 −2 1 3 −1
Also 𝐵 −1 𝐵 = −4 [ ][ ] singular matrix and (𝐴𝐵)−1 exist.
−2 3 2 −2
1 (−2)(3) + (1)(2) (−2)(−1) + (1)(−2) 0 8
= [ ] Now, 𝐴𝑑𝑗𝐴𝐵 = [ ]
−4 (−2)(3) + (3)(2) (−2)(−1) + (3)(−2) −6 −16
1 −6 + 2 2 − 2 As
= [ ] 1
−4 −6 + 6 2 − 6 (𝐴𝐵)−1 = 𝐴𝑑𝑗𝐴𝐵
1 −4 0 1 0 |𝐴𝐵|
= −4 [ ]=[ ] = 𝐼 →(2)
0 −4 0 1 Putting values
From (1) and (2), we have 1 0 8
𝐿. 𝐻. 𝑆 = (𝐴𝐵)−1 = 48 [ ]…(1)
𝐵𝐵 −1 = 𝐵 −1 𝐵 = 𝐼. −6 −16
Q#5) Determine whether the given matrices are 𝑅. 𝐻. 𝑆 = 𝐵 −1 𝐴−1
multiplicative inverse of each other or not. First, we find 𝐵 −1 and 𝐴−1
(i). [
𝟑 𝟓
] and [
𝟕 −𝟓
] |𝐴| = | 4 0|
𝟒 𝟕 −𝟒 𝟑 −1 2
Sol: = (4)(2) − (0)(−1)
3 5 7 −5 =8−0=8
[ ][ ] Which is not zero and hence, matrix 𝐴 is Non-
4 7 −4 3
(3)(7) + (5)(−4) (3)(−5) + (5)(3) singular matrix and 𝐴−1 exist.
=[ ]
(4)(7) + (7)(−4) (4)(−5) + (7)(3) 2 0
Now, 𝐴𝑑𝑗𝐴 = [ ]
21 − 20 −15 + 15 1 4
=[ ] 1
28 − 28 −20 + 21 𝐴−1 = 𝐴𝑑𝑗𝐴
1 0 |𝐴|
=[ ]
0 1 Putting values
Yes, the given matrices are multiplicative inverse 1 2 0
of each other. 𝐴−1 = [ ]
8 1 4
𝟏 𝟐 −𝟑 𝟐 −4 −2
(ii). [ ] and [ ] Also, |𝐵| = | |
𝟐 𝟑 𝟐 −𝟏 1 −1
Sol: = (−4)(−1) − (−2)(1)
16 | P a g e
Class 9th Chapter 1 www.notes.pk.com
=4+2=6 2 0
Now, 𝐴𝑑𝑗𝐴 = [ ]
Which is not zero and hence, matrix 𝐵 is Non- 1 4
1
singular matrix and 𝐵 −1 exist. 𝐴−1 = 𝐴𝑑𝑗𝐴
−1 2 |𝐴|
Now, 𝐴𝑑𝑗𝐵 = [ ] Putting values
−`1 −4
1 1 2 0
𝐵 −1 = 𝐴𝑑𝑗𝐵 𝐴−1 = [ ]
|𝐵| 8 1 4
Putting values 3 1
Also, |𝐷| = | |
1 −1 2 −2 2
𝐵 −1 = [ ] = (3)(2) − (1)(−2)
6 −`1 −4
𝑅. 𝐻. 𝑆 = 𝐵 −1 𝐴−1 =6+2=8
1 −1 Which is not zero and hence, matrix 𝐷 is Non-
2 1 2 0
= [ ] [ ] singular matrix and 𝐷−1 exist.
6 −`1 −4 8 1 4
1 2 −1
−1 2 2 0 Now, 𝐴𝑑𝑗𝐷 = [ ]
= [ ][ ] −2 3
8 × 6 −`1 −4 1 4 1
1 (−1)(2) + (2)(1) (−1)(0) + (2)(4) 𝐷−1 = 𝐴𝑑𝑗𝐷
= [ ] |𝐷|
48 (−1)(2) + (−4)(1) (−1)(0) + (−4)(4)
1 −2 + 2 0 + 8 Putting values
= [ ] 1 2 −1
48 −2 − 4 0 − 16 𝐷−1 = [ ]
1 0 8 8 −2 3
= 48 [ ]…(2) 𝑅. 𝐻. 𝑆 = 𝐴−1 𝐷−1
−6 −16
From (1) and (2), we have 1 2 0 1 2 −1
= [ ] [ ]
(𝑨𝑩)−𝟏 = 𝑩−𝟏 𝑨−𝟏 8 1 4 8 −2 3
(ii). (𝑫𝑨)−𝟏 = 𝑨−𝟏 𝑫−𝟏 1 2 0 2 −1
= [ ][ ]
Sol: L.H.S=(𝑫𝑨)−𝟏 8 × 8 1 4 −2 3
1 (2)(2) + (0)(−2) (2)(−1) + (0)(3)
First we find = [ ]
3 1 4 0 64 (1)(2) + (4)(−2) (1)(−1) + (4)(3)
𝐷𝐴 = [ ][ ] 1 4 + 0 −2 + 0
−2 2 −1 2 = [ ]
(3)(4) + (1)(−1) (1)(0) + (1)(2) 64 2 + 8 −1 + 12
=[ ] 1 4 −2
(−2)(4) + (2)(−1) (−2)(0) + (2)(2) = 64 [ ]…(2)
12 − 1 0 + 2 10 11
=[ ] From (1) and (2), we have
−8 − 2 0 − 4
11 2 (𝐷𝐴)−1 = 𝐴−1 𝐷−1
=[ ]
−10 4
Now, we find the its determinant
|𝐷𝐴| = | 11 2|
−10 4
= (11)(4) − (2)(−10)
= 44 + 20 = 64
Which is not zero and hence, matrix 𝐷𝐴 is Non-
singular matrix and (𝐷𝐴)−1 exist.
4 −2
Now, 𝐴𝑑𝑗𝐷𝐴 = [ ]
10 11
1
(𝐷𝐴)−1 = 𝐴𝑑𝑗𝐷𝐴
|𝐷𝐴|
Putting values
1 4 −2
𝐿. 𝐻. 𝑆 = (𝐷𝐴)−1 = 64 [ ]…(1)
10 11
−1 −1
𝑅. 𝐻. 𝑆 = 𝐴 𝐷
First, we find 𝐷−1 and 𝐴−1
|𝐴| = | 4 0|
−1 2
= (4)(2) − (0)(−1)
=8−0=8
Which is not zero and hence, matrix 𝐴 is Non-
singular matrix and 𝐴−1 exist.

17 | P a g e
Class 9th Chapter 1 www.notes.pk.com
Exercise 1.6 |𝐴| = 4 + 6 = 10
Which is non-zero, so solution exists and
Q#1) Use matrices, to solve the following system 4 −2
of linear equations by: 𝐴𝑥 = [ ]
6 2
(a). the matrix inverse method 4 −2
⇒ |𝐴𝑥 | = | |
(b). the Cramer’s rule 6 2
(i). 𝟐𝒙 − 𝟐𝒚 = 𝟒 ; 𝟑𝒙 + 𝟐𝒚 = 𝟔 |𝐴𝑥 | = (4)(2) − (−2)(6)
Sol: (a). the matrix inverse method |𝐴𝑥 | = 8 + 12 = 20
In matrix form Also,
2 −2 𝑥 4 2 4
[ ] [𝑦 ] = [ ] 𝐴𝑦 = [ ]
3 2 6 3 6
2 4
𝐴𝑋 = 𝐵 ⇒ 𝑋 = 𝐴−1 𝐵 →(1) ⇒ |𝐴𝑦 | = | |
𝑥 3 6
2 −2 4 = (2)(6) − (4)(3)
Where𝐴 = [ ], 𝑋 = [𝑦] and 𝐵 = [ ]
3 2 6
Now, we find 𝐴−1 using |𝐴𝑦 | = 12 − 12 = 0
1 Now
𝐴−1 = |𝐴| 𝐴𝑑𝑗𝐴 →(2) |𝐴 | 20
𝑥 = |𝐴|𝑥 ⇒ x = 10 = 2
|𝐴| = |2 −2| |𝐴𝑦 | 0
3 2 And 𝑦 = ⇒ y = 10 = 0
= (2)(2) − (−2)(3) |𝐴|

= 4 + 6 = 10 Hence, 𝑥 = 2 and 𝑦 = 0
Which is not zero and hence, matrix 𝐴 is Non- (ii). 𝟐𝒙 + 𝒚 = 𝟑 ; 𝟔𝒙 + 𝟓𝒚 = 𝟏
singular matrix and 𝐴−1 exist. Sol: (a). the matrix inverse method
2 2 In matrix form
Now, 𝐴𝑑𝑗𝐴 = [ ] 2 1 𝑥 3
−3 2 [ ] [𝑦] = [ ]
Putting values in eq. (2), we have 6 5 1
1
𝐴−1 = |𝐴| 𝐴𝑑𝑗𝐴 𝐴𝑋 = 𝐵 ⇒ 𝑋 = 𝐴−1 𝐵…(1)
2 −2 𝑥 3
1 2 2 Where𝐴 = [ ], 𝑋 = [𝑦] and 𝐵 = [ ]
⇒ 𝐴−1 = [ ] 3 2 1
10 −3 2 Now, we find 𝐴−1 using
Now, put values in eq. (1) 1
𝐴−1 = |𝐴| 𝐴𝑑𝑗𝐴 →(2)
𝑋 = 𝐴−1 𝐵
1 2 2 4 |𝐴| = |2 1|
⇒X= [ ][ ] 6 5
10 −3 2 6 = (2)(5) − (1)(6)
1 (2)(4) + (2)(6)
= [ ] = 10 − 6 = 4
10 (−3)(4) + (2)(6) Which is not zero and hence, matrix 𝐴 is Non-
1 8 + 12
= [ ] singular matrix and 𝐴−1 exist.
10 −12 + 12 5 −1
1 20 Now, 𝐴𝑑𝑗𝐴 = [ ]
= [ ] −6 2
10 0 Putting values in eq. (2), we have
𝑥 2 1
⇒ [𝑦 ] = [ ] 𝐴−1 = |𝐴| 𝐴𝑑𝑗𝐴
0
⇒ x = 2, y = 0 1 5 −1
(b). the Cramer’s rule ⇒ 𝐴−1 = [ ]
4 −6 2
In matrix form Now, put values in eq. (1)
2 −2 𝑥 4 𝑋 = 𝐴−1 𝐵
[ ][ ] = [ ]
3 2 𝑦 6 1 5 −1 3
⇒X= [ ][ ]
4 −6 2 1
2 −2 4 −2 1 (5)(3) + (−1)(1)
Where 𝐴 = [ ], 𝐴𝑥 = [ ] and
3 2 6 2 = [ ]
2 4 4 (−6)(3) + (2)(1)
𝐴𝑦 = [ ] 1 15 − 1
3 6 = [ ]
First of all we find |𝐴|, |𝐴𝑥 |𝑎𝑛𝑑 |𝐴𝑦 | 4 −18 + 2
2 −2 14
𝐴=[ ] 1 14
3 2
2 −2 = [ ]=[ 4 ]
⇒ |𝐴| = | | 4 −16 16
3 2 −
4
= (2)(2) − (−2)(3)

18 | P a g e
Class 9th Chapter 1 www.notes.pk.com
𝑥 7 −1 −2
Now, 𝐴𝑑𝑗𝐴 = [ ]
⇒ [𝑦 ] = [ 2 ] −3 4
−4 Putting values in eq. (2), we have
7 𝐴−1 = 𝐴𝑑𝑗𝐴
1
⇒ x = , y = −4 |𝐴|
2 1 −1 −2
(b). the Cramer’s rule ⇒ 𝐴−1 = [ ]
In matrix form −10 −3 4
2 1 𝑥 3 Now, put values in eq. (1)
[ ][ ] = [ ] 𝑋 = 𝐴−1 𝐵
6 5 𝑦 1
2 1 3 1 1 −1 −2 8
Where𝐴 = [ ], 𝐴 = [ ] and ⇒X= [ ][ ]
6 5 𝑥 1 5 −10 −3 4 −1
2 3 1 (−1)(8) + (−2)(−1)
𝐴𝑦 = [ ] = [ ]
6 1 −10 (−3)(8) + (4)(−1)
First of all we find |𝐴|, |𝐴𝑥 |𝑎𝑛𝑑 |𝐴𝑦 | 1 −8 + 2
2 1 = [ ]
𝐴=[ ] −10 −24 − 4
6 5 −6
2 1
⇒ |𝐴| = | | 1 −6
6 5 = [ ] = [−10]
= (2)(5) − (1)(6) −10 −28 −28
= 10 − 6 = 4 −10
Which is non-zero, so solution exists and 3
𝑥
𝐴𝑥 = [
3 1
] ⇒ [𝑦] = [5]
1 5 7
3 1 5
⇒ |𝐴𝑥 | = | |
1 5 3 7
|𝐴𝑥 | = (3)(5) − (1)(1) ⇒ x = ,y =
5 5
|𝐴𝑥 | = 15 − 1 = 14 (b). the Cramer’s rule
Also, In matrix form
2 3 4 2 𝑥 8
𝐴𝑦 = [ ] [ ][ ] = [ ]
6 1 3 −1 𝑦 −1
2 3 4 2 8 2
⇒ |𝐴𝑦 | = | | Where𝐴 = [ ], 𝐴 = [ ] and 𝐴𝑦 =
6 1 3 −1 𝑥 −1 −1
= (2)(1) − (3)(6) 4 8
[ ]
|𝐴𝑦 | = 2 − 18 = −16 3 −1
Now First of all we find |𝐴|, |𝐴𝑥 |𝑎𝑛𝑑 |𝐴𝑦 |
|𝐴 | 14 7 4 2
𝑥 = |𝐴|𝑥 ⇒ x = 4 = 2 𝐴=[ ]
3 −1
|𝐴𝑦 | −16 4 2
And 𝑦 = ⇒y= = −4 ⇒ |𝐴| = | |
|𝐴| 4 3 −1
7
Hence, 𝑥 = 2 and 𝑦 = −4 = (4)(−1) − (2)(3)
= −4 − 6 = −10
(iii). 𝟒𝒙 + 𝟐𝒚 = 𝟖 ; 𝟑𝒙 − 𝒚 = −𝟏
Which is non-zero, so solution exists and
Sol: (a). the matrix inverse method
8 2
In matrix form 𝐴𝑥 = [ ]
−1 −1
4 2 𝑥 8 8 2
[ ][ ] = [ ] ⇒ |𝐴𝑥 | = | |
3 −1 𝑦 −1 −1 −1
𝐴𝑋 = 𝐵 ⇒ 𝑋 = 𝐴−1 𝐵 →(1) |𝐴𝑥 | = (8)(−1) − (2)(−1)
4 2 𝑥 8 |𝐴𝑥 | = −8 + 2 = −6
Where𝐴 = [ ], 𝑋 = [𝑦] and 𝐵 = [ ]
3 −1 −1 Also,
Now, we find 𝐴−1 using 4 8
1
𝐴−1 = |𝐴| 𝐴𝑑𝑗𝐴…(2) 𝐴𝑦 = [ ]
3 −1
4 8
|𝐴| = |4 2 | ⇒ |𝐴𝑦 | = |
3 −1
|
3 −1 = (4)(−1) − (8)(3)
= (4)(−1) − (2)(3)
= −4 − 6 = −10 |𝐴𝑦 | = −4 − 24 = −28
Which is not zero and hence, matrix 𝐴 is Non- Now
|𝐴 | −6 3
singular matrix and 𝐴−1 exist. 𝑥 = |𝐴|𝑥 ⇒ x = −10 = 5

19 | P a g e
Class 9th Chapter 1 www.notes.pk.com
And 𝑦 =
|𝐴𝑦 | −28 7
⇒ y = −10 = 5 −6 2
𝐴𝑥 = [ ]
|𝐴| −10 −2
3 7 −6 2
Hence, 𝑥 = 5 and 𝑦 = 5 ⇒ |𝐴𝑥 | = | |
−10 −2
(iv). 𝟑𝒙 − 𝟐𝒚 = −𝟔 ; 𝟓𝒙 − 𝟐𝒚 = −𝟏𝟎 |𝐴𝑥 | = (−6)(−2) − (2)(−10)
Sol: (a). the matrix inverse method |𝐴𝑥 | = 12 − 20 = −8
In matrix form Also,
3 −2 𝑥 −6 3 −6
[ ][ ] = [ ] 𝐴𝑦 = [ ]
5 −2 𝑦 −10 5 −10
𝐴𝑋 = 𝐵 ⇒ 𝑋 = 𝐴−1 𝐵 →(1) 3 −6
3 −2 𝑥 −6 ⇒ |𝐴𝑦 | = | |
Where𝐴 = [ ], 𝑋 = [𝑦] and 𝐵 = [ ] 5 −10
5 −2 −10 = (3)(−10) − (−6)(5)
Now, we find 𝐴−1 using |𝐴𝑦 | = −30 + 30 = 0
1
𝐴−1 = |𝐴| 𝐴𝑑𝑗𝐴 →(2) Now
|𝐴 | −8
|𝐴| = |3 −2| 𝑥 = |𝐴|𝑥 ⇒ x = 4 = −2
5 −2 |𝐴𝑦 | 0
= (3)(−2) − (−2)(5) And 𝑦 = |𝐴|
⇒y=4=0
= −6 + 10 = 4 Hence, 𝑥 = −2 and 𝑦 = 0
Which is not zero and hence, matrix 𝐴 is Non- (iii). 𝟑𝒙 − 𝟐𝒚 = 𝟒 ; −𝟔𝒙 + 𝟒𝒚 = 𝟕
singular matrix and 𝐴−1 exist. Sol: The matrix inverse method
−2 2
Now, 𝐴𝑑𝑗𝐴 = [ ] In matrix form
−5 3 3 −2 𝑥 4
Putting values in eq. (2), we have [ ] [𝑦 ] = [ ]
1 −6 4 7
𝐴−1 = |𝐴| 𝐴𝑑𝑗𝐴 𝐴𝑋 = 𝐵 ⇒ 𝑋 = 𝐴−1 𝐵 →(1)
1 −2 2 3 −2 𝑥 4
⇒ 𝐴−1 = [ ] Where𝐴 = [ ], 𝑋 = [𝑦] and 𝐵 = [ ]
4 −5 3 −6 4 7
Now, put values in eq. (1) Now, we find 𝐴−1 using
1
𝑋 = 𝐴−1 𝐵 𝐴−1 = |𝐴| 𝐴𝑑𝑗𝐴 →(2)
1 −2 2 −6
⇒X= [ ][ ] |𝐴| = | 3 −2|
4 −5 3 −10 −6 4
1 (−2)(−6) + (2)(−10) = (3)(4) − (−2)(−6)
= [ ]
4 (−5)(−6) + (3)(−10) = 12 − 12 = 0
1 12 − 20 Which is zero and hence, matrix 𝐴 is singular
= [ ]
4 30 − 30 matrix and 𝐴−1 does not exist. No solution
−8 possible.
1 −8
= [ ]=[ 4 ] (vi). 𝟒𝒙 + 𝒚 = 𝟗 ; −𝟑𝒙 − 𝒚 = −𝟓
4 0 0 Sol: (a). the matrix inverse method
4 In matrix form
𝑥 −2
⇒ [𝑦 ] = [ ] 4 1 𝑥 9
0 [ ] [𝑦 ] = [ ]
−3 −1 −5
⇒ x = −2, y = 0 𝐴𝑋 = 𝐵 ⇒ 𝑋 = 𝐴−1 𝐵 →(1)
(b). the Cramer’s rule 4 1 𝑥 9
In matrix form Where𝐴 = [ ], 𝑋 = [𝑦] and 𝐵 = [ ]
−3 −1 −5
3 −2 𝑥 −6 Now, we find 𝐴−1 using
[ ][ ] = [ ]
5 −2 𝑦 −10 1
𝐴−1 = 𝐴𝑑𝑗𝐴 →(2)
3 2 −6 2 |𝐴|
Where𝐴 = [ ], 𝐴 = [ ] and
5 −2 𝑥 −10 −2 |𝐴| = | 4 1
|
3 −6 −3 −1
𝐴𝑦 = [ ]
5 −10 = (4)(−1) − (−3)(1)
First of all we find |𝐴|, |𝐴𝑥 |𝑎𝑛𝑑 |𝐴𝑦 | = −4 + 3 = −1
3 2 Which is not zero and hence, matrix 𝐴 is Non-
𝐴=[ ]
5 −2 singular matrix and 𝐴−1 exist.
3 −2
⇒ |𝐴| = | | −1 −1
5 −2 Now, 𝐴𝑑𝑗𝐴 = [ ]
= (3)(−2) − (−2)(5) 3 4
Putting values in eq (2), we have
= −6 + 10 = 4 1
Which is non-zero, so solution exists and 𝐴−1 = |𝐴| 𝐴𝑑𝑗𝐴

20 | P a g e
Class 9th Chapter 1 www.notes.pk.com
1 −1 −1 2 −2 𝑥 4
⇒ 𝐴−1 = [ ] Where𝐴 = [ ], 𝑋 = [𝑦] and 𝐵 = [ ]
−1 3 4 −5 −2 −10
−1
Now, put values in eq. (1) Now, we find 𝐴 using
1
𝑋 = 𝐴−1 𝐵 𝐴−1 = |𝐴| 𝐴𝑑𝑗𝐴 →(2)
1 −1 −1 9
⇒X= [ ][ ] |𝐴| = | 2 −2|
−1 3 4 −5 −5 −2
1 (−1)(9) + (−1)(−5) = (2)(−2) − (−2)(−5)
= [ ]
−1 (3)(9) + (4)(−5) = −4 − 10 = −14
1 −9 + 5 Which is not zero and hence, matrix 𝐴 is Non-
= [ ]
−1 27 − 20 singular matrix and 𝐴−1 exist.
−4 −2 2
1 −4 Now, 𝐴𝑑𝑗𝐴 = [ ]
= [ ] = [−1] 5 2
−1 7 7 Putting values in eq. (2), we have
1
−1 𝐴−1 = |𝐴| 𝐴𝑑𝑗𝐴
𝑥 4
⇒ [𝑦 ] = [ ] 1 −2 2
−7 ⇒ 𝐴−1 = [ ]
⇒ x = 4, y = −7 −14 5 2
(b). the Cramer’s rule Now, put values in eq. (1)
In matrix form 𝑋 = 𝐴−1 𝐵
4 1 𝑥 9 1 −2 2 4
[ ] [𝑦 ] = [ ] ⇒X= [ ][ ]
−3 −1 −5 −14 5 2 −10
4 1 9 1 1 (−2)(4) + (2)(−10)
Where𝐴 = [ ], 𝐴𝑥 = [ ] and = [ ]
−3 −1 −5 −1 −14 (5)(4) + (2)(−10)
4 9 1 −8 − 20
𝐴𝑦 = [ ]
−3 −5 = [ ]
First of all we find |𝐴|, |𝐴𝑥 |𝑎𝑛𝑑 |𝐴𝑦 | −14 20 − 20
−28
4 1 1 −28
𝐴=[ ]
−3 −1 = [ ] = [−14]
4 1 −14 0 0
⇒ |𝐴| = | |
−3 −1 −14
= (4)(−1) − (−3)(1) 𝑥 2
⇒ [𝑦 ] = [ ]
= −4 + 3 = −1 0
Which is non-zero, so solution exists and ⇒ x = 2, y = 0
9 1 (b). the Cramer’s rule
𝐴𝑥 = [ ] In matrix form
−5 −1
9 1 𝟐 −𝟐 𝒙 𝟒
⇒ |𝐴𝑥 | = | | [ ] [ 𝒚] = [ ]
−5 −1 −𝟓 −𝟐 −𝟏𝟎
|𝐴𝑥 | = (9)(−1) − (1)(−5) 2 −2 4 −2
Where𝐴 = [ ], 𝐴𝑥 = [ ] and
|𝐴𝑥 | = −9 + 5 = −4 −5 −2 −10 −2
2 4
Also, 𝐴𝑦 = [ ]
4 9 −5 −10
|𝐴𝑦 | = [ ] First of all we find |𝐴|, |𝐴𝑥 |𝑎𝑛𝑑 |𝐴𝑦 |
−3 −5
4 9 2 −2
⇒ |𝐴𝑦 | = | | 𝐴=[ ]
−3 −5 −5 −2
= (4)(−5) − (9)(−3) 2 −2
⇒ |𝐴| = | |
𝐴𝑦 = −20 + 27 = 7 −5 −2
= (2)(−2) − (−2)(−5)
Now = −4 − 10 = −14
|𝐴 | −4
𝑥 = |𝐴|𝑥 ⇒ x = −1 = 4 Which is non-zero, so solution exists and
|𝐴𝑦 | 7 4 −2
And 𝑦 = ⇒ y = −1 = 7 𝐴𝑥 = [ ]
|𝐴| −10 −2
Hence, 𝑥 = 4 and 𝑦 = 7 4 −2
⇒ |𝐴𝑥 | = | |
(vii). 𝟐𝒙 − 𝟐𝒚 = 𝟒 ; −𝟓𝒙 − 𝟐𝒚 = −𝟏𝟎 −10 −2
|𝐴𝑥 | = (4)(−2) − (−2)(−10)
Sol: (a). the matrix inverse method
|𝐴𝑥 | = −8 − 20 = −28
In matrix form
Also,
2 −2 𝑥 4
[ ][ ] = [ ] 2 4
−5 −2 𝑦 −10 𝐴𝑦 = [ ]
𝐴𝑋 = 𝐵 ⇒ 𝑋 = 𝐴−1 𝐵 →(1) −5 −10

21 | P a g e
Class 9th Chapter 1 www.notes.pk.com
2 4 𝑥 4
⇒ |𝐴𝑦 | = | | ⇒ [𝑦 ] = [ ]
−5 −10 2
= (2)(−10) − (4)(−5) ⇒ x = 4, y = 2
|𝐴𝑦 | = −20 + 20 = 0 (b). the Cramer’s rule
Now In matrix form
3 −4 𝑥 4
|𝐴 | −28
𝑥 = |𝐴|𝑥 ⇒ x = −14 = 2 [ ][ ] = [ ]
1 2 𝑦 8
|𝐴𝑦 | 3 −4 4 −4
And 𝑦 =
0
⇒ y = −14 = 0 Where𝐴 = [ ], 𝐴𝑥 = [ ] and
|𝐴| 1 2 8 2
Hence, 𝑥 = 2 and 𝑦 = 0 3 4
𝐴𝑦 = [ ]
(viii). 𝟑𝒙 − 𝟒𝒚 = 𝟒 ; 𝒙 + 𝟐𝒚 = 𝟖 1 8
Sol: (a). the matrix inverse method First of all, we find |𝐴|, |𝐴𝑥 |𝑎𝑛𝑑 |𝐴𝑦 |
In matrix form 3 −4
3 −4 𝑥 4 𝐴=[ ]
[ ] [𝑦 ] = [ ] 1 2
1 2 8 3 −4
𝐴𝑋 = 𝐵 ⇒ 𝑋 = 𝐴−1 𝐵 →(1) ⇒ |𝐴| = | |
3 −4 𝑥 4
1 2
Where𝐴 = [ ], 𝑋 = [𝑦] and 𝐵 = [ ] = (3)(2) − (−4)(1)
1 2 8
Now, we find 𝐴−1 using = 6 + 4 = 10
1 Which is non-zero, so solution exists and
𝐴−1 = |𝐴| 𝐴𝑑𝑗𝐴…(2)
4 −4
|𝐴| = |3 −4| 𝐴𝑥 = [ ]
1 2 8 2
= (3)(2) − (−4)(1) 4 −4
⇒ |𝐴𝑥 | = | |
= 6 + 4 = 10 8 2
Which is not zero and hence, matrix 𝐴 is Non- = (4)(2) − (−4)(8)
singular matrix and 𝐴−1 exist. |𝐴𝑥 | = 8 + 32 = 40
2 4 Also,
Now, 𝐴𝑑𝑗𝐴 = [ ]
−1 3 3 4
Putting values in eq.(2), we have 𝐴𝑦 = [ ]
1 1 8
𝐴−1 = |𝐴| 𝐴𝑑𝑗𝐴 3 4
1 2 4
⇒ |𝐴𝑦 | = | |
⇒ 𝐴−1 = [ ]
1 8
10 −1 3 = (3)(8) − (4)(1)
Now, put values in eq. (1) |𝐴𝑦 | = 24 − 4 = 20
𝑋 = 𝐴−1 𝐵
1 2 4 4 Now
⇒X= [ ][ ] |𝐴𝑥 | 40
10 −1 3 8 𝑥= ⇒x= =4
1 (2)(4) + (4)(8) |𝐴| 10
= [ ] |𝐴𝑦 | 20
10 (−1)(4) + (3)(8) And 𝑦 = ⇒y= =2
1 8 + 32 |𝐴| 10
= [
10 −4 + 24
] Hence, 𝑥 = 4 and 𝑦 = 2
40
1 40
= [ ] = [10]
10 20 20
10

A project of: www.notespk.com


Contact or Suggest Us: info@notespk.com

22 | P a g e
MATHEMATICS 9th Science Group

7/18/2020
Chapter 2.
REAL AND COMPLEX
NUMBERS

A project of: www.notespk.com


Contact or Suggest Us: info@notespk.com
Contents
Exercise #2.1............................................................................ 1
Exercise #2.2............................................................................ 4
Exercise # 2.3........................................................................... 5
Exercise # 2.4........................................................................... 6
Exercise # 2.5........................................................................... 7
Class 9th Chapter 2 www.notes.pk.com
Rational number: Question.2. Convert the following fractions
A number which can be written in the form of into decimal fraction.
𝑝 17
, 𝑤ℎ𝑒𝑟𝑒 𝑝, 𝑞 𝜖 𝑍 Λ ≠ 0 is called a rational (i) 25
𝑞
number. Solution.
e.g. 4 ,
3 22 2
,6. 𝟎. 𝟔𝟖
7 19
Irrational number: (ii) 4
A real number which cannot be written in the Solution.
𝑝
form of 𝑞 , 𝑤ℎ𝑒𝑟𝑒 𝑝, 𝑞 𝜖 𝑍 Λ ≠ 0 is called an 𝟒. 𝟕𝟓
57
(iii)
irrational number. 8

e.g. √2 , √5 Solution.
𝟕. 𝟏𝟐𝟓
Real number: 205
The field of all rational and irrational numbers is (iv) 18
called the real numbers, or simply the "reals," Solution.
and denoted ℝ . 𝟏𝟏. 𝟑𝟖𝟖𝟗
5
Terminating decimal: (v) 8
A decimal which has only a finite number of digits Solution.
in its decimal part, is called terminating decimal. 𝟎. 𝟔𝟐𝟓
e.g. 202.04, 0.25, 0.5 example of terminating 25
(vi) 38
decimal.
Recurring decimal: Solution.
𝟎. 𝟔𝟓𝟕𝟖𝟗
A decimal in which one or more digits repeats
Question.3. Which of the following statements are
indefinitely is called recurring decimal or periodic
true and which are false?
decimal. 𝟐
(i). 𝟑 𝒊𝒔 𝒂𝒏 𝒊𝒓𝒓𝒂𝒕𝒊𝒐𝒏𝒂𝒍 𝒏𝒖𝒎𝒃𝒆𝒓.
e.g. 0.33333 , 21.134134 … …
Solution.
Exercise 2.1 False.
(ii). 𝝅 𝒊𝒔 𝒂𝒏 𝒊𝒓𝒓𝒂𝒕𝒊𝒐𝒏𝒂𝒍 𝒏𝒖𝒎𝒃𝒆𝒓.
Question.1. Identify which of the following are
Solution.
rational and irrational numbers
True.
(i). √𝟑 𝟏
(iii). 𝟗 𝒊𝒔 𝒂 𝒕𝒆𝒓𝒎𝒊𝒏𝒂𝒕𝒊𝒏𝒈 𝒇𝒓𝒂𝒄𝒕𝒊𝒐𝒏.
Solution.
Is an irrational number. Solution.
𝟏 False.
(ii). 𝟔 𝟑
(iv). 𝟒 𝒊𝒔 𝒕𝒆𝒓𝒎𝒊𝒏𝒂𝒕𝒊𝒏𝒈 𝒇𝒓𝒂𝒄𝒕𝒊𝒐𝒏.
Solution.
Is a rational number. Solution.
(iii). 𝝅 True.
𝟒
Solution. (v). 𝟓 𝒊𝒔 𝒂 𝒓𝒆𝒄𝒖𝒓𝒓𝒊𝒏𝒈 𝒇𝒓𝒂𝒄𝒕𝒊𝒐𝒏..
Is an irrational number. Solution.
𝟏𝟓
(iv). False.
𝟕
Solution. Question.4. Represent the following numbers on
Is a rational number. the number line
𝟐
(v). 7.25 (i) 𝟑
Solution. Solution.
Is a rational number.
(vi). √𝟐𝟗 -4 -3 -2 -1 0 1 2 3 4
Solution. 𝟒
Is an irrational number. (ii). − 𝟓
Solution.

1|Page
Class 9th Chapter 2 www.notes.pk.com
𝟓
𝒙=
-4 -3 -2 -1 0 1 2 3 4 𝟗
𝟓
𝟑 ̅=
𝟎. 𝟓
(iii). 𝟏 𝟗
𝟒
Solution. 𝑨𝒏𝒔𝒘𝒆𝒓.
̅̅̅̅
(ii). 𝟎. 𝟏𝟑
-4 -3 -2 -1 0 1 2 3 4 Solution.
𝟓 Let
(iv). −𝟐 𝟖
𝒙 = 𝟎. ̅̅̅̅
𝟏𝟑
Solution.
That is
𝒙 = 𝟎. 𝟏𝟑𝟏𝟑𝟏𝟑𝟏𝟑 … → (𝒊)
-4 -3 -2 -1 0 1 2 3 4
Only two digits 13 is being repeated, multiply
(v). √𝟓 by 100 on both sides of (𝒊), we have
Solution. 𝟏𝟎𝟎𝒙 = (𝟎. 𝟏𝟑𝟏𝟑𝟏𝟑𝟏𝟑 … ) × 𝟏𝟎𝟎
𝟏𝟎𝟎𝒙 = 𝟏𝟑. 𝟏𝟑𝟏𝟑𝟏𝟑𝟏𝟑 … → (𝒊𝒊)
-4 -3 -2 -1 0 1 2 3 4
Subtracting (𝒊)𝒇𝒓𝒐𝒎 (𝒊𝒊), 𝒘𝒆 𝒉𝒂𝒗𝒆
𝟑
Question.5. Give a rational number between 𝟒 𝟏𝟎𝟎𝒙 − 𝒙
and 𝟗 .
𝟓 = 𝟏𝟑. 𝟏𝟑𝟏𝟑𝟏𝟑𝟏𝟑 … . −𝟎. 𝟏𝟑𝟏𝟑𝟏𝟑𝟏𝟑 …
𝟗𝟗𝒙 = 𝟏𝟑
Solution. 𝟏𝟑
The mean of the numbers is between given 𝒙=
𝟗𝟗
numbers. Therefore 𝟏𝟑
𝟑 𝟓 ̅̅̅̅ =
𝟎. 𝟏𝟑
+ 𝟗𝟗
𝒓𝒆𝒒𝒖𝒊𝒓𝒆𝒅 𝒏𝒖𝒎𝒃𝒆𝒓 𝒊𝒔 = 𝟒 𝟗 𝑨𝒏𝒔𝒘𝒆𝒓.
𝟐
𝟐𝟕 + 𝟐𝟎 ̅̅̅̅
(iii). 𝟎. 𝟔𝟕
= 𝟑𝟔 Solution.
𝟐 Let
𝟒𝟕 𝒙 = 𝟎. ̅̅̅̅
𝟔𝟕
= 𝟑𝟔 That is
𝟐
𝟒𝟕 𝒙 = 𝟎. 𝟔𝟕𝟔𝟕𝟔𝟕𝟔𝟕 … → (𝒊)
= Only two digits 67 is being repeated, multiply
𝟑𝟔 × 𝟐
𝟒𝟕 by 100 on both sides of (𝒊), we have
=
𝟕𝟐 𝟏𝟎𝟎𝒙 = (𝟎. 𝟔𝟕𝟔𝟕𝟔𝟕𝟔𝟕 … ) × 𝟏𝟎𝟎
𝑨𝒏𝒔𝒘𝒆𝒓. 𝟏𝟎𝟎𝒙 = 𝟔𝟕. 𝟔𝟕𝟔𝟕𝟔𝟕𝟔𝟕 … → (𝒊𝒊)
𝑸𝒖𝒆𝒔𝒕𝒊𝒐𝒏.6. Express the following recurring
p Subtracting (𝒊)𝒇𝒓𝒐𝒎 (𝒊𝒊), 𝒘𝒆 𝒉𝒂𝒗𝒆
decimals as the rational number , where p, q
q 𝟏𝟎𝟎𝒙 − 𝒙
are integers and q  0. = 𝟔𝟕. 𝟔𝟕𝟔𝟕𝟔𝟕𝟔𝟕 … . −𝟎. 𝟔𝟕𝟔𝟕𝟔𝟕𝟔𝟕 …
̅
(i). 𝟎. 𝟓
𝟗𝟗𝒙 = 𝟔𝟕
Solution.
Let 𝟔𝟕
̅ 𝒙=
𝒙 = 𝟎. 𝟓 𝟗𝟗
That is 𝟔𝟕
𝒙 = 𝟎. 𝟓𝟓𝟓𝟓 … → (𝒊) ̅̅̅̅ =
𝟎. 𝟔𝟕
𝟗𝟗
Only one digit 5 is being repeated, multiply by
10 on both sides of (𝒊), we have 𝑨𝒏𝒔𝒘𝒆𝒓.
𝟏𝟎𝒙 = (𝟎. 𝟓𝟓𝟓𝟓 … ) × 𝟏𝟎
𝟏𝟎𝒙 = 𝟓. 𝟓𝟓𝟓𝟓 … → (𝒊𝒊)
Subtracting (𝒊)𝒇𝒓𝒐𝒎 (𝒊𝒊), 𝒘𝒆 𝒉𝒂𝒗𝒆
𝟏𝟎𝒙 − 𝒙 = 𝟓. 𝟓𝟓𝟓𝟓 … . −𝟎. 𝟓𝟓𝟓𝟓 … Properties of Real Numbers:
𝟗𝒙 = 𝟓 Binary Operations:

2|Page
Class 9th Chapter 2 www.notes.pk.com
A binary operation in a set A is a rule usually 1
𝑎 𝑎𝑛𝑑 𝑎𝑟𝑒 𝑐𝑎𝑙𝑙𝑒𝑑 𝑡ℎ𝑒 𝑖𝑑𝑒𝑡𝑖𝑣𝑒 𝑖𝑛𝑣𝑒𝑟𝑠𝑒 𝑜𝑓 𝑒𝑎𝑐ℎ
denoted by ∗ that assigns to any pair of elements 𝑎
of A to another element of A. e.g. two important 𝑜𝑡ℎ𝑒𝑟.
binary operations are addition and multiplication
in a set of real numbers. Commutative Law for Multiplication:

Addition Laws: ∀ 𝑎 , 𝑏 ∈ ℜ 𝑡ℎ𝑒𝑛 𝑎𝑏 = 𝑏𝑎.


Closure Law of Addition: Properties of Equality:
∀ 𝑎 , 𝑏 ∈ ℜ 𝑡ℎ𝑒𝑛 𝑎 + 𝑏 Reflexive property:
∈ℜ ∀ 𝑠𝑡𝑎𝑛𝑑𝑠 𝑓𝑜𝑟 𝑎𝑙𝑙.
∀ 𝑎 ∈ ℜ 𝑡ℎ𝑒𝑛 𝑎 = 𝑎
Associative Law of Addition:
Symmetric Property:
∀ 𝑎 , 𝑏 , 𝑐 ∈ ℜ 𝑡ℎ𝑒𝑛 𝑎 + (𝑏 + 𝑐) = (𝑎 + 𝑏) + 𝑐.
∀ 𝑎 , 𝑏 ∈ ℜ 𝑎𝑛𝑑 𝑖𝑓 𝑎 = 𝑏 𝑡ℎ𝑒𝑛 𝑏 = 𝑎.
Additive Identity:
Transitive Property:
∀ 𝑎 ∈ ℜ , ∃ 0 ∈ ℜ 𝑠𝑢𝑐ℎ 𝑡ℎ𝑎𝑡 𝑎 + 0 = 0 + 𝑎 = 𝑎.
∀ 𝑎 , 𝑏 , 𝑐 ∈ ℜ , 𝑖𝑓 𝑎 = 𝑏 𝑎𝑛𝑑 𝑏 = 𝑐 𝑡ℎ𝑒𝑛 𝑎 = 𝑐.
∃ 𝑆𝑡𝑎𝑛𝑑𝑠 𝑓𝑜𝑟 𝑡ℎ𝑒𝑟𝑒 𝑒𝑥𝑖𝑠𝑡 𝑎𝑛𝑑 0 𝑖𝑠 𝑐𝑎𝑙𝑙𝑒𝑑 𝑡ℎ𝑒 𝑖𝑑𝑖𝑡𝑖𝑣𝑒
Additive Property:
𝑖𝑑𝑒𝑛𝑡𝑖𝑡𝑦.
∀ 𝑎 , 𝑏 , 𝑐 ∈ ℜ , 𝑎 = 𝑏 𝑡ℎ𝑒𝑛 𝑎 + 𝑐 = 𝑏 + 𝑐.
Additive Inverse:
Multiplicative Property:
∀ 𝑎 ∈ ℜ , ∃ − 𝑎 ∈ ℜ 𝑠𝑢𝑐ℎ 𝑡ℎ𝑎𝑡 𝑎 + (−𝑎) ∀ 𝑎 , 𝑏 , 𝑐 ∈ ℜ , 𝑎 = 𝑏 𝑡ℎ𝑒𝑛 𝑎𝑐 = 𝑏𝑐.
= −𝑎 + 𝑎 = 0.
Cancellation Property w.r.t. addition:
−𝑎 𝑎𝑛𝑑 𝑎 𝑎𝑟𝑒 𝑐𝑎𝑙𝑙𝑒𝑑 𝑡ℎ𝑒 𝑖𝑑𝑒𝑡𝑖𝑣𝑒 𝑖𝑛𝑣𝑒𝑟𝑠𝑒 𝑜𝑓 𝑒𝑎𝑐ℎ ∀ 𝑎 , 𝑏 , 𝑐 ∈ ℜ , 𝑎 + 𝑐 = 𝑏 + 𝑐 𝑡ℎ𝑒𝑛 𝑎 = 𝑏.
𝑜𝑡ℎ𝑒𝑟. Cancellation Property w.r.t. Multiplication:
Commutative Law for Addition: ∀ 𝑎 , 𝑏 , 𝑐 ∈ ℜ , 𝑎𝑐 = 𝑏𝑐 𝑡ℎ𝑒𝑛 𝑎 = 𝑏.

∀ 𝑎 , 𝑏 ∈ ℜ 𝑡ℎ𝑒𝑛 𝑎 + 𝑏 = 𝑏 + 𝑎. Distributive property of multiplication over


addition.
Multiplication Laws:
𝑎(𝑏 + 𝑐) = 𝑎𝑏 + 𝑎𝑐
Closure Law of Multiplication:
Distributive property of multiplication over
∀ 𝑎 , 𝑏 ∈ ℜ 𝑡ℎ𝑒𝑛 𝑎𝑏 Subtraction.
∈ℜ ∀ 𝑠𝑡𝑎𝑛𝑑𝑠 𝑓𝑜𝑟 𝑎𝑙𝑙.
𝑎(𝑏 − 𝑐) = 𝑎𝑏 − 𝑎𝑐
Associative Law of Multiplication:
Properties of Inequalities (Order
∀ 𝑎 , 𝑏 , 𝑐 ∈ ℜ 𝑡ℎ𝑒𝑛 𝑎(𝑏𝑐) = (𝑎𝑏)𝑐. properties):
Trichotomy Property:
Multiplicative Identity:
∀ 𝑎 ,𝑏 ∈ ℜ
∀ 𝑎 ∈ ℜ , ∃ 1 ∈ ℜ 𝑠𝑢𝑐ℎ 𝑡ℎ𝑎𝑡 𝑎. 1 = 1. 𝑎 = 𝑎. 𝑒𝑖𝑡ℎ𝑒𝑟 𝑎 = 𝑏 𝑜𝑟 𝑎 > 𝑏 𝑜𝑟 𝑎 < 𝑏.
Transitive Property:
∃ 𝑆𝑡𝑎𝑛𝑑𝑠 𝑓𝑜𝑟 𝑡ℎ𝑒𝑟𝑒 𝑒𝑥𝑖𝑠𝑡 𝑎𝑛𝑑 1 𝑖𝑠 𝑐𝑎𝑙𝑙𝑒𝑑 𝑡ℎ𝑒 ∀ 𝑎 ,𝑏 ∈ ℜ
(i). 𝑖𝑓 𝑎 > 𝑏 𝑎𝑛𝑑 𝑏 > 𝑐 𝑡ℎ𝑒𝑛 𝑎 > 𝑐.
𝑖𝑑𝑖𝑡𝑖𝑣𝑒 𝑖𝑑𝑒𝑛𝑡𝑖𝑡𝑦.
(ii). 𝑖𝑓 𝑎 < 𝑏 𝑎𝑛𝑑 𝑏 < 𝑐 𝑡ℎ𝑒𝑛 𝑎 < 𝑐.
Multiplicative Inverse: Additive Property:
∀ 𝑎 ,𝑏 ∈ ℜ
1 1 1 (i). 𝑖𝑓 𝑎 > 𝑏 𝑡ℎ𝑒𝑛 𝑎 + 𝑐 > 𝑏 + 𝑐.
∀ 𝑎 ∈ ℜ , ∃ 𝑎′ = ∈ ℜ 𝑠𝑢𝑐ℎ 𝑡ℎ𝑎𝑡 𝑎. = . 𝑎
𝑎 𝑎 𝑎 (ii). 𝑖𝑓 𝑎 < 𝑏 𝑎𝑛𝑑 𝑡ℎ𝑒𝑛 𝑎 + 𝑐 < 𝑏 + 𝑐.
= 1. Multiplicative Properties:
∀ 𝑎 ,𝑏 ,𝑐 ∈ ℜ
𝐼𝑓 𝑐 > 0
3|Page
Class 9th Chapter 2 www.notes.pk.com
(i). 𝑖𝑓 𝑎 > 𝑏 𝑡ℎ𝑒𝑛 𝑎𝑐 > 𝑏𝑐. Question.3. Give the name of property used in
(ii). 𝑖𝑓 𝑎 < 𝑏 𝑎𝑛𝑑 𝑡ℎ𝑒𝑛 𝑎𝑐 < 𝑏𝑐. the following.
𝐼𝑓 𝑐 < 0 (i). √𝟐𝟒 + 𝟎 = √𝟐𝟒
(iii). 𝑖𝑓 𝑎 > 𝑏 𝑡ℎ𝑒𝑛 𝑎𝑐 < 𝑏𝑐. Solution.
(iv). 𝑖𝑓 𝑎 < 𝑏 𝑎𝑛𝑑 𝑡ℎ𝑒𝑛 𝑎𝑐 > 𝑏𝑐. 𝐴𝑑𝑑𝑖𝑡𝑖𝑣𝑒 𝑖𝑑𝑒𝑛𝑡𝑖𝑡𝑦.
𝟐 𝟕 𝟐 𝟐 𝟕
(ii). − 𝟑 (𝟓 + 𝟐) = (− 𝟑) (𝟓) + (− 𝟑) (𝟐)
Exercise 2.2 Solution.
Question.1. Identify the property used in the
𝐷𝑖𝑠𝑡𝑟𝑖𝑏𝑢𝑡𝑖𝑣𝑒 𝑝𝑟𝑜𝑝𝑒𝑟𝑡𝑦 𝑜𝑓 𝑚𝑢𝑙𝑡𝑖𝑝𝑙𝑖𝑐𝑎𝑡𝑖𝑜𝑛 𝑜𝑣𝑒𝑟
following.
𝑎𝑑𝑑𝑖𝑡𝑖𝑜𝑛.
(i). 𝒂 + 𝒃 = 𝒃 + 𝒂
(iii). 𝝅 + (−𝝅) = 𝟎
Solution.
Solution.
𝐶𝑜𝑚𝑚𝑢𝑡𝑎𝑡𝑖𝑣𝑒 𝑝𝑟𝑜𝑝𝑒𝑟𝑡𝑦 𝑤. 𝑟. 𝑡 𝐴𝑑𝑑𝑖𝑡𝑖𝑜𝑛.
𝐴𝑑𝑑𝑖𝑡𝑖𝑣𝑒 𝐼𝑛𝑣𝑒𝑟𝑠𝑒.
(ii). (𝒂𝒃)𝒄 = 𝒂(𝒃𝒄)
(iv). √𝟑. √𝟑 𝒊𝒔 𝒂 𝒓𝒆𝒂𝒍 𝒏𝒖𝒎𝒃𝒆𝒓.
Solution.
Solution.
𝐴𝑠𝑠𝑜𝑐𝑖𝑎𝑡𝑖𝑣𝑒 𝑝𝑟𝑜𝑝𝑒𝑟𝑡𝑦 𝑤. 𝑟. 𝑡 𝑀𝑢𝑙𝑡𝑖𝑝𝑙𝑖𝑐𝑎𝑡𝑖𝑜𝑛.
𝐶𝑙𝑜𝑠𝑢𝑟𝑒 𝑙𝑎𝑤 𝑓𝑜𝑟 𝑚𝑢𝑙𝑡𝑖𝑝𝑙𝑖𝑐𝑎𝑡𝑖𝑜𝑛.
(iii). 𝟕 × 𝟏 = 𝟕 𝟓 𝟖
Solution. (v). (− 𝟖) (− 𝟓) = 𝟏
𝑀𝑢𝑙𝑡𝑖𝑝𝑙𝑖𝑐𝑎𝑡𝑖𝑣𝑒 𝑖𝑑𝑒𝑛𝑡𝑖𝑡𝑦. Solution.
(iv). 𝒙 > 𝒚 𝒐𝒓 𝒙 = 𝒚 𝒐𝒓 𝒙 < 𝒚 𝑀𝑢𝑙𝑡𝑖𝑝𝑙𝑖𝑐𝑎𝑡𝑖𝑣𝑒 𝐼𝑛𝑣𝑒𝑟𝑠𝑒.
Solution. Radicals and Radicands:
𝑇𝑟𝑖𝑐ℎ𝑜𝑡𝑟𝑜𝑚𝑦 𝑃𝑟𝑜𝑝𝑒𝑟𝑡𝑦.
(v). 𝒂𝒃 = 𝒃𝒂 If 𝑛 is a positive integer greater than 1 and 𝑎 is a
Solution. real number , then any real number 𝑥 such that
𝐶𝑜𝑚𝑚𝑢𝑡𝑎𝑡𝑖𝑣𝑒 𝑝𝑟𝑜𝑝𝑒𝑟𝑡𝑦 𝑤. 𝑟. 𝑡 𝑀𝑢𝑙𝑡𝑖𝑝𝑙𝑖𝑐𝑎𝑡𝑖𝑜𝑛. 𝑥𝑛 = 𝑎 is called the nth root of
(vi). 𝒂 + 𝒃 = 𝒃 + 𝒄 => 𝒂 = 𝒃 𝑎 , 𝑎𝑛𝑑 𝑖𝑛 𝑠𝑦𝑚𝑏𝑜𝑙𝑠 𝑖𝑠 𝑤𝑟𝑖𝑡𝑡𝑒𝑛 𝑎𝑠
Solution. 𝑛
1
𝐶𝑎𝑛𝑐𝑒𝑙𝑙𝑎𝑡𝑖𝑜𝑛 𝐿𝑎𝑤 𝑤. 𝑟. 𝑡 𝐴𝑑𝑑𝑖𝑡𝑖𝑜𝑛. 𝑥 = √𝑎 𝑜𝑟 𝑥 = (𝑎)𝑛
(vii). 𝟓 + (−𝟓) = 𝟎 𝑛
And √𝑎 is called radical, the symbol √ is called
Solution.
the radical sign , n is called the index of the radical
𝐴𝑑𝑑𝑖𝑡𝑖𝑣𝑒 𝐼𝑛𝑣𝑒𝑟𝑠𝑒.
𝟏 and the real number 𝑎 under the radical sign is
(viii). 𝟕 × 𝟕 called the radicand or base.
Solution.
𝑀𝑢𝑙𝑡𝑖𝑝𝑙𝑖𝑐𝑎𝑡𝑖𝑣𝑒 𝐼𝑛𝑣𝑒𝑟𝑠𝑒. Radical and Exponent Form:
(ix). 𝒂 > 𝒃 => 𝒂𝒄 > 𝒃𝒄 ( 𝒄 > 𝟎) 𝑛
𝑥 = √𝑎 𝑖𝑠 𝑐𝑎𝑙𝑙𝑒𝑑 𝑟𝑒𝑑𝑖𝑐𝑎𝑙 𝑓𝑜𝑟𝑚 𝑎𝑛𝑑 𝑎
Solution. 1
𝑀𝑢𝑙𝑡𝑝𝑙𝑖𝑐𝑎𝑡𝑖𝑣𝑒 𝑝𝑟𝑜𝑝𝑒𝑟𝑡. = 𝑥 𝑛 𝑖𝑠 𝑐𝑎𝑙𝑙𝑒𝑑 𝑒𝑥𝑝𝑜𝑛𝑒𝑛𝑡 𝑓𝑜𝑟𝑚.
Question.2. Fill in the following blanks by Some Properties of Radicals:
stating the properties of real numbers used.
𝑛 𝑛 𝑛
𝟑𝒙 + 𝟑(𝒚 − 𝒙) (i). √𝑎𝑏 = √𝑎 √𝑏
Solution. 𝑛
𝑛 𝑎 √𝑎
Given (ii). √𝑏 = 𝑛
√𝑏
3𝑥 + 3(𝑦 − 𝑥) = 3𝑥 + 3𝑦 − 3𝑥
𝐷𝑖𝑠𝑡𝑟𝑖𝑏𝑢𝑡𝑖𝑣𝑒 𝑝𝑟𝑜𝑝𝑒𝑟𝑡𝑦 𝑤. 𝑟. 𝑡 𝑚𝑢𝑙𝑖𝑝𝑙𝑖𝑐𝑎𝑡𝑖𝑜𝑛 𝑜𝑣𝑒𝑟 𝑛
(iii). √ √𝑎 =
𝑚 𝑛𝑚
√𝑎
𝑠𝑢𝑏𝑡𝑟𝑎𝑐𝑡𝑖𝑜𝑛.
𝑚
= 3𝑥 − 3𝑥 𝑛
(iv). √𝑎𝑚 = ( √𝑎)
𝑛

+ 3𝑦 𝑐𝑜𝑚𝑚𝑢𝑡𝑎𝑡𝑖𝑣𝑒 𝑝𝑟𝑜𝑝𝑒𝑟𝑡𝑦 𝑤. 𝑟. 𝑡 𝑎𝑑𝑑𝑖𝑡𝑖𝑜𝑛.


𝑛
= 0 + 3𝑦 𝑎𝑑𝑑𝑖𝑡𝑖𝑣𝑒 𝑖𝑛𝑣𝑒𝑟𝑠𝑒 𝑝𝑟𝑜𝑝𝑒𝑟𝑡𝑦. (v). √𝑎𝑛 = 𝑎
= 3𝑦 𝑎𝑑𝑑𝑖𝑡𝑖𝑣𝑒 𝑖𝑑𝑒𝑛𝑡𝑖𝑡𝑦.
Answer.

4|Page
Class 9th Chapter 2 www.notes.pk.com
1 1
Exercise # 2.3 = (24 )4 × 24
4
Question.1. Write each radical expression in = 2 × √2
4
exponential notation and each exponential = 2 √2
expression in radical notation. Do not simplify. Answer.
1
3
(i). √−64 = (−64)3 5 3
(iii). √32
Solution.
3
1 Solution.
√−64 = (−64)3 1
3 5 3 3 5
(ii). 2 5 √ =( )
Solution. 32 32
3 1
5
25 = √23 3 5
1 = ( 5)
(iii). −73 2
1
Solution. 35
1
= 1
25×5
3
−73 = − √7
5
2
√3
(iv). 𝑦 −3 =
Solution. 2
2
Answer.
𝑦 −3 = √𝑦 −2
3
3 −8
(iv). √ 27
Question.2. Tell whether the following
statements are true or false? Solution.
1 1
(i). 5 = √5
5 3 −8 −23 3
Solution. √ =( 3 )
27 3
1 5
𝐹𝑎𝑙𝑠𝑒 𝑏𝑒𝑐𝑎𝑢𝑠𝑒 55 = √5 𝑖𝑠 𝑡𝑟𝑢𝑒. 1
2 −23×3
(ii). 2 = √4
3
3 = 1
Solution. 33×3
2
−2
3 3 =
𝑇𝑟𝑢𝑒 𝑏𝑒𝑐𝑎𝑢𝑠𝑒 23 = √22 = √4 𝑖𝑠 𝑡𝑟𝑢𝑒. 3
(iii). √49 = √7 Answer.
Solution. Base and Exponents:
In the exponential form
𝐹𝑎𝑙𝑠𝑒 𝑏𝑒𝑐𝑎𝑢𝑠𝑒 √49 = √72 = 7𝑖𝑠 𝑡𝑟𝑢𝑒.
3 𝑎𝑛 ( 𝑟𝑒𝑎𝑑 𝑎𝑠 𝑎 𝑡𝑜 𝑡ℎ𝑒 𝑛𝑡ℎ 𝑝𝑜𝑤𝑒𝑟) we call ′′𝑎′′ as
(iv). √𝑥 27 = 𝑥 3
the base and ′′𝑛′′ as the exponent or power.
Solution.
27 Laws of Exponents:
3
𝐹𝑎𝑙𝑠𝑒 𝑏𝑒𝑐𝑎𝑢𝑠𝑒 √𝑥 27 = 𝑥 3 = 𝑥 9 𝑖𝑠 𝑡𝑟𝑢𝑒. If 𝑎, 𝑏 ∈
Question.3. Simplify the following radical 𝑅 𝑎𝑛𝑑 𝑚 , 𝑛 𝑎𝑟𝑒 𝑝𝑜𝑠𝑖𝑡𝑖𝑣𝑒 𝑖𝑛𝑡𝑒𝑔𝑒𝑟𝑠, 𝑡ℎ𝑒𝑛
expressions. (i). 𝑎𝑚 . 𝑎𝑛 = 𝑎𝑚+𝑛
3
(i). √−125 (ii). (𝑎𝑚 )𝑛 = 𝑎𝑚𝑛
Solution. (iii). (𝑎𝑏)𝑛 = 𝑎𝑛 𝑏 𝑛
3 3 𝑎 𝑛 𝑎𝑛
√−125 = √−53 (iv). (𝑏) = 𝑏𝑛
1
= (−5)3×3 𝑎𝑚
(v). = 𝑎𝑚−𝑛
= −5 𝑎𝑛

4
Answer. (vi). 𝑎0 = 1 , 𝑤ℎ𝑒𝑟𝑒 𝑎 ≠ 0
(ii). √32
1
Solution. (vii). 𝑎−𝑛 = , 𝑤ℎ𝑒𝑟𝑒 𝑎 ≠ 0
4 4
𝑎𝑛
√32 = √24 × 2
1
= (24 × 2)4

5|Page
Class 9th Chapter 2 www.notes.pk.com
Exercise # 2.4 𝑥18 𝑧12
=
Question.1. Use laws of exponents to simplify 𝑦6
Answer.
2 1
(243)−3 (32)−5 (81)𝑛 .35 −(3)4𝑛−1 (243)
(i). (iv). (9)2𝑛 .33
√(196)−1
Solution. Solution.
2 1 2 1 (81)𝑛 . 35 − (3)4𝑛−1 (243)
(243)−3 (32)−5 (35 )−3 (25 )− 5
= (9)2𝑛 . 33
1
√(196)−1 (142 )−1×2 (34 )𝑛 . 35 − (3)4𝑛−1 (3)5
10 =
(32 )2𝑛 . 33
3− 3 2− 1
= (3)4𝑛 . 35 − (3)4𝑛−1 (3)5
14−1 =
10 (3)4𝑛 . 33
3− 3 2− 1 34𝑛+5 − 34𝑛−1+5
= =
(2 × 7)−1 34𝑛+3
10 4𝑛+5
3− 3 2− 1 3 − 34𝑛+4
= −1 =
2 × 7−1 34𝑛+3
4𝑛+4 1
10 3 (3 − 1)
3− 3 = 4𝑛+3
= −1 3
7 = 34𝑛+4−4𝑛−3 (2)
7
= 9+1 = 31 (2)
3 3 =6
7 Answer
= 9 1
33+3 Question.2. Show that
7 𝒃+𝒄
= 9 𝒙𝒂 𝒂+𝒃 𝒙𝒃 𝒙𝒄 𝒄+𝒂
1 ( 𝒃) × ( 𝒄) × ( 𝒂) =𝟏
33 × 33 𝒙 𝒙 𝒙
7 Solution.
= 3 𝑏+𝑐
33 × √3 𝑥 𝑎 𝑎+𝑏 𝑥𝑏 𝑥 𝑐 𝑐+𝑎
7 𝐿. 𝐻. 𝑆 = ( 𝑏 ) × ( 𝑐) × ( 𝑎)
= 3 𝑥 𝑥 𝑥
27√3 𝐿. 𝐻. 𝑆 = (𝑥 𝑎−𝑏 ) 𝑎+𝑏 𝑏−𝑐
× (𝑥 ) 𝑏+𝑐
× (𝑥 𝑐−𝑎 )𝑐+𝑎
Answer.
𝐿. 𝐻. 𝑆 = 𝑥 (𝑎−𝑏)(𝑎+𝑏) × 𝑥 (𝑏−𝑐)(𝑏+𝑐) × 𝑥 (𝑐−𝑎)(𝑐+𝑎)
(ii). (2𝑥 5 𝑦 −4 )(−8𝑥 −3 𝑦 2 ) 2 2 2 2 2 2
𝐿. 𝐻. 𝑆 = 𝑥 𝑎 −𝑏 × 𝑥 𝑏 −𝑐 × 𝑥 𝑐 −𝑎
Solution. 2 2 2 2 2 2
(2𝑥 5 𝑦 −4 )(−8𝑥 −3 𝑦 2 ) = (2)(−8)𝑥 5 . 𝑦 −4 . 𝑥 −3 𝑦 2 𝐿. 𝐻. 𝑆 = 𝑥 𝑎 −𝑏 +𝑏 −𝑐 +𝑐 −𝑎
= −16 𝑥 5−3 . 𝑦 −4+2 𝐿. 𝐻. 𝑆 = 𝑥 0 = 1
= −16 𝑥 2 . 𝑦 −2 Hence Proved.
16𝑥 2 Question.3. Simplify
=− 2 1 1
23 (27)3 (60)2
1
𝑦 (i). 1 1 1
Answer. (180)2 (4)−3 94
−3 Solution.
𝑥 −2 𝑦 −1 𝑧 −4
(iii). ( 𝑥 4 𝑦 −3 𝑧 0
) 1 1 1 1 1 1
23 (27)3 (60)2 23 (33 )3 (22 . 3.5)2
Solution. 1 1 1 = 1 1 1
−2 −1 −4 −3
𝑥 𝑦 𝑧 𝑦 −1+3 −3 (180)2 (4)−3 94 (22 . 32 . 5)2 (22 )−3 (32 )4
( 4 −3 0 ) = ( ) 1 1 1 1
𝑥 𝑦 𝑧 𝑥 4+2 𝑧 0+4 23 31 . 22×2 . 32 . 52
−3 = 1 1 1 2 1
𝑦2 22×2 . 32×2 . 52 . 2−3 32×4
= ( 6 4) 1 1 1
𝑥 𝑧
3
23 31 . 21 . 32 . 52
𝑥6𝑧4 = 1 2 1
= ( 2 ) 21 . 31 . 52 . 2−3 32
𝑦
6×3 4×3
𝑥 𝑧
=
𝑦 2×3
6|Page
Class 9th Chapter 2 www.notes.pk.com
1 Complex Numbers:
23
= 2
The numbers of the form 𝑥 + 𝑖𝑦 , where 𝑥, 𝑦 ∈
2−3 ℜ ,are called complex numbers, here x is
1 2 called real part and y is called imaginary part
= 23+3
1+2 of the complex number.
=2 3 Remarks:
=
3
23
1. Every real number is a complex number
with 0 as its imaginary part.
=2
Conjugate Complex Numbers:
Answer. 𝑖𝑓 𝑍 = 𝑎 + 𝑖𝑏 𝑏𝑒 𝑎 𝑐𝑜𝑚𝑝𝑙𝑒𝑥 𝑛𝑢𝑚𝑏𝑒𝑟 𝑡ℎ𝑒𝑛 𝑍̅ =
2
(216)3 (25)2
1
𝑎 − 𝑖𝑏 is the conjugate of the complex number
(ii). √ 1 𝑍 = 𝑎 + 𝑖𝑏.
(0.04)−2
Remarks:
Solution.
1. A real number is self-Conjugate.
2 1 2 1 Equality of Two Complex Numbers:
(216)3 (25)2 (63 )3 (52 )2
= Two complex numbers 𝑎 + 𝑏𝑖 𝑎𝑛𝑑 𝑐 +
1 1

(0.04) 2 4 −2 𝑑𝑖 𝑎𝑟𝑒 𝑠𝑎𝑖𝑑 𝑡𝑜 𝑏𝑒 𝑒𝑞𝑢𝑎𝑙 𝑖𝑓 𝑎 = 𝑐 𝑎𝑛𝑑 𝑏 = 𝑑.
(100)
That is
𝑎 + 𝑖𝑏 = 𝑐 + 𝑖𝑑 ⇒ 𝑎 = 𝑏 𝑎𝑛𝑑 𝑐 = 𝑑.
62 . 51
= 1
1 −2 Exercise # 2.5
( )
25
Question.1. Evaluate
62 . 51 (i). 𝒊𝟕
=√ 1
(25)2 Solution.
𝒊𝟕 = 𝒊𝟔 . 𝒊
62 .5
=√ 1 = (𝒊𝟐 )𝟑 . 𝒊
(52 )2 = (−𝟏)𝟑 . 𝒊
= (−𝟏). 𝒊
62 . 5
=√ = −𝒊
5 𝑨𝒏𝒔𝒘𝒆𝒓.
= √36 (ii). 𝒊𝟓𝟎
=6 𝑺𝒐𝒍𝒖𝒕𝒊𝒐𝒏.
Answer. 𝒊𝟓𝟎 = (𝒊𝟐 )𝟐𝟓
3
(iii). 52 ÷ (52 )3 = (−𝟏)𝟐𝟓
Solution. = −𝟏
3 58 𝑨𝒏𝒔𝒘𝒆𝒓.
52 ÷ (52 )3 = 56
(𝒊𝒊𝒊). 𝒊𝟏𝟐
= 58−6 𝑺𝒐𝒍𝒖𝒕𝒊𝒐𝒏.
= 52 𝒊𝟏𝟐 = (𝒊𝟐 )𝟔
= 25
= (−𝟏)𝟔
Answer.
2
=𝟏
(iv). (𝑥 3 )2 ÷ 𝑥 3 Answer.
Solution. (iv). (−𝒊)𝟖
2 𝑥6 Solution.
(𝑥 3 )2 ÷ 𝑥 3 =
𝑥8 (−𝒊)𝟖 = 𝒊𝟖
1
= = (𝒊𝟐 )𝟒
𝑥 8−6
1 = (−𝟏)𝟒
= =𝟏
𝑥2
Answer. Answer.
(v). (−𝒊)𝟓
7|Page
Class 9th Chapter 2 www.notes.pk.com
Solution. Answer.
(−𝒊)𝟓= −𝒊 𝟓 (ii). −𝟏 + 𝟐𝒊
= −𝒊𝟒 . 𝒊 Solution.
= −(𝒊𝟐 )𝟐 . 𝒊 𝑆𝑢𝑝𝑝𝑜𝑠𝑒 𝑍 = −𝟏 + 𝟐𝒊
= −(−𝟏)𝟐 . 𝒊 𝑅𝑒(𝑍) = −1 , 𝐼𝑚(𝑍) = 2
= −(𝟏). 𝒊 Answer.
= −𝒊 (iii). −𝟑𝒊 + 𝟐
Answer. Solution.
(vi). 𝒊𝟐𝟕 𝑆𝑢𝑝𝑝𝑜𝑠𝑒 𝑍 = −𝟑𝒊 + 𝟐 = 𝟐 − 𝟑𝒊
Solution. 𝑅𝑒(𝑍) = 2 , 𝐼𝑚(𝑍) = −3
𝒊𝟐𝟕 = 𝒊𝟐𝟔 . 𝒊 Answer.
= (𝒊𝟐 )𝟏𝟑 . 𝒊 (iv). −𝟐 − 𝟐𝒊
= (−𝟏)𝟏𝟑 . 𝒊 Solution.
= (−𝟏). 𝒊 𝑆𝑢𝑝𝑝𝑜𝑠𝑒 𝑍 = −𝟐𝒊 − 𝟐 = −𝟐 − 𝟐𝒊
= −𝟏 𝑅𝑒(𝑍) = −2 , 𝐼𝑚(𝑍) = −2
Answer. Answer.
Question.2. Write the conjugate of the following (v). −𝟑𝒊
numbers. Solution.
(i). 𝟐 + 𝟑𝒊 𝑆𝑢𝑝𝑝𝑜𝑠𝑒 𝑍 = −𝟑𝒊 = 𝟎 − 𝟑𝒊
Solution. 𝑅𝑒(𝑍) = 0 , 𝐼𝑚(𝑍) = −3
𝑆𝑢𝑝𝑝𝑜𝑠𝑒 𝑍 = 2 + 3𝑖 Answer.
𝑍̅ = ̅̅̅̅̅̅̅̅
2 + 3𝑖 = 2 − 3𝑖 (vi). 𝟐 + 𝟎𝒊
Answer. Solution.
(ii). 𝟑 − 𝟓𝒊 𝑆𝑢𝑝𝑝𝑜𝑠𝑒 𝑍 = 𝟐 + 𝟎𝒊
Solution. 𝑅𝑒(𝑍) = 2 , 𝐼𝑚(𝑍) = 0
𝑆𝑢𝑝𝑝𝑜𝑠𝑒 𝑍 = 𝟑 − 𝟓𝒊 Answer.
𝑍̅ = ̅̅̅̅̅̅̅̅
𝟑 − 𝟓𝒊 = 𝟑 + 𝟓𝒊 Question.4. Find the value of 𝒙 and 𝒚 if
Answer. 𝒙 + 𝒊𝒚 + 𝟏 = 𝟒 − 𝟑𝒊
(iii). −𝒊 Solution.
Solution. Given that
𝑆𝑢𝑝𝑝𝑜𝑠𝑒 𝑍 = −𝒊 𝒙 + 𝟏 + 𝒊𝒚 = 𝟒 − 𝟑𝒊
𝑍̅ = −𝒊
̅̅̅̅ = +𝒊 Separating real and imaginary parts
Answer. 𝒙+𝟏=𝟒 , 𝒚 = −𝟑
(iv). −𝟑 + 𝟒𝒊 𝒙=𝟒−𝟏 , 𝒚 = −𝟑
𝒙=𝟑 , 𝒚 = −𝟑
Solution.
Answer.
𝑆𝑢𝑝𝑝𝑜𝑠𝑒 𝑍 = −𝟑 + 𝟒𝒊
Operations on Complex Numbers:
𝑍̅ = ̅̅̅̅̅̅̅̅̅̅̅
−𝟑 + 𝟒𝒊 = −𝟑 − 𝟒𝒊
The symbols a,b,c,d,k, where used, represent
Answer. real numbers
(v). −𝟒 − 𝒊 Addition of Two Complex Numbers:
Solution. (𝑎 + 𝑖𝑏) + (𝑐 + 𝑖𝑑) = (𝑎 + 𝑏) + 𝑖(𝑐 + 𝑑).
𝑆𝑢𝑝𝑝𝑜𝑠𝑒 𝑍 = −𝟒 − 𝒊 Scalar Multiplication:
𝑍̅ = ̅̅̅̅̅̅̅̅̅̅
−𝟒 − 𝒊 = −𝟒 + 𝒊 𝑘(𝑎 + 𝑖𝑏) = 𝑘𝑎 + 𝑖𝑘𝑏.
Answer. Subtraction of Two Complex Numbers:
Question.3. Write the real and imaginary part of (𝑎 + 𝑖𝑏) − (𝑐 + 𝑖𝑑) = (𝑎 − 𝑏) + 𝑖(𝑐 − 𝑑).
the following numbers. Multiplication of Two Complex Numbers:
(i). 𝟏 + 𝒊 (𝑎 + 𝑖𝑏)(𝑐 + 𝑖𝑑) = (𝑎𝑐 − 𝑏𝑑) + 𝑖(𝑎𝑑 + 𝑏𝑐).
Solution. Division of two Complex Numbers:
𝑆𝑢𝑝𝑝𝑜𝑠𝑒 𝑍 = 1 + 𝑖 (𝑎 + 𝑖𝑏) 𝑎𝑐 − 𝑏𝑑 𝑏𝑐 − 𝑎𝑑
𝑅𝑒(𝑍) = 1 , 𝐼𝑚(𝑍) = 1 = 2 + 𝑖
(𝑐 + 𝑖𝑑) 𝑐 + 𝑑 2 𝑐 2 + 𝑑2

8|Page
Class 9th Chapter 2 www.notes.pk.com
Exercise # 2.6 Solution.
Question.1. Identify the following statements as 𝟐(𝟓 + 𝟒𝒊) − 𝟑(𝟕 + 𝟒𝒊) = 𝟏𝟎 + 𝟖𝒊 − 𝟐𝟏 − 𝟏𝟐𝒊
true or false. = −𝟏𝟏 − 𝟑𝒊
(i). √−3 × √−3 = 3 Answer.
Solution. (iii). −𝟏(−𝟑 + 𝟓𝒊) − (𝟒 + 𝟗𝒊)
Solution.
𝐹𝑎𝑙𝑠𝑒 𝑏𝑒𝑐𝑎𝑢𝑠𝑒 √−3 × √−3 = √3𝑖 × √3𝑖
2 −𝟏(−𝟑 + 𝟓𝒊) − (𝟒 + 𝟗𝒊) = 𝟑 − 𝟓𝒊 − 𝟒 − 𝟗𝒊
= (√3) 𝑖 2 = −3 = −𝟏 − 𝟏𝟒𝒊
(ii). 𝑖 73 = −𝑖 Answer.
Solution. (iv). 𝟐𝒊𝟐 + 𝟔𝒊𝟑 + 𝟑𝒊𝟏𝟔 − 𝟔𝒊𝟏𝟗 + 𝟒𝒊𝟐𝟓
𝐹𝑎𝑙𝑠𝑒 𝑏𝑒𝑐𝑎𝑢𝑠𝑒 𝑖 73 = 𝑖 72 . 𝑖 = (𝑖 2 )36 . 𝑖 Solution.
= (−1)36 . 𝑖 = 𝑖 𝟐𝒊𝟐 + 𝟔𝒊𝟑 + 𝟑𝒊𝟏𝟔 − 𝟔𝒊𝟏𝟗 + 𝟒𝒊𝟐𝟓
(iii). 𝑖 10 = −1 = 𝟐(−𝟏) + 𝟔𝒊𝟐 𝒊 + 𝟑𝒊𝟏𝟔 − 𝟔𝒊𝟏𝟖 𝒊
Solution. + 𝟒𝒊𝟐𝟒 𝒊
𝑇𝑟𝑢𝑒 𝑏𝑒𝑐𝑎𝑢𝑠𝑒 𝑖 10 = (𝑖 2 )5 = (−1)5 = −1 = 𝟐(−𝟏) + 𝟔(−𝟏)𝒊 + 𝟑(𝒊𝟐 )𝟖 − 𝟔(𝒊𝟐 )𝟗 𝒊
(iv). 𝐶𝑜𝑚𝑝𝑙𝑒𝑥 𝑐𝑜𝑛𝑗𝑢𝑔𝑎𝑡𝑒 𝑜𝑓 (−6𝑖 + + 𝟒(𝒊𝟐 )𝟏𝟐 𝒊
𝑖 2 ) 𝑖𝑠 (−1 + 6𝑖) = −𝟐 − 𝟔𝒊 + 𝟑(−𝟏)𝟖 − 𝟔(−𝟏)𝟗 𝒊 + 𝟒(−𝟏)𝟏𝟐 𝒊
Solution. = −𝟐 − 𝟔𝒊 + 𝟑(𝟏) − 𝟔(−𝟏)𝒊 + 𝟒(𝟏)𝒊
𝑇𝑟𝑢𝑒 𝑏𝑒𝑐𝑎𝑢𝑠𝑒 ̅̅̅̅̅̅̅̅̅̅̅
−6𝑖 + 𝑖2 = ̅̅̅̅̅̅̅̅̅̅
−6𝑖 − 1 = −1 + 6𝑖 = −𝟐 − 𝟔𝒊 + 𝟑 + 𝟔𝒊 + 𝟒𝒊
(v). Difference of a complex number 𝑧 = 𝑎 + = 𝟏 + 𝟒𝒊
𝑏𝑖 𝑎𝑛𝑑 𝑖𝑡𝑠 𝑐𝑜𝑛𝑗𝑢𝑔𝑎𝑡𝑒 𝑖𝑠 𝑎 𝑟𝑒𝑎𝑙 𝑛𝑢𝑚𝑏𝑒𝑟. Question.3. Simplify and write your answer in
Solution. the form 𝒂 + 𝒃𝒊.
𝐹𝑎𝑙𝑠𝑒 𝑏𝑒𝑐𝑎𝑢𝑠𝑒 𝑍 − 𝑍̅ = (𝑎 + 𝑏𝑖) − (𝑎 − 𝑏𝑖) (i). (−𝟕 + 𝟑𝒊)(−𝟑 + 𝟐𝒊)
= 𝑎 + 𝑏𝑖 − 𝑎 + 𝑏𝑖 = 2𝑏𝑖 Solution.
(vi). If (𝑎 − 1) − (𝑏 + 3)𝑖 = 5 + 8𝑖 𝑡ℎ𝑒𝑛 𝑎 = (−𝟕 + 𝟑𝒊)(−𝟑 + 𝟐𝒊) = 𝟐𝟏 − 𝟏𝟒𝒊 − 𝟗𝒊 + 𝟔𝒊𝟐
6 𝑎𝑛𝑑 𝑏 = −11. = 𝟐𝟏 − 𝟏𝟒𝒊 − 𝟗𝒊 − 𝟔
Solution. = 𝟏𝟓 − 𝟐𝟑𝒊
True because Comparing real and imaginary parts Answer.
in given equation (ii). (𝟐 − √−𝟒)(𝟑 − √−𝟒)
𝑎 − 1 = 5 , − (𝑏 + 3) = 8 Solution.
𝑎 = 5+1 , 𝑏 + 3 = −8 (𝟐 − √−𝟒)(𝟑 − √−𝟒) = (𝟐 − 𝟐𝒊)(𝟑 − 𝟐𝒊)
𝑎=6 , 𝑏 = −8 − 3 = 𝟐(𝟑 − 𝟐𝒊) − 𝟐𝒊(𝟑 − 𝟐𝒊)
𝑎=6 , 𝑏 = −11 = 𝟔 − 𝟒𝒊 − 𝟔𝒊 + 𝟒𝒊𝟐
(vii) Product of a complex number and its = 𝟔 − 𝟏𝟎𝒊 − 𝟒
conjugate is always a non-negative real number. = 𝟐 − 𝟏𝟎𝒊
Solution. Answer.
𝟐
𝑻𝒓𝒖𝒆 𝒃𝒆𝒄𝒂𝒖𝒔𝒆 𝒇𝒐𝒓 𝒂 𝒄𝒐𝒎𝒑𝒍𝒆𝒙 𝒏𝒖𝒎𝒆𝒓 𝒁 (iii). (√𝟓 − 𝟑𝒊)
= 𝒂 + 𝒃𝒊 Solution.
𝒁. 𝒁̅ = (𝒂 + 𝒃𝒊) . (𝒂 − 𝒃𝒊) = 𝒂𝟐 − (𝒃𝒊)𝟐 𝟐 𝟐
(√𝟓 − 𝟑𝒊) = (√𝟓) + (𝟑𝒊)𝟐 − 𝟐(√𝟓)(𝟑𝒊)
= 𝒂𝟐 + 𝒃𝟐
Is a real number. = 𝟓 + 𝟗𝒊𝟐 − 𝟔√𝟓𝒊
Question.2. Express each complex number in = 𝟓 − 𝟗 − 𝟔√𝟓𝒊
the standard form 𝒂 + 𝒃𝒊 where ‘a’ and ‘b’ = −𝟒 − 𝟔√𝟓𝒊
are real numbers. Answer.
(i). (2 + 3𝑖) + (7 − 2𝑖) ̅̅̅̅̅̅̅̅
(iv). (𝟐 − 𝟑𝒊)(𝟑 − 𝟐𝒊)
Solution. Solution.
(2 + 3𝑖) + (7 − 2𝑖) = 2 + 3𝑖 + 7 − 2𝑖 ̅̅̅̅̅̅̅̅
(𝟐 − 𝟑𝒊)(𝟑 − 𝟐𝒊) = (𝟐 − 𝟑𝒊)(𝟑 + 𝟐𝒊)
=𝟗+𝒊 = 𝟐(𝟑 + 𝟐𝒊) − 𝟑𝒊(𝟑 + 𝟐𝒊)
Answer. = 𝟔 + 𝟒𝒊 − 𝟗𝒊 − 𝟔𝒊𝟐
(ii). 𝟐(𝟓 + 𝟒𝒊) − 𝟑(𝟕 + 𝟒𝒊) = 𝟔 − 𝟓𝒊 + 𝟔
9|Page
Class 9th Chapter 2 www.notes.pk.com
= 𝟏𝟐 − 𝟓𝒊 𝟐 − 𝟔𝒊 𝟒 + 𝒊 𝟐 − 𝟔𝒊 𝟑 − 𝒊 𝟒 + 𝒊 𝟑 − 𝒊
− = × − ×
Answer. 𝟑+𝒊 𝟑+𝒊 𝟑+𝒊 𝟑−𝒊 𝟑+𝒊 𝟑−𝒊
Question.4. Simplify and write your answer in 𝟐(𝟑 − 𝒊) − 𝟔𝒊(𝟑 − 𝒊) 𝟒(𝟑 − 𝒊) + 𝒊(𝟑 − 𝒊)
= −
the form of 𝒂 + 𝒃𝒊. 𝟑𝟐 − 𝒊𝟐 𝟑𝟐 − 𝒊𝟐
(i). − 𝟏+𝒊
𝟐 𝟔 − 𝟐𝒊 − 𝟏𝟖𝒊 + 𝟔𝒊𝟐
=
𝟗+𝟏
Solution.
𝟏𝟐 − 𝟒𝒊 + 𝟑𝒊 − 𝒊𝟐
𝟐 −𝟐 𝟏−𝒊 −
− = × 𝟗+𝟏
𝟏+𝒊 𝟏+𝒊 𝟏−𝒊 𝟔 − 𝟐𝟎𝒊 − 𝟔 𝟏𝟐 − 𝒊 + 𝟏
−𝟐 + 𝟐𝒊 = −
= 𝟐 𝟏𝟎 𝟏𝟎
𝟏 − 𝒊𝟐 −𝟐𝟎𝒊 𝟏𝟑 − 𝒊
−𝟐 + 𝟐𝒊 = −
= 𝟏𝟎 𝟏𝟎
𝟏+𝟏 −𝟐𝟎𝒊 − 𝟏𝟑 + 𝒊
−𝟐 + 𝟐𝒊 =
= 𝟏𝟎
𝟐 −𝟐𝟎𝒊 − 𝟏𝟑 + 𝒊
𝟐 𝟐𝒊 =
=− + 𝟏𝟎
𝟐 𝟐 −𝟏𝟑 − 𝟏𝟗𝒊
= −𝟏 + 𝒊 =
𝟏𝟎
Answer. 𝟏𝟑 𝟏𝟗
(ii).
𝟐+𝟑𝒊 =− − 𝒊
𝟒−𝒊 𝟏𝟎 𝟏𝟎
Solution. Answer.
𝟐 + 𝟑𝒊 𝟐 + 𝟑𝒊 𝟒 + 𝒊 𝟏+𝒊 𝟐
= × (v). (𝟏−𝒊)
𝟒−𝒊 𝟒−𝒊 𝟒+𝒊
𝟐(𝟒 + 𝒊) + 𝟑𝒊(𝟒 + 𝒊) Solution.
= 𝟏+𝒊 𝟐 𝟏+𝒊 𝟏+𝒊 𝟐
𝟒𝟐 − 𝒊𝟐 ( ) =( × )
𝟖 + 𝟐𝒊 + 𝟏𝟐𝒊 + 𝟑𝒊𝟐 𝟏−𝒊 𝟏−𝒊 𝟏+𝒊
= 𝟏(𝟏 + 𝒊) + 𝒊(𝟏 + 𝒊) 𝟐
𝟏𝟔 + 𝟏
𝟖 + 𝟏𝟒𝒊 − 𝟑 =( )
𝟏𝟐 − 𝒊𝟐
= 𝟐
𝟏𝟕 𝟏 + 𝒊 + 𝒊 + 𝒊𝟐
𝟒 + 𝟏𝟒𝒊 =( )
= 𝟏+𝟏
𝟏𝟕
𝟒 𝟏𝟒 𝟏 + 𝟐𝒊 − 𝟏 𝟐
= + 𝒊 =( )
𝟏𝟕 𝟏𝟕 𝟐
Answer. 𝟐𝒊 𝟐
𝟗−𝟕𝒊 =( )
(iii). 𝟐
𝟑+𝒊
= 𝒊𝟐
Solution.
𝟗 − 𝟕𝒊 𝟗 − 𝟕𝒊 𝟑 − 𝒊 = −𝟏
= × Answer.
𝟑+𝒊 𝟑+𝒊 𝟑−𝒊 𝟏
𝟗(𝟑 − 𝒊) − 𝟕𝒊(𝟑 − 𝒊) (vi). (𝟐+𝟑𝒊)(𝟏−𝒊)
=
𝟑𝟐 − 𝒊𝟐 Solution.
𝟐𝟕 − 𝟗𝒊 − 𝟐𝟏𝒊 + 𝟕𝒊𝟐 𝟏 𝟏
= =
𝟗+𝟏 (𝟐 + 𝟑𝒊)(𝟏 − 𝒊) 𝟐(𝟏 − 𝒊) + 𝟑𝒊(𝟏 − 𝒊)
𝟐𝟕 − 𝟑𝟎𝒊 − 𝟕 𝟏
= =
𝟏𝟎 𝟐 − 𝟐𝒊 + 𝟑𝒊 − 𝟑𝒊𝟐
𝟐𝟎 − 𝟑𝟎𝒊 𝟏
= =
𝟏𝟎 𝟐+𝒊+𝟑
𝟐𝟎 𝟑𝟎
= − 𝒊 𝟏
𝟏𝟎 𝟏𝟎 =
𝟓+𝒊
= 𝟐 − 𝟑𝒊 𝟏 𝟓−𝒊
Answer. = ×
𝟐−𝟔𝒊 𝟒+𝒊 𝟓+𝒊 𝟓−𝒊
(iv). − 𝟑+𝒊 𝟓−𝒊
𝟑+𝒊 = 𝟐
Solution. 𝟓 − (𝒊)𝟐

10 | P a g e
Class 9th Chapter 2 www.notes.pk.com
𝟓−𝒊 −𝟒 − 𝟐𝟐𝒊
= 𝒁=
𝟐𝟓 + 𝟏 𝟒 + 𝟏𝟔
𝟓−𝒊 −𝟒 − 𝟐𝟐𝒊
= 𝒁=
𝟐𝟔 𝟐𝟎
𝟓 𝒊 𝟒 𝟐𝟐
= − 𝒁=− − 𝒊
𝟐𝟔 𝟐𝟔 𝟐𝟎 𝟐𝟎
Answer. 𝟏 𝟏𝟏
𝒁=− − 𝒊
Question.5. 𝟓 𝟏𝟎
Calculate (a) 𝒁 ̅ (𝒃)𝒁 + 𝒁
̅ (𝒄) 𝒁 − ̅ = ̅̅̅̅̅̅̅̅̅̅̅̅
(a). 𝒁
𝟏 𝟏𝟏 𝟏 𝟏𝟏
− 𝟓 − 𝟏𝟎 𝒊 = − 𝟓 + 𝟏𝟎 𝒊
̅ (𝒅) 𝒁𝒁
𝒁 ̅ for each of the following 𝟏 𝟏𝟏 𝟏 𝟏𝟏
̅ =− − 𝒊+− + 𝒊
(b). 𝒁 + 𝒁
(i). 𝒁 = −𝒊 𝟓 𝟏𝟎 𝟓 𝟏𝟎
𝟏 𝟏 −𝟏 − 𝟏 𝟐 𝟐
Solution. 𝒁+𝒁 ̅=− − = =− =−
(a). 𝒁̅ = ̅̅̅
−𝒊̅ = 𝒊 𝟓 𝟓 𝟓 𝟓 𝟓
̅ = −𝒊 + 𝒊 = 𝟎 𝟐
(b). 𝒁 + 𝒁 𝒁+𝒁 ̅=−
̅ = (−𝒊) − (𝒊) = −𝒊 − 𝒊 = −𝟐𝒊
(c). 𝒁 − 𝒁 𝟓
𝟏 𝟏𝟏 𝟏 𝟏𝟏
̅ = (−𝒊)(𝒊) = −𝒊𝟐 = 𝟏 ̅
(c). 𝒁 − 𝒁 = (− 𝟓 − 𝟏𝟎 𝒊) − (− 𝟓 + 𝟏𝟎 𝒊)
(d). 𝒁𝒁
(ii). 𝒁 = 𝟐 + 𝒊 𝟏 𝟏𝟏 𝟏 𝟏𝟏
𝒁−𝒁 ̅=− − 𝒊+ − 𝒊
Solution. 𝟓 𝟏𝟎 𝟓 𝟏𝟎
(a). 𝒁̅ = ̅̅̅̅̅̅̅
𝟐+𝒊=𝟐−𝒊 𝟏𝟏 𝟏𝟏
𝒁−𝒁 ̅=− 𝒊− 𝒊
(b). 𝒁 + 𝒁̅ =𝟐+𝒊+𝟐−𝒊=𝟒 𝟏𝟎 𝟏𝟎
̅ = (𝟐 + 𝒊) − (𝟐 − 𝒊) = 𝟐 + 𝒊 − 𝟐 + −𝟏𝟏 − 𝟏𝟏
(c). 𝒁 − 𝒁 𝒁−𝒁 ̅= 𝒊
𝒊 = 𝟐𝒊 𝟏𝟎
𝟐𝟐
(d). 𝒁𝒁̅ = (𝟐 + 𝒊)(𝟐 − 𝒊) = 𝟐𝟐 − 𝒊𝟐 = 𝟒 + 𝟏 = 𝒁−𝒁 ̅=− 𝒊
𝟏𝟎
𝟓 𝟏𝟏
𝟏+𝒊 𝒁−𝒁 ̅=− 𝒊
(iii). 𝒁 = 𝟏−𝒊 𝟓
Solution.
𝟏+𝒊 ̅ = (− 𝟏 − 𝟏𝟏 𝒊) (− 𝟏 + 𝟏𝟏 𝒊)
(d). 𝒁𝒁
𝒁= 𝟓 𝟏𝟎 𝟓 𝟏𝟎
𝟏−𝒊 𝟏 𝟐 𝟏𝟏 𝟐
𝟏+𝒊 𝟏+𝒊 ̅
𝒁𝒁 = (− ) − ( 𝒊)
𝒁= × 𝟓 𝟏𝟎
𝟏−𝒊 𝟏+𝒊 𝟏 𝟏𝟐𝟏 𝟐
𝟏 + 𝒊 + 𝒊 + 𝒊𝟐 ̅=
𝒁𝒁 − 𝒊
𝒁= 𝟐𝟓 𝟏𝟎𝟎
𝟏+𝟏 𝟏 𝟏𝟐𝟏
𝟏 + 𝟐𝒊 − 𝟏 ̅=
𝒁𝒁 +
𝒁= 𝟐𝟓 𝟏𝟎𝟎
𝟐 𝟒 + 𝟏𝟐𝟏
𝟐𝒊 𝒁𝒁̅=
𝒁= 𝟏𝟎𝟎
𝟐 𝟏𝟐𝟓
𝒁=𝒊 ̅=
𝒁𝒁
𝟏𝟎𝟎
̅ = 𝜾̅ = −𝒊
(a). 𝒁 𝟓
̅ =𝒊−𝒊=𝟎 𝒁𝒁̅=
(b). 𝒁 + 𝒁 𝟒
̅ = (𝒊) − (−𝒊) = 𝒊 + 𝒊 = 𝟐𝒊
(c). 𝒁 − 𝒁 Answer.
(d). 𝒁𝒁̅ = (𝒊)(−𝒊) = −𝒊𝟐 = 𝟏 Question.6. If 𝒛 = 𝟐 + 𝟑𝒊 , 𝒘 = 𝟓 −
𝟒−𝟑𝒊
(iv). 𝒁 = 𝟐+𝟒𝒊 𝟒𝒊 , 𝒔𝒉𝒐𝒘 𝒕𝒉𝒂𝒕
Solution. (i). ̅̅̅̅̅̅̅̅
𝒛 + 𝒘 = 𝒛̅ + 𝒘 ̅
𝟒 − 𝟑𝒊 Solution.
𝒁= 𝑳. 𝑯. 𝑺 = ̅̅̅̅̅̅̅̅
𝒛+𝒘
𝟐 + 𝟒𝒊
𝟒 − 𝟑𝒊 𝟐 − 𝟒𝒊 𝑳. 𝑯. 𝑺 = ̅̅̅̅̅̅̅̅̅̅̅̅̅̅̅̅̅̅̅̅
𝟐 + 𝟑𝒊 + 𝟓 − 𝟒𝒊
𝒁= × 𝑳. 𝑯. 𝑺 = ̅̅̅̅̅̅̅
𝟖−𝒊
𝟐 + 𝟒𝒊 𝟐 − 𝟒𝒊
𝟖 − 𝟏𝟔𝒊 − 𝟔𝒊 + 𝟏𝟐𝒊𝟐 𝑳. 𝑯. 𝑺 = 𝟖 + 𝒊 − − − (𝟏)
𝒁= 𝑹. 𝑯. 𝑺 = 𝒛̅ + 𝒘̅
𝟐𝟐 − (𝟒𝒊)𝟐
𝟖 − 𝟐𝟐𝒊 − 𝟏𝟐 𝑹. 𝑯. 𝑺 = 𝟐 + 𝟑𝒊 + ̅̅̅̅̅̅̅̅
̅̅̅̅̅̅̅̅ 𝟓 − 𝟒𝒊
𝒁= 𝑹. 𝑯. 𝑺 = 𝟐 − 𝟑𝒊 + 𝟓 + 𝟒𝒊
𝟒 − 𝟏𝟔𝒊𝟐
11 | P a g e
Class 9th Chapter 2 www.notes.pk.com
𝑹. 𝑯. 𝑺 = 𝟖 + 𝒊 − − − (𝟐) ̅̅̅̅̅̅̅̅̅̅̅̅̅̅̅
−𝟐 + 𝟐𝟑𝒊
𝑳. 𝑯. 𝑺 = ( )
From (1) and (2) , we have 𝟒𝟏
𝑳. 𝑯. 𝑺 = 𝑹. 𝑯. 𝑺 ̅̅̅̅̅̅̅̅̅̅̅̅̅̅̅̅̅̅
𝟐 𝟐𝟑
𝑳. 𝑯. 𝑺 = (− + 𝒊)
Hence Proved. 𝟒𝟏 𝟒𝟏
(ii). ̅̅̅̅̅̅̅̅
𝒛 − 𝒘 = 𝒛̅ − 𝒘 ̅ 𝟐 𝟐𝟑
𝑳. 𝑯. 𝑺 = − − 𝒊 − − − (𝟏)
Solution. 𝟒𝟏 𝟒𝟏
𝑳. 𝑯. 𝑺 = ̅̅̅̅̅̅̅̅
𝒛−𝒘 𝒛̅
𝑹. 𝑯. 𝑺 =
̅̅̅̅̅̅̅̅̅̅̅̅̅̅̅̅̅̅̅̅̅̅̅̅̅
𝑳. 𝑯. 𝑺 = (𝟐 + 𝟑𝒊) − (𝟓 − 𝟒𝒊) 𝒘
̅
(𝟐̅̅̅̅̅̅̅̅
+ 𝟑𝒊)
𝑳. 𝑯. 𝑺 = ̅̅̅̅̅̅̅̅̅̅̅̅̅̅̅̅̅̅̅̅
𝟐 + 𝟑𝒊 − 𝟓 + 𝟒𝒊 𝑹. 𝑯. 𝑺 =
𝑳. 𝑯. 𝑺 = ̅̅̅̅̅̅̅̅̅̅̅
−𝟑 + 𝟕𝒊 ̅̅̅̅̅̅̅̅
𝟓 − 𝟒𝒊
𝟐 − 𝟑𝒊
𝑳. 𝑯. 𝑺 = −𝟑 − 𝟕𝒊 − − − (𝟏) 𝑹. 𝑯. 𝑺 =
𝑹. 𝑯. 𝑺 = 𝒛̅ − 𝒘 ̅ 𝟓 + 𝟒𝒊
𝟐 − 𝟑𝒊 𝟓 − 𝟒𝒊
𝑹. 𝑯. 𝑺 = (𝟐 ̅̅̅̅̅̅̅̅
+ 𝟑𝒊) − (𝟓 ̅̅̅̅̅̅̅̅
− 𝟒𝒊) 𝑹. 𝑯. 𝑺 = ×
𝟓 + 𝟒𝒊 𝟓 − 𝟒𝒊
𝑹. 𝑯. 𝑺 = (𝟐 − 𝟑𝒊) − (𝟓 + 𝟒𝒊)
𝟏𝟎 − 𝟖𝒊 − 𝟏𝟓𝒊 + 𝟏𝟐𝒊𝟐
𝑹. 𝑯. 𝑺 = 𝟐 − 𝟑𝒊 − 𝟓 − 𝟒𝒊 𝑹. 𝑯. 𝑺 =
𝟓𝟐 − (𝟒𝒊)𝟐
𝑹. 𝑯. 𝑺 = −𝟑 − 𝟕𝒊 − − − (𝟐)
𝟏𝟎 − 𝟐𝟑𝒊 − 𝟏𝟐
From (1) and (2) , we have 𝑹. 𝑯. 𝑺 =
𝟐𝟓 − 𝟏𝟔𝒊𝟐
𝑳. 𝑯. 𝑺 = 𝑹. 𝑯. 𝑺 −𝟐 − 𝟐𝟑𝒊
Hence Proved. 𝑹. 𝑯. 𝑺 =
𝟐𝟓 + 𝟏𝟔
(iii). 𝒛𝒘
̅̅̅̅ = 𝒛̅ 𝒘 ̅ −𝟐 − 𝟐𝟑𝒊
Solution. 𝑹. 𝑯. 𝑺 =
𝟒𝟏
𝑳. 𝑯. 𝑺 = 𝒛𝒘 ̅̅̅̅ 𝟐 𝟐𝟑
̅̅̅̅̅̅̅̅̅̅̅̅̅̅̅̅̅̅̅̅̅̅ 𝑹. 𝑯. 𝑺 = − − 𝒊 − − − (𝟐)
𝑳. 𝑯. 𝑺 = (𝟐 + 𝟑𝒊)(𝟓 − 𝟒𝒊) 𝟒𝟏 𝟒𝟏
𝑳. 𝑯. 𝑺 = ̅̅̅̅̅̅̅̅̅̅̅̅̅̅̅̅̅̅̅̅̅̅̅̅̅̅̅̅
𝟏𝟎 − 𝟖𝒊 + 𝟏𝟓𝒊 − 𝟏𝟐𝒊𝟐 From (1) and (2) , we have
𝑳. 𝑯. 𝑺 = ̅̅̅̅̅̅̅̅̅̅̅̅̅̅̅̅̅
𝟏𝟎 + 𝟕𝒊 + 𝟏𝟐 𝑳. 𝑯. 𝑺 = 𝑹. 𝑯. 𝑺
𝑳. 𝑯. 𝑺 = ̅̅̅̅̅̅̅̅̅̅
𝟐𝟐 + 𝟕𝒊 Hence Proved.
𝟏
𝑳. 𝑯. 𝑺 = 𝟐𝟐 − 𝟕𝒊 − − − (𝟏) (v). (𝒛 + 𝒛̅) 𝒊𝒔 𝒂 𝒓𝒆𝒂𝒍 𝒑𝒂𝒓𝒕 𝒐𝒇 𝒛.
𝟐
𝑹. 𝑯. 𝑺 = 𝒛̅𝒘 ̅ Solution.
̅̅̅̅̅̅̅̅
𝑹. 𝑯. 𝑺 = (𝟐 + 𝟑𝒊)(𝟓 ̅̅̅̅̅̅̅̅
− 𝟒𝒊) 𝟏 𝟏
(𝟐 (𝒛 + 𝒛̅) = (𝟐 + 𝟑𝒊 + ̅̅̅̅̅̅̅̅
𝟐 + 𝟑𝒊)
𝑹. 𝑯. 𝑺 = − 𝟑𝒊)(𝟓 + 𝟒𝒊) 𝟐 𝟐
𝑹. 𝑯. 𝑺 = 𝟏𝟎 + 𝟖𝒊 − 𝟏𝟓𝒊 − 𝟏𝟐𝒊𝟐 𝟏
= (𝟐 + 𝟑𝒊 + 𝟐 − 𝟑𝒊)
𝑹. 𝑯. 𝑺 = 𝟏𝟎 − 𝟕𝒊 + 𝟏𝟐 𝟐
𝑹. 𝑯. 𝑺 = 𝟐𝟐 − 𝟕𝒊 − − − (𝟐) 𝟏
= (𝟒)
From (1) and (2) , we have 𝟐
𝑳. 𝑯. 𝑺 = 𝑹. 𝑯. 𝑺 = 𝟐 𝒘𝒉𝒊𝒄𝒉 𝒊𝒔 𝒓𝒆𝒂𝒍 𝒑𝒂𝒓𝒕 𝒐𝒇 𝒛.
Hence Proved. Hence Proved.
𝟏
̅̅̅̅̅
𝒛 𝒛̅ (vi). (𝒛 − 𝒛̅) 𝒊𝒔 𝒂 𝒊𝒎𝒂𝒈𝒊𝒏𝒂𝒓𝒚 𝒑𝒂𝒓𝒕 𝒐𝒇 𝒛.
(iv). (𝒘) = 𝒘̅ 𝟐𝒊
Solution.
Solution. 𝟏 𝟏
̅̅̅̅̅̅
𝒛 (𝒛 − 𝒛̅) = ̅̅̅̅̅̅̅̅
((𝟐 + 𝟑𝒊) + (𝟐 + 𝟑𝒊))
𝑳. 𝑯. 𝑺 = ( ) 𝟐𝒊 𝟐𝒊
𝒘 𝟏
̅̅̅̅̅̅̅̅̅̅̅
𝟐 + 𝟑𝒊 = ((𝟐 + 𝟑𝒊) − (𝟐 − 𝟑𝒊))
𝑳. 𝑯. 𝑺 = ( ) 𝟐𝒊
𝟓 − 𝟒𝒊 𝟏
̅̅̅̅̅̅̅̅̅̅̅̅̅̅̅̅̅̅̅̅̅̅̅
𝟐 + 𝟑𝒊 𝟓 + 𝟒𝒊 = (𝟐 + 𝟑𝒊 − 𝟐 + 𝟑𝒊)
𝑳. 𝑯. 𝑺 = ( × ) 𝟐𝒊
𝟓 − 𝟒𝒊 𝟓 + 𝟒𝒊 𝟏
̅̅̅̅̅̅̅̅̅̅̅̅̅̅̅̅̅̅̅̅̅̅̅̅̅̅̅̅̅̅̅̅ = (𝟔𝒊)
𝟏𝟎 + 𝟖𝒊 + 𝟏𝟓𝒊 + 𝟏𝟐𝒊𝟐 𝟐𝒊
𝑳. 𝑯. 𝑺 = ( ) = 𝟑 𝒘𝒉𝒊𝒄𝒉 𝒊𝒔 𝒊𝒎𝒂𝒈𝒊𝒏𝒂𝒓𝒚 𝒑𝒂𝒓𝒕 𝒐𝒇 𝒛.
𝟓𝟐 − (𝟒𝒊)𝟐
̅̅̅̅̅̅̅̅̅̅̅̅̅̅̅̅̅̅̅̅̅̅
𝟏𝟎 + 𝟐𝟑𝒊 − 𝟏𝟐 Hence Proved.
𝑳. 𝑯. 𝑺 = ( ) Question.7. Solve the following equations for
𝟐𝟓 − 𝟏𝟔𝒊𝟐
̅̅̅̅̅̅̅̅̅̅̅̅̅̅̅
−𝟐 + 𝟐𝟑𝒊 real 𝒙 𝒂𝒏𝒅 𝒚.
𝑳. 𝑯. 𝑺 = ( ) (i). (𝟐 − 𝟑𝒊)(𝒙 + 𝒊𝒚) = 𝟒 + 𝒊
𝟐𝟓 + 𝟏𝟔
12 | P a g e
Class 9th Chapter 2 www.notes.pk.com
Solution. Given that 𝒙 = −𝟏
(𝟐 − 𝟑𝒊)(𝒙 + 𝒊𝒚) = 𝟒 + 𝒊 Hence required 𝒙 = −𝟏 𝒂𝒏𝒅 𝒚 = 𝟎.
𝟐(𝒙 + 𝒊𝒚) − 𝟑𝒊(𝒙 + 𝒊𝒚) = 𝟒 + 𝒊 (iii). (𝟑 + 𝟒𝒊)𝟐 − 𝟐(𝒙 − 𝒊𝒚) = 𝒙 + 𝒚𝒊
𝟐𝒙 + 𝟐𝒊𝒚 − 𝟑𝒊𝒙 − 𝟑𝒊𝟐 𝒚 = 𝟒 + 𝒊 Solution. Given that
𝟐𝒙 + 𝟐𝒊𝒚 − 𝟑𝒊𝒙 + 𝟑𝒚 = 𝟒 + 𝒊 (𝟑 + 𝟒𝒊)𝟐 − 𝟐(𝒙 − 𝒊𝒚) = 𝒙 + 𝒚𝒊
𝟐𝒙 + 𝟑𝒚 + (𝟐𝒚 − 𝟑𝒙)𝒊 = 𝟒 + 𝒊 (𝟑) + (𝟒𝒊)𝟐 + 𝟐(𝟑)(𝟒𝒊) − 𝟐𝒙 + 𝟐𝒊𝒚 = 𝒙 + 𝒚𝒊
𝟐

Comparing real and imaginary parts, we have 𝟗 + 𝟏𝟔𝒊𝟐 + 𝟏𝟐𝒊 − 𝟐𝒙 + 𝟐𝒊𝒚 = 𝒙 + 𝒚𝒊


𝟐𝒙 + 𝟑𝒚 = 𝟒 − − − (𝒊) , 𝟐𝒚 − 𝟑𝒙 𝟗 − 𝟏𝟔 + 𝟏𝟐𝒊 − 𝟐𝒙 + 𝟐𝒊𝒚 = 𝒙 + 𝒚𝒊
= 𝟏 − − − (𝒊𝒊) −𝟕 − 𝟐𝒙 + (𝟏𝟐 + 𝟐𝒚)𝒊 = 𝒙 + 𝒚𝒊
𝟑 × (𝒊) + 𝟐 × (𝒊𝒊), 𝒘𝒆 𝒉𝒂𝒗𝒆 Comparing real and imaginary parts, we have
𝟑(𝟐𝒙 + 𝟑𝒚) + 𝟐(𝟐𝒚 − 𝟑𝒙) = 𝟑(𝟒) + 𝟐(𝟏) −𝟕 − 𝟐𝒙 = 𝒙 , 𝟏𝟐 + 𝟐𝒚 = 𝒚
𝟔𝒙 + 𝟗𝒚 + 𝟒𝒚 − 𝟔𝒙 = 𝟏𝟐 + 𝟐 𝒙 + 𝟐𝒙 = −𝟕 = , 𝟐𝒚 − 𝒚 = 𝟏𝟐
𝟏𝟑𝒚 = 𝟏𝟒 𝟑𝒙 = −𝟕 , 𝒚 = 𝟏𝟐
𝟏𝟒 𝟕
𝒚= 𝒙 = − , 𝒚 = 𝟏𝟐
𝟏𝟑 𝟑
Using value of 𝒚 𝒊𝒏 𝒆𝒒𝒖𝒂𝒕𝒊𝒐𝒏 (𝒊), 𝒘𝒆 𝒉𝒂𝒗𝒆 𝟕
Hence required 𝒙 = − 𝟑 𝒂𝒏𝒅 𝒚 = 𝟏𝟐.
𝟏𝟒
𝟐𝒙 + 𝟑 ( ) = 𝟒
𝟏𝟑
𝟒𝟐
𝟐𝒙 + =𝟒
𝟏𝟑
𝟒𝟐
𝟐𝒙 = 𝟒 −
𝟏𝟑
𝟒 × 𝟏𝟑 − 𝟒𝟐

Contact or Suggest Us: info@notespk.com


𝟐𝒙 =
𝟏𝟑
𝟓𝟐 − 𝟒𝟐

A project of: www.notespk.com


𝟐𝒙 =
𝟏𝟑
𝟏𝟎
𝟐𝒙 =
𝟏𝟑
𝟏𝟎
𝒙=
𝟏𝟑 × 𝟐
𝟓
𝒙=
𝟏𝟑
𝟓 𝟏𝟒
Hence required 𝒙 = 𝟏𝟑 𝒂𝒏𝒅 𝒚 = 𝟏𝟑.
(ii). (𝟑 − 𝟐𝒊)(𝒙 + 𝒊𝒚) = 𝟐(𝒙 − 𝟐𝒚𝒊) + 𝟐𝒊 − 𝟏
Solution. Given that
(𝟑 − 𝟐𝒊)(𝒙 + 𝒊𝒚) = 𝟐(𝒙 − 𝟐𝒚𝒊) + 𝟐𝒊 − 𝟏
𝟑(𝒙 + 𝒊𝒚) − 𝟐𝒊(𝒙 + 𝒊𝒚) = 𝟐𝒙 − 𝟒𝒚𝒊 + 𝟐𝒊 − 𝟏
𝟑𝒙 + 𝟑𝒊𝒚 − 𝟐𝒊𝒙 − 𝟐𝒊𝟐 𝒚 = 𝟐𝒙 − 𝟏 + 𝟐𝒊 − 𝟒𝒚𝒊
𝟑𝒙 + 𝟑𝒊𝒚 − 𝟐𝒊𝒙 + 𝟐𝒚 = 𝟐𝒙 − 𝟏 + (𝟐 − 𝟒𝒚)𝒊
𝟑𝒙 + 𝟐𝒚 + (𝟑𝒚 − 𝟐𝒙)𝒊 = 𝟐𝒙 − 𝟏 + (𝟐 − 𝟒𝒚)𝒊
Comparing real and imaginary parts, we have
𝟑𝒙 + 𝟐𝒚 = 𝟐𝒙 − 𝟏 , (𝟑𝒚 − 𝟐𝒙) = 𝟐 − 𝟒𝒚
𝟑𝒙 − 𝟐𝒙 + 𝟐𝒚 = −𝟏 , −𝟐𝒙 + 𝟑𝒚 + 𝟒𝒚 = 𝟐
𝒙 + 𝟐𝒚 = −𝟏 − − − (𝒊) , −𝟐𝒙 + 𝟕𝒚
= 𝟐 − − − (𝒊𝒊)
𝟐 × (𝒊) + (𝒊𝒊), 𝒘𝒆 𝒉𝒂𝒗𝒆
𝟐(𝒙 + 𝟐𝒚) + (−𝟐𝒙 + 𝟕𝒚) = 𝟐(−𝟏) + 𝟐
𝟐𝒙 + 𝟒𝒚 − 𝟐𝒙 + 𝟕𝒚 = −𝟐 + 𝟐
𝟏𝟏𝒚 = 𝟎
𝒚=𝟎
Using value of 𝒚 𝒊𝒏 𝒆𝒒𝒖𝒂𝒕𝒊𝒐𝒏 (𝒊), 𝒘𝒆 𝒉𝒂𝒗𝒆
𝒙 + 𝟐(𝟎) = −𝟏
13 | P a g e
MATHEMATICS 9th SCIENCE GROUP

7/18/2020
Chapter 3.
LOGARITHM

A project of: www.notespk.com


Contact or Suggest Us: info@notespk.com
Contents
Exercise 3.1 ..................................................................................... 1
Example 3.2 ..................................................................................... 4
Example3.3 ...................................................................................... 9
Example 3.4 ................................................................................... 11
Class 9th Chapter 3 www.notes.pk.com
Scientific Notation: Solution.
A number written in the form 𝑎 × 10𝑛 , 𝑤ℎ𝑒𝑟𝑒 1 ≤ 𝑎 ≤ 10 𝟎. 𝟎𝟎𝟕𝟒 = 𝟎. 𝟎𝟎𝟕 ∧ 𝟒
and n is an integer, is called the scientific notation.
𝟎. 𝟎𝟎𝟕𝟒 = 𝟕. 𝟒 × 𝟏𝟎−𝟑
Example:
Answer.
Write each of the following ordinary numbers in
scientific notation. (viii). 𝟔𝟎, 𝟎𝟎𝟎, 𝟎𝟎𝟎
Solution: Solution.
(i) 30600= 3.06 × 104 𝟔𝟎, 𝟎𝟎𝟎, 𝟎𝟎𝟎 = 𝟔 ∧ 𝟎, 𝟎𝟎𝟎, 𝟎𝟎𝟎.
(𝑚𝑜𝑣𝑒 𝑑𝑒𝑐𝑖𝑚𝑎𝑙 𝑝𝑜𝑖𝑛𝑡 𝑓𝑜𝑢𝑟 𝑝𝑙𝑎𝑐𝑒𝑠 𝑡𝑜 𝑡ℎ𝑒 𝑙𝑒𝑓𝑡) 𝟔𝟎, 𝟎𝟎𝟎, 𝟎𝟎𝟎 = 𝟔. 𝟎 × 𝟏𝟎𝟕
(ii) 0.000058 = 5.8 × 10−5 Answer.
(𝑚𝑜𝑣𝑒 𝑑𝑖𝑐𝑒𝑚𝑎𝑙 𝑝𝑜𝑖𝑛𝑡 𝑓𝑖𝑣𝑒 𝑝𝑙𝑎𝑐𝑒𝑠 𝑡𝑜 𝑡ℎ𝑒 𝑟𝑖𝑔ℎ𝑡) (ix). 𝟎. 𝟎0000000395
Example:
Solution.
Change each of the following numbers from scientific
notation to ordinary notation. 𝟎. 𝟎𝟎𝟎𝟎𝟎𝟎𝟎𝟎𝟑𝟗𝟓 = 𝟎. 𝟎𝟎𝟎𝟎𝟎𝟎𝟎𝟎𝟑 ∧ 𝟗𝟓
Solution: 𝟎. 𝟎𝟎𝟎𝟎𝟎𝟎𝟎𝟎𝟑𝟗𝟓 = 𝟑. 𝟗𝟓 × 𝟏𝟎−𝟗
(i) 6.35 × 106 = 6350000 Answer.
(𝑀𝑜𝑣𝑒 𝑡ℎ𝑒 𝑑𝑒𝑐𝑖𝑚𝑎𝑙 𝑝𝑜𝑖𝑛𝑡 𝑠𝑖𝑥 𝑝𝑙𝑎𝑐𝑒𝑠 𝑡𝑜 𝑡ℎ𝑒 𝑟𝑖𝑔ℎ𝑡) 𝟐𝟕𝟓,𝟎𝟎𝟎
(x).
(ii) 7.61 × 10−4 = 0.000761 𝟎.𝟎𝟎𝟐𝟓
(𝑀𝑜𝑣𝑒 𝑡ℎ𝑒 𝑑𝑒𝑐𝑖𝑚𝑎𝑙 𝑝𝑜𝑖𝑛𝑡 𝑓𝑜𝑢𝑟 𝑝𝑙𝑎𝑐𝑒𝑠 𝑡𝑜 𝑡ℎ𝑒 𝑙𝑒𝑓𝑡) Solution.
𝟐𝟕𝟓,𝟎𝟎𝟎 𝟐∧𝟕𝟓𝟎𝟎𝟎.
Exercise 3.1 =
𝟎.𝟎𝟎𝟐𝟓 𝟎.𝟎𝟎𝟐∧𝟓
Question.1. Express each of the following
numbers in scientific notation. 𝟐𝟕𝟓, 𝟎𝟎𝟎 𝟐. 𝟕𝟓 × 𝟏𝟎𝟓
=
(i). 𝟓𝟕𝟎𝟎 𝟎. 𝟎𝟎𝟐𝟓 𝟐. 𝟓 × 𝟏𝟎−𝟑
𝟐. 𝟕𝟓
Solution. = × 𝟏𝟎𝟓 × 𝟏𝟎𝟑
𝟓𝟕𝟎𝟎 = 𝟓 ∧ 𝟕𝟎𝟎. 𝟐. 𝟓
𝟓𝟕𝟎𝟎 = 𝟓. 𝟕 × 𝟏𝟎𝟑 = 𝟏. 𝟏 × 𝟏𝟎𝟓+𝟑
Answer. = 𝟏. 𝟏 × 𝟏𝟎𝟖
(ii). 𝟒𝟗, 𝟖𝟎𝟎, 𝟎𝟎𝟎 Answer.
Solution. Question.2. Express the following numbers in
𝟒𝟗, 𝟖𝟎𝟎, 𝟎𝟎𝟎 = 𝟒 ∧ 𝟗, 𝟖𝟎𝟎, 𝟎𝟎𝟎. ordinary notation.
𝟒𝟗, 𝟖𝟎𝟎, 𝟎𝟎𝟎 = 𝟒. 𝟗𝟖 × 𝟏𝟎𝟕 (i). 𝟔 × 𝟏𝟎−𝟒
Answer. Solution.
(iii). 𝟗𝟔, 𝟎𝟎𝟎, 𝟎𝟎𝟎 𝟔
𝟔 × 𝟏𝟎−𝟒 = 𝟒
Solution. 𝟏𝟎
𝟔
𝟗𝟔, 𝟎𝟎𝟎, 𝟎𝟎𝟎 = 𝟗 ∧ 𝟔, 𝟎𝟎𝟎, 𝟎𝟎𝟎. =
𝟗𝟔, 𝟎𝟎𝟎, 𝟎𝟎𝟎 = 𝟗. 𝟔 × 𝟏𝟎𝟕 𝟏𝟎𝟎𝟎𝟎
= 𝟎. 𝟎𝟎𝟎𝟔
Answer.
Answer.
(iv). 𝟒𝟏𝟔. 𝟗
(ii). 𝟓. 𝟎𝟔 × 𝟏𝟎𝟏𝟎
Solution.
Solution.
𝟒𝟏𝟔. 𝟗 = 𝟒 ∧ 𝟏𝟔. 𝟗
𝟓. 𝟎𝟔 × 𝟏𝟎𝟏𝟎 = 𝟓. 𝟎𝟔 × 𝟏𝟎, 𝟎𝟎𝟎, 𝟎𝟎𝟎, 𝟎𝟎𝟎
𝟒𝟏𝟔. 𝟗 = 𝟒. 𝟏𝟔𝟗 × 𝟏𝟎𝟐
= 𝟓𝟎, 𝟔𝟎𝟎, 𝟎𝟎𝟎, 𝟎𝟎𝟎.
Answer.
Answer.
(v). 𝟖𝟑, 𝟎𝟎𝟎
(iii). 𝟗. 𝟎𝟏𝟖 × 𝟏𝟎−𝟔
Solution.
Solution.
𝟖𝟑, 𝟎𝟎𝟎 = 𝟖 ∧ 𝟑, 𝟎𝟎𝟎.
𝟗. 𝟎𝟏𝟖
𝟖𝟑, 𝟎𝟎𝟎 = 𝟖. 𝟑 × 𝟏𝟎𝟒 𝟗. 𝟎𝟏𝟖 × 𝟏𝟎−𝟔 =
𝟏𝟎𝟔
Answer. 𝟗. 𝟎𝟏𝟖
(vi). 𝟎. 𝟎𝟎𝟔𝟒𝟑 =
𝟏, 𝟎𝟎𝟎, 𝟎𝟎𝟎
Solution. = 𝟎. 𝟎𝟎𝟎𝟎𝟎𝟗𝟎𝟏𝟖
𝟎. 𝟎𝟎𝟔𝟒𝟑 = 𝟎. 𝟎𝟎𝟔 ∧ 𝟒𝟑 Answer.
𝟎. 𝟎𝟎𝟔𝟒𝟑 = 𝟔. 𝟒𝟑 × 𝟏𝟎−𝟑 (iv). 𝟕. 𝟖𝟔𝟓 × 𝟏𝟎𝟖
Answer. Solution.
(vii). 𝟎. 𝟎𝟎𝟕𝟒
1|Page
Class 9th Chapter 3 www.notes.pk.com
𝟕. 𝟖𝟔𝟓 × 𝟏𝟎𝟖 = 𝟕. 𝟖𝟔𝟓 × 𝟏𝟎𝟎, 𝟎𝟎𝟎, 𝟎𝟎𝟎 number 3692. The number at the intersection of this
= 𝟕𝟖𝟔, 𝟓𝟎𝟎, 𝟎𝟎𝟎. row and the mean difference column corresponding to
Answer. 7 is 13. Hence the sum of 3692 and 13 gives the
Logarithm of real numbers: mantissa of the logarithm of 0.0023476 as o.3705
𝑖𝑓 𝑎 𝑥 = 𝑦 𝑡ℎ𝑒𝑛 𝑥 𝑖𝑠 𝑐𝑎𝑙𝑙𝑒𝑑 𝑡ℎ𝑒 𝑙𝑜𝑔𝑎𝑟𝑖𝑡ℎ𝑚 𝑜𝑓 𝑦 𝑡𝑜 Example:
𝑡ℎ𝑒 𝑏𝑎𝑠𝑒 "a" 𝑎𝑛𝑑 𝑖𝑡𝑠 𝑤𝑟𝑖𝑡𝑡𝑒𝑛 𝑎𝑠 log 𝑎 𝑦 = 𝑥 𝑤ℎ𝑒𝑟𝑒 1. Find log 278.23
𝑎 > 0, 𝑎 ≠ 1 𝑎𝑛𝑑 𝑦 > 0 2. Log0.07058
𝑖. 𝑒 𝑡ℎ𝑒 𝑙𝑜𝑔𝑎𝑟𝑖𝑡ℎ𝑚 𝑜𝑓 𝑎 𝑛𝑢𝑚𝑏𝑒𝑟 𝑦 𝑡𝑜 𝑡ℎ𝑒 𝑏𝑎𝑠𝑒 "a" Solution:
𝑖𝑠 𝑡ℎ𝑒 𝑖𝑛𝑑𝑒𝑥 𝑥 𝑜𝑓 𝑡ℎ𝑒 𝑝𝑜𝑤𝑒𝑟 𝑡𝑜 𝑤ℎ𝑖𝑐ℎ 𝑎 𝑚𝑢𝑠𝑡 𝑏𝑒 1. 278.22 𝑐𝑎𝑛 𝑏𝑒 𝑟𝑜𝑢𝑛𝑑𝑒𝑑 𝑜𝑓𝑓 𝑎𝑠 278.22
𝑟𝑎𝑖𝑠𝑒𝑑 𝑡𝑜 𝑔𝑒𝑡 𝑡ℎ𝑎𝑡 𝑛𝑢𝑚𝑏𝑒𝑟 𝑦. The characteristic is 2 and the mantissa, using log
𝑡ℎ𝑒 𝑟𝑒𝑙𝑎𝑡𝑖𝑜𝑛 𝑎 𝑥 = 𝑦 𝑎𝑛𝑑 log 𝑎 𝑦 = 𝑥 𝑎𝑟𝑒 𝑒𝑞𝑢𝑖𝑣𝑎𝑙𝑒𝑛 tables, is .4443
When one relation is given, it can be converted into ∵ 𝑙𝑜𝑔278.23 = 2.4443
the other. Thus 2. The characteristic of log𝑜. 07058 𝑖𝑠 − 2
𝑎 𝑥 = 𝑦 ⟺ log 𝑎 𝑦 = 𝑥 which is written as 2̅ by convention.
Example: find 𝒍𝒐𝒈𝟒 𝟐 𝒊. 𝒆. 𝒇𝒊𝒏𝒅 𝒍𝒐𝒈 𝒐𝒇 𝟐 𝒕𝒐 𝒕𝒉𝒆 Using log tables the mantissa is .8487, so that
Base 4. 𝑙𝑜𝑔0.07058 = 2̅. 8487
Solution: Example:
Let log 4 2 = 𝑥 Find the numbers whose logarithms are
𝑡ℎ𝑒𝑛 𝑖𝑡𝑠 𝑒𝑥𝑝𝑜𝑛𝑒𝑛𝑒𝑛𝑖𝑎𝑙 𝑓𝑜𝑟𝑚 𝑖𝑠 4𝑥 = 2 (i) 1.3247
𝑖. 𝑒 22𝑥 = 21 ⇒ 2𝑥 = 1 ̅. 𝟏𝟑𝟐𝟒
(ii) 𝟐
1 1 Solution:
∵ 𝑥 = ⇒ 𝑙𝑜𝑔4 2 = (i) 1.3247
2 2
antilog1.3247 = 21.12
Deductions from Definition of logarithm
1. Since 𝑎0 = 1, 𝑙𝑜𝑔𝑎 1 = 0 (ii) 2̅. 1324
2. 𝑆𝑖𝑛𝑐𝑒 𝑎1 = 𝑎 , 𝑙𝑜𝑔𝑎 𝑎 = 1 𝑎𝑛𝑡𝑖𝑙𝑜𝑔(2̅. 1324) 𝑖𝑠 0.01356
Common logarithm:
If the base of logarithm is taken as 10 then logarithm is
called Common Logarithm.
Example 3.2
Characteristic: Question.1. Find the common logarithm of the
The integral part of the logarithm of any number is following numbers:
called the characteristic. (i). 𝟐𝟑𝟐. 𝟗𝟐
Mantissa: the fractional part of the logarithm of a Solution.
number is called the mantissa. Mantissa is always 𝑪𝒉𝒂𝒓𝒂𝒄𝒕𝒆𝒓𝒊𝒔𝒕𝒊𝒄𝒔 = 𝟐
positive.
𝑴𝒂𝒏𝒕𝒊𝒔𝒂 = 𝟎. 𝟑𝟔𝟕𝟐
Example: find the mantissa of the logarithm of
𝟒𝟑. 𝟐𝟓𝟒 𝑳𝒐𝒈(𝟐𝟑𝟐. 𝟗𝟐) = 𝟐. 𝟑𝟔𝟕𝟐
Solution: Answer.
Rounding off 43.254 𝑤𝑒 𝑐𝑜𝑛𝑠𝑖𝑑𝑒𝑟 𝑜𝑛𝑙𝑦 𝑡ℎ𝑒 𝑓𝑜𝑢𝑟 (ii). 𝟐𝟗. 𝟑𝟐𝟔
Significant digits4325. Solution.
(i) We first locate the row corresponding to 43 in 𝑪𝒉𝒂𝒓𝒂𝒄𝒕𝒆𝒓𝒊𝒔𝒕𝒊𝒄𝒔 = 𝟏
the log tables and 𝑴𝒂𝒏𝒕𝒊𝒔𝒂 = 𝟎. 𝟒𝟔𝟕𝟐
(ii) Proceed horizontally till we reach the column 𝑳𝒐𝒈(𝟐𝟗. 𝟑𝟐𝟔) = 𝟏. 𝟒𝟔𝟕𝟐
corresponding to 2. The number at the
Answer.
intersection is 6355.
(iii) Again proceeding horizontally till the mean (iii). 𝟎. 𝟎𝟎𝟎𝟑𝟐
difference column corresponding to 5 Solution.
intersects this row. We get the number 5 at 𝑪𝒉𝒂𝒓𝒂𝒄𝒕𝒆𝒓𝒊𝒔𝒕𝒊𝒄𝒔 = −𝟒
the intersection. 𝑴𝒂𝒏𝒕𝒊𝒔𝒂 = 𝟎. 𝟓𝟎𝟓𝟏
(iv) Adding the two numbers 6355 and 5 we get 𝑳𝒐𝒈(𝟎. 𝟎𝟎𝟎𝟑𝟐) = 𝟒̅. 𝟓𝟎𝟓𝟏
.6360 as the mantissa of the logarithm of 43.25 Answer.
Example:
(iv). 𝟎. 𝟑𝟐𝟎𝟔
Find the mantissa of the logarithm of 𝟎. 𝟎𝟎𝟐𝟑𝟒𝟕
Solution.
Solution:
Here also, we consider only the four Signiant digits 𝑪𝒉𝒂𝒓𝒂𝒄𝒕𝒆𝒓𝒊𝒔𝒕𝒊𝒄𝒔 = −𝟏
2347 𝑴𝒂𝒏𝒕𝒊𝒔𝒂 = 𝟎. 𝟓𝟎𝟓𝟗
We first locate the row corresponding to 23 in the ̅. 𝟓𝟎𝟓𝟗
𝑳𝒐𝒈(𝟎. 𝟑𝟐𝟎𝟔) = 𝟏
logarithm tables and proceeding to 4 the resulting Answer.
pg. 4
Class 9th Chapter 3 www.notes.pk.com
Question.2. If 𝒍𝒐𝒈𝟑𝟏. 𝟎𝟗 = 𝟏. 𝟒𝟗𝟐𝟔 , find the (i). 𝒍𝒐𝒈𝟑 𝟖𝟏 = 𝑳
values of the following Solution.
(i). 𝒍𝒐𝒈𝟑. 𝟏𝟎𝟗 𝒍𝒐𝒈𝟑 𝟖𝟏 = 𝑳
Solution. Exponential Form
𝑪𝒉𝒂𝒓𝒂𝒄𝒕𝒆𝒓𝒊𝒔𝒕𝒊𝒄𝒔 = 𝟎 𝟑𝑳 = 𝟖𝟏
𝑴𝒂𝒏𝒕𝒊𝒔𝒂 = 𝟎. 𝟒𝟗𝟐𝟔 𝟑𝑳 = 𝟑𝟒
𝑳𝒐𝒈(𝟑. 𝟏𝟎𝟗) = 𝟎. 𝟒𝟗𝟐𝟔 => 𝑳 = 𝟒.
Answer. (ii). 𝒍𝒐𝒈𝒂 𝟔 = 𝟎. 𝟓
(ii). 𝒍𝒐𝒈𝟑𝟏𝟎. 𝟗 Solution.
Solution. 𝒍𝒐𝒈𝒂 𝟔 = 𝟎. 𝟓
𝑪𝒉𝒂𝒓𝒂𝒄𝒕𝒆𝒓𝒊𝒔𝒕𝒊𝒄𝒔 = 𝟐 Exponential Form
𝑴𝒂𝒏𝒕𝒊𝒔𝒂 = 𝟎. 𝟒𝟗𝟐𝟔 𝒂𝟎.𝟓 = 𝟔
𝑳𝒐𝒈(𝟑𝟏𝟎. 𝟗) = 𝟐. 𝟒𝟗𝟐𝟔 𝟏
𝒂𝟐 = 𝟔
Answer.
Squaring on both sides , we have
(iii). 𝒍𝒐𝒈𝟎. 𝟎𝟎𝟑𝟏𝟎𝟗 𝟏 𝟐
Solution. (𝒂𝟐 ) = 𝟔𝟐
𝑪𝒉𝒂𝒓𝒂𝒄𝒕𝒆𝒓𝒊𝒔𝒕𝒊𝒄𝒔 = −𝟑
𝒂 = 𝟑𝟔.
𝑴𝒂𝒏𝒕𝒊𝒔𝒂 = 𝟎. 𝟒𝟗𝟐𝟔
̅. 𝟒𝟗𝟐𝟔 (iii). 𝒍𝒐𝒈𝟓 𝒏 = 𝟐
𝑳𝒐𝒈(𝟎. 𝟎𝟎𝟑𝟏𝟎𝟗) = 𝟑
Solution.
Answer.
𝒍𝒐𝒈𝟓 𝒏 = 𝟐
(iv). 𝒍𝒐𝒈𝟎. 𝟑𝟏𝟎𝟗
Exponential Form
Solution.
𝟓𝟐 = 𝒏
𝑪𝒉𝒂𝒓𝒂𝒄𝒕𝒆𝒓𝒊𝒔𝒕𝒊𝒄𝒔 = −𝟏
𝟐𝟓 = 𝒏
𝑴𝒂𝒏𝒕𝒊𝒔𝒂 = 𝟎. 𝟒𝟗𝟐𝟔
̅. 𝟒𝟗𝟐𝟔 𝒏 = 𝟐𝟓.
𝑳𝒐𝒈(𝟎. 𝟑𝟏𝟎𝟗) = 𝟏
(iv). 𝟏𝟎𝒑 = 𝟒𝟎
Answer.
Solution.
Question.3. Find the number whose common
𝟏𝟎𝒑 = 𝟒𝟎
logarithms are:
Logarithm Form
(i). 𝟑. 𝟓𝟔𝟐𝟏 𝒍𝒐𝒈𝟏𝟎 𝟒𝟎 = 𝒑
Solution. 𝒑 = 𝒍𝒐𝒈𝟏𝟎 𝟒𝟎
Since it is log of any number. So,
𝒑 = 𝟏. 𝟔𝟎𝟐𝟏
𝑪𝒉𝒂𝒓𝒂𝒄𝒕𝒆𝒓𝒊𝒔𝒕𝒊𝒄𝒔 = 𝟑 Question.5. Evaluate
𝑴𝒂𝒏𝒕𝒊𝒔𝒂 = 𝟎. 𝟓𝟔𝟐𝟏 𝟏
𝑴𝒂𝒏𝒕𝒊𝒔𝒂 𝒊𝒏 𝒂𝒏𝒕𝒊𝒍𝒐𝒈 = 𝟑. 𝟔𝟒𝟖𝟒 (i). 𝒍𝒐𝒈𝟐 𝟏𝟐𝟖
Characteristics change the place of decimal. Solution.
So Let
𝑨𝒏𝒕𝒊 − 𝒍𝒐𝒈(𝟑. 𝟓𝟔𝟐𝟏) = 𝟑𝟔𝟒𝟖. 𝟒 𝟏
𝒙 = 𝒍𝒐𝒈𝟐
Answer. 𝟏𝟐𝟖
(ii). 𝟏̅. 𝟕𝟒𝟐𝟕 Exponential Form
𝟏
Solution. 𝟐𝒙 =
Since it is log of any number. So, 𝟏𝟐𝟖
𝟏
𝑪𝒉𝒂𝒓𝒂𝒄𝒕𝒆𝒓𝒊𝒔𝒕𝒊𝒄𝒔 = −𝟏 𝟐𝒙 =
𝑴𝒂𝒏𝒕𝒊𝒔𝒂 = 𝟎. 𝟕𝟒𝟐𝟕 𝟐×𝟐×𝟐×𝟐×𝟐×𝟐×𝟐
𝑴𝒂𝒏𝒕𝒊𝒔𝒂 𝒊𝒏 𝒂𝒏𝒕𝒊𝒍𝒐𝒈 = 𝟓. 𝟓𝟐𝟗𝟕 𝟐𝒙 = 𝟐−𝟕
Characteristics change the place of decimal.
So =≫ 𝒙 = −𝟕
𝑨𝒏𝒕𝒊 − 𝒍𝒐𝒈(𝟏̅. 𝟕𝟒𝟐𝟕) = 𝟎. 𝟓𝟓𝟑𝟎
Answer. Answer.
Question.4. what replacement for the unknown in
each of the following will make the statement (ii). 𝒍𝒐𝒈 𝟓𝟏𝟐 𝒕𝒐 𝒕𝒉𝒆 𝒃𝒂𝒔𝒆 𝟐√𝟐.
true? Solution.
pg. 5
Class 9th Chapter 3 www.notes.pk.com
𝒙
Let (iii). 𝒍𝒐𝒈𝟔𝟒 𝟖 = 𝟐
𝒙 = 𝒍𝒐𝒈𝟐√𝟐 𝟓𝟏𝟐 Solution.
Exponential Form 𝒙
𝒙 𝒍𝒐𝒈𝟔𝟒 𝟖 =
(𝟐√𝟐) = 𝟓𝟏𝟐 𝟐
𝟏 𝒙 Exponential Form
𝒙
(𝟐 × 𝟐𝟐 ) = 𝟐×𝟐×𝟐×𝟐×𝟐×𝟐×𝟐×𝟐×𝟐 (𝟔𝟒)𝟐 = 𝟖
𝟏 𝒙 𝒙
(𝟐𝟏+𝟐 ) = 𝟐𝟗 (𝟖 × 𝟖)𝟐 = 𝟖
𝟐+𝟏 𝒙 𝒙
(𝟐 𝟐 ) = 𝟐𝟗 (𝟖𝟐 )𝟐 = 𝟖
𝟑 𝒙
(𝟐𝟐 ) = 𝟐𝟗 𝟖𝒙 = 𝟖𝟏
𝟑𝒙
𝟐 𝟐 = 𝟐𝟗 ==> 𝒙 = 𝟏
𝟑𝒙
==> =𝟗
𝟐 Answer.

𝟑𝒙 = 𝟏𝟖 (iv). 𝒍𝒐𝒈𝒙 𝟔𝟒 = 𝟐
Solution.
𝟏𝟖 𝒍𝒐𝒈𝒙 𝟔𝟒 = 𝟐
𝒙=
𝟑 Exponential Form
(𝒙)𝟐 = 𝟔𝟒
𝒙=𝟔
Taking square root on both sides
Answer. √𝒙𝟐 = √𝟔𝟒
𝒙=𝟖
Question.6. Evaluate the value of ′′𝒙′′ from the Answer.
following statements.
(i). 𝒍𝒐𝒈𝟐 𝒙 = 𝟓 (v). 𝒍𝒐𝒈𝟑 𝒙 = 𝟒
Solution.

Contact or Suggest Us: info@notespk.com


A project of: www.notespk.com
Solution.
𝒍𝒐𝒈𝟐 𝒙 = 𝟓 𝒍𝒐𝒈𝟑 𝒙 = 𝟒
Exponential Form Exponential Form
𝟐𝟓 = 𝒙 𝟑𝟒 = 𝒙
𝒙 = 𝟐𝟓 𝒙= 𝟑×𝟑×𝟑×𝟑
𝒙 = 𝟖𝟏
𝒙= 𝟐×𝟐×𝟐×𝟐×𝟐 Answer.

𝒙 = 𝟑𝟐Answer.

(ii). 𝒍𝒐𝒈𝟖𝟏 𝟗 = 𝒙
Solution.
𝒍𝒐𝒈𝟖𝟏 𝟗 = 𝒙
Exponential Form
𝟖𝟏𝒙 = 𝟗
(𝟗 × 𝟗)𝒙 = 𝟗

𝟗𝟐𝒙 = 𝟗𝟏

==> 𝟐𝒙 = 𝟏

𝟏
𝒙=
𝟐

Answer.
pg. 6
Class 9th Chapter 3 www.notes.pk.com
Note:
Laws of Logarithm (i) 𝑙𝑜𝑔𝑎 (𝑚𝑛) ≠ 𝑙𝑜𝑔𝑎 𝑚 × 𝑙𝑜𝑔𝑎 𝑛
(i) log 𝑎 (𝑚𝑛) = log 𝑎 𝑚 + log 𝑎 𝑛 (ii) 𝑙𝑜𝑔𝑎 𝑚 + 𝑙𝑜𝑔𝑎 𝑛 ≠ 𝑙𝑜𝑔𝑎 (𝑚 + 𝑛)
𝑚
(ii) log 𝑎 ( ) = log 𝑎 𝑚 − log 𝑎 𝑛 (iii) 𝑙𝑜𝑔𝑎 (𝑚𝑛𝑝) = 𝑙𝑜𝑔𝑎 𝑚 + 𝑙𝑜𝑔𝑎 𝑛 + 𝑙𝑜𝑔𝑛 𝑝+. ..
𝑛 Example:
(iii) log 𝑎 𝑚𝑛 = 𝑛𝑙𝑜𝑔𝑎 𝑚 291.3
(iv) log 𝑎 𝑛 = log 𝑏 𝑛 × log 𝑎 𝑏 𝐸𝑣𝑎𝑙𝑢𝑎𝑡𝑒
log 𝑏 𝑛 42.36
𝑜𝑟 = 291.3 291.3
log 𝑏 𝑎 𝑙𝑒𝑡 𝑥 = 𝑠𝑜 𝑡ℎ𝑎𝑡 𝑙𝑜𝑔𝑥 = log
42.36 42.36
(i)
𝐥𝐨𝐠 𝒂 (𝒎𝒏) = 𝐥𝐨𝐠 𝒂 𝒎 + 𝐥𝐨𝐠 𝒂 𝒏 Then log 𝑥 = 𝑙𝑜𝑔291.33 − 𝑙𝑜𝑔42.36, …
Proof: 𝑚
(log 𝑎 = log 𝑎 𝑚 − log 𝑎 𝑛)
𝐿𝑒𝑡 log 𝑎 𝑚 = 𝑥 𝑎𝑛𝑑 log 𝑎 𝑛 = 𝑦 𝑛
Writing in exponential form log 𝑥 = 2.4643 − 1.6269 = 0,8374
𝑎 𝑥 = 𝑚 𝑎𝑛𝑑 𝑎 𝑦 = 𝑛 Thus 𝑥 = 𝑎𝑛𝑡𝑖𝑙𝑜𝑔𝑜. 8374 = 6.877
∵ 𝑎 𝑥 × 𝑎 𝑦 = 𝑚𝑛 Example:
𝑖. 𝑒 𝑎 𝑥+𝑦 = 𝑚𝑛 0.0002913
𝐸𝑣𝑎𝑙𝑢𝑎𝑡𝑒
𝑜𝑟 log 𝑎 (𝑚𝑛) = 𝑥 + 𝑦 = log 𝑎 𝑚 + log 𝑎 𝑛 0.04236
ℎ𝑒𝑛𝑐𝑒 log 𝑎 (𝑚𝑛) = log 𝑎 𝑚 + log 𝑎 𝑛 Solution:
𝑡ℎ𝑒 𝑟𝑢𝑙𝑒 𝑔𝑖𝑣𝑒𝑛 𝑎𝑏𝑜𝑣𝑒 𝑖𝑠 𝑢𝑠𝑒𝑓𝑢𝑙 𝑖𝑛 𝑓𝑖𝑛𝑑𝑖𝑛𝑔𝑡ℎ𝑒 𝟎.𝟎𝟎𝟎𝟐𝟗𝟏𝟑
𝒍𝒆𝒕 𝒚 = 𝟎.𝟎𝟒𝟐𝟑𝟔
𝒔𝒐 𝒕𝒉𝒂𝒕
Product of two or more numbers using logarithms 𝟎. 𝟎𝟎𝟐𝟗𝟏𝟑
Example: 𝒍𝒐𝒈𝒚 = 𝒍𝒐𝒈 ( )
𝟎. 𝟎𝟒𝟐𝟑𝟔
Evaluate 𝟐𝟗𝟏. 𝟑 × 𝟒𝟐. 𝟑𝟔 Then 𝒍𝒐𝒈𝒚 = 𝒍𝒐𝒈 𝒐. 𝒐𝒐𝟐𝟗𝟏𝟑 − 𝒍𝒐𝒈𝟎. 𝟎𝟒𝟐𝟑𝟔
Solution: 𝒍𝒐𝒈𝒚 = 𝟑 ̅. 𝟒𝟔𝟒𝟑 − 𝟐 ̅. 𝟔𝟐𝟔𝟗
𝑙𝑒𝑡 𝑥 = 291.3 × 42.36 ̅
= 𝟑 + (𝟎. 𝟒𝟔𝟒𝟑 − 𝟎. 𝟔𝟐𝟔𝟗) − 𝟐 ̅
Then𝑙𝑜𝑔𝑥 = 𝑙𝑜𝑔(291.3 × 42.36) =𝟑̅ − 𝟎. 𝟏𝟔𝟐𝟔 − 𝟐 ̅
= 𝑙𝑜𝑔291.3 + 𝑙𝑜𝑔42.36 =𝟑 ̅ + (𝟏 − 𝟎. 𝟏𝟔𝟐𝟔) − 𝟏 − 𝟐 ̅
(𝑙𝑜𝑔𝑎 𝑚𝑛 = 𝑙𝑜𝑔𝑎 𝑚 + 𝑙𝑜𝑔𝑎 𝑛) (adding and subtraction 1 )
= 2.4643 + 1.6269 = 4.0912 =𝟐 ̅. 𝟖𝟑𝟕𝟒
𝑥 = 𝑎𝑛𝑡𝑖𝑙𝑜𝑔4.0912 = 12340 [∵ 𝟑 ̅−𝟏−𝟐 ̅ = −𝟑 − 𝟏 − (−𝟐) = −𝟐 = 𝟐 ̅]
Example: ̅
Therefore , 𝒚 = 𝒂𝒏𝒕𝒊𝒍𝒐𝒈𝟐. 𝟖𝟑𝟕𝟒
𝐸𝑣𝑎𝑙𝑢𝑎𝑡𝑒 0.2913 × 0.004236
𝒚 = 𝟎. 𝟎𝟔𝟖𝟕𝟕
Solution:
(iii) 𝒍𝒐𝒂 (𝒎𝒏 ) = 𝒏𝒍𝒐𝒈𝒂 𝒎
Let 𝑦 = 0.2913 × 0.004236
Proof:
then𝑙𝑜𝑔𝑦 = log 0.2913 + 𝑙𝑜𝑔0.004236
𝒍𝒆𝒕 𝒍𝒐𝒈𝒂 𝒎𝒏 = 𝒙, 𝒊. 𝒆 𝒂𝒙 = 𝒎𝒏
𝑙𝑜𝑔𝑦 = 1̅. 4643 + 3̅. 6269 And𝒍𝒐𝒈𝒂 𝒎 = 𝒚, 𝒊. 𝒆 𝒂𝒚 = 𝒎
𝑙𝑜𝑔𝑦 = 3̅. 0912
𝒕𝒉𝒆𝒏 𝒂𝒙 = 𝒎𝒏 = (𝒂𝒚 )𝒏
𝑦 = 𝑎𝑛𝑡𝑖𝑙𝑜𝑔3̅. 0912 𝒊. 𝒆 𝒂𝒙 = (𝒂𝒚 )𝒏 = 𝒂𝒚𝒏 ⇒ 𝒙 = 𝒏𝒚
𝑦 = 0.001234 𝒊. 𝒆 𝒍𝒐𝒈𝒂 𝒎𝒏 = 𝒏𝒍𝒐𝒈𝒂 𝒎
𝑚
(𝒊𝒊) log 𝑎 ( ) = log 𝑎 𝑚. log 𝑎 𝑛 Example:
𝑛
𝟑
𝑺𝒐𝒍𝒖𝒕𝒊𝒐𝒏: 𝑬𝒗𝒂𝒍𝒖𝒂𝒕𝒆 𝟒√(𝟎. 𝟎𝟏𝟔𝟑)𝟑 = (𝟎. 𝟎𝟏𝟔𝟑)𝟒
Let 𝑙𝑜𝑔𝑎 𝑚 = 𝑥 𝑎𝑛𝑑 𝑙𝑜𝑔𝑎 𝑛 = 𝑦 Solution:
So that 𝑎 𝑥 = 𝑚 𝑎𝑛𝑑 𝑎 𝑦 = 𝑛 𝟑
𝑎𝑥 𝑚 𝑚 𝒍𝒆𝒕 𝒚 = 𝟒√(𝟎. 𝟎𝟏𝟔𝟑)𝟑 = (𝟎. 𝟎𝟏𝟔𝟑)𝟒
∵ 𝑦 = ⇒ 𝑎 𝑥−𝑦 = 𝟑
𝑎 𝑛 𝑛 𝒍𝒐𝒕𝒚 = (𝒍𝒐𝒈𝟎. 𝟎𝟏𝟔𝟑)
𝑚 𝟒
𝑖. 𝑒 𝑙𝑜𝑔𝑎 ( ) = 𝑥 − 𝑦 = 𝑙𝑜𝑔𝑎 𝑚 − 𝑙𝑜𝑔𝑎 𝑛 𝟑
𝑛 = ×𝟐 ̅. 𝟐𝟏𝟐𝟐
𝑚
Hence 𝑙𝑜𝑔𝑎 ( 𝑛 ) = 𝑙𝑜𝑔𝑎 𝑚 − 𝑙𝑜𝑔𝑎 𝑛 𝟒
̅. 𝟔𝟑𝟔𝟔
𝟔
Note: =
𝑚 log𝑎 𝑚 𝟒
(i) log 𝑎 ( 𝑛 ) ≠ log𝑎 𝑛
̅ + 𝟐. 𝟔𝟑𝟔𝟔
𝟖
=
(ii) log 𝑎 𝑚 − log 𝑎 𝑛 ≠ log 𝑎 (𝑚 − 𝑛) 𝟒
1 =𝟐̅ + 𝟎. 𝟔𝟓𝟗𝟐 = 𝟐̅. 𝟔𝟓𝟗𝟐
(iii) log 𝑎 (𝑛) = log 𝑎 𝑙 − log 𝑎 𝑛 = − log 𝑎 𝑛 …
̅
Hence𝒚 = 𝒂𝒏𝒕𝒊𝒍𝒐𝒈𝟐. 𝟔𝟓𝟗𝟐
∵ log 𝑎 1 = 0
= 𝟎. 𝟎𝟒𝟓𝟔𝟐
(𝒊𝒗)𝑪𝒉𝒂𝒏𝒈𝒆 𝒐𝒇 𝒃𝒂𝒔𝒆 𝒇𝒐𝒓𝒎𝒖𝒍𝒂:
𝒍𝒐𝒈𝒃 𝒏
𝒍𝒐𝒈𝒂 𝒏 = 𝒍𝒐𝒈𝒃 𝒏 × 𝒍𝒐𝒈𝒂 𝒃 𝒐𝒓
𝒍𝒐𝒈𝒃 𝒂
Proof:
pg. 7
Class 9th Chapter 3 www.notes.pk.com
𝑙𝑒𝑡 𝑙𝑜𝑔𝑏 𝑛 = 𝑥 𝑠𝑜 𝑡ℎ𝑎𝑡 𝑛 = 𝑏 𝑥 1
(22)3
Taking log to the base 𝑎, 𝑤𝑒 ℎ𝑎𝑣𝑒 (v) log 53
𝑙𝑜𝑔𝑎 𝑛 = 𝑙𝑜𝑔𝑎 𝑏 𝑥 = 𝑥𝑙𝑜𝑔𝑎 𝑏 = 𝑙𝑜𝑔𝑏 𝑛𝑙𝑜𝑔𝑎 𝑏 1
1
Thus 𝑙𝑜𝑔𝑎 𝑛 = 𝑙𝑜𝑔𝑏 𝑛𝑙𝑜𝑔𝑎 𝑏 → (𝑖) Sol: log
(22)3
= log(22)3 − log 53
Putting 𝑛 = 𝑎 𝑖𝑛 𝑡ℎ𝑒 𝑎𝑏𝑜𝑣𝑒 𝑟𝑒𝑠𝑢𝑙𝑡, 𝑤𝑒 𝑔𝑒𝑡 53

𝑙𝑜𝑔𝑏 𝑎 × 𝑙𝑜𝑔𝑎 𝑏 = 𝑙𝑜𝑔𝑎𝑎 = 1 1


1 (22)3 1
or 𝑙𝑜𝑔𝑎 𝑏 = 𝑙𝑜𝑔 𝑎 log 3 = log 22 − 3 log 5
𝑏 5 3
hence equation(𝑖) 𝑔𝑖𝑣𝑒𝑠
𝑙𝑜𝑔 𝑛 25×47
𝑙𝑜𝑔𝑎 𝑛 = 𝑙𝑜𝑔𝑏 𝑎 → (𝑖𝑖) (iii) log ( 29
)
𝑏
Using the above rule, a natural logarithm can be 25×47
converted to a common logarithm and vice versa. Sol: log ( 29
) = log 25 + log 47 − log 29
𝑙𝑜𝑔10 𝑛
𝑙𝑜𝑔𝑒 𝑛 = 𝑙𝑜𝑔10 𝑛 × 𝑙𝑜𝑔𝑒 10 𝑜𝑟 Q#2) Express log 𝑥 − 2 log 𝑥 + 3 log(𝑥 + 1) −
𝑙𝑜𝑔10 𝑒
𝑙𝑜𝑔𝑒 𝑛 log(𝑥 2 − 1) as a single logarithm.
𝑙𝑜𝑔10 𝑛 = 𝑙𝑜𝑔𝑒 𝑛 × 𝑙𝑜𝑔10 𝑒 𝑜𝑟
𝑙𝑜𝑔𝑒 10 Sol: log 𝑥 − 2 log 𝑥 + 3 log(𝑥 + 1) − log(𝑥 2 − 1)
The valves of 𝑙𝑜𝑔𝑒 10 𝑎𝑛𝑑 𝑙𝑜𝑔10 𝑒 𝑎𝑟𝑒 𝑎𝑣𝑎𝑖𝑙𝑎𝑏𝑙𝑒
From the tables. = log 𝑥 − log 𝑥 2 + log(𝑥 + 1)3 − log(𝑥 2 − 1)
1
𝑙𝑜𝑔𝑒 10 = = 2.3026 and 𝑥 (𝑥 + 1)3
0.4343
𝑙𝑜𝑔10 𝑒 = 𝑙𝑜𝑔2.718 = 0.4343 = log ( )
𝑥 2 (𝑥 2 − 1)
Example:
𝑪𝒂𝒍𝒄𝒖𝒍𝒂𝒕𝒆 𝒍𝒐𝒈𝟐 𝟑 × 𝒍𝒐𝒈𝟑 𝟖 (𝑥 + 1)3
Solution: = log ( )
𝑥 (𝑥 − 1)(𝑥 + 1)
We know that
log 𝑏 𝑛 (𝑥 + 1)2
log 𝑎 𝑛 = = log ( )
log 𝑏 𝑎 𝑥 (𝑥 − 1)
𝑙𝑜𝑔3 𝑙𝑜𝑔8
∵ log 2 3 × log 3 8 = × Q#3) Write the following in the single logarithm.
𝑙𝑜𝑔2 𝑙𝑜𝑔3
𝑙𝑜𝑔8 𝑙𝑜𝑔23
= (i) log 21 + log 5
𝑙𝑜𝑔2 𝑙𝑜𝑔2
3𝑙𝑜𝑔2 Sol: log 21 + log 5 = log(21 × 𝟓)
= = 333
𝑙𝑜𝑔2
(ii) log 25 − 2 log 3

Example3.3 Sol: log 2 5 − 2 log 3 = log 25 − log 32


25
= log
. Which of the following into sum of difference. 32
(i) log(𝐴 × 𝑩) (iii) 2 log 𝑥 − 3 log 𝑦
Sol: log(𝐴 × 𝑩) = log 𝐴 + log 𝐵 Sol: 2 log 𝑥 − 3 log 𝑦 = log 𝑥 2 − log 𝑦 3
15.2
(ii) log (30.5) 𝑥2
= log
𝑦3
15.2
Sol: log (30.5) = log 15.2 − log 30.5
(iv) log 5 + log 6 − log 2
21×5 5×6
(iii) log ( 8
) Sol: log 5 + log 6 − log 2 = log ( )
2
21×5
Sol: log ( 8
) = log 21 + log 5 − log 8 Q#4) calculate the following:

3 7
(i). log 3 2 × log 2 81
(iv) log √15
Sol: log 3 2 × log 2 81
1
7 1 7 log 𝑛
3
Sol: log (15) = 3 (log 15) (using log 𝑎 𝑛 = log𝑏 𝑎)
𝑏

1 log 2 log 81
= (log 7 − log 15) log 3 2 × log 2 81 = ×
3 log 3 log 2

pg. 9
Class 9th Chapter 3 www.notes.pk.com
log 34 (iv).
8
log 3
=
log 3
8
Sol: log = log 8 − log 3 = log 23 − 𝑙𝑜𝑔3
4 log 3 3
=
log 3 = 3 log 2 − log 3 = 3 (0.3010) − 0.4171
=4 = 0.9030 − 0.4171
(i). log 5 3 × log 3 25 = 0.4259
Sol: ). log 5 3 × log 3 25 (v). log 30
log 𝑛
(using log 𝑎 𝑛 = log𝑏 𝑎) Sol: log 30 = log(2 × 5 × 3) =
𝑏

log 3 log 25 = log 2 + log 5 + log 3


log 5 3 × log 3 25 = log 5 × log 3
= 0.3010 + 0.6990 + 0.4171
2
log 5
= = 1.4771
log 5
Application of logarithm
2 log 5 Example:
= Show that
log 5
𝟏𝟔 𝟐𝟓 𝟖𝟏
=2 𝟕 𝐥𝐨𝐠 + 𝟓 𝐥𝐨𝐠 + 𝐥𝐨𝐠 = 𝒍𝒐𝒈𝟐
𝟏𝟓 𝟐𝟒 𝟖𝟎
Solution:
Q#5) If log 2 = 0.3010, log 3 = 0.4171 , and log 5 = 𝟏𝟔 𝟐𝟓 𝟖𝟏
0.6990, then find the values of the following: 𝐿. 𝐻. 𝑆 = 𝟕 𝒍𝒐𝒈
+ 𝟓 𝒍𝒐𝒈 + 𝒍𝒐𝒈
𝟏𝟓 𝟐𝟒 𝟖𝟎
(i). log 32 = 𝟕[𝒍𝒐𝒈𝟏𝟔 − 𝒍𝒐𝒈𝟏𝟓] + 𝟓[𝒍𝒐𝒈𝟐𝟓 − 𝒍𝒐𝒈𝟐𝟒] +
𝟑[𝒍𝒐𝒈𝟖𝟏 − 𝒍𝒐𝒈𝟖𝟎]
Sol: log 32 = log 25 = 5 log 2 = 5 (0.3010) = 𝟕[𝒍𝒐𝒈𝟐 − 𝒍𝒐𝒈(𝟑 × 𝟓)] + 𝟓[𝒍𝒐𝒈𝟓𝟐 − 𝒍𝒐𝒈
𝟒

(𝟐𝟑 × 𝟑)] + 𝟑[𝒍𝒐𝒈𝟑𝟒 − 𝒍𝒐𝒈(𝟐𝟒 × 𝟓)]


= 1.5050 = 𝟕[𝟒𝒍𝒐𝒈𝟐 − 𝒍𝒐𝒈𝟑 − 𝒍𝒐𝒈𝟓]
+𝟓[𝟐𝒍𝒐𝒈𝟓 − 𝟑𝒍𝒐𝒈𝟐 − 𝒍𝒐𝒈𝟑] + 𝟑[𝟒𝒍𝒐𝒈𝟑 − 𝟒𝒍𝒐𝒈𝟐
(ii). log 24
− 𝒍𝒐𝒈𝟓]
Sol: log 24 = log(23 × 3) = log 23 + log 3 = (28 − 15 − 12)𝑙𝑜𝑔2 + (−7 − 5 + 12)𝑙𝑜𝑔3
+(−7 + 10 − 3)𝑙𝑜𝑔5
= 3 log 2 + log 3 = 3 (0.3010) + (0.4171) = 𝑙𝑜𝑔2 + 0 + 0 = 𝑙𝑜𝑔2 = 𝑅. 𝐻. 𝑆
Example
= 0.9030 + 0.4171 = 1.3801
𝟎.𝟎𝟕𝟗𝟐𝟐𝟐×(𝟏𝟖.𝟗𝟗)𝟐
Evaluate 𝟑√ (𝟓.𝟕𝟗)𝟒 ×𝟎.𝟗𝟒𝟕𝟒𝟒
1
(iii). log √3 3 Solution:
3 𝟎.𝟎𝟕𝟗𝟐𝟐𝟐×(𝟏𝟖.𝟗𝟗)𝟐
1 Let 𝑦 = √ (𝟓.𝟕𝟗)𝟒 ×𝟎.𝟗𝟒𝟕𝟒𝟒
1 10 10 2
Sol: log √3 3 = log √ 3 = log ( 3 ) 1
𝟎. 𝟎𝟕𝟗𝟐𝟐𝟐 × (𝟏𝟖. 𝟗𝟗)𝟐 3
1 10 ( )
(𝟓. 𝟕𝟗)𝟒 × 𝟎. 𝟗𝟒𝟕𝟒𝟒
= log
2 3 1 𝟎. 𝟎𝟕𝟗𝟐𝟐𝟐 × (𝟏𝟖. 𝟗𝟗)𝟐
𝑙𝑜𝑔𝑦 = log ( )
1 3 (𝟓. 𝟕𝟗)𝟒 × 𝟎. 𝟗𝟒𝟕𝟒𝟒
= (log 10 − log 3) 1
2 = [log{0.07921 × (18.99)}
3
1 − log{(5.79)2 × 0.9474}]
= (log(2 × 5) − log 3)
2 1
= [𝑙𝑜𝑔0.07921 + 2𝑙𝑜𝑔18.99 − 4𝑙𝑜𝑔5.79
1 3
= (log 2 + log 5 − log 3) − 𝑙𝑜𝑔0.9474]
2 1
= [2̅. 8988 + 2(1.2786) − 4(0.7627) − 1̅. 9765]
1 3
= (0.3010 + 0.6990 − 0.4171) 1
2 = [2̅. 8988 + 2.5572 − 3.0508 + 1 − 0.9765]
3
1 1
= (0.5229) = (2̅. 4287)
2 3
1
= 0.2615 = (3̅ + 1.4287)
3
pg. 10
Class 9th Chapter 3 www.notes.pk.com
= 1̅ + 0.4762 = 1̅. 4762 = −0.1688 + 0.9547 + 1.6308
𝑦 = 𝑎𝑛𝑡𝑖𝑙𝑜𝑔1̅. 4762 = 0.299333 = 2.4163
Example: log 𝑥 = 2.4163
Given 𝐴 = 𝐴𝑜 𝑒 −𝑘𝑑 𝑖𝑓 𝑘 = 2 𝑤ℎ𝑎𝑡 𝑠ℎ𝑜𝑢𝑙𝑑 𝑏𝑒 𝑡ℎ𝑒 Taking anti-log on both side, we have
𝐴
Value of 𝑑 𝑡𝑜 𝑚𝑎𝑘𝑒 𝐴 = 0 ? 𝑥 = 𝐴𝑛𝑡𝑖𝑙𝑜𝑔(2.4163)
2
Solution: 𝑥 = 261
5 7
𝐴
Given that 𝐴 = 𝐴0 𝑒 −𝑘𝑑 ⇒ 𝐴 = 𝑒 −𝑘𝑑 (iv) √2.709 × √1.239
5 7
0
𝐴0 1 Sol: Let 𝑥 = √2.709 × √1.239
Subtracting 𝑘 = 2 𝑎𝑛𝑑 𝐴 = 𝑤𝑒 𝑔𝑒𝑡 2 = 𝑒 −2𝑑
2
, Taking log on both sides
Taking common log on both sides 5 7
𝑙𝑜𝑔𝑥 = log(√2.709 × √1.239)
log10 1 − log10 2 = −2𝑑𝑙𝑜𝑔10 𝑒 1 1
Where 𝑒 = 2.718 = log(2.709)5 + log(1.239)7
0 − 0.3010 = −2𝑑(0.4343) 1 1
= [log(2.709)] + [log(1.239)]
0.3010 5 7
𝑑= = 0.3465 1 1
2 × 0.4343 = [0.4328] + [0.1931]
5 7
Example 3.4 =
0.4328 0.1931
+
5 7
1. Using log tables to find the value of. = 0.0866 + 0.0133
(i) 𝟎. 𝟖𝟏𝟕𝟔 × 𝟏𝟑. 𝟔𝟒 log 𝑥 = 0.0999
Sol: Let 𝑥 = 0.8176 × 13.64 Taking anti-log on both sides, we have
Taking log on both sides 𝑥 = 𝐴𝑛𝑡𝑖𝑙𝑜𝑔(0.0999)
log 𝑥 = log(0.8176 × 13.64) 𝑥 = 1.258
(1.23)(0.6975)
= log 0.8176 + log 13.64 (v)
(0.0075)(1278)
(In log 0.8176 , the ch. Is 1̅ we find the log(8.176) (1.23)(0.6975)
which is 0.9125, so combine both that is Sol: Let 𝑥 = (0.0075)(1278)
log 0.8176= 1̅ + 0.9125=1̅. 9125) Taking log on both sides
= 1̅. 9125 + 1.1348 (1.23)(0.6975)
log 𝑥 = log ( )
= −1 + 0.9125 + 1.1348 (0.0075)(1278)
= −0.0875 + 1.1348 = log(1.23) + log(0.6975) − log(0.0075)
log 𝑥 = 1.0473 − log(1279)
Taking anti-log on both sides, we have = 0.0899 + 1̅. 8435 − 3̅. 8751 − 3.1065
𝑥 = 𝐴𝑛𝑡𝑖𝑙𝑜𝑔(1.0473) = 0.0899 + (−1 + 0.8435) − (−3 + 0.8751
𝑥 = 11.15 − 3.1065
1
(ii) (789.5)8 = 0.0899 + (−0.1565) − (−2.1249) − 3.1065
1 = 0.0899 − 0.1565 + 2.1249 − 3.1065
Sol: Let 𝑥 = (789.5)8 log 𝑥 = −1.0482
Taking log on both sides Adding and subtracting 2 on R.H.S
1
𝑙𝑜𝑔𝑥 = log(789.5)8 log 𝑥 = −2 + 2 − 1.0482
1 log 𝑥 = 2̅ + (2 − 1.0482)
= [log(789.5)] log 𝑥 = 2̅ + (0.9518)
8
1 2.8974
= 8 [2.8974] = 8 log 𝑥 = 2̅. 9518
Taking anti-log on both side, we have
log 𝑥 = 0.3622
Taking anti-log on both sides, we have 𝑥 = 𝐴𝑛𝑡𝑖𝑙𝑜𝑔(2̅. 9518)
𝑥 = 𝐴𝑛𝑡𝑖𝑙𝑜𝑔(0.3622) (Here 𝐴𝑛𝑡𝑖𝑙𝑜𝑔(0.9518) = 8.50 but Ch. 2̅ indicates
𝑥 = 2.302 that point will move two digits to left side)
0.678×9.01 𝑥 = 0.0850
(iii)
0.0234 3 0.7214×20.37
0.678×9.01 (iii) √
Sol: Let 𝑥 = 0.0234
60.8

Taking log on both sides 3 0.7214×20.37


Sol: Let 𝑥 = √ 60.8

0.678 × 9.01 Taking log on both sides


log 𝑥 = log ( )
0.0234 3 0.7214 × 20.37
= log(0.678) + log(9.01) − log(0.0234) log 𝑥 = log ( √ )
60.8
= 1̅. 8312 + 0.9547 − 2̅. 3692
1
= (−1 + 0.83120) + 0.9547 − (−2 + 0.3692) 0.7214 × 20.37 3
= (−0.1688) + 0.9547 − (−1.6308) = log ( )
60.8
pg. 11
Class 9th Chapter 3 www.notes.pk.com
1 = 7.9245 − 0.6259 − 10.3552
= [log(0.7214) + log(20.37) − log(60.8)]
3 log 𝑥 = −3.0566
1 Adding and subtracting 4 on R.H.S
= [1̅. 8582 + 1.3090 − 1.7839]
3 log 𝑥 = −4 + 4 − 3.0566
1
= [−1 + 0.8582 + 1.3090 − 1.7839] log 𝑥 = 4̅ + (4 − 3.0566)
3 log 𝑥 = 4̅ + (0.9434)
1
= [−0.1418 + 1.3090 − 1.7839] log 𝑥 = 4̅. 9434
3
1 Taking anti-log on both side, we have
= [−0.6157] 𝑥 = 𝐴𝑛𝑡𝑖𝑙𝑜𝑔(4̅. 9434)
3
0.6157 (Here 𝐴𝑛𝑡𝑖𝑙𝑜𝑔(0.9434) = 8.778 but Ch. 4̅ indicates
=− that point will move four digits to left side)
3
log 𝑥 = −0.2056 𝑥 = 0.0008778
Adding and subtracting 1 on R.H.S Q#2) A gas is expanding according to the law 𝒑𝒗𝒏 =
𝟓
log 𝑥 = −1 + 1 − 0.2056 𝑪. Find 𝑪, when 𝒑 = 𝟖𝟎, 𝒗 = 𝟑. 𝟏 𝒂𝒏𝒅 𝒏 = 𝟒.
log 𝑥 = 1̅ + (1 − 0.2056) Sol: 𝑝𝑣 𝑛 = 𝐶
log 𝑥 = 1̅ + (0.7944) Taking log on both sides
log 𝑥 = 1̅. 7944 log( 𝑝𝑣 𝑛 ) = log 𝐶
Taking anti-log on both side, we have log 𝐶 = log 𝑝 + log 𝑣 𝑛
𝑥 = 𝐴𝑛𝑡𝑖𝑙𝑜𝑔(1̅. 7944) log 𝐶 = log 𝑝 + 𝑛 log 𝑣
(Here 𝐴𝑛𝑡𝑖𝑙𝑜𝑔(0.7944) = 6.229 but Ch. 1̅ indicates Putting values
that point will move one digits to left side) 5
𝑥 = 0.6229 log 𝐶 = log 80 + log 3.1
4
3
83× √92 5
(v) 5 log 𝐶 = 1.9030 + (0.4914)
127× √246
3
4
Sol: Let 𝑥 =
83× √92 log 𝐶 = 1.9030 + 0.6143
5
127× √246 log 𝐶 = 2.5173
Taking log on both sides Taking anti-log on both side, we have
3
83 × √92 𝐶 = 𝐴𝑛𝑡𝑖𝑙𝑜𝑔(2.5173)
log 𝑥 = log ( 5 )
127 × √246 𝐶 = 329.2
1 1 𝑞
= log(83) + log(92)3 − log(127) − log(246)5 Q#3) The formula 𝑝 = 90 (5)−10 applies to the
1 1 demand of a product, where 𝑞 is the number of units
= log(83) + log(92) − log(127) − log(246) and 𝑝 is the price of one unit. How many units will be
3 5
1 1 demanded if the price is Rs. 18.00?
= 1.9191 + (1.9638) − 2.1038 − (2.3909) 𝑞
3 5 Sol: 𝑝 = 90 (5)−10
1.9638 2.3909
= 1.9191 + − 2.1038 − Taking log on both sides
3 5 𝑞
= 1.9191 + 0.6546 − 2.1038 − 0.4782 log( 𝑝) = log(90 (5)−10 )
𝑞
log 𝑥 = 1.9917 log 𝑝 = log 90 + log((5)−10 )
Taking anti-log on both side, we have 𝑞
𝑥 = 𝐴𝑛𝑡𝑖𝑙𝑜𝑔(1.9917) log 𝑝 = log 90 − log 5
10
𝑥 = 0.9811 Putting values
(438)3 √0.056 𝑞
(viii) log 18 = log 90 − log 5
(388)4 10
(438)3 √0.056 𝑞
Sol: Let 𝑥 = 1.2553 = 1.9542 − (0.6990)
(388)4 10
Taking log on both sides 𝑞
1.2553 − 1.9542 = − (0.6990)
(438)3 √0.056 10
log 𝑥 = log ( ) 𝑞
(388)4 −0.6990 = − (0.6990)
10
1 𝑞
= log(438)3 + log(0.056)2 − log(388)4 1=
10
1 𝑞 = 10 units
= 3 log(438) + log(0.056) − 4 log(388)
2 22
1 Q#4) If 𝐴 = 𝜋𝑟 2 , find 𝐴, when 𝜋 = 7
and 𝑟 = 15
= 3(2.6415) + (2̅. 7482) − 4(2.5888) Sol: 𝐴 = 𝜋𝑟 2
2
1 Taking log on both sides
= 7.9245 + (−2 + 0.7482) − 10.3552
2 log( 𝐴) = log(𝜋𝑟 2 )
1
= 7.9245 + (−1.2518) − 10.3552
2
pg. 12
Class 9th Chapter 3 www.notes.pk.com
2
22𝑟
log( 𝐴) = log ( )
7

log 𝐴 = log 22 + log(𝑟)2 − log 7


log 𝐴 = log 22 + 2 log 𝑟 − log 7
Putting values
log 𝐴 = log 22 + 2 log 15 − log 7
log 𝐴 = 1.3424 + 2(1.1761) − 0.8451
log 𝐴 = 1.3424 + 2.3522 − 0.8451
log 𝐴 = 2.8495
Taking anti-log on both side, we have
𝐴 = 𝐴𝑛𝑡𝑖𝑙𝑜𝑔(2.8495)
𝐴 = 707.1 Sq. units
1 22
Q#5) If 𝐴 = 3 𝜋𝑟 2 ℎ, find 𝐴, when 𝜋 = 7
, 𝑟 = 2.5 and
ℎ = 4.2
1
Sol: 𝐴 = 𝜋𝑟 2 ℎ
3
Taking log on both sides
1
log( 𝐴) = log ( 𝜋𝑟 2 ℎ)
3
22𝑟 2 ℎ
log( 𝐴) = log ( )
21
log 𝐴 = log 22 + log(𝑟)2 + log ℎ − log 21
log 𝐴 = log 22 + 2 log 𝑟 + log ℎ − log 21
Putting values
log 𝐴 = log 22 + 2 log 2.5 + log 4.2 − log 21
log 𝐴 = 1.3424 + 2(0.3979) + 0.6232 − 1.3222
log 𝐴 = 1.3424 + 0.7958 + 0.6232 − 1.3222
log 𝐴 = 1.4392
Taking anti-log on both side, we have
𝐴 = 𝐴𝑛𝑡𝑖𝑙𝑜𝑔(1.4392)
𝐴 = 27.49 cubic units

A project of: www.notespk.com


Contact or Suggest Us: info@notespk.com

pg. 13
MATHEMATICS 9th Science Group

7/18/2020
Chapter 4.
ALGEBRAIC EXPRESSIONS AND
ALGEBRAIC FORMULAS

A project of: www.notespk.com


Contact or Suggest Us: info@notespk.com
Contents
EXERCISE 4.1 ....................................................................................... 1
EXERCISE 4.2 ....................................................................................... 3
EXERCISE 4.3 ....................................................................................... 6
EXERCISE 4.4 ....................................................................................... 8
Class 9th Chapter 4 www.notes.pk.com

Algebraic Expressions (iii). 𝒙𝟐 − 𝟑𝒙 + √𝟐


Algebra is a generalization of arithmetic. Recall Sol: Yes, it is a polynomial because all powers are
that when operations of addition and subtraction non-negative integers.
are applied to algebraic terms, we obtain an 𝟑𝒙
(iv). 𝟐𝒙−𝟏 + 𝟖
algebraic expression. For instance, 𝟓𝒙𝟐 − 𝟑𝒙 +
𝟐 𝟑 Sol: No, it is not a polynomial because it contains
, 𝟑𝒙𝒚 + 𝒙 (𝒙 ≠ 𝟎) are algebraic expressions. 𝟑𝒙
√𝒙 the term 𝟐𝒙−𝟏.
Polynomials
Q#2) State whether each of the following
it is a polynomial A polynomial in the variable x is
expressions is a rational expression or not.
an algebraic expression of the form 𝟑√𝒙
𝑷(𝒙) = 𝒂𝒏 𝒙𝒏 + 𝒂𝒏−𝟏 𝒙𝒏−𝟏 + 𝒂𝒏−𝟐 𝒙𝒏−𝟐 + … + (i). 𝟑
√𝒙+𝟓
𝒂𝟏 𝒙 + 𝒂𝟎 , 𝒂𝒏 ≠ 𝟎 …… (i) Sol: It is not rational expression.
where 𝒏, the highest power of 𝒙, is a non-negative 𝒙𝟑 −𝟐𝒙𝟐 +√𝟑
(ii). 𝟐+𝟑𝒙−𝒙𝟐
integer called the degree of the polynomial and
each coefficient 𝒂𝒏 , is a real number. The Sol: It is not rational expression.
𝒙𝟐 +𝟔𝒙+𝟗
coefficient 𝒂𝒏 of the highest power of 𝒙 is called the (iii). 𝒙𝟐 −𝟗
leading coefficient Sol: It a rational expression.
of the polynomial. 𝟐𝒙𝟒 𝒚𝟐 + 𝒙𝟐 𝒚𝟐 + 𝟖𝒙 is a 𝟐√𝒙+𝟑
(iv). 𝟐
polynomial in two variables 𝒙 and 𝒚 having degree √𝒙−𝟑
6 (4+2=6). Sol: It is not rational expression.
Rational Expression Q#3) Reduce the following rational expressions to
𝒑(𝒙) the lowest form.
The quotient 𝒒(𝒙) of two polynomials, 𝒑(𝒙) and
𝟏𝟐𝟎 𝒙𝟐 𝒚𝟑 𝒛𝟓
(i).
𝒒(𝒙), where 𝒒(𝒙) 𝟑𝟎 𝒙𝟑 𝒚 𝒛𝟐
𝟏𝟐𝟎 𝒙𝟐 𝒚𝟑 𝒛𝟓
is a non-zero polynomial, is called a rational Sol: 𝟑𝟎 𝒙𝟑𝒚 𝒛𝟐 = 𝟒𝒙𝟐−𝟑 𝒚𝟑−𝟏 𝒛𝟓−𝟐
expression.
𝟐𝒙+𝟓 = 𝟒𝒙−𝟏 𝒚𝟐 𝒛𝟑
For example, , 𝟓𝒙− ≠ 𝟎 is a rational
𝟓𝒙−𝟏 𝟒 𝒚𝟐 𝒛𝟑
expression. =
𝒙
Note: 𝟖𝒂 (𝒙+𝟏)
(ii).
Every polynomial 𝒑(𝒙) can be regarded as a 𝟐(𝒙𝟐 −𝟏)
𝒑(𝒙) 𝟖𝒂 (𝒙+𝟏) 𝟒𝒂 (𝒙+𝟏)
rational expression, since we can write 𝒑(𝒙) as . Sol: =
𝟏 𝟐(𝒙𝟐 −𝟏) (𝒙−𝟏)(𝒙+𝟏)
Thus, every polynomial is a rational expression, but 𝟒𝒂
=
every rational expression need not be a polynomial. (𝒙 − 𝟏)
Algebraic formulas 𝟒𝒂
=
(i).(𝒂 + 𝒃)𝟐 = 𝒂𝟐 + 𝒃𝟐 + 𝟐𝒂𝒃 𝒙−𝟏
(𝒙+𝒚)𝟐 −𝟒𝒙𝒚
(ii).(𝒂 − 𝒃)𝟐 = 𝒂𝟐 + 𝒃𝟐 − 𝟐𝒂𝒃 (iii). (𝒙−𝒚)𝟐
(iii). 𝒙𝟐 − 𝒚𝟐 = (𝒙 − 𝒚)(𝒙 + 𝒚) (𝒙+𝒚)𝟐 −𝟒𝒙𝒚 𝒙𝟐 +𝒚𝟐 +𝟐𝒙𝒚−𝟒𝒙𝒚
(iv) (𝒙 + 𝒚)𝟑 = 𝒙𝟑 + 𝒚𝟑 + 𝟑𝒙𝒚(𝒙 + 𝒚) Sol: (𝒙−𝒚) 𝟐 = 𝒙𝟐 +𝒚𝟐 −𝟐𝒙𝒚
(v) (𝒙 − 𝒚)𝟑 = 𝒙𝟑 − 𝒚𝟑 − 𝟑𝒙𝒚(𝒙 − 𝒚) 𝟐 𝟐
𝒙 + 𝒚 − 𝟐𝒙𝒚
𝟏 𝟑 𝟏 𝟏 == =𝟏
(vi) (𝒙 + ) = 𝒙𝟑 + + 𝟑 (𝒙 + ) 𝒙𝟐 + 𝒚𝟐 − 𝟐𝒙𝒚
𝒙 𝒙𝟑 𝒙
(vii) 𝒙 + 𝒚 = (𝒙 + 𝒚)(𝒙𝟐 − 𝒙𝒚 + 𝒚𝟐 )
𝟑 𝟑
(𝒙𝟑 −𝒚𝟑 )(𝒙𝟐 −𝟐𝒙𝒚+𝒚𝟐 )
(viii) 𝒙𝟑 − 𝒚𝟑 = (𝒙 − 𝒚)(𝒙𝟐 + 𝒙𝒚 + 𝒚𝟐 ) (iv). (𝒙−𝒚)(𝒙𝟐 +𝒙𝒚+𝒚𝟐 )
(𝒙 −𝒚𝟑 )(𝒙𝟐 −𝟐𝒙𝒚+𝒚𝟐 )
𝟑 (𝒙𝟑 −𝒚𝟑 )(𝒙𝟐 −𝟐𝒙𝒚+𝒚𝟐 )
Sol: (𝒙−𝒚)(𝒙𝟐+𝒙𝒚+𝒚𝟐) = (𝒙𝟑 −𝒚𝟑 )
EXERCISE4.1 (𝒙 = 𝟐
− 𝒚)
Q#1) Identify whether the following algebraic (𝒙+𝟐)(𝒙𝟐 −𝟏)
expressions are polynomials (Yes or No). (v). (𝒙+𝟏)(𝒙𝟐−𝟒)
𝟏 (𝒙+𝟐)(𝒙𝟐 −𝟏)
(i). 𝟑𝒙𝟐 + 𝒙 − 𝟓 Sol: (𝒙+𝟏)(𝒙𝟐−𝟒) =
(𝒙+𝟐)(𝒙−𝟏)(𝒙+𝟏)
(𝒙+𝟏)(𝒙𝟐 −𝟐𝟐 )
Sol: No, it is not a polynomial because it contains (𝒙 + 𝟐)(𝒙 − 𝟏) 𝒙 − 𝟏
𝟏
. = =
the term 𝒙 (𝒙 − 𝟐)(𝒙 + 𝟐) 𝒙 − 𝟐
(ii). 𝟑𝒙𝟑 − 𝟒𝒙𝟐 − 𝒙√𝒙 + 𝟑 𝒙𝟐 −𝟒𝒙+𝟒
(vi).
𝟐𝒙𝟐 −𝟖
Sol: No, it is not a polynomial because it contains 𝒙𝟐 −𝟒𝒙+𝟒 𝒙𝟐 −𝟐(𝒙)(𝟐)+(𝟐)𝟐
the term 𝒙√𝒙. Sol: 𝟐𝒙𝟐 −𝟖
= 𝟐(𝒙𝟐 −𝟒)

1|Page
Class 9th Chapter 4 www.notes.pk.com
(𝒙 − 𝟐)𝟐 𝒙𝟐 𝒚𝟑 − 𝟓𝒛𝟒 (𝟒)𝟐 (−𝟐)𝟑 − 𝟓(−𝟏)𝟒
= =
𝟐(𝒙𝟐 − 𝟐𝟐 ) 𝒙𝒚𝒛 (𝟒)(−𝟐)(−𝟏)
(𝒙 − 𝟐)𝟐 (𝟏𝟔)(−𝟖) − 𝟓(𝟏)
= =
𝟐(𝒙 − 𝟐)(𝒙 + 𝟐) 𝟖
𝒙−𝟐 −𝟏𝟐𝟖 − 𝟓
= =
𝟐(𝒙 + 𝟐) 𝟖
𝟔𝟒𝒙𝟓 −𝟔𝟒𝒙
−𝟏𝟑𝟑 𝟓
(vii). (𝟖𝒙𝟐+𝟖)(𝟐𝒙+𝟐) = = −𝟏𝟔
𝟖 𝟖
𝟔𝟒𝒙𝟓 −𝟔𝟒𝒙 𝟔𝟒𝒙(𝒙𝟒 −𝟏) Q#5) Perform the indicated operation and simplify.
Sol: (𝟖𝒙𝟐+𝟖)(𝟐𝒙+𝟐) = 𝟖(𝒙𝟐+𝟏)𝟐(𝒙+𝟏) 𝟏𝟓 𝟒
(i). 𝟐𝒙−𝟑𝒚 − 𝟑𝒚−𝟐𝒙
𝟐 𝟐 𝟐
𝟒𝒙((𝒙 ) − (𝟏) ) 𝟏𝟓 𝟒 𝟏𝟓 𝟒
= Sol: 𝟐𝒙−𝟑𝒚 − 𝟑𝒚−𝟐𝒙 = 𝟐𝒙−𝟑𝒚 − −(𝟐𝒙−𝟑𝒚)
(𝒙𝟐 + 𝟏)(𝒙 + 𝟏)
𝟒𝒙(𝒙𝟐 + 𝟏)(𝒙𝟐 − 𝟏) 𝟏𝟓 𝟒
= = +
(𝒙𝟐 + 𝟏)(𝒙 + 𝟏) 𝟐𝒙 − 𝟑𝒚 𝟐𝒙 − 𝟑𝒚
𝟒𝒙(𝒙 − 𝟏)(𝒙 + 𝟏) 𝟏𝟓 + 𝟒
= =
(𝒙 + 𝟏) 𝟐𝒙 − 𝟑𝒚
= 𝟒𝒙(𝒙 − 𝟏) 𝟏𝟗
=
𝟗𝒙𝟐 −(𝒙𝟐 −𝟒)
𝟐
𝟐𝒙 − 𝟑𝒚
(viii). 𝟒+𝟑𝒙−𝒙𝟐 𝟏+𝟐𝒙 𝟏−𝟐𝒙
𝟐 𝟐
(ii). 𝟏−𝟐𝒙 − 𝟏+𝟐𝒙
𝟗𝒙 −(𝒙𝟐 −𝟒)
𝟐 (𝟑𝒙)𝟐 −(𝒙𝟐 −𝟒)
Sol: = 𝟏+𝟐𝒙 𝟏−𝟐𝒙 (𝟏+𝟐𝒙)𝟐 −(𝟏−𝟐𝒙)𝟐
𝟒+𝟑𝒙−𝒙𝟐 𝟒+𝟑𝒙−𝒙𝟐 Sol: 𝟏−𝟐𝒙 − 𝟏+𝟐𝒙
= (𝟏−𝟐𝒙)(𝟏+𝟐𝒙)
(𝟑𝒙 − (𝒙𝟐 − 𝟒))(𝟑𝒙 + (𝒙𝟐 − 𝟒)) [(𝟏)𝟐 + (𝟐𝒙)𝟐 + 𝟐(𝟏)(𝟐𝒙)] − [(𝟏)𝟐 + (𝟐𝒙)𝟐 − 𝟐(𝟏)(𝟐𝒙)]
= =
𝟒 + 𝟑𝒙 − 𝒙𝟐 (𝟏)𝟐 − (𝟐𝒙)𝟐
(𝟑𝒙 − 𝒙 + 𝟒)(𝟑𝒙 + 𝒙𝟐 − 𝟒)
𝟐
𝟏 + 𝟒𝒙𝟐 + 𝟒𝒙 − 𝟏 − 𝟒𝒙𝟐 + 𝟒𝒙
= =
𝟒 + 𝟑𝒙 − 𝒙𝟐 𝟏 − 𝟒𝒙𝟐
= 𝟑𝒙 + 𝒙𝟐 − 𝟒 𝟖
𝒙𝟑 𝒚−𝟐𝒛 =
Q#4) Evaluate (a). for 𝟏 − 𝟒𝒙𝟐
𝒙𝒛 𝒙𝟐 −𝟐𝟓 𝒙+𝟓
(i). 𝒙 = 𝟑, 𝒚 = −𝟏 𝒂𝒏𝒅 𝒛 = −𝟐 (iii). −
𝒙𝟐 −𝟑𝟔 𝒙+𝟔
𝒙𝟑 𝒚−𝟐𝒛 𝒙𝟐 −𝟐𝟓 𝒙+𝟓 𝒙𝟐 −𝟓𝟐 𝒙+𝟓
Sol: As given 𝒙𝒛
Sol 𝒙𝟐 −𝟑𝟔
− 𝒙+𝟔 = 𝒙𝟐−𝟔𝟐 − 𝒙+𝟔
Putt 𝒙 = 𝟑, 𝒚 = −𝟏 𝒂𝒏𝒅 𝒛 = −𝟐 in above (𝒙 + 𝟓)(𝒙 − 𝟓) 𝒙 + 𝟓
= −
𝒙𝟑 𝒚 − 𝟐𝒛 (𝟑)𝟑 (−𝟏) − 𝟐(−𝟐) (𝒙 + 𝟔)(𝒙 − 𝟔) 𝒙 + 𝟔
= (𝒙 + 𝟓)(𝒙 − 𝟓) − (𝒙 − 𝟔)(𝒙 + 𝟓)
𝒙𝒛 (𝟑)(−𝟐)
=
(𝟐𝟕)(−𝟏) + 𝟒 (𝒙 + 𝟔)(𝒙 − 𝟔)
=
−𝟔 (𝒙 − 𝟐𝟓) − (𝒙𝟐 + 𝟓𝒙 − 𝟔𝒙 − 𝟑𝟎)
𝟐
−𝟐𝟕 + 𝟒 =
= 𝒙𝟐 − 𝟑𝟔
−𝟔 (𝒙 − 𝟐𝟓) − (𝒙𝟐 − 𝒙 − 𝟑𝟎)
𝟐
−𝟐𝟑 𝟐𝟑 𝟓 =
= = =𝟑 𝒙𝟐 − 𝟑𝟔
−𝟔 𝟔 𝟔 𝒙𝟐 − 𝟐𝟓 − 𝒙𝟐 + 𝒙 + 𝟑𝟎
(ii). 𝒙 = −𝟏, 𝒚 = −𝟗 𝒂𝒏𝒅 𝒛 = 𝟒 =
𝒙𝟑 𝒚−𝟐𝒛
𝒙𝟐 − 𝟑𝟔
Sol: As given 𝒙+𝟓
𝒙𝒛 = 𝟐
Putt 𝒙 = −𝟏, 𝒚 = −𝟗 𝒂𝒏𝒅 𝒛 = 𝟒 in above 𝒙 − 𝟑𝟔
𝒙 𝒚 𝟐𝒙𝒚
𝒙𝟑 𝒚 − 𝟐𝒛 (−𝟏)𝟑 (−𝟗) − 𝟐(𝟒) (iv). 𝒙−𝒚 − 𝒙+𝒚 − 𝒙𝟐−𝒚𝟐
= 𝒙 𝒚 𝟐𝒙𝒚 𝒙 𝒚 𝟐𝒙𝒚
𝒙𝒛 (−𝟏)(𝟒) Sol: − − 𝟐 𝟐 = − −
(−𝟏)(−𝟗) − 𝟖 𝒙−𝒚 𝒙+𝒚 𝒙 −𝒚 𝒙−𝒚 𝒙+𝒚 (𝒙−𝒚)(𝒙+𝒚)
= 𝒙(𝒙 + 𝒚) − 𝒚(𝒙 − 𝒚) − 𝟐𝒙𝒚
−𝟒 =
+𝟗 − 𝟖 (𝒙 − 𝒚)(𝒙 + 𝒚)
= 𝒙𝟐 + 𝒙𝒚 − 𝒙𝒚 + 𝒚𝟐 − 𝟐𝒙𝒚
−𝟒 =
𝟏 𝟏 (𝒙 − 𝒚)(𝒙 + 𝒚)
= =−
−𝟒 𝟒 (𝒙 − 𝒚)𝟐
𝒙𝟐 𝒚𝟑 −𝟓𝒛𝟒 =
(b). for 𝒙 = 𝟒, 𝒚 = −𝟐 𝒂𝒏𝒅 𝒛 = −𝟏 (𝒙 − 𝒚)(𝒙 + 𝒚)
𝒙𝒚𝒛
𝒙−𝒚
𝒙𝟐 𝒚𝟑 −𝟓𝒛𝟒 =
Sol: As given 𝒙+𝒚
𝒙𝒚𝒛
𝒙−𝟐 𝒙+𝟐
Putt 𝒙 = 𝟒, 𝒚 = −𝟐 𝒂𝒏𝒅 𝒛 = −𝟏 in above (v). 𝒙𝟐+𝟔𝒙+𝟗 − 𝟐𝒙𝟐 −𝟏𝟖

2|Page
Class 9th Chapter 4 www.notes.pk.com
𝒙−𝟐
Sol: 𝒙𝟐+𝟔𝒙+𝟗 −
𝒙+𝟐 𝒙−𝟐
= (𝒙)𝟐+𝟐(𝒙)(𝟑)+(𝟑𝟐) −
𝒙+𝟐 (𝒙 − 𝒚)(𝒙 + 𝒚)(+𝒙𝟐 𝒚𝟐 + 𝒚𝟒 ) 𝟏
𝟐𝒙𝟐 −𝟏𝟖 𝟐(𝒙𝟐 −𝟗) = × 𝟒
𝒙−𝟐 𝒙+𝟐 (𝒙 − 𝒚)(𝒙 + 𝒚) (𝒙 + 𝒙 𝒚𝟐 + 𝒚𝟒 )
𝟐
= 𝟐
− =𝟏
(𝒙 + 𝟑) 𝟐(𝒙𝟐 − 𝟑𝟐 )
𝒙𝟐 −𝟏 𝒙+𝟓
𝒙−𝟐 𝒙+𝟐 (iv). 𝒙𝟐+𝟐𝒙+𝟏 . 𝟏−𝒙
= 𝟐

(𝒙 + 𝟑) 𝟐(𝒙 − 𝟑)(𝒙 + 𝟑) 𝒙𝟐 −𝟏 𝒙+𝟓 (𝒙−𝟏)(𝒙+𝟏) 𝒙+𝟓
Sol: 𝒙𝟐+𝟐𝒙+𝟏 . 𝟏−𝒙 = . 𝟏−𝒙
𝟐(𝒙 − 𝟐)(𝒙 − 𝟑) − (𝒙 + 𝟐)(𝒙 + 𝟑) (𝒙+𝟏)𝟐
= (𝒙 − 𝟏) 𝒙+𝟓
𝟐(𝒙 − 𝟑)(𝒙 + 𝟑)𝟐 = .
𝟐(𝒙 − 𝟑𝒙 − 𝟐𝒙 + 𝟔) − (𝒙𝟐 + 𝟑𝒙 + 𝟐𝒙 + 𝟔)
𝟐 (𝒙 + 𝟏) −(𝒙 − 𝟏)
= (𝒙 + 𝟓)
𝟐(𝒙 − 𝟑)(𝒙 + 𝟑)𝟐 =−
𝟐(𝒙 − 𝟓𝒙 + 𝟔) − (𝒙𝟐 + 𝟓𝒙 + 𝟔)
𝟐 (𝒙 + 𝟏)
= 𝒙𝟐 +𝒙𝒚 𝒙𝟐 +𝒙𝒚
(v). 𝒚(𝒙+𝒚) . 𝒚(𝒙+𝒚) ÷ 𝒙𝒚−𝟐𝒚
𝒙𝟐 −𝒙
𝟐(𝒙 − 𝟑)(𝒙 + 𝟑)𝟐
𝟐𝒙 − 𝟏𝟎𝒙 + 𝟏𝟐 − 𝒙𝟐 − 𝟓𝒙 − 𝟔
𝟐
𝒙𝟐 +𝒙𝒚 𝒚𝟐 +𝒙𝒚 𝒙𝟐 −𝒙
= Sol: . ÷
𝟐(𝒙 − 𝟑)(𝒙 + 𝟑)𝟐 𝒚(𝒙+𝒚) 𝒚(𝒙+𝒚) 𝒙𝒚−𝟐𝒚

𝒙𝟐 − 𝟏𝟓𝒙 + 𝟔 𝒙(𝒙 + 𝒚) 𝒙(𝒙 + 𝒚) 𝒙𝒚 − 𝟐𝒚


= = . × 𝟐
𝟐(𝒙 − 𝟑)(𝒙 + 𝟑)𝟐 𝒚(𝒙 + 𝒚) 𝒚(𝒙 + 𝒚) 𝒙 −𝒙
𝟏 𝟏 𝟐 𝟒 𝒙 𝒙 𝒚(𝒙 − 𝟐)
(iv). − − 𝟐 − 𝟒 = . ×
𝒙−𝟏 𝒙+𝟏 𝒙 +𝟏 𝒙 −𝟏 𝒚 𝒚 𝒙(𝒙 − 𝟏)
𝟏 𝟏 𝟐 𝟒
Sol: 𝒙−𝟏 − 𝒙+𝟏
− 𝒙𝟐 +𝟏
− 𝒙𝟒 −𝟏
𝒙 (𝒙 − 𝟐)
𝟏 𝟏 𝟐 𝟒
= ×
𝒚 (𝒙 − 𝟏)
= − − 𝟐 −
𝒙 − 𝟏 𝒙 + 𝟏 𝒙 + 𝟏 (𝒙 − 𝟏)(𝒙 + 𝟏)(𝒙𝟐 + 𝟏) 𝒙(𝒙 − 𝟐)
𝟏(𝒙 + 𝟏)(𝒙𝟐 + 𝟏) − 𝟏(𝒙 − 𝟏)(𝒙𝟐 + 𝟏) − 𝟐(𝒙 + 𝟏)(𝒙 − 𝟏) − 𝟒 =
= 𝒚(𝒙 − 𝟏)
(𝒙 − 𝟏)(𝒙 + 𝟏)(𝒙𝟐 + 𝟏)
Algebraic formulas
(𝒙𝟑 + 𝒙 + 𝒙𝟐 + 𝟏) − (𝒙𝟑 + 𝒙 − 𝒙𝟐 − 𝟏) − 𝟐(𝒙𝟐 − 𝟏) − 𝟒 (i).(𝑎 + 𝑏)2 + (𝑎 − 𝑏)2 = 2(𝑎2 + 𝑏 2 )
= (ii).(𝑎 + 𝑏)2 − (𝑎 − 𝑏)2 = 4𝑎𝑏
𝒙𝟒 − 𝟏
𝒙𝟑 + 𝒙 + 𝒙𝟐 + 𝟏 − 𝒙𝟑 − 𝒙 + 𝒙𝟐 + 𝟏 − 𝟐𝒙𝟐 + 𝟐 − 𝟒 (iii). (𝑎 + 𝑏 + 𝑐)2 = 𝑎2 + 𝑏 2 + 𝑐 2 + 2(𝑎𝑏 + 𝑎𝑐 +
=
𝒙𝟒 − 𝟏 𝑐𝑎)
(iv) (𝑥 + 𝑦)3 = 𝑥 3 + 𝑦 3 + 3𝑥𝑦(𝑥 + 𝑦)
𝟐𝒙𝟐 + 𝟒 − 𝟐𝒙𝟐 − 𝟒 (v) (𝑥 − 𝑦)3 = 𝑥 3 − 𝑦 3 − 3𝑥𝑦(𝑥 − 𝑦)
=
𝒙𝟒 − 𝟏 1 3 1 1
𝟎 (vi) (𝑥 + 𝑥) = 𝑥 3 + 𝑥 3 + 3 (𝑥 + 𝑥)
= 𝟒 =𝟎 (vii) 𝑥 3 + 𝑦 3 = (𝑥 + 𝑦)(𝑥 2 − 𝑥𝑦 + 𝑦 2 )
𝒙 −𝟏
Q#6) Perform the indicated operation and simplify. (viii) 𝑥 3 − 𝑦 3 = (𝑥 − 𝑦)(𝑥 2 + 𝑥𝑦 + 𝑦 2 )
𝟓𝒙+𝟐
(i). (𝒙𝟐 − 𝟒𝟗).
𝒙+𝟕
Sol: (𝒙𝟐 −
𝟓𝒙+𝟐
𝟒𝟗). 𝒙+𝟕 = (𝒙𝟐 − 𝟕𝟐 ). 𝒙+𝟕
𝟓𝒙+𝟐 EXERCISE 4.2
𝟓𝒙 + 𝟐 Q#1).
= (𝒙 − 𝟕)(𝒙 + 𝟕). (i) If 𝑎 + 𝑏 = 10 and 𝑎 − 𝑏 = 6, then find the value of
𝒙+𝟕
= (𝒙 − 𝟕)(𝟓𝒙 + 𝟐) 𝑎2 + 𝑏 2
𝟒𝒙−𝟏𝟐 𝟏𝟖−𝟐𝒙𝟐 Solution: As given 𝑎 + 𝑏 = 10 and 𝑎 − 𝑏 = 6
(ii).𝒙𝟐 −𝟗
÷ 𝒙𝟐+𝟔𝒙+𝟗 We find 𝑎2 + 𝑏 2 =?
𝟒𝒙−𝟏𝟐 𝟏𝟖−𝟐𝒙𝟐 𝟒(𝒙−𝟑) (𝒙)𝟐 +𝟐(𝒙)(𝟑)+(𝟑𝟐 ) Using the identity
Sol: 𝒙𝟐 −𝟗 ÷ 𝒙𝟐+𝟔𝒙+𝟗 = 𝟐
𝒙 −𝟑 𝟐 × 𝟐(𝟗−𝒙𝟐 )
𝟐
(𝑎 + 𝑏)2 + (𝑎 − 𝑏)2 = 2(𝑎2 + 𝑏 2 )
𝟒(𝒙 − 𝟑) + 𝟑) (𝒙 Put values
= ×
(𝒙 − 𝟑)(𝒙 + 𝟑) 𝟐(𝟑 − 𝒙)(𝟑 + 𝒙) (10)2 + (6)2 = 2(𝑎2 + 𝑏 2 )
𝟐 𝟏 𝟐 100 + 36 = 2(𝑎2 + 𝑏 2 )
= × =
𝟏 (𝟑 − 𝒙) 𝟑 − 𝒙 136 = 2(𝑎2 + 𝑏 2 )
𝒙𝟔 −𝒚𝟔
(iii). 𝒙𝟐 −𝒚𝟐 ÷ (𝒙𝟒 + 𝒙𝟐 𝒚𝟐 + 𝒚𝟒 ) 136
𝑎2 + 𝑏 2 =
𝒙𝟔 −𝒚𝟔 2
Sol: 𝒙𝟐 −𝒚𝟐
÷ (𝒙𝟒 + 𝒙𝟐 𝒚𝟐 + 𝒚𝟒 ) 𝑎2 + 𝑏 2 = 68
(𝒙𝟐 )𝟑 − (𝒚𝟐 )𝟑 𝟏 Which is required.
= × 𝟒 (ii) If 𝒂 + 𝒃 = 𝟓 and 𝒂 − 𝒃 = √𝟏𝟕, then find the
(𝒙 − 𝒚)(𝒙 + 𝒚) (𝒙 + 𝒙 𝒚𝟐 + 𝒚𝟒 )
𝟐

(𝒙𝟐 − 𝒚𝟐 )((𝒙𝟐 )𝟐 + (𝒙𝟐 )(𝒚𝟐 ) + (𝒚𝟐 )𝟐 ) value of 𝒂𝒃


= Solution: As given 𝑎 + 𝑏 = 5 and 𝑎 − 𝑏 = √17
(𝒙 − 𝒚)(𝒙 + 𝒚)
𝟏 We find ab=?
× 𝟒
(𝒙 + 𝒙 𝒚𝟐 + 𝒚𝟒 )
𝟐 Using the identity
3|Page
Class 9th Chapter 4 www.notes.pk.com
(𝑎 + 𝑏)2 − (𝑎 − 𝑏)2 = 4𝑎𝑏 Solution: As given 𝑥 + 𝑦 + 𝑧 = 12 and𝑥 2 + 𝑦 2 +
Put values 𝑧 2 = 64
2
(5)2 − (√17) = 4𝑎𝑏 We find 𝑥𝑦 + 𝑦𝑧 + 𝑧𝑥 =?
25 − 17 = 4𝑎𝑏 Using the identity
8 = 4𝑎𝑏 (𝑥 + 𝑦 + 𝑧)2 = 𝑥 2 + 𝑦 2 + 𝑧 2 + 2(𝑥𝑦 + 𝑦𝑧 + 𝑧𝑥)
8 Put values
𝑎𝑏 = (12)2 = 64 + 2( 𝑥𝑦 + 𝑦𝑧 + 𝑧𝑥)
4
𝑎𝑏 = 2 144 = 64 + 2( 𝑥𝑦 + 𝑦𝑧 + 𝑧𝑥)
Which is required. 144 − 64 = 2( 𝑥𝑦 + 𝑦𝑧 + 𝑧𝑥)
Q#2) If 𝒂𝟐 + 𝒃𝟐 + 𝒄𝟐 = 𝟒𝟓 and 𝒂 + 𝒃 + 𝒄 = −𝟏, 80 = 2( 𝑥𝑦 + 𝑦𝑧 + 𝑧𝑥)
then find the value of 𝒂𝒃 + 𝒃𝒄 + 𝒄𝒂 80
= ( 𝑥𝑦 + 𝑦𝑧 + 𝑧𝑥)
Solution: As given 𝑎2 + 𝑏 2 + 𝑐 2 = 45 and 𝑎 + 𝑏 + 2
𝑥𝑦 + 𝑦𝑧 + 𝑧𝑥 = 40
𝑐 = −1
Which is required.
We find 𝑎𝑏 + 𝑏𝑐 + 𝑐𝑎=?
Using the identity Q#6) If 𝒙 + 𝒚 = 𝟕 and 𝒙𝒚 = 𝟏𝟐, then find the value
(𝑎 + 𝑏 + 𝑐)2 = 𝑎2 + 𝑏 2 + 𝑐 2 + 2(𝑎𝑏 + 𝑎𝑐 + 𝑐𝑎) of 𝒙𝟑 + 𝒚𝟑
Put values Solution: As given 𝑥 + 𝑦 = 7 and 𝑥𝑦 = 12
(−1)2 = 45 + 2(𝑎𝑏 + 𝑏𝑐 + 𝑐𝑎) We find 𝑥 3 + 𝑦 3 =?
1 = 45 + 2(𝑎𝑏 + 𝑏𝑐 + 𝑐𝑎) Using the identity
1 − 45 = 2(𝑎𝑏 + 𝑏𝑐 + 𝑐𝑎) (𝑥 + 𝑦)3 = 𝑥 3 + 𝑦 3 + 3𝑥𝑦(𝑥 + 𝑦)
−44 = 2(𝑎𝑏 + 𝑏𝑐 + 𝑐𝑎) Put values
44 (7)3 = 𝑥 3 + 𝑦 3 + 3(12)(7)
𝑎𝑏 + 𝑏𝑐 + 𝑐𝑎 = − 343 = 𝑥 3 + 𝑦 3 + 252
2
𝑎𝑏 + 𝑏𝑐 + 𝑐𝑎 = −22 343 − 252 = 𝑥 3 + 𝑦 3
Which is required. 𝑥 3 + 𝑦 3 = 91
Q#3) If 𝒎 + 𝒏 + 𝒑 = 𝟏𝟎 and 𝒎𝒏 + 𝒏𝒑 + 𝒎𝒑 = 𝟐𝟕, Which is required.
then find the value of 𝒎𝟐 + 𝒏𝟐 + 𝒑𝟐 Q#7) If 𝟑𝒙 + 𝟒𝒚 = 𝟏𝟏 and 𝒙𝒚 = 𝟏𝟐, then find the
Solution: As given 𝑚 + 𝑛 + 𝑝 = 10 and 𝑚𝑛 + 𝑛𝑝 + value of 𝟐𝟕𝒙𝟑 + 𝟔𝟒𝒚𝟑
𝑚𝑝 = 27 Solution: As given 3𝑥 + 4𝑦 = 11 and 𝑥𝑦 = 12
We find 𝑚2 + 𝑛2 + 𝑝2 =? We find 27𝑥 3 + 64𝑦 3 =?
Using the identity Using the identity
(𝑚 + 𝑛 + 𝑝)2 = 𝑚2 + 𝑛2 + 𝑝2 + 2(𝑚𝑛 + 𝑛𝑝 + 𝑚𝑝) (𝑥 + 𝑦)3 = 𝑥 3 + 𝑦 3 + 3𝑥𝑦(𝑥 + 𝑦)
Put values It becomes
(10)2 = 𝑚2 + 𝑛2 + 𝑝2 + 2(27) (3𝑥 + 4𝑦)3 = (3𝑥)3 + (4𝑦)3
100 = 𝑚2 + 𝑛2 + 𝑝2 + 54 + 3(3𝑥)(4𝑦)(3𝑥 + 4𝑦)
100 − 54 = 𝑚2 + 𝑛2 + 𝑝2 (3𝑥 + 4𝑦) = 27𝑥 + 64𝑦 3 + 36(𝑥𝑦)(3𝑥 + 4𝑦)
3 3

𝑚2 + 𝑛2 + 𝑝2 = 46 (11)3 = 27𝑥 3 + 64𝑦 3 + 36(12)(11)


Which is required. 1331 = 27𝑥 3 + 64𝑦 3 + 4752
Q#4) If 𝒙𝟐 + 𝒚𝟐 + 𝒛𝟐 = 𝟕𝟖 and 𝒙𝒚 + 𝒚𝒛 + 𝒛𝒙 = 𝟓𝟗, 1331 − 4752 = 27𝑥 3 + 64𝑦 3
then find the value of 𝒙 + 𝒚 + 𝒛 27𝑥 + 64𝑦 3 = −3421
3

Solution: As given 𝑥 2 + 𝑦 2 + 𝑧 2 = 78 and 𝑥𝑦 + Q#8) If 𝒙 − 𝒚 = 𝟒 and 𝒙𝒚 = 𝟐𝟏, then find the value
𝑦𝑧 + 𝑧𝑥 = 59 of 𝒙𝟑 − 𝒚𝟑
We find 𝑥 + 𝑦 + 𝑧 =? Solution: As given 𝑥 − 𝑦 = 4 and 𝑥𝑦 = 21
Using the identity We find 𝑥 3 − 𝑦 3 =?
(𝑥 + 𝑦 + 𝑧)2 = 𝑥 2 + 𝑦 2 + 𝑧 2 + 2(𝑥𝑦 + 𝑦𝑧 + 𝑧𝑥) Using the identity
Put values (𝑥 − 𝑦)3 = 𝑥 3 − 𝑦 3 − 3𝑥𝑦(𝑥 − 𝑦)
(𝑥 + 𝑦 + 𝑧)2 = 78 + 2(59) Put values
(𝑥 + 𝑦 + 𝑧)2 = 78 + 118 (4)3 = 𝑥 3 − 𝑦 3 − 3(21)(4)
(𝑥 + 𝑦 + 𝑧)2 = 196 64 = 𝑥 3 + 𝑦 3 − 252
On taking square root, we get 64 + 252 = 𝑥 3 + 𝑦 3
√(𝑥 + 𝑦 + 𝑧)2 = ±√196 𝑥 3 + 𝑦 3 = 316
𝑥 + 𝑦 + 𝑧 = ±14 Which is required value.
Which is required. Q#9) If 𝟓𝒙 − 𝟔𝒚 = 𝟏𝟑 and 𝒙𝒚 = 𝟔, then find the
Q#5) If 𝒙𝟐 + 𝒚𝟐 + 𝒛𝟐 = 𝟕𝟖 and 𝒙𝒚 + 𝒚𝒛 + 𝒛𝒙 = 𝟓𝟗, value of 𝟏𝟐𝟓𝒙𝟑 − 𝟐𝟏𝟔𝒚𝟑
then find the value of 𝒙 + 𝒚 + 𝒛 Solution: As given 5𝑥 − 6𝑦 = 13 and 𝑥𝑦 = 6

4|Page
Class 9th Chapter 4 www.notes.pk.com
We find 27𝑥 3 + 64𝑦 3 =? 1 3 1 1
Using the identity (3𝑥 + ) = (3𝑥)3 + 3
+ (3𝑥 + )
3𝑥 (3𝑥) 3𝑥
(𝑥 + 𝑦)3 = 𝑥 3 + 𝑦 3 + 3𝑥𝑦(𝑥 + 𝑦) 1 3
1 1
It becomes (3𝑥 + ) = 27𝑥 3 + 3
+ (3𝑥 + )
3𝑥 27𝑥 3𝑥
(5𝑥 − 6𝑦)3 = (5𝑥)3 − (6𝑦)3 − 3(5𝑥)(6𝑦)(5𝑥 1
− 6𝑦) (5)3 = 27𝑥 3 + + 3(5)
27𝑥 3
(5𝑥 − 6𝑦)3 = 125𝑥 3 − 216𝑦 3 − 90(𝑥𝑦)(5𝑥 − 6𝑦) 1
125 = 27𝑥 3 + + 15
(13)3 = 125𝑥 3 − 216𝑦 3 − 90(6)(13) 27𝑥 3
2197 = 125𝑥 3 − 216𝑦 3 − 7020 1
125 − 15 = 27𝑥 3 +
2197 + 7020 = 125𝑥 3 − 216𝑦 3 27𝑥 3
125𝑥 3 − 216𝑦 3 = 9217 1
27𝑥 3 + = 110
Which is required value. 27𝑥 3
𝟏
Q#13) If (𝟓𝒙 − 𝟓𝒙) = 𝟔 then find the value of
𝟏 𝟏 𝟏
Q#10) If 𝒙 + = 𝟑 then find the value of 𝒙𝟑 + 𝟏𝟐𝟓𝒙𝟑 − 𝟏𝟐𝟓𝒙𝟑
𝒙 𝒙𝟑
1 1
Solution: As given 𝑥 + 𝑥 = 3 Solution: As given 5𝑥 − 5𝑥 = 6
1 1
We find 𝑥 3 + 𝑥 3 =? We find 125𝑥 3 − 125𝑥 3 =?
Using the identity Using the identity
1 3 1 1 1 3 1 1
(𝑥 + ) = 𝑥 3 + 3 + 3 (𝑥 + ) (𝑥 − ) = 𝑥 3 − 3 − 3 (𝑥 − )
𝑥 𝑥 𝑥 𝑥 𝑥 𝑥
Put values It becomes
1 1 3 1 1
(3)3 = 𝑥 3 + 3 + 3(3) (5𝑥 − ) = (3𝑥)3 − − (5𝑥 − )
𝑥 5𝑥 (3𝑥) 3 5𝑥
1 3
27 = 𝑥 3 + 3 + 9 1 1 1
𝑥 (5𝑥 − ) = 125𝑥 3 − + (5𝑥 − )
1 5𝑥 125𝑥 3 5𝑥
27 − 9 = 𝑥 3 + 3 1
𝑥 (6)3 = 125𝑥 3 − − 3(6)
1 125𝑥 3
3 1
𝑥 + 3 = 18
𝑥 216 = 125𝑥 3 − − 18
Which is required value. 125𝑥 3
1 1 1
Q#11) If 𝑥 − 𝑥 = 7 then find the value of 𝑥 3 − 𝑥 3 216 + 18 = 125𝑥 3 −
125𝑥 3
1 1
Sol: As given 𝑥 − 𝑥 = 7 125𝑥 3 − = 234
1 125𝑥 3
We find 𝑥 3 − 𝑥 3 =? Q#15)
Using the identity (i). 𝑥 3 − 𝑦 3 − 𝑥 + 𝑦
1 3 1 1 Sol: 𝑥 3 − 𝑦 3 − 𝑥 + 𝑦
(𝑥 − ) = 𝑥 3 − 3 − 3 (𝑥 − )
𝑥 𝑥 𝑥 = (𝑥 − 𝑦)(𝑥 2 + 𝑥𝑦 + 𝑦 2 ) − (𝑥 − 𝑦)
Put values = (𝑥 − 𝑦)(𝑥 2 + 𝑥𝑦 + 𝑦 2 − 1)
1 1
(7)3 = 𝑥 3 − 3 − 3(7) (ii). 8𝑥 3 −
27𝑦 3
𝑥 1
1 Sol: 8𝑥 3 − 27𝑦3
343 = 𝑥 3 − 3 − 21
𝑥 1 3
1 = (2𝑥)3 −( )
343 + 21 = 𝑥 3 − 3 3𝑦
𝑥
1 1 1 1 2
3
𝑥 − 3 = 364 = (2𝑥 − ) ((2𝑥)2 + (2𝑥) ( ) + ( ) )
𝑥 3𝑦 3𝑦 3𝑦
𝟏
Q#12) If (𝟑𝒙 + 𝟑𝒙) = 𝟓 then find the value of 1 2𝑥 1
= (2𝑥 − ) (4𝑥 2 + + 2)
𝟏 3𝑦 3𝑦 9𝑦
𝟐𝟕𝒙𝟑 + 𝟐𝟕𝒙𝟑
1
Q#16) Find the product, using formulas.
Solution: As given3𝑥 + 3𝑥 = 5 (i). (𝒙𝟐 + 𝒚𝟐 )(𝒙𝟒 − 𝒙𝟐 𝒚𝟐 + 𝒚𝟒 )
1
We find 27𝑥 3 + 27𝑥3 =? Solution: (𝑥 2 + 𝑦 2 )(𝑥 4 − 𝑥 2 𝑦 2 + 𝑦 4 )
Using the identity = (𝑥 2 + 𝑦 2 )[(𝑥 2 )2 − (𝑥 2 )(𝑦 2 ) + (𝑦 2 )2 ]
Using identity
1 3 1 1
(𝑥 + ) = 𝑥 3 + 3 + 3 (𝑥 + ) 𝑥 3 + 𝑦 3 = (𝑥 + 𝑦)(𝑥 2 − 𝑥𝑦 + 𝑦 2 )
𝑥 𝑥 𝑥
= (𝑥 2 )3 + (𝑦 2 )3
It becomes
= 𝑥6 + 𝑦6
5|Page
Class 9th Chapter 4 www.notes.pk.com
(ii). (𝑥 3 − 𝑦 3 )(𝑥 6 + 𝑥 3 𝑦 3 + 𝑦 6 ) Surd:
Sol: (𝑥 3 − 𝑦 3 )(𝑥 6 + 𝑥 3 𝑦 3 + 𝑦 6 ) An irrational radical with rational radicand is called a
= (𝑥 3 − 𝑦 3 )[(𝑥 3 )2 + (𝑥 3 )(𝑦 3 ) + (𝑦 3 )2 ] surd.
𝑛 𝑛
Using identity That is √𝑎 surd if 𝑎 is rational and √𝑎 is irrational.
𝑥 3 − 𝑦 3 = (𝑥 − 𝑦)(𝑥 2 + 𝑥𝑦 + 𝑦 2 ) 4
For example, √2,√5, √10
= (𝑥 3 )3 − (𝑦 3 )3 Also, √𝜋 is not a surd because 𝜋 is not rational.
= 𝑥9 − 𝑦9 √10 + √2 is not a surd because 10 + √2 is not a
(iii). (𝑥 − 𝑦)(𝑥 + 𝑦)(𝑥 2 + 𝑦 2 )(𝑥 2
rational number.
+𝑥𝑦 + 𝑦 2 )(𝑥 2
−𝑥𝑦 + 𝑦 2 )(𝑥 4 − 𝑥 2 𝑦 2 + 𝑦 4 )
Sol: (𝑥 − 𝑦)(𝑥 + 𝑦)(𝑥 2 + 𝑦 2 )(𝑥 2 EXERCISE 4.3
+𝑥𝑦 + 𝑦 2 )(𝑥 2 1. Express each of the following surd in the simplest
−𝑥𝑦 + 𝑦 2 )(𝑥 4 − 𝑥 2 𝑦 2 + 𝑦 4 ) foam.
= [(𝑥 − 𝑦)(𝑥 2 + 𝑥𝑦 + 𝑦 2 )][(𝑥 + 𝑦)(𝑥 2 (i) √𝟏𝟖𝟎
−𝑥𝑦 + 𝑦 2 )][(𝑥 2 + 𝑦 2 )((𝑥 2 )2 − (𝑥 2 )(𝑦 2 ) + (𝑦 2 )2 )] Solution: √180 = √2 × 2 × 3 × 3 × 5
Using identity = √22 × 32 × 5
𝑥 3 + 𝑦 3 = (𝑥 + 𝑦)(𝑥 2 − 𝑥𝑦 + 𝑦 2 ) = 2 × 3√5
= [𝑥 3 − 𝑦 3 ][𝑥 3 + 𝑦 3 ][(𝑥 2 )3 + (𝑦 2 )3 ]
= 6√5
= [(𝑥 3 )2 − (𝑦 3 )2 ][𝑥 6 + 𝑦 6 ]
(ii) 𝟑√𝟏𝟔𝟐
= (𝑥 − 𝑦 )(𝑥 6 + 𝑦 6 )
6 6

= (𝑥 6 )2 − (𝑦 6 )2 Solution: 3√162 = 3√2 × 3 × 3 × 3 × 3


= 𝑥 12 − 𝑦12 = 3√2 × 32 × 32
(iv). (2𝑥 2 − 1)(2𝑥 2 + 1)(4𝑥 4 + 2𝑥 2 + 1)(4𝑥 4 − = 3 × 3 × 3√2
2𝑥 2 + 1) = 27√2
Sol: (2𝑥 2 − 1)(2𝑥 2 + 1)(4𝑥 4 + 2𝑥 2 + 1)(4𝑥 4 − 𝟑𝟑
(iii) 𝟒 √𝟏𝟐𝟖
2𝑥 2 + 1) 33 33
= [(2𝑥 2 − 1)((2𝑥 2 )2 + (2𝑥 2 )(1) + (1)2 ][(2𝑥 2 Solution: 4 √128 = 4 √2 × 2 × 2 × 2 × 2 × 2 × 2
+ 1)((2𝑥 2 )2 − (2𝑥 2 )(1) + (1)2 ] 33
2 )3
= √23 × 23 × 2
= [(2𝑥 − (1)3 ][(2𝑥 2 )3 + (1)3 ] 4
= (8𝑥 6 − 1)(8𝑥 6 + 1) 3 3
= (2 × 2√2)
= (8𝑥 6 )2 − (1)2 4
3 3
= 64𝑥 12 − 1 = (4√2)
4
𝟑
= 𝟑√𝟐
𝟓
(iv) √𝟗𝟔𝒙𝟔 𝒚𝟕 𝒛𝟖
5
Solution: √96𝑥 6 𝑦 7 𝑧 8
5
= √2 × 2 × 2 × 2 × 2 × 3 × 𝑥 5 × 𝑦 5 × 𝑧 5 × 𝑥 × 𝑦 2 × 𝑧 3
5
= √25 × 𝑥 5 × 𝑦 5 × 𝑧 5 × 3 × 𝑥 × 𝑦 2 × 𝑧 3
5
= 2 × 𝑥 × 𝑦 × 𝑧 √3 × 𝑥 × 𝑦 2 × 𝑧 3
5
= 2𝑥𝑦𝑧 √3𝑥𝑦 2 𝑧 3
Q#2) Simplify
√18
(i).
√3√2
√18 √2×3×3 √2√3√3
Solution: = = = √3
√3√2 √3√2 √3√2
√21√9
(ii).
√63
√21√9 √3×7√3×3 √3√7√3√3
Solution: = 3×3×7 = = √3
√63 √ √3√3√7
5 5 10 15
(iii). √243𝑥 𝑦 𝑧
5
Solution: √243𝑥 5 𝑦10 𝑧15
5
= √3 × 3 × 3 × 3 × 3 × 𝑥 5 × 𝑦 5 × 𝑦 5 × 𝑧 5 × 𝑧 5 × 𝑧 5
5
= √35 × 𝑥 5 × 𝑦 5 × 𝑦 5 × 𝑧 5 × 𝑧 5 × 𝑧 5
1
= (35 × 𝑥 5 × 𝑦 5 × 𝑦 5 × 𝑧 5 × 𝑧 5 × 𝑧 5 )5

6|Page
Class 9th Chapter 4 www.notes.pk.com
1 1 1 1 1 1 1 2
= 35×5 × 𝑥 5×5 × 𝑦 5×5 × 𝑦 5×5 × 𝑧 5×5 × 𝑧 5×5 × 𝑧 5×5 Solution: (√5 + √3)
2 2
=3×𝑥×𝑦×𝑦×𝑧×𝑧×𝑧 = (√5) + (√3) + 2(√5)(√3)
= 3𝑥𝑦 2 𝑧 3
43 = 5 + 3 + 2√3 × 5 = 8 + 2√15
(iv) 5 √125 (iii). (√5 + √3)(√5 − √3)
43 43
Solution: 5 √1258 = 5 √5 × 5 × 5 Solution: (√5 + √3)(√5 − √3)
43 2 2
= √53 = (√5) − (√3)
5 =5−3=2
4 1 1
= (5) (iv). (√2 + 3)(√2 − )
5 √ √3
=4 Solution: (√2 +
1 1
)(√2 − 3)
√3 √
(v). √21 × √7 × √3
2 1 2
Solution: √21 × √7 × √3 = √7 × 3 × √7 × √3 = (√2) − ( )
= √7 × √3 × √7 × √3 √3
2 2
1 6−1 5
= (√7) × (√3) =2− = =
3 3 3
= 7 × 3 = 21 (i). (√𝑥 + √𝑦)(√𝑥 − √𝑦)(𝑥 + 𝑦)(𝑥 2 + 𝑦 2 )
Q#3) Simplify by combining similar terms. Solution: (√𝑥 + √𝑦)(√𝑥 − √𝑦)(𝑥 + 𝑦)(𝑥 2 + 𝑦 2 )
(i). √𝟒𝟓 − 𝟑√𝟐𝟎 + 𝟒√𝟓 2 2
= ((√𝑥) − (√𝑦) ) (𝑥 + 𝑦)(𝑥 2 + 𝑦 2 )
Solution: √45 − 3√20 + 4√5 = √3 × 3 × 5 −
3√2 × 2 × 5 + 4√5 = (𝑥 − 𝑦)(𝑥 + 𝑦)(𝑥 2 + 𝑦 2 )
= ((𝑥)2 − (𝑦)2 )(𝑥 2 + 𝑦 2 )
= √32 × 5 − 3√22 × 5 + 4√5
= (𝑥 2 )2 − (𝑦 2 )2
= 3√5 − 3 × 2√5 + 4√5 = 𝑥4 − 𝑦4
= 3√5 − 6√5 + 4√5
= √5(3 − 6 + 4)
= √5(1)
= √5
(ii). 4√12 + 5√27 − 3√75 + √300
Solution: 4√12 + 5√27 − 3√75 + √300
= 4√2 × 2 × 3 + 5√3 × 3 × 3 − 3√5 × 5 × 3
+ √2 × 2 × 5 × 5 × 3
= 4√22 × 3 + 5√32 × 3 − 3√52 × 3
+ √22 × 52 × 3
= 4 × 2√3 + 5 × 3√3 − 3 × 5√3 + 2 × 5√3
= 8√5 + 15√5 − 15√5 + 10√3
= √3(8 + 15 − 15 + 10)
= √5(18)
= 18√5
(iii). √3(2√3 + 3√3)
Solution: √3(2√3 + 3√3) = √3(5√3)
2
= 5(√3) = 5(3) = 15
(iv). 2(6√5 − 3√5)
Solution: 2(6√5 − 3√5) = 2(3√5)
= 6√5
Q#4) Simplify
(i). (3 + √3)(3 − √3)
Sol: (3 + √3)(3 − √3)
2
= (3)2 − (√3)
=9−3=6
2
(ii). (√5 + √3)

7|Page
Class 9th Chapter 4 www.notes.pk.com
Surd: 𝟑√𝟑
An irrational radical with rational radicand is called a =
𝟒×𝟑
surd. √𝟑
𝑛 𝑛
That is √𝑎 surd if 𝑎 is rational and √𝑎 is irrational. =
𝟒
4 𝟏𝟒
For example, √2,√5, √10 (ii)
√𝟗𝟖
Also, √𝜋 is not a surd because 𝜋 is not rational. 𝟏𝟒 𝟏𝟒 √𝟗𝟖
Solution: = ×
√10 + √2 is not a surd because 10 + √2 is not a √𝟗𝟖 √𝟗𝟖 √𝟗𝟖
rational number. 𝟏𝟒√𝟗𝟖
= 𝟐
Monomial surd: (√𝟗𝟖)
A surd which contains a single term is called a
𝟏𝟒√𝟗𝟖
monomial surd. =
𝟗𝟖
e.g., √2, √5 etc.
√𝟗𝟖
Binomial surd: =
A surd which contains sum of two monomial surds or 𝟕
𝟔
sum of a monomial surd and a rational number is (iii)
√𝟖√𝟐𝟕
called a binomial surd. 𝟔 𝟔 𝟔 √𝟐𝟏𝟔
Solution: = = ×
√𝟖√𝟐𝟕 √𝟐𝟏𝟔 √𝟐𝟏𝟔 √𝟐𝟏𝟔
e.g., √2 + √7 + or √12 − √7 or √10 − √2 etc.
We can extend this to the definition of a trinomial 𝟔√𝟐𝟏𝟔
= 𝟐
surd. (√𝟐𝟏𝟔)
Rationalizing factor of the other 𝟔√𝟔 × 𝟔 × 𝟔
If the product of two surds is a rational number, then =
𝟐𝟏𝟔
each surd is called the rationalizing factor of the other. 𝟔 × 𝟔√𝟔
Rationalization =
𝟐𝟏𝟔
The process of multiplying a given surd by its
√𝟔
rationalizing factor to get a rational number as product =
is called rationalization of the given surd. 𝟔
𝟏
Conjugate surd (iv)
𝟑+𝟐√𝟓
Two binomial surds of second order differing only in 𝟏 𝟏 𝟑−𝟐√𝟓
Solution: = ×
sign connecting their terms are called conjugate surds. 𝟑+𝟐√𝟓 𝟑+𝟐√𝟓 𝟑−𝟐√𝟓

Thus (√𝑎 + √𝑏) and (√𝑎 − √𝑏) are conjugate surds 𝟑 − 𝟐√𝟓
= 𝟐
of each other. (𝟑)𝟐 − (𝟐√𝟓)
The conjugate of 𝑥 + √𝑦 is 𝑥 − √𝑦. 𝟑 − 𝟐√𝟓
The product of the conjugate surds √𝒙 + √𝒚and =
𝟗 − (𝟒 × 𝟓)
√𝒙 − √𝒚, 𝟑 − 𝟐√𝟓
𝟐 𝟐 =
(√𝒙 + √𝒚)(√𝒙 − √𝒚) = (√𝒙) − (√𝒚) = 𝒙 − 𝒚 𝟗 − 𝟐𝟎
𝟑 − 𝟐√𝟓
is a rational quantity independent of any radical. =
Similarly, the product of 𝒙 + 𝒎√𝒚 nd its conjugate −𝟏𝟏
𝟏
𝒙 − 𝒎√𝒚 has =− (𝟑 − 𝟐√𝟓)
𝟏𝟏
𝟐 𝟏𝟓
(𝒙 + 𝒎√𝒚)(𝒙 − 𝒎√𝒚) = (𝒙)𝟐 − (𝒎√𝒚) (v)
√𝟑𝟏−𝟒
= 𝒙𝟐 − 𝒎𝟐 𝒚 Solution:
𝟏𝟓
=
𝟏𝟓
×
√𝟑𝟏+𝟒
and have no radical. For example, √𝟑𝟏−𝟒 √𝟑𝟏−𝟒 √𝟑𝟏+𝟒
𝟐 𝟏𝟓(√𝟑𝟏 + 𝟒)
(𝟒 + √𝟑)(𝟒 − √𝟑) = (𝟒)𝟐 − (√𝟑) = 𝟏𝟔 − 𝟑 = 𝟏𝟑 = 𝟐
, which is a rational number. (√𝟑𝟏) − (𝟒)𝟐
𝟏𝟓(√𝟑𝟏 + 𝟒)
=
EXERCISE 4.4 𝟑𝟏 − 𝟏𝟔
𝟏𝟓(√𝟑𝟏 + 𝟒)
1. Rationalize the denominator of the following. =
𝟑 𝟏𝟓
(i) 𝟒√𝟑
= √𝟑𝟏 + 𝟒
𝟑 𝟑 √𝟑 𝟐
Sol: 𝟒√𝟑 = 𝟒√𝟑 × (vi)
√𝟑 √𝟓−√𝟑
𝟑√𝟑 Solution:
𝟐
=
𝟐
×
√𝟓+√𝟑
= 𝟐 √𝟓−√𝟑 √𝟓−√𝟑 √𝟓+√𝟑
𝟒(√𝟑)

8|Page
Class 9th Chapter 4 www.notes.pk.com
𝟐(√𝟓 + √𝟑) Taking conjugate, we get
= 𝟐
(√𝟓) − (√𝟑)
𝟐 𝒛̅ = ̅̅̅̅̅̅̅̅̅
𝟐 + √𝟑
𝒛̅ = 𝟐 − √𝟑
𝟐(√𝟓 + √𝟑)
= (iv). 𝟐 + √𝟓
𝟓−𝟑
𝟐(√𝟓 + √𝟑) Solution: Let 𝒛 = 𝟐 + √𝟓
= Taking conjugate, we get
𝟐
̅̅̅̅̅̅̅̅̅̅
= √𝟓 + √𝟑 𝒛̅ = 𝟐 + √𝟓
𝒛̅ = 𝟐 − √𝟓
√𝟑−𝟏 (v). 𝟓 + √𝟕
(vi)
√𝟑+𝟏
√𝟑−𝟏 √𝟑−𝟏 √𝟑−𝟏 Solution: Let 𝒛 = 𝟓 + √𝟕
Solution: = × Taking conjugate, we get
√𝟑+𝟏 √𝟑+𝟏 √𝟑−𝟏
𝒛̅ = ̅̅̅̅̅̅̅̅̅
𝟐
(√𝟑 − 𝟏) 𝟓 + √𝟕
= 𝟐 𝒛̅ = 𝟓 − √𝟕
(√𝟑) − (𝟏)𝟐
𝟐 (vi). 𝟒 − √𝟏𝟓
(√𝟑) + (𝟏)𝟐 − 𝟐(√𝟑)(𝟏)
= Solution: Let 𝒛 = 𝟒 − √𝟏𝟓
𝟑−𝟏 Taking conjugate, we get
𝟑 + 𝟏 − 𝟐√𝟑 ̅̅̅̅̅̅̅̅̅̅̅
= 𝒛̅ = 𝟒 − √𝟏𝟓
𝟐
𝟒 − 𝟐√𝟑 𝒛̅ = 𝟒 + √𝟏𝟓
= (vii). 𝟕 − √𝟔
𝟐
𝟐(𝟐 − √𝟑) Solution: Let 𝒛 = 𝟕 − √𝟔
= Taking conjugate, we get
𝟐
= 𝟐 − √𝟑 𝒛̅ = ̅̅̅̅̅̅̅̅̅
𝟕 − √𝟔
√𝟓+√𝟑 𝒛̅ = 𝟕 + √𝟔
(vi)
√𝟓−√𝟑
√𝟓+√𝟑 √𝟓+√𝟑 √𝟓+√𝟑
(viii). 𝟗 + √𝟐
Sol: = × Solution: Let 𝒛 = 𝟗 + √𝟐
√𝟓−√𝟑 √𝟓−√𝟑 √𝟓+√𝟑
𝟐
(√𝟓 + √𝟑) Taking conjugate, we get
= 𝟐 𝟐 𝒛̅ = ̅̅̅̅̅̅̅̅̅
𝟗 + √𝟐
(√𝟓) − (√𝟑)
𝟐 𝟐 𝒛̅ = 𝟗 − √𝟐
(√𝟓) + (√𝟑) + 𝟐(√𝟓)(√𝟑) Q#3)
=
𝟓−𝟑 (i). If 𝒙 = 𝟐 − √𝟑 find 𝒙
𝟏
𝟓 + 𝟑 + 𝟐√𝟏𝟓
= Solution: 𝒙 = 𝟐 − √𝟑
𝟐
𝟏 𝟏 𝟏 𝟐+√𝟑
𝟖 + 𝟐√𝟏𝟓 And 𝒙 = 𝟐−√𝟑 = 𝟐−√𝟑 × 𝟐+√𝟑
=
𝟐 𝟐 + √𝟑
𝟐(𝟒 + √𝟏𝟓) = 𝟐
= (𝟐)𝟐 − (√𝟑)
𝟐
= 𝟒 + √𝟏𝟓 𝟐 + √𝟑
=
Q#2) Find the conjugate of 𝒙 + √𝒚. 𝟒−𝟑
𝟐 + √𝟑
(i). 𝟑 + √𝟕 =
Solution: Let 𝒛 = 𝟑 + √𝟕 𝟏
= 𝟐 + √𝟑
Taking conjugate, we get 𝟏
(ii). If 𝒙 = 𝟒 − √𝟏𝟕 find 𝒙
𝒛̅ = ̅̅̅̅̅̅̅̅̅
𝟑 + √𝟕
𝒛̅ = 𝟑 − √𝟕 Solution: 𝒙 = 𝟒 − √𝟏𝟕
𝟏 𝟏 𝟏 𝟒+√𝟏𝟕
(ii). 𝟒 − √𝟓 And 𝒙 = 𝟒−√𝟏𝟕 = 𝟒−√𝟏𝟕 × 𝟒+√𝟏𝟕
Solution: Let 𝒛 = 𝟒 − √𝟓 𝟒 + √𝟏𝟕
Taking conjugate, we get = 𝟐
̅̅̅̅̅̅̅̅̅ (𝟒)𝟐 − (√𝟏𝟕)
𝒛̅ = 𝟒 − √𝟓
𝟒 + √𝟏𝟕
𝒛̅ = 𝟒 + √𝟓 =
𝟏𝟔 − 𝟏𝟕
(iii). 𝟐 + √𝟑
𝟒 + √𝟏𝟕
Solution: Let 𝒛 = 𝟐 + √𝟑 =
−𝟏
9|Page
Class 9th Chapter 4 www.notes.pk.com
= −(𝟒 + √𝟏𝟕) (𝟐 − √𝟑) 𝟐(√𝟓 + √𝟑) (𝟐 − √𝟓)
= 𝟐
+ 𝟐 𝟐
+ 𝟐
= −𝟒 − √𝟏𝟕 (𝟐)𝟐 − (√𝟑) (√𝟓) − (√𝟑) (𝟐)𝟐 − (√𝟓)
𝟏
(iii). If 𝒙 = √𝟑 + 𝟐 find 𝒙 (𝟐 − √𝟑) 𝟐(√𝟓 + √𝟑) (𝟐 − √𝟓)
= + +
Solution: 𝒙 = √𝟑 + 𝟐 𝟒−𝟑 𝟓−𝟑 𝟒−𝟓
And 𝒙 =
𝟏 𝟏
=
𝟏
×
√𝟑−𝟐 (𝟐 − √𝟑) 𝟐(√𝟓 + √𝟑) (𝟐 − √𝟓)
√𝟑+𝟐 √𝟑+𝟐 √𝟑−𝟐 = + +
𝟏 𝟐 −𝟏
√𝟑 − 𝟐 = (𝟐 − √𝟑) + (√𝟓 + √𝟑) − (𝟐 − √𝟓)
= 𝟐
(√𝟑) − (𝟐)𝟐 = 𝟐 − √𝟑 + √𝟓 + √𝟑 − 𝟐 + √𝟓
√𝟑 − 𝟐 = 𝟐√𝟓
= 𝟐 𝟏 𝟑
𝟑−𝟒 (iii) + −
√𝟓+√𝟑 √𝟑+√𝟐 √𝟓+√𝟐
√𝟑 − 𝟐
= Solution:
−𝟏 𝟐 𝟏 𝟑
= −(√𝟑 − 𝟐) + −
√𝟓 + √𝟑 √𝟑 + √𝟐 √𝟓 + √𝟐
= −√𝟑 + 𝟐 = 𝟐 − √𝟑
𝟐 √𝟓 − √𝟑 𝟏 √𝟑 − √𝟐
Q#4) Simplify =( × )+( × )
𝟏+√𝟐 𝟏−√𝟐 √𝟓 + √𝟑 √𝟓 − √𝟑 √𝟑 + √𝟑 √𝟑 − √𝟐
(vi) +
√𝟓+√𝟑 √𝟓−√𝟑 𝟑 √𝟓 − √𝟐
Solution: −( × )
√𝟓 + √𝟑 √𝟓 − √𝟐
𝟏 + √𝟐 𝟏 − √𝟐
+ 𝟐(√𝟓 − √𝟑) (√𝟑 − √𝟐)
√𝟓 + √𝟑 √𝟓 − √𝟑 = 𝟐 𝟐
+ 𝟐 𝟐
𝟏 + √𝟐 √𝟓 − √𝟑 (√𝟓) − (√𝟑) (√𝟑) − (√𝟐)
=( × ) 𝟑(√𝟓 − √𝟐)
√𝟓 + √𝟑 √𝟓 − √𝟑 − 𝟐 𝟐
𝟏 − √𝟐 √𝟓 + √𝟑 (√𝟓) − (√𝟐)
+( × )
√𝟓 − √𝟑 √𝟓 + √𝟑 𝟐(√𝟓 − √𝟑) (√𝟑 − √𝟐) 𝟑(√𝟓 − √𝟐)
= + −
(𝟏 + √𝟐)(√𝟓 − √𝟑) (𝟏 − √𝟐)(√𝟓 + √𝟑) 𝟓−𝟑 𝟑−𝟐 𝟓−𝟐
= 𝟐 𝟐
+ 𝟐 𝟐 𝟐(√𝟓 − √𝟑) (√𝟑 − √𝟐) 𝟑(√𝟓 − √𝟐)
(√𝟓) − (√𝟑) (√𝟓) − (√𝟑) = + −
𝟐 𝟏 𝟑
[(𝟏)(√𝟓) − (𝟏)(√𝟑) + (√𝟐)(√𝟓) − (√𝟐)(√𝟑)] = (√𝟓 − √𝟑) + (√𝟑 − √𝟐) − (√𝟓 − √𝟐)
=
𝟓−𝟑
= √𝟓 − √𝟑 + √𝟑 − √𝟐 − √𝟓 + √𝟐
[(𝟏)(√𝟓) + (𝟏)( √𝟑) − (√𝟐)(√𝟓) − (√𝟐)(√𝟑)]
+ =𝟎
𝟓−𝟑 Q#5)
[√𝟓−√𝟑+√𝟏𝟎−√𝟔] [√𝟓+√𝟑−√𝟏𝟎−√𝟔]
= 𝟐
+ 𝟐 𝟏 𝟏 𝟐
(i). If 𝒙 = 𝟐 + √𝟑 find 𝒙 − 𝒙 and (𝒙 − 𝒙)

(√𝟓 − √𝟑 + √𝟏𝟎 − √𝟔) + (√𝟓 + √𝟑 − √𝟏𝟎 − √𝟔) Solution: 𝒙 = 𝟐 + √𝟑


= 𝟏 𝟏 𝟏 𝟐−√𝟑
𝟐 And 𝒙 = 𝟐+√𝟑 = 𝟐+√𝟑 × 𝟐−√𝟑
√𝟓 − √𝟑 + √𝟏𝟎 − √𝟔 + √𝟓 + √𝟑 − √𝟏𝟎 − √𝟔 𝟐 − √𝟑
= =
𝟐 𝟐
𝟐√𝟓 − 𝟐√𝟔 (𝟐)𝟐 − (√𝟑)
= 𝟐 − √𝟑
𝟐 =
𝟐(√𝟓 − √𝟔) 𝟒−𝟑
= 𝟐 − √𝟑
𝟐 =
= √𝟓 − √𝟔 𝟏
𝟏
(ii) 𝟐+√𝟑 +
𝟐
+
𝟏 = 𝟐 − √𝟑
√𝟓−√𝟑 𝟐+√𝟓 𝟏
Solution: Now, 𝒙 − 𝒙 = (𝟐 + √𝟑) − (𝟐 − √𝟑)
𝟏 𝟐 𝟏 = 𝟐 + √𝟑 − 𝟐 + √𝟑
+ + 𝟏
𝟐 + √𝟑 √𝟓 − √𝟑 𝟐 + √𝟓 𝒙 − = 𝟐√𝟑
𝟏 𝟐 − √𝟑 𝟐 √𝟓 + √𝟑 𝒙
=( × )+( × ) 𝟏 𝟐 𝟐
𝟐 + √𝟑 𝟐 − √𝟑 √𝟓 − √𝟑 √𝟓 + √𝟑 Also, (𝒙 − 𝒙) = (𝟐√𝟑) = 𝟒 × 𝟑 = 𝟏𝟐
𝟏 𝟐 − √𝟓 √𝟓−√𝟐 𝟏 𝟏 𝟏
+( × ) (i). If 𝒙 = find 𝒙 + 𝒙 , 𝒙𝟐 + 𝒙𝟐 and 𝒙𝟑 + 𝒙𝟑
√𝟓+√𝟐
𝟐 + √𝟓 𝟐 − √𝟓 √𝟓−√𝟐 √𝟓−√𝟐
Solution: 𝒙 = ×
√𝟓+√𝟐 √𝟓−√𝟐

10 | P a g e
Class 9th Chapter 4 www.notes.pk.com
𝟐
(√𝟓 − √𝟐)
= 𝟐 𝟐 √𝟑 − 𝟏 √𝟑 − 𝟏 √𝟑 + 𝟏 √𝟑 + 𝟏
(√𝟓) − (√𝟐) ( × ) +( × )
𝟐 𝟐 √𝟑 + 𝟏 √𝟑 − 𝟏 √𝟑 − 𝟏 √𝟑 + 𝟏
(√𝟓) + (√𝟐) − 𝟐(√𝟓)(√𝟐) = 𝒂 + √𝟑𝒃
=
𝟓−𝟐 𝟐 𝟐
𝟓 + 𝟐 − 𝟐√𝟏𝟎 (√𝟑 − 𝟏) (√𝟑 − 𝟏)
= ( 𝟐
)+( 𝟐
) = 𝒂 + √𝟑𝒃
𝟑 (√𝟑) − (𝟏)𝟐 (√𝟑) − (𝟏)𝟐
𝟏
= (𝟕 − 𝟐√𝟏𝟎)
𝟑 𝟐
𝟏 √𝟓+√𝟐 √𝟓+√𝟐 √𝟓+√𝟐 (√𝟑) + (𝟏)𝟐 − 𝟐(√𝟑)(𝟏)
And = = ×
𝒙 √𝟓−√𝟐 √𝟓−√𝟐 √𝟓+√𝟐 𝟑−𝟏
𝟐 𝟐
(√𝟓 + √𝟐) (√𝟑) + (𝟏)𝟐 + 𝟐(√𝟑)(𝟏)
= 𝟐 𝟐 +
(√𝟓) − (√𝟐) 𝟑−𝟏
𝟐 𝟐 = 𝒂 + √𝟑𝒃
(√𝟓) + (√𝟐) + 𝟐(√𝟓)(√𝟐) 𝟑 + 𝟏 + 𝟐√𝟑 𝟑 + 𝟏 − 𝟐√𝟑
= + = 𝒂 + √𝟑𝒃
𝟓−𝟐 𝟐 𝟐
𝟓 + 𝟐 + 𝟐√𝟏𝟎 𝟒 + 𝟐√𝟑 + 𝟒 − 𝟐√𝟑
=
𝟑 = 𝒂 + √𝟑𝒃
𝟏 𝟐
= (𝟕 + 𝟐√𝟏𝟎) 𝟖
𝟑 = 𝒂 + √𝟑𝒃
𝟏 𝟏 𝟏 𝟐
Now, 𝒙 + 𝒙 = 𝟑 (𝟕 − 𝟐√𝟏𝟎) + 𝟑 (𝟕 + 𝟐√𝟏𝟎) 𝟒 + 𝟎√𝟑 = 𝒂 + √𝟑𝒃
𝟏 On comparing, we get
= (𝟕 − 𝟐√𝟏𝟎 + 𝟕 + 𝟐√𝟏𝟎)
𝟑 𝒂 = 𝟒 and 𝒃 = 𝟎

Contact or Suggest Us: info@notespk.com


A project of: www.notespk.com
𝟏 𝟏𝟒
𝒙+ =
𝒙 𝟑
Using identity
𝟏 𝟐 𝟏
(𝒙 + ) = 𝒙𝟐 + 𝟐 + 𝟐
𝒙 𝒙
Putting values
𝟏𝟒 𝟐 𝟏
( ) = 𝒙𝟐 + 𝟐 + 𝟐
𝟑 𝒙
𝟏𝟗𝟔 𝟏
= 𝒙𝟐 + 𝟐 + 𝟐
𝟗 𝒙
𝟏 𝟏𝟗𝟔
𝒙𝟐 + 𝟐 = −𝟐
𝒙 𝟗
𝟏 𝟏𝟗𝟔 − 𝟏𝟖
𝒙𝟐 + 𝟐 =
𝒙 𝟗
𝟐
𝟏 𝟏𝟕𝟖
𝒙 + 𝟐=
𝒙 𝟗
Also using the identity
𝟏 𝟑 𝟏 𝟏
(𝒙 + ) = 𝒙𝟑 + 𝟑 + 𝟑 (𝒙 + )
𝒙 𝒙 𝒙
Putting values
𝟏𝟒 𝟑 𝟏 𝟏𝟒
( ) = 𝒙𝟑 + 𝟑 + 𝟑 ( )
𝟑 𝒙 𝟑
𝟐𝟕𝟒𝟒 𝟑 𝟏
𝟐𝟕
= 𝒙 + 𝒙𝟑
+ 𝟏𝟒
𝟏 𝟐𝟕𝟒𝟒
𝒙𝟑 + 𝟑 = − 𝟏𝟒
𝒙 𝟐𝟕
𝟏 𝟐𝟑𝟔𝟔
𝒙𝟑 + 𝟑 = Complied by Shumaila Amin
𝒙 𝟐𝟕
Q#6) Determine the rational numbers 𝒂 and 𝒃 if,
√𝟑−𝟏 √𝟑+𝟏
+ = 𝒂 + √𝟑𝒃
√𝟑+𝟏 √𝟑−𝟏
Solution:
√𝟑 − 𝟏 √𝟑 + 𝟏
+ = 𝒂 + √𝟑𝒃
√𝟑 + 𝟏 √𝟑 − 𝟏
11 | P a g e
MATHEMATICS 9th Science Group

7/18/2020
Chapter 5.
Factorization

A project of: www.notespk.com


Contact or Suggest Us: info@notespk.com
Contents
Exercise 5.1 .............................................................. 1
Exercise 5.2 .............................................................. 4
Exercise 5.3 .............................................................. 6
EXERCISE 5.4 .......................................................... 10
Class 9th Chapter 5 www.notes.pk.com

Introduction: (v). 𝟑𝒙𝟑 𝒚(𝒙 − 𝟑𝒚) − 𝟕𝒙𝟐 𝒚𝟐 (𝒙 − 𝟑𝒚)


Factorization plays an important role in Solution.
mathematics as it helps to reduce the study of 𝟑𝒙𝟑 𝒚(𝒙 − 𝟑𝒚) − 𝟕𝒙𝟐 𝒚𝟐 (𝒙 − 𝟑𝒚)
complicated expressions to the study of simpler = 𝒙𝟐 𝒚(𝒙 − 𝟑𝒚)(𝟑𝒙 − 𝟕𝒚)
expressions. In this unit, we will deal different Answer.
types of factorization of polynimials.
Factorization: (vi). 2𝑥𝑦 3 (𝑥 2 + 5) + 8𝑥𝑦 2 (𝑥 2 + 5)
If a polynomial 𝒑(𝒙) 𝒄𝒂𝒏 be expressed as Solution.
𝒑(𝒙) = 𝒈(𝒙)𝒉(𝒙), then each of the polynomial 2𝑥𝑦 3 (𝑥 2 + 5) + 8𝑥𝑦 2 (𝑥 2 + 5) = 2𝑥𝑦 2 (𝑥 2 +
𝒈(𝒙)𝒂𝒏𝒅 𝒉(𝒙) is called a factor of 𝒇(𝒙). 5)(𝑦 + 4)
Factorization of the expression of the type 𝒌𝒂 + Answer.
𝒌𝒃 + 𝒌𝒄. Question.2.
We will take common k from every term of the (i). 5𝑎𝑥 − 3𝑎𝑦 − 5𝑏𝑥 + 3𝑏𝑦
expression Solution.
𝒌𝒂 + 𝒌𝒃 + 𝒌𝒄 = 𝒌(𝒂 + 𝒃 + 𝒄) 5𝑎𝑥 − 3𝑎𝑦 − 5𝑏𝑥 + 3𝑏𝑦
Factorization of the type 𝒂𝒄 + 𝒂𝒅 + 𝒃𝒄 + 𝒃𝒅 = 𝑎(5𝑥 − 3𝑦) − 𝑏(5𝑥 − 3𝑦)
𝒂𝒄 + 𝒂𝒅 + 𝒃𝒄 + 𝒃𝒅 = 𝒂(𝒄 + 𝒅) + 𝒃(𝒄 + 𝒅) = (5𝑥 − 3𝑦)(𝑎 − 𝑏)
= (𝒄 + 𝒅)(𝒂 + 𝒃) Answer.
Factorization of the type 𝒂𝟐 ± 𝟐𝒂𝒃 + 𝒃𝟐 (ii). 3𝑥𝑦 + 2𝑦 − 12𝑥 − 8
(𝒊). 𝒂𝟐 + 𝟐𝒂𝒃 + 𝒃𝟐 = (𝒂 + 𝒃)𝟐 Solution.
= (𝒂 + 𝒃)(𝒂 + 𝒃) 3𝑥𝑦 + 2𝑦 − 12𝑥 − 8 = 𝑦(3𝑥 + 2) − 4(3𝑥 + 2)
(𝒊𝒊). 𝒂𝟐 − 𝟐𝒂𝒃 + 𝒃𝟐 = (𝒂 − 𝒃)𝟐 = (3𝑥 + 2)(𝑦 − 4)
= (𝒂 − 𝒃)(𝒂 − 𝒃) Answer.
Factorization of the type 𝒂𝟐 − 𝒃𝟐 (iii). 𝑥 3 + 3𝑥𝑦 2 − 2𝑥 2 𝑦 − 6𝑦 3
𝒂𝟐 − 𝒃𝟐 = (𝒂 + 𝒃)(𝒂 − 𝒃) Solution.
Factorization of the type 𝒂𝟐 ± 𝟐𝒂𝒃 + 𝒃𝟐 − 𝒄𝟐 𝑥 3 + 3𝑥𝑦 2 − 2𝑥 2 𝑦 − 6𝑦 3
𝒂𝟐 ± 𝟐𝒂𝒃 + 𝒃𝟐 − 𝒄𝟐 = (𝒂 ± 𝒃)𝟐 − 𝒄𝟐 = 𝑥(𝑥 2 + 3𝑦 2 ) − 2𝑦(𝑥 2 + 3𝑦 2 )
= (𝒂 ± 𝒃 + 𝒄)(𝒂 ± 𝒃 − 𝒄) = (𝑥 2 + 3𝑦 2 )(𝑥 − 2𝑦)
Answer.
Exercise 5.1 (iv). (𝑥 2 − 𝑦 2 )𝑧 + (𝑦 2 − 𝑧 2 )𝑥
Factorize.Question.1. Solution.
(i). 𝟐𝒂𝒃𝒄 − 𝟒𝒂𝒃𝒙 + 𝟐𝒂𝒃𝒅 (𝑥 2 − 𝑦 2 )𝑧 + (𝑦 2 − 𝑧 2 )𝑥
Solution. = 𝑥2𝑧 − 𝑦2𝑧 + 𝑦2𝑥 − 𝑧2𝑥
𝟐𝒂𝒃𝒄 − 𝟒𝒂𝒃𝒙 + 𝟐𝒂𝒃𝒅 = 𝟐𝒂𝒃(𝒄 − 𝟐𝒙 + 𝒅) = 𝑥2𝑧 + 𝑦2𝑥 − 𝑦2𝑧 − 𝑧2𝑥
Answer. = 𝑥(𝑥𝑧 + 𝑦 2 ) − 𝑧(𝑦 2 + 𝑥𝑧)
= (𝑥𝑧 + 𝑦 2 )(𝑥 − 𝑧)
(ii). 𝟗𝒙𝒚 − 𝟏𝟐𝒙𝟐 𝒚 + 𝟏𝟖𝒚𝟐
Answer.
Solution.
Question.3.
𝟗𝒙𝒚 − 𝟏𝟐𝒙𝟐 𝒚 + 𝟏𝟖𝒚𝟐 = 𝟑𝒚(𝟑𝒙 − 𝟒𝒙𝟐 + 𝟔𝒚)
(i). 144𝑎2 + 24𝑎 + 1
Answer.
Solution.
(iii). −𝟑𝒙𝟐 𝒚 − 𝟑𝒙 + 𝟗𝒙𝒚𝟐
144𝑎2 + 24𝑎 + 1 = (12𝑎)2 + 2(12𝑎)(1) + (1)2
Solution.
= (𝟏𝟐𝒂 + 𝟏)𝟐
−𝟑𝒙𝟐 𝒚 − 𝟑𝒙 + 𝟗𝒙𝒚𝟐 = −𝟑𝒙(𝒚 + 𝟏 − 𝟑𝒚𝟐 ) 𝑎2 𝑏2
Answer. (ii). 𝑏2 − 2 + 𝑎2
(iv). 𝟓𝒂𝒃𝟐 𝒄𝟑 − 𝟏𝟎𝒂𝟐 𝒃𝟑 𝒄 − 𝟐𝟎𝒂𝟑 𝒃𝒄𝟐 Solution.
Solution. 𝑎2 𝑏2 𝒂 𝟐 𝒂 𝒃 𝒃 𝟐
− 2 + = ( ) − 𝟐 ( ) ( ) + ( )
𝟓𝒂𝒃𝟐 𝒄𝟑 − 𝟏𝟎𝒂𝟐 𝒃𝟑 𝒄 − 𝟐𝟎𝒂𝟑 𝒃𝒄𝟐 = 𝟓𝒂𝒃𝒄(𝒃𝒄𝟐 − 𝑏2 𝑎2 𝒃 𝒃 𝒂 𝒂
𝟐𝒂𝒃𝟐 − 𝟒𝒂𝟐 𝒄) 𝒂 𝒃 𝟐
Answer. =( − )
𝒃 𝒂
1|Page
Class 9th Chapter 5 www.notes.pk.com
(iii).(𝒙 + 𝒚)𝟐 − 𝟏𝟒𝒛(𝒙 + 𝒚) + 𝟒𝟗𝒛𝟐 Solution.
Solution. 𝟒𝒙𝟐 − 𝒚𝟐 − 𝟐𝒚 − 𝟏 = 𝟒𝒙𝟐 − (𝒚𝟐 + 𝟐𝒚 + 𝟏)
(𝒙 + 𝒚)𝟐 − 𝟏𝟒𝒛(𝒙 + 𝒚) + 𝟒𝟗𝒛𝟐 = 𝟒𝒙𝟐 − [(𝒚)𝟐 + 𝟐(𝒚)(𝟏) + (𝟏)𝟐 ]
= (𝒙 + 𝒚)𝟐 − 𝟐(𝒙 + 𝒚)(𝟕𝒛) = (𝟐𝒙)𝟐 − (𝒚 + 𝟏)𝟐
+ (𝟕𝒛)𝟐 = [𝟐𝒙 + (𝒚 + 𝟏)][𝟐𝒙 − (𝒚 + 𝟏)]
= (𝒙 + 𝒚 − 𝟕𝒛)𝟐 = (𝟐𝒙 + 𝒚 + 𝟏)(𝟐𝒙 − 𝒚 − 𝟏)
(iv).𝟏𝟐𝒙𝟐 − 𝟑𝟔𝒙 + 𝟐𝟕 (iv). 𝒙 − 𝒚𝟐 − 𝟒𝒙 − 𝟐𝒚 + 𝟑
𝟐

Solution. Solution.
𝟏𝟐𝒙𝟐 − 𝟑𝟔𝒙 + 𝟐𝟕 = 𝟑(𝟒𝒙𝟐 − 𝟏𝟐𝒙 + 𝟗) 𝒙𝟐 − 𝒚𝟐 − 𝟒𝒙 − 𝟐𝒚 + 𝟑
= 𝟑[(𝟐𝒙)𝟐 − 𝟐(𝟐𝒙)(𝟑) + (𝟑)𝟐 ] = 𝒙𝟐 − 𝟒𝒙 − 𝒚𝟐 − 𝟐𝒚 + 𝟑
= 𝟑(𝟐𝒙 − 𝟑)𝟐 = 𝒙𝟐 − 𝟒𝒙 + 𝟒 − 𝒚𝟐 − 𝟐𝒚 − 𝟏
Question.4. = (𝒙𝟐 − 𝟒𝒙 + 𝟒) − (𝒚𝟐 + 𝟐𝒚 + 𝟏)
(i). 𝟑𝒙𝟐 − 𝟕𝟓𝒚𝟐 = [(𝒙)𝟐 − 𝟐(𝒙)(𝟐) + (𝟐)𝟐 ]
Solution. − [(𝒚)𝟐 + 𝟐(𝒚)(𝟏) + (𝟏)𝟐 ]
𝟑𝒙𝟐 − 𝟕𝟓𝒚𝟐 = 𝟑(𝒙𝟐 − 𝟐𝟓𝒚𝟐 ) = (𝒙 − 𝟐)𝟐 − (𝒚 + 𝟏)𝟐
= 𝟑[(𝒙)𝟐 − (𝟓𝒚)𝟐 ] = [(𝒙 − 𝟐) + (𝒚 + 𝟏)][(𝒙 − 𝟐) − (𝒚 + 𝟏)]
= 𝟑(𝒙 + 𝟓𝒚)(𝒙 − 𝟓𝒚) = (𝒙 − 𝟐 + 𝒚 + 𝟏)(𝒙 − 𝟐 − 𝒚 − 𝟏)
(ii). 𝒙(𝒙 − 𝟏) − 𝒚(𝒚 − 𝟏) = (𝒙 + 𝒚 − 𝟏)(𝒙 − 𝒚 − 𝟑)
Solution. (v). 𝟐𝟓𝒙 − 𝟏𝟎𝒙 + 𝟏 − 𝟑𝟔𝒛𝟐
𝟐

𝒙(𝒙 − 𝟏) − 𝒚(𝒚 − 𝟏) = 𝒙𝟐 − 𝒙 − 𝒚𝟐 + 𝒚 Solution.


= 𝒙 𝟐 − 𝒚𝟐 − 𝒙 + 𝒚 𝟐𝟓𝒙𝟐 − 𝟏𝟎𝒙 + 𝟏 − 𝟑𝟔𝒛𝟐
= (𝒙 + 𝒚)(𝒙 − 𝒚) − 𝟏(𝒙 − 𝒚) = [(𝟓𝒙)𝟐 − 𝟐(𝟓𝒙)(𝟏) + (𝟏)𝟐 ]
= (𝒙 − 𝒚)(𝒙 + 𝒚 − 𝟏) − (𝟔𝒛)𝟐
(iii).𝟏𝟐𝟖𝒂𝒎 − 𝟐𝟒𝟐𝒂𝒏𝟐
𝟐 = (𝟓𝒙 − 𝟏)𝟐 − (𝟔𝒛)𝟐
Solution. = (𝟓𝒙 − 𝟏 + 𝟔𝒛)(𝟓𝒙 − 𝟏 − 𝟓𝒛)
𝟏𝟐𝟖𝒂𝒎𝟐 − 𝟐𝟒𝟐𝒂𝒏𝟐 = 𝟐𝒂(𝟔𝟒𝒎𝟐 − 𝟏𝟐𝟏𝒏𝟐 ) = (𝟓𝒙 + 𝟔𝒛 − 𝟏)(𝟓𝒙 − 𝟓𝒛 − 𝟏)
= 2𝑎[(8𝑚)2 − (11𝑛)2 ] (vi). 𝒙 − 𝒚𝟐 − 𝟒𝒙𝒛 + 𝟒𝒛𝟐
𝟐

= 2𝑎(8𝑚 + 11𝑛)(8𝑚 − 11𝑛) Solution.


(iv).𝟑𝒙 − 𝟐𝟒𝟑𝒙𝟑 𝒙𝟐 − 𝒚𝟐 − 𝟒𝒙𝒛 + 𝟒𝒛𝟐 = 𝒙𝟐 − 𝟒𝒙𝒛 + 𝟒𝒛𝟐 − 𝒚𝟐
Solution. = (𝒙)𝟐 − 𝟐(𝒙)(𝒛) + (𝟐𝒛)𝟐 − 𝒚𝟐
𝟑𝒙 − 𝟐𝟒𝟑𝒙𝟑 = 𝟑𝒙(𝟏 − 𝟖𝟏𝒙𝟐 ) = (𝒙 − 𝟐𝒛)𝟐 − (𝒚)𝟐
= 𝟑𝒙[(𝟏)𝟐 − (𝟗𝒙)𝟐 ] = (𝒙 − 𝟐𝒛 + 𝒚)(𝒙 − 𝟐𝒛 − 𝒚)
= 𝟑𝒙(𝟏 + 𝟗𝒙)(𝟏 − 𝟗𝒙) = (𝒙 + 𝒚 − 𝟐𝒛)(𝒙 − 𝒚 − 𝟐𝒛)
Question.5.
(i). 𝒙𝟐 − 𝒚𝟐 − 𝟔𝒚 − 𝟗
Solution.
𝒙𝟐 − 𝒚𝟐 − 𝟔𝒚 − 𝟗 = 𝒙𝟐 − (𝒚𝟐 + 𝟔𝒚 + 𝟗)
= 𝒙𝟐 − [(𝒚)𝟐 + 𝟐(𝒚)(𝟑) + (𝟑)𝟐 ]
= 𝒙𝟐 − (𝒚 + 𝟑)𝟐
= [𝒙 + (𝒚 + 𝟑)][𝒙 − (𝒚 + 𝟑)]
= (𝒙 + 𝒚 + 𝟑)(𝒙 − 𝒚 − 𝟑)
𝟐 𝟐
(ii). 𝒙 − 𝒂 + 𝟐𝒂 − 𝟏
Solution.
𝒙𝟐 − 𝒂𝟐 + 𝟐𝒂 − 𝟏 = 𝒙𝟐 − (𝒂𝟐 − 𝟐𝒂 + 𝟏)
= 𝒙𝟐 − [(𝒂)𝟐 − 𝟐(𝒂)(𝟏) + (𝟏)𝟐 ]
= (𝒙)𝟐 − (𝒂 − 𝟏)𝟐
= [𝒙 + (𝒂 − 𝟏)][𝒙 − (𝒂 − 𝟏)]
= (𝒙 + 𝒂 − 𝟏)(𝒙 − 𝒂 + 𝟏)
(iii). 𝟒𝒙 − 𝒚𝟐 − 𝟐𝒚 − 𝟏
𝟐

2|Page
Class 9th Chapter 5 www.notes.pk.com
(a) Factorization of the Expression of the = (𝒚 + 𝒄)(𝒚 + 𝒅) + 𝒌
types: = 𝒚𝟐 + 𝒚𝒅 + 𝒚𝒄 + 𝒌
𝒂 + 𝒂 𝒃 + 𝒃𝟒 𝒐𝒓 𝒂𝟒 + 𝟒𝒃𝟒
𝟒 𝟐 𝟐 = 𝒚𝟐 + (𝒅 + 𝒄)𝒚 + 𝒌
Explanation: For 𝒂𝟒 + 𝒂𝟐 𝒃𝟐 + 𝒃𝟒 This the same type (b).
𝒂𝟒 + 𝒂𝟐 𝒃𝟐 + 𝒃𝟒 = 𝒂𝟒 + 𝒃𝟒 + 𝒂𝟐 𝒃𝟐 Explanation: For (𝒙 + 𝒂)(𝒙 + 𝒃)(𝒙 + 𝒄)(𝒙 +
= (𝒂𝟐 )𝟐 + (𝒃𝟐 )𝟐 + 𝟐(𝒂𝟐 )(𝒃𝟐 ) − 𝟐(𝒂𝟐 )(𝒃𝟐 ) 𝒅) + 𝒌
+ 𝒂𝟐 𝒃𝟐 (𝒙 + 𝒂)(𝒙 + 𝒃)(𝒙 + 𝒄)(𝒙 + 𝒅) + 𝒌
= (𝒂𝟐 + 𝒃𝟐 )𝟐 − 𝟐𝒂𝟐 𝒃𝟐 + 𝒂𝟐 𝒃𝟐 We will multiply the pair for which 𝒂 + 𝒃 = 𝒄 +
= (𝒂𝟐 + 𝒃𝟐 )𝟐 − 𝒂𝟐 𝒃𝟐 𝒅, then
= (𝒂𝟐 + 𝒃𝟐 )𝟐 − (𝒂𝒃)𝟐 = [(𝒙 + 𝒂)(𝒙 + 𝒃)][(𝒙 + 𝒄)(𝒙 + 𝒅)] + 𝒌
= (𝒂𝟐 + 𝒃𝟐 + 𝒂𝒃)(𝒂𝟐 + 𝒃𝟐 − 𝒂𝒃) = [𝒙𝟐 + 𝒃𝒙 + 𝒂𝒙 + 𝒂𝒃][𝒙𝟐 + 𝒅𝒙 + 𝒄𝒙 + 𝒄𝒅]
Explanation: For 𝒂𝟒 + 𝟒𝒃𝟒 +𝒌
𝒂𝟒 + 𝟒𝒃𝟒 = (𝒂𝟐 )𝟐 + (𝟐𝒃𝟐 )𝟐 + 𝟐(𝒂𝟐 )(𝟐𝒃𝟐 ) = (𝒙 + (𝒃 + 𝒂)𝒙 + 𝒂𝒃)(𝒙𝟐 + (𝒅 + 𝒄)𝒙 + 𝒄𝒅)
𝟐

− 𝟐(𝒂𝟐 )(𝟐𝒃𝟐 ) +𝒌
= (𝒂𝟐 + 𝟐𝒃𝟐 )𝟐 − 𝟒𝒂𝟐 𝒃𝟐 As 𝒂 + 𝒃 = 𝒄 + 𝒅 , 𝒕𝒉𝒆𝒏
= (𝒂𝟐 + 𝟐𝒃𝟐 )𝟐 − (𝟐𝒂𝒃)𝟐 = [𝒙𝟐 + (𝒄 + 𝒅)𝒙 + 𝒂𝒃][𝒙𝟐 + (𝒄 + 𝒅)𝒙 + 𝒄𝒅]
= (𝒂𝟐 + 𝟐𝒃𝟐 + 𝟐𝒂𝒃)(𝒂𝟐 + 𝟐𝒃𝟐 − 𝟐𝒂𝒃) +𝒌
(b) Factorization of the Expression of the Suppose that
types: 𝒙𝟐 + (𝒄 + 𝒅)𝒙
𝒙𝟐 + 𝒑𝒙 + 𝒒 = 𝒚 , 𝒕𝒉𝒆𝒏 𝒂𝒃𝒐𝒗𝒆 𝒆𝒙𝒑𝒓𝒆𝒔𝒔𝒊𝒐𝒏 𝒃𝒆𝒄𝒐𝒎𝒆𝒔
Explanation: = (𝒚 + 𝒂𝒃)(𝒚 + 𝒄𝒅) + 𝒌
𝒙𝟐 + 𝒑𝒙 + 𝒒 = 𝒙𝟐 + (𝒔 + 𝒓)𝒙 + 𝒒 , = 𝒚𝟐 + 𝒚𝒄𝒅 + 𝒚𝒂𝒃 + 𝒂𝒃𝒄𝒅 + 𝒌
𝒘𝒉𝒆𝒓𝒆 𝒔 + 𝒓 = 𝒑 𝒂𝒏𝒅 𝒔 × 𝒓 = 𝒒 = 𝒚𝟐 + (𝒄𝒅 + 𝒂𝒃)𝒚 + 𝒂𝒃𝒄𝒅 + 𝒌
= 𝒙𝟐 + 𝒔𝒙 + 𝒓𝒙 + 𝒔 × 𝒓 This the same type (b).
= 𝒙(𝒙 + 𝒔) + 𝒓(𝒙 + 𝒔) Explanation: For (𝒂𝒙𝟐 + 𝒃𝒙 + 𝒄)(𝒂𝒙𝟐 + 𝒃𝒙 +
= (𝒙 + 𝒔)(𝒙 + 𝒓) 𝒅) + 𝒌𝒙𝟐
(c) Factorization of the Expression of the (𝒙 + 𝒂)(𝒙 + 𝒃)(𝒙 + 𝒄)(𝒙 + 𝒅) + 𝒌𝒙𝟐
types: We will multiply the pair for which 𝒂 + 𝒃 = 𝒄 +
𝒂𝒙𝟐 + 𝒃𝒙 + 𝒄, 𝒂 ≠ 𝟎 𝒅, then
𝒂𝒙𝟐 + 𝒃𝒙 + 𝒄 = 𝒂𝒙𝟐 + (𝒔 + 𝒓)𝒙 + 𝒄, = [(𝒙 + 𝒂)(𝒙 + 𝒃)][(𝒙 + 𝒄)(𝒙 + 𝒅)] + 𝒌𝒙𝟐
𝒔 + 𝒓 = 𝒃 𝒂𝒏𝒅 𝒔 × 𝒓 = 𝒂𝒄 = [𝒙𝟐 + 𝒃𝒙 + 𝒂𝒙 + 𝒂𝒃][𝒙𝟐 + 𝒅𝒙 + 𝒄𝒙 + 𝒄𝒅]
𝒔×𝒓 + 𝒌𝒙𝟐
= 𝒂𝒙𝟐 + 𝒔𝒙 + 𝒓𝒙 +
𝒂 = (𝒙𝟐 + (𝒃 + 𝒂)𝒙 + 𝒂𝒃)(𝒙𝟐 + (𝒅 + 𝒄)𝒙 + 𝒄𝒅)
𝒔
= 𝒙(𝒂𝒙 + 𝒔) + 𝒓 (𝒙 + ) + 𝒌𝒙𝟐
𝒂 As 𝒂 + 𝒃 = 𝒄 + 𝒅 , 𝒕𝒉𝒆𝒏
𝒂𝒙 + 𝒔
= 𝒙(𝒂𝒙 + 𝒔) + 𝒓 ( ) = [𝒙𝟐 + (𝒄 + 𝒅)𝒙 + 𝒂𝒃][𝒙𝟐 + (𝒄 + 𝒅)𝒙 + 𝒄𝒅]
𝒂
𝒓 + 𝒌𝒙𝟐
= 𝒙(𝒂𝒙 + 𝒔) + (𝒂𝒙 + 𝒔)
𝒂 Suppose that
𝒓 𝒙𝟐 + (𝒄 + 𝒅)𝒙
= (𝒂𝒙 + 𝒔) (𝒙 + )
𝒂 = 𝒚 , 𝒕𝒉𝒆𝒏 𝒂𝒃𝒐𝒗𝒆 𝒆𝒙𝒑𝒓𝒆𝒔𝒔𝒊𝒐𝒏 𝒃𝒆𝒄𝒐𝒎𝒆𝒔
(d) Factorization of the Expression of the
= (𝒚 + 𝒂𝒃)(𝒚 + 𝒄𝒅) + 𝒌𝒙𝟐
types:
= 𝒚𝟐 + 𝒚𝒄𝒅 + 𝒚𝒂𝒃 + 𝒂𝒃𝒄𝒅 + 𝒌𝒙𝟐
(i). (𝒂𝒙 + 𝒃𝒙 + 𝒄)(𝒂𝒙𝟐 + 𝒃𝒙 + 𝒅) + 𝒌
𝟐
= 𝒚𝟐 + (𝒄𝒅 + 𝒂𝒃)𝒚 + 𝒂𝒃𝒄𝒅 + 𝒌𝒙𝟐
(ii). (𝒙 + 𝒂)(𝒙 + 𝒃)(𝒙 + 𝒄)(𝒙 + 𝒅) + 𝒌 After simplification it also becomes type (b).
(iii). (𝒙 + 𝒂)(𝒙 + 𝒃)(𝒙 + 𝒄)(𝒙 + 𝒅) + 𝒌𝒙𝟐 (e). Factorization of the Expression of the type:
Explanation: For (𝒂𝒙𝟐 + 𝒃𝒙 + 𝒄)(𝒂𝒙𝟐 + 𝒃𝒙 + (i). 𝒂𝟑 + 𝟑𝒂𝟐 𝒃 + 𝟑𝒂𝒃𝟐 + 𝒃𝟑
𝒅) + 𝒌 (ii). 𝒂𝟑 − 𝟑𝒂𝟐 𝒃 + 𝟑𝒂𝒃𝟐 − 𝒃𝟑
(𝒂𝒙𝟐 + 𝒃𝒙 + 𝒄)(𝒂𝒙𝟐 + 𝒃𝒙 + 𝒅) + 𝒌
Explanation For 𝒂𝟑 + 𝟑𝒂𝟐 𝒃 + 𝟑𝒂𝒃𝟐 + 𝒃𝟑
We will suppose 𝒂𝒙𝟐 + 𝒃𝒙 =
𝒂𝟑 + 𝟑𝒂𝟐 𝒃 + 𝟑𝒂𝒃𝟐 + 𝒃𝟑 = (𝒂 + 𝒃)𝟑
𝒚 , 𝒕𝒉𝒆𝒏 𝒂𝒃𝒐𝒗𝒆 𝒃𝒆𝒄𝒐𝒎𝒆𝒔
3|Page
Class 9th Chapter 5 www.notes.pk.com
It’s a very famous formula. = (2𝑥 2 + 9)2 − 36𝑥 2
Explanation For 𝒂𝟑 − 𝟑𝒂𝟐 𝒃 + 𝟑𝒂𝒃𝟐 − 𝒃𝟑 = (2𝑥 2 + 9)2 − (6𝑥)2
𝒂𝟑 − 𝟑𝒂𝟐 𝒃 + 𝟑𝒂𝒃𝟐 − 𝒃𝟑 = (𝒂 − 𝒃)𝟑 = (2𝑥 2 + 9 + 6𝑥)(2𝑥 2 + 9 − 6𝑥)
It’s a very famous formula. = (2𝑥 2 + 6𝑥 + 9)(2𝑥 2 − 6𝑥 + 9)
(f). Factorization of the Expression of the type: (v). 𝑥 4 + 𝑥 2 + 25
(i).𝒂𝟑 + 𝒃𝟑 Solution.
(ii). 𝒂𝟑 − 𝒃𝟑 𝑥 4 + 𝑥 2 + 25 = 𝑥 4 + 25 + 𝑥 2
We will use, well known formulas for these = (𝑥 ) + (5)2 + 2(𝑥 2 )(5) − 2(𝑥 2 )(5) + 𝑥 2
2 2

types = (𝑥 2 + 5)2 − 10𝑥 2 + 𝑥 2


(i). 𝒂𝟑 + 𝒃𝟑 = (𝒂 + 𝒃)(𝒂𝟐 − 𝒂𝒃 + 𝒃𝟐 ) = (𝑥 2 + 5)2 − 9𝑥 2
(ii). 𝒂𝟑 − 𝒃𝟑 = (𝒂 − 𝒃)(𝒂𝟐 + 𝒂𝒃 + 𝒃𝟐 ) = (𝑥 2 + 5)2 − (3𝑥)2
= (𝑥 2 + 5 + 3𝑥)(𝑥 2 + 5 − 3𝑥)
= (𝑥 2 + 3𝑥 + 5)(𝑥 2 − 3𝑥 + 5)
Exercise 5.2 (v). 𝑥 4 + 4𝑥 2 + 16
Factorize Solution.
Question.1. 𝑥 4 + 4𝑥 2 + 16 = 𝑥 4 + 16 + 4𝑥 2
𝟏
(i). 𝒙𝟒 + 𝒙𝟒 − 𝟑 = (𝑥 2 )2 + (4)2 + 2(𝑥 2 )(4) − 2(𝑥 2 )(4) + 4𝑥 2
Solution. = (𝑥 2 + 4)2 − 8𝑥 2 + 4𝑥 2
𝟏 𝟏 𝟐 𝟏 = (𝑥 2 + 4)2 − 4𝑥 2
𝒙 + 𝟒 − 𝟑 = (𝒙 ) + ( 𝟐 ) − 𝟐(𝒙𝟐 ) ( 𝟐 )
𝟒 𝟐 𝟐
= (𝑥 2 + 4)2 − (2𝑥)2
𝒙 𝒙 𝒙
𝟏 = (𝑥 + 4 + 2𝑥)(𝑥 2 + 4 − 2𝑥)
2
+ 𝟐(𝒙𝟐 ) ( 𝟐 ) − 𝟑 = (𝑥 2 + 2𝑥 + 4)(𝑥 2 − 2𝑥 + 4)
𝒙
𝟐
𝟏 Question.2.
= (𝒙𝟐 − 𝟐 ) + 𝟐 − 𝟑 (i). 𝑥 2 + 14𝑥 + 48
𝒙
𝟏 𝟐 Solution.
= (𝒙 − 𝟐 ) − (𝟏)𝟐
𝟐
𝑥 2 + 14𝑥 + 48 = 𝑥 2 + 8𝑥 + 6𝑥 + 48
𝒙
𝟏 𝟏 = 𝑥(𝑥 + 8) + 6(𝑥 + 8)
= (𝒙𝟐 − 𝟐 + 𝟏)(𝒙𝟐 − 𝟐 − 𝟏)
𝒙 𝒙 = (𝑥 + 8)(𝑥 + 6)
(ii). 3𝑥 4 + 12𝑦 4 2
(ii). 𝑥 − 21𝑥 + 108
Solution. Solution.
3𝑥 4 + 12𝑦 4 = 3(𝑥 4 + 4𝑦 4 ) 𝑥 2 − 21𝑥 + 108 = 𝑥 2 − 12𝑥 − 9𝑥 + 108
= 3[(𝑥 2 )2 + (2𝑦 2 )2 + 2(𝑥 2 )(2𝑦 2 ) = 𝑥(𝑥 − 12) − 9(𝑥 − 12)
− 2(𝑥 2 )(2𝑦 2 )] = (𝑥 − 12)(𝑥 − 9)
= 3[(𝑥 + 2𝑦 2 )2 − 4𝑥 2 𝑦 2 ]
2 2
(iii). 𝑥 − 11𝑥 − 42
= 3[(𝑥 2 + 2𝑦 2 )2 − (2𝑥𝑦)2 ] Solution.
= 3(𝑥 2 + 2𝑦 2 + 2𝑥𝑦)(𝑥 2 + 2𝑦 2 − 2𝑥𝑦) 𝑥 2 − 11𝑥 − 42 = 𝑥 2 − 14𝑥 + 3𝑥 − 42
= 3(𝑥 2 + 2𝑥𝑦 + 2𝑦 2 )(𝑥 2 − 2𝑥𝑦 + 2𝑦 2 ) = 𝑥(𝑥 − 14) + 3(𝑥 − 14)
(iii). 𝑎4 + +3𝑎2 𝑏 2 + 4𝑏 4 = (𝑥 − 14)(𝑥 + 3)
2
Solution. (iii). 𝑥 + 𝑥 − 132
𝑎4 + +3𝑎2 𝑏 2 + 4𝑏 4 = 𝑎4 + 4𝑏 4 + 3𝑎2 𝑏 2 Solution.
= (𝑎2 )2 + (2𝑏 2 )2 + 2(𝑎2 )(2𝑏 2 ) − 2(𝑎2 )(2𝑏 2 ) 𝑥 2 + 𝑥 − 132 = 𝑥 2 + 12𝑥 − 11𝑥 − 132
+ 3𝑎2 𝑏 2 = 𝑥(𝑥 + 12) − 11(𝑥 + 12)
= (𝑎2 + 2𝑏 2 )2 − 4𝑎2 𝑏 2 + 3𝑎2 𝑏 2 = (𝑥 + 12)(𝑥 − 11)
= (𝑎2 + 2𝑏 2 )2 − 𝑎2 𝑏 2 Question.3.
= (𝑎2 + 2𝑏 2 )2 − (𝑎𝑏)2 (i). 4𝑥 2 + 12𝑥 + 5
= (𝑎2 + 2𝑏 2 + 𝑎𝑏)(𝑎2 + 2𝑏 2 − 𝑎𝑏) Solution.
(iv). 4𝑥 4 + 81 4𝑥 2 + 12𝑥 + 5 = 4𝑥 2 + 10𝑥 + 2𝑥 + 5
Solution. = 2𝑥(2𝑥 + 5) + 1(2𝑥 + 5)
4𝑥 4 + 81 = (2𝑥 2 )2 + (9)2 + 2(2𝑥 2 )(9) = (2𝑥 + 5)(2𝑥 + 1)
− 2(2𝑥 2 )(9) 2
(ii). 30𝑥 + 7𝑥 − 15
4|Page
Class 9th Chapter 5 www.notes.pk.com
Solution. = (𝑦 + 4)(𝑦 + 6) − 3
30𝑥 2 + 7𝑥 − 15 = 30𝑥 2 + 25𝑥 − 18𝑥 − 15 = 𝑦 2 + 6𝑦 + 4𝑦 + 24 − 3
= 5𝑥(6𝑥 + 5) − 3(6𝑥 + 5) = 𝑦 2 + 10𝑦 + 21
= (6𝑥 + 5)(5𝑥 − 3) = 𝑦 2 + 7𝑦 + 3𝑦 + 21
2 = 𝑦(𝑦 + 7) + 3(𝑦 + 7)
(iii). 24𝑥 − 65𝑥 + 21
Solution. = (𝑦 + 7)(𝑦 + 3)
24𝑥 2 − 65𝑥 + 21 = 24𝑥 2 − 56𝑥 − 9𝑥 + 21 = (𝑥 + 5𝑥 + 7)(𝑥 2 + 5𝑥 + 3)
2

= 8𝑥(3𝑥 − 7) − 3(3𝑥 − 7) (ii). (𝑥 2 − 4𝑥)(𝑥 2 − 4𝑥 − 1) − 20


= (3𝑥 − 7)(8𝑥 − 3) Solution.
2
(iv). 5𝑥 − 16𝑥 − 21 (𝑥 2 − 4𝑥)(𝑥 2 − 4𝑥 − 1) − 20
Solution. Suppose that 𝑥 2 − 4𝑥 = 𝑦
5𝑥 2 − 16𝑥 − 21 = 5𝑥 2 − 21𝑥 + 5𝑥 − 21 = (𝑦)(𝑦 − 1) − 20
= 𝑥(5𝑥 − 21) + 1(5𝑥 − 21) = 𝑦 2 − 𝑦 − 20
= (5𝑥 − 21)(𝑥 + 1) = 𝑦 2 − 5𝑦 + 4𝑦 − 20
(v). 4𝑥 2 − 17𝑥𝑦 + 4𝑦 2 = 𝑦(𝑦 − 5) + 4(𝑦 − 5)
Solution. = (𝑦 − 5)(𝑦 + 4)
4𝑥 2 − 17𝑥𝑦 + 4𝑦 2 = 4𝑥 2 − 16𝑥𝑦 − 𝑥𝑦 + 4𝑦 2 = (𝑥 − 4𝑥 − 5)(𝑥 2 − 4𝑥 + 4)
2

= 4𝑥(𝑥 − 4𝑦) − 𝑦(𝑥 − 4𝑦) = (𝑥 2 − 5𝑥 + 𝑥 − 5)(𝑥 2 − 2𝑥 − 2𝑥 + 4)


= (𝑥 − 4𝑦)(4𝑥 − 𝑦) = [𝑥(𝑥 − 5) + 1(𝑥 − 5)][𝑥(𝑥 − 2) − 2(𝑥 − 2)]
(vi). 3𝑥 − 38𝑥𝑦 − 13𝑦 2
2 = (𝑥 − 5)(𝑥 + 1)(𝑥 − 2)(𝑥 − 2)
Solution. = (𝑥 − 5)(𝑥 + 1)(𝑥 − 2)2
3𝑥 2 − 38𝑥𝑦 − 13𝑦 2 (iii). (𝑥 + 2)(𝑥 + 3)(𝑥 + 4)(𝑥 + 5) − 15
= 3𝑥 2 − 39𝑥𝑦 + 𝑥𝑦 − 13𝑦 2 Solution.
= 3𝑥(𝑥 − 13𝑦) + 𝑦(𝑥 − 13𝑦) (𝑥 + 2)(𝑥 + 3)(𝑥 + 4)(𝑥 + 5) − 15
= (𝑥 − 13𝑦)(3𝑥 + 𝑦) = (𝑥 + 2)(𝑥 + 5)(𝑥 + 3)(𝑥 + 4)
(vii). 5𝑥 + 33𝑥𝑦 − 14𝑦 2
2 − 15
Solution. = (𝑥 + 5𝑥 + 2𝑥 + 10)(𝑥 2 + 4𝑥 + 3𝑥 + 12)
2

5𝑥 2 + 33𝑥𝑦 − 14𝑦 2 − 15
= 5𝑥 2 + 35𝑥𝑦 − 2𝑥𝑦 − 14𝑦 2 = (𝑥 + 7𝑥 + 10)(𝑥 2 + 7𝑥 + 12) − 15
2

= 5𝑥(𝑥 + 7𝑦) − 2𝑦(𝑥 + 7𝑦) Suppose that 𝑥 2 + 7𝑥 = 𝑦


= (𝑥 + 7𝑦)(5𝑥 − 2𝑦) = (𝑦 + 10)(𝑦 + 12) − 15
1 2 1 = 𝑦 2 + 12𝑦 + 10𝑦 + 120 − 15
(viii). (5𝑥 − 𝑥) + 4 (5𝑥 − 𝑥) + 4 , 𝑥 ≠ 0.
= 𝑦 2 + 22𝑦 + 105
Solution. = 𝑦 2 + 15𝑦 + 7𝑦 + 105
1 2 1 = 𝑦(𝑦 + 15) + 7(𝑦 + 15)
(5𝑥 − ) + 4 (5𝑥 − ) + 4
𝑥 𝑥 = (𝑦 + 15)(𝑦 + 7)
1
Suppose that 5𝑥 − 𝑥 = 𝑦 (𝑥 2 + 7𝑥 + 15)(𝑥 2 + 7𝑥 + 7)
(iv). (𝑥 + 4)(𝑥 − 5)(𝑥 + 6)(𝑥 − 7) − 504
1 2 1
(5𝑥 − ) + 4 (5𝑥 − ) + 4 = 𝑦 2 + 4𝑦 + 4 Solution.
𝑥 𝑥
= 𝑦 2 + 2𝑦 + 2𝑦 + 4 (𝑥 + 4)(𝑥 − 5)(𝑥 + 6)(𝑥 − 7) − 504
= 𝑦(𝑦 + 2) + 2(𝑦 + 2) (𝑥 + 2)(𝑥 + 5)(𝑥 + 3)(𝑥 + 4) − 1
= (𝑦 + 2)(𝑦 + 2) = (𝑥 + 4)(𝑥 − 5)(𝑥 + 6)(𝑥 − 7)
= (𝑦 + 2)2 − 504
1 2 = (𝑥 2 − 5𝑥 + 4𝑥 − 20)(𝑥 2 − 7𝑥 + 6𝑥 − 42)
= (5𝑥 − + 2) − 504
𝑥
Question.4 = (𝑥 − 𝑥 − 20)(𝑥 2 − 𝑥 − 42) − 504
2

(i). (𝑥 2 + 5𝑥 + 4)(𝑥 2 + 5𝑥 + 6) − 3 Suppose that 𝑥 2 − 𝑥 = 𝑦


Solution. = (𝑦 − 20)(𝑦 − 42) − 504
= 𝑦 2 − 42𝑦 − 20𝑦 + 840 − 504
(𝑥 2 + 5𝑥 + 4)(𝑥 2 + 5𝑥 + 6) − 3
= 𝑦 2 − 62𝑦 + 336
Suppose that 𝑥 2 + 5𝑥 = 𝑦
5|Page
Class 9th Chapter 5 www.notes.pk.com
= 𝑦 2 − 56𝑦 − 6𝑦 + 336 = (2𝑥 − 5𝑦)3
= 𝑦(𝑦 − 56) − 6(𝑦 − 56) Question.6.
= (𝑦 − 56)(𝑦 − 6) (i). 27 + 8𝑥 3
= (𝑥 − 𝑥 − 56)(𝑥 2 − 𝑥 − 6)
2
Solution.
= (𝑥 2 − 8𝑥 + 7𝑥 − 56)(𝑥 2 − 3𝑥 + 2𝑥 − 6) 27 + 8𝑥 3 = (3)3 + (2𝑥)3
[𝑥(𝑥 − 8) + 7(𝑥 − 9)][𝑥(𝑥 − 3) + 2(𝑥 − 3)] = (3 + 2𝑥)[(3)2 − (3)(2𝑥) + (2𝑥)2 ]
= (𝑥 − 9)(𝑥 + 7)(𝑥 − 3)(𝑥 + 2) = (3 + 2𝑥)(9 − 6𝑥 + 4𝑥 2 )
(v). (𝑥 + 1)(𝑥 + 2)(𝑥 + 3)(𝑥 + 6) − 3𝑥 2 = (2𝑥 + 3)(4𝑥 2 − 6𝑥 + 9)
Solution. (ii). 125𝑥 3 − 216𝑦 3
(𝑥 + 1)(𝑥 + 2)(𝑥 + 3)(𝑥 + 6) − 3𝑥 2 Solution.
= (𝑥 + 1)(𝑥 + 6)(𝑥 + 2)(𝑥 + 3) 125𝑥 3 − 216𝑦 3 = (5𝑥)3 − (6𝑦)3
− 3𝑥 2 = (5𝑥 − 6𝑦)[(5𝑥)2 + (5𝑥)(6𝑦) + (6𝑦)2 ]
= (𝑥 2 + 6𝑥 + 𝑥 + 6)(𝑥 2 + 3𝑥 + 2𝑥 + 6) − 3𝑥 2 = (5𝑥 − 6𝑦)(25𝑥 2 + 30𝑥𝑦 + 36𝑦 2 )
= (𝑥 2 + 7𝑥 + 6)(𝑥 2 + 5𝑥 + 6) − 3𝑥 2 (iii). 64𝑥 3 + 27𝑦 3
= (𝑥 2 + 6 + 7𝑥)(𝑥 2 + 6 + 5𝑥) − 3𝑥 2 Solution.
Suppose that 𝑥 2 + 6 = 𝑦 64𝑥 3 + 27𝑦 3 = (4𝑥)3 + (3𝑦)3
= (𝑦 + 7𝑥)(𝑦 + 5𝑥) − 3𝑥 2 = (4𝑥 + 3𝑦)[(4𝑥)2 − (4𝑥)(3𝑦) + (3𝑦)2 ]
= 𝑦 2 + 5𝑥𝑦 + 7𝑥𝑦 + 35𝑥 2 − 3𝑥 2 = (4𝑥 + 3𝑦)(16𝑥 2 − 12𝑥𝑦 + 9𝑦 2 )
= 𝑦 2 + 12𝑥𝑦 + 32𝑥 2 (iv). 8𝑥 3 + 125𝑦 3
= 𝑦 2 + 8𝑥𝑦 + 4𝑥𝑦 + 32𝑥 2 Solution.
= 𝑦(𝑦 + 8𝑥) + 4𝑥(𝑦 + 8𝑥) 8𝑥 3 + 125𝑦 3 = (2𝑥)3 + (5𝑦)3
= (𝑦 + 8𝑥)(𝑦 + 4𝑥) = (2𝑥 + 5𝑦)[(2𝑥)2 − (2𝑥)(5𝑦) + (5𝑦)2 ]
= (𝑥 + 6 + 8𝑥)(𝑥 2 + 6 + 4𝑥)
2
= (2𝑥 + 5𝑦)(4𝑥 2 − 10𝑥𝑦 + 25𝑦 2 )
= (𝑥 2 + 8𝑥 + 6)(𝑥 2 + 4𝑥 + 6) Remainder Theorem and Factor Theorem:
Question.5. Remainder Theorem:
(i). 𝑥 3 + 48𝑥 − 12𝑥 2 − 64 If a polynomial 𝑝(𝑥) is divided by a linear divisor
Solution. (𝑥 − 𝑎), then
𝑥 3 + 48𝑥 − 12𝑥 2 − 64 the remainder is 𝑝(𝑎).
= 𝑥 3 − 12𝑥 2 + 48𝑥 − 64 Zero of a Polynomial:
= (𝑥)3 − 3(𝑥)2 (4) + 3(𝑥)(4)2 − (4)3 If a specific number 𝑥 = 𝑎 is substituted for the
= (𝑥 − 4)3 variable 𝑥 in a
(ii). 8𝑥 3 + 60𝑥 2 + 150𝑥 + 125 polynomial 𝑝(𝑥) so that the value 𝑝(𝑎) is zero,
Solution. then 𝑥 = 𝑎 is called a zero
8𝑥 3 + 60𝑥 2 + 150𝑥 + 125 of the polynomial 𝑝(𝑥).
= 8𝑥 3 + 60𝑥 2 + 150𝑥 + 53 Factor Theorem:
= (2𝑥)3 + 3(2𝑥)2 (5) + 3(2𝑥)(5)2 + (5)3 The polynomial (𝑥 − 𝑎) is a factor of the
= (2𝑥 + 5)3 polynomial 𝑝(𝑥) if and
(iii). 𝑥 3 − 18𝑥 2 + 108𝑥 − 216 only if 𝑝(𝑎) = 0.
Solution.
𝑥 3 − 18𝑥 2 + 108𝑥 − 216 Exercise 5.3
= 𝑥 3 − 18𝑥 2 + 108𝑥 − 63
Question.1. Use the remainder theorem to find
= (𝑥)3 − 3(𝑥)2 (6) + 3(𝑥)(6)2
the remainder when
− (6)3
(i). 𝟑𝒙𝟑 − 𝟏𝟎𝒙𝟐 + 𝟏𝟑𝒙 −
= (𝑥 − 6)3
𝟔 𝒊𝒔 𝒅𝒊𝒗𝒊𝒅𝒆𝒅 𝒃𝒚 (𝒙 − 𝟐)
(iv). 8𝑥 3 − 125𝑦 3 − 60𝑥 2 𝑦 + 150𝑥𝑦 2
Solution.
Solution.
Suppose that 𝒑(𝒙) = 𝟑𝒙𝟑 − 𝟏𝟎𝒙𝟐 + 𝟏𝟑𝒙 −
8𝑥 3 − 125𝑦 3 − 60𝑥 2 𝑦 + 150𝑥𝑦 2
𝟔 𝒂𝒏𝒅
= 8𝑥 3 − 60𝑥 2 𝑦 + 150𝑥𝑦 2
𝒙−𝟐=𝟎
− 125𝑦 3
𝒙=𝟐
= (2𝑥)3 − 3(2𝑥)2 (5𝑦) + 3(2𝑥)(5𝑦)2 − (5𝑦)3
6|Page
Class 9th Chapter 5 www.notes.pk.com
Then 𝟏
𝑹𝒆𝒎𝒂𝒊𝒏𝒅𝒆𝒓 = 𝒑 (− )
𝑹𝒆𝒎𝒂𝒊𝒏𝒅𝒆𝒓 = 𝒑(𝟐) 𝟐
𝟑
= 𝟑(𝟐)𝟑 − 𝟏𝟎(𝟐)𝟐 + 𝟏𝟑(𝟐) − 𝟔 𝟏
= (𝟐 (− ) − 𝟏)
𝑅𝑒𝑚𝑎𝑖𝑛𝑑𝑒𝑟 = 3(8) − 10(4) + 26 − 6 𝟐
𝟐
= 24 − 40 + 20 𝟏
= 44 − 40 + 𝟔 (𝟑 + 𝟒 (− )) − 𝟏𝟎
𝟐
=4 𝑅𝑒𝑚𝑎𝑖𝑛𝑑𝑒𝑟 = (−1 − 1) + 6(3 − 2)2 − 10
3
Hence required Remainder is 4. = (−2)3 + 6(1)2 − 10
(ii). 𝟒𝒙𝟑 − 𝟒𝒙 + 𝟑 𝒊𝒔 𝒅𝒊𝒗𝒊𝒅𝒆𝒅 𝒃𝒚 (𝟐𝒙 − 𝟏) = −8 + 6 − 10
Solution. = 6 − 18
Suppose that 𝒑(𝒙) = 𝟒𝒙𝟑 − 𝟒𝒙 + 𝟑 𝒂𝒏𝒅 = −12
𝟐𝒙 − 𝟏 = 𝟎 Hence required Remainder is −12.
𝟐𝒙 = 𝟏 (v). 𝒙𝟑 − 𝟑𝒙𝟐 + 𝟒𝒙 − 𝟏𝟒 𝒊𝒔 𝒅𝒊𝒗𝒊𝒅𝒆𝒅 𝒃𝒚 (𝒙 +
𝟏 𝟐)
𝒙=
𝟐 Solution.
Then
Suppose that 𝒑(𝒙) = 𝒙𝟑 − 𝟑𝒙𝟐 + 𝟒𝒙 − 𝟏𝟒 𝒂𝒏𝒅
𝟏 𝟏 𝟑 𝟏 𝒙+𝟐=𝟎
𝑹𝒆𝒎𝒂𝒊𝒏𝒅𝒆𝒓 = 𝒑 ( ) = 𝟒 ( ) − 𝟒 ( ) + 𝟑
𝟐 𝟐 𝟐 𝒙 = −𝟐
1
𝑅𝑒𝑚𝑎𝑖𝑛𝑑𝑒𝑟 = 4 ( ) − 2 + 3 Then
8 𝑹𝒆𝒎𝒂𝒊𝒏𝒅𝒆𝒓 = 𝒑(−𝟐)
1
= +1 = (−𝟐)𝟑 − 𝟑(−𝟐)𝟐 + 𝟒(−𝟐)
2
1+2 − 𝟏𝟒
= 𝑅𝑒𝑚𝑎𝑖𝑛𝑑𝑒𝑟 = −8 − 3(4) − 8 − 14
2
3 = −8 − 12 − 8 − 14
= = −42
2
3 Hence required Remainder is −42.
Hence required Remainder is 2.
Question.2.
(iii). 𝟔𝒙𝟒 + 𝟐𝒙𝟑 − 𝒙 + 𝟐 𝒊𝒔 𝒅𝒊𝒗𝒊𝒅𝒆𝒅 𝒃𝒚 (𝒙 + 𝟐)
Solution. (i). If (𝒙 + 𝟐) is a factor of 𝒙𝟐 − 𝟒𝒌𝒙 − 𝟒𝒌𝟐 , then
find the value(s) of k.
Suppose that 𝒑(𝒙) = 𝟔𝒙𝟒 + 𝟐𝒙𝟑 − 𝒙 + 𝟐 𝒂𝒏𝒅
Solution.
𝒙+𝟐=𝟎
𝒙 = −𝟐 Suppose
Then 𝑝(𝑥) = 3𝑥 2 − 4𝑘𝑥 − 4𝑘 2
𝑹𝒆𝒎𝒂𝒊𝒏𝒅𝒆𝒓 = 𝒑(−𝟐) And 𝑥+2=0
= 𝟔(−𝟐)𝟒 + 𝟐(−𝟐)𝟑 − (−𝟐) + 𝟐 𝑥 = −2.
𝑅𝑒𝑚𝑎𝑖𝑛𝑑𝑒𝑟 = 6(16) + 2(−8) + 2 + 2 Since (𝑥 + 2) is factor of the polynomial 𝑝(𝑥), So
= 96 − 16 + 4 𝑝(𝑎) = 0
= 100 − 16 3(−2) − 4𝑘(−2) − 4𝑘 2 = 0
2

= 84 3(4) + 8𝑘 − 4𝑘 2 = 0
Hence required Remainder is 84. 12 + 8𝑘 − 4𝑘 2 = 0
(iv). (𝟐𝒙 − 𝟏)𝟑 + 𝟔(𝟑 + 𝟒𝒙)𝟐 − 4𝑘 2 − 8𝑘 − 12 = 0
𝟏𝟎 𝒊𝒔 𝒅𝒊𝒗𝒊𝒅𝒆𝒅 𝒃𝒚 (𝟐𝒙 + 𝟏) 4(𝑘 2 − 2𝑘 − 3) = 0
Solution. 𝑘 2 − 2𝑘 − 3 = 0
𝑘 2 − 3𝑘 + 𝑘 − 3 = 0
Suppose that 𝒑(𝒙) = (𝟐𝒙 − 𝟏)𝟑 + 𝟔(𝟑 +
𝑘(𝑘 − 3) + 1(𝑘 − 3) = 0
𝟒𝒙)𝟐 − 𝟏𝟎 𝒂𝒏𝒅
(𝑘 − 3)(𝑘 + 1) = 0
𝟐𝒙 + 𝟏 = 𝟎
𝑘−3=0 , 𝑘+1=0
𝟐𝒙 = −𝟏
𝑘 = 3 , 𝑘 = −1.
𝟏
𝒙=− (ii). If (𝒙 − 𝟏) is a factor of 𝒙𝟑 − 𝒌𝒙𝟐 + 𝟏𝟏𝒙 − 𝟔 ,
𝟐
Then then find the value(s) of k.
Solution.
7|Page
Class 9th Chapter 5 www.notes.pk.com
Suppose 72
3 2 𝑚=− = −24.
𝑝(𝑥) = 𝑥 − 𝑘𝑥 + 11𝑥 − 6 3
And 𝑥−1=0 Question.5.
𝑥 = 1. Determine the value of 𝒌 if 𝒑(𝒙) = 𝒌𝒙𝟑 + 𝟒𝒙𝟐 +
Since (𝑥 − 1) is factor of the polynomial 𝑝(𝑥), So 𝟑𝒙 − 𝟒 and
𝑝(1) = 0 𝒒(𝒙) = 𝒙𝟑 − 𝟒𝒙 + 𝒌 leaves the same remainder
(1) − 𝑘(1)2 + 11(1) − 6 = 0
3 when divided by (𝒙 − 𝟑).
1 − 𝑘 + 11 − 6 = 0 Solution.
6−𝑘 =0 Let 𝑝(𝑥) = 𝑘𝑥 3 + 4𝑥 2 + 3𝑥 − 4.
𝑘 = 6. By remainder theorem, 𝑝(𝑥) is divided by (𝑥 − 3)
Question.3. , then remainder is
Without actual long division determine whether 𝑝(3) = 𝑘(3)3 + 4(3)2 + 3(3) − 4
(i). (𝒙 − 𝟐)𝒂𝒏𝒅 (𝒙 − 𝟑) are factors of 𝒑(𝒙) = 𝑝(3) = 𝑘(27) + 4(9) + 9 − 4
𝒙𝟑 − 𝟏𝟐𝒙𝟐 + 𝟒𝟒𝒙 − 𝟒𝟖. 𝑝(3) = 27𝑘 + 36 + 5
Solution. 𝑝(3) = 27𝑘 + 41
Suppose that Also by remainder theorem, 𝑞(𝑥) is divided by
𝑝(𝑥) = 𝑥 3 − 12𝑥 2 + 44𝑥 − 48 (𝑥 − 3), then remainder is
And 𝑥−2= 0, 𝑥−3=0 𝑞(3) = (3)3 − 4(3) + 𝑘
𝑥 =2, 𝑥=3 𝑞(3) = 27 − 12 + 𝑘
Remainder for 𝑥 − 2 𝑖𝑠 𝑞(3) = 15 + 𝑘
𝑝(2) = (2)3 − 12(2)2 + 44(2) − 48 By Given Condition, we have
= 8 − 12(4) + 88 − 48 𝑝(3) = 𝑞(3)
= 8 − 48 + 88 − 48 27𝑘 + 41 = 15 + 𝑘
= 96 − 96 27𝑘 − 𝑘 = 15 − 41
=0 26𝑘 = −26
Hence (𝑥 − 2)𝑖𝑠 𝑡ℎ𝑒 𝑓𝑎𝑐𝑡𝑜𝑟 𝑜𝑓 𝑝(𝑥). 𝑘 = −1.
Remainder for 𝑥 − 3 𝑖𝑠 Question.6.
𝑝(3) = (3)3 − 12(3)2 + 44(3) − 48 The remainder after dividing the polynomial
= 27 − 12(9) + 132 − 48 𝒑(𝒙) = 𝒙𝟑 + 𝒂𝒙𝟐 + 𝟕 by (𝒙 + 𝟏) is 𝟐𝒃. Calculate
= 27 − 108 + 132 − 48 the value of 𝒂 𝒂𝒏𝒅 𝒃 𝒊𝒇 𝒕𝒉𝒊𝒔 expression leaves a
= 159 − 156 remainder of (𝒃 + 𝟓) on being divided by (𝒙 − 𝟐).
=3≠0 Solution.
Hence (𝑥 − 3)𝑖𝑠 𝑛𝑜𝑡 𝑓𝑎𝑐𝑡𝑜𝑟 𝑜𝑓 𝑝(𝑥). Let 𝑝(𝑥) = 𝑥 3 + 𝑎𝑥 2 + 7 𝑏𝑦 (𝑥 + 1).
Question.4. By remainder theorem, 𝑝(𝑥) is divided by (𝑥 + 1)
For what value of 𝒎 is the polynomial 𝒑(𝒙) = , then remainder is 2b
𝟒𝒙𝟑 − 𝟕𝒙𝟐 + 𝟔𝒙 − 𝟑𝒎 exactly divisible by 𝑝(−1) = 2𝑏
(𝒙 + 𝟐)? (−1) + 𝑎(−1)2 + 7 = 2𝑏
3

Solution. −1 + 𝑎 + 7 = 2𝑏
Suppose that 𝑎 + 6 = 2𝑏
𝑝(𝑥) = 4𝑥 3 − 7𝑥 2 + 6𝑥 − 3𝑚 𝑎 = 2𝑏 − 6 − − − (𝑖)
And 𝑥+2=0 By remainder theorem, 𝑝(𝑥) is divided by (𝑥 − 2)
𝑥 = −2. , then remainder is 𝑏 + 5
Remainder for 𝑥 + 2 𝑖𝑠 𝑝(2) = 𝑏 + 5
𝑝(−2) = 4(−2)3 − 7(−2)2 + 6(−2) − 3𝑚 (2) + 𝑎(2)2 + 7 = 𝑏 + 5
3

= 4(−8) − 7(4) − 12 − 3𝑚 8 + 𝑎(4) + 7 = 𝑏 + 5


= −32 − 28 − 12 − 3𝑚 4𝑎 + 15 = 𝑏 + 5
= −72 − 3𝑚 4𝑎 = 𝑏 − 10 − − − (𝑖𝑖)
For the given condition 𝑝(−2) = 0 Using (i) in (ii), we have
−72 − 3𝑚 = 0 4(2𝑏 − 6) = 𝑏 − 10
−3𝑚 = 72 8𝑏 − 24 = 𝑏 − 10
8|Page
Class 9th Chapter 5 www.notes.pk.com
8𝑏 − 𝑏 = −10 + 24 𝑚 = −2.
7𝑏 = 14 Hence 𝑙 = 2 𝑎𝑛𝑑 𝑚 = −2.
14 Question.8.
𝑏=
7 The expression 𝒍𝒙𝟑 + 𝒎𝒙𝟐 − 𝟒 𝒍𝒆𝒂𝒗𝒆𝒔 remainder
𝑏=2 −𝟑 𝒂𝒏𝒅 𝟏𝟐 𝒓𝒆𝒔𝒑𝒆𝒄𝒕𝒊𝒗𝒆𝒍𝒚 when divided by the
Using 𝑏 = 2 𝑖𝑛 (𝑖𝑖), 𝑤𝑒 ℎ𝑎𝑣𝑒 (𝒙 − 𝟏)𝒂𝒏𝒅 (𝒙 + 𝟐) respectively. Calculate the
4𝑎 = 2 − 10 values of 𝒍 𝒂𝒏𝒅 𝒎.
4𝑎 = −8 Solution.
8
𝑎=− Let 𝑝(𝑥) = 𝑙𝑥 3 + 𝑚𝑥 2 − 4.
4
𝑎 = −2 By remainder theorem, 𝑝(𝑥) is divided by (𝑥 − 1)
Hence 𝑎 = −2 𝑎𝑛𝑑 𝑏 = 2. , then remainder is 0.
Question.7. 𝑝(1) = −3
𝑙(1) + 𝑚(1)2 − 4 = −3
3
The polynomial 𝒙𝟑 + 𝒍𝒙𝟐 + 𝒎𝒙 + 𝟐𝟒 has a factor
𝑙 + 𝑚 = −3 + 4
(𝒙 + 𝟒) and it leaves a remainder of 𝟑𝟔 when
𝑙+𝑚 =1
divided by (𝒙 − 𝟐). Find the values of 𝒍 𝒂𝒏𝒅 𝒎.
𝑙 = 1 − 𝑚 − − − (𝑖)
Solution.
By remainder theorem, 𝑝(𝑥) is divided by (𝑥 + 2)
Let 𝑝(𝑥) = 𝑥 3 + 𝑙𝑥 2 + 𝑚𝑥 + 24.
, then remainder is 12.
By remainder theorem, 𝑝(𝑥) is divided by (𝑥 + 4)
𝑝(−2) = 12
, then remainder is 0.
𝑙(−2) + 𝑚(−2)2 − 4 = 12
3
𝑝(−4) = 0
−8𝑙 + 4𝑚 − 4 = 12
(−4) + 𝑙(−4)2 + 𝑚(−4) + 24 = 0
3
−8𝑙 + 4𝑚 = 12 + 4
−64 + 16𝑙 − 4𝑚 + 24 = 0 −8𝑙 + 4𝑚 = 16
16𝑙 − 4𝑚 − 40 = 0 4(−2𝑙 + 𝑚) = 16
4(4𝑙 − 𝑚 − 10) = 0 16
4𝑙 − 𝑚 − 10 = 0 −2𝑙 + 𝑚 =
4
4𝑙 − 𝑚 = 10 − − − (𝑖). −2𝑙 + 𝑚 = 4 − − − (𝑖𝑖)
By remainder theorem, 𝑝(𝑥) is divided by (𝑥 − 2) Using (i) in (ii) , we have
, then remainder is 36. −2(1 − 𝑚) + 𝑚 = 4
𝑝(2) = 36 −2 + 2𝑚 + 𝑚 = 4
(2) + 𝑙(2)2 + 𝑚(2) + 24 = 36
3
3𝑚 = 4 + 2
8 + 4𝑙 + 2𝑚 + 24 = 36 3𝑚 = 6
4𝑙 + 2𝑚 + 32 = 36 6
4𝑙 + 2𝑚 = 36 − 32 𝑚=
3
4𝑙 + 2𝑚 = 4 𝑚 = 2.
2(2𝑙 + 𝑚) = 4 Using value of 𝑚 in (ii), we have
4 −2𝑙 + 2 = 4
2𝑙 + 𝑚 =
2 −2𝑙 = 4 − 2
2𝑙 + 𝑚 = 2 − − − (𝑖𝑖) −2𝑙 = 2
Using (i) in (ii), we have 2
4𝑙 − 𝑚 = 10 𝑙=−
2
2𝑙 + 𝑚 = 2 𝑙 = −1.
_________________ Hence 𝑙 = −1 𝑎𝑛𝑑 𝑚 = 2.
6𝑙 = 12 Question.9.
12 The expression 𝒂𝒙𝟑 − 𝟗𝒙𝟐 + 𝒃𝒙 + 𝟑𝒂 is exactly
𝑙=
6 divisible by 𝒙𝟐 − 𝟓𝒙 + 𝟔. Find the values of
𝑙 = 2.
𝒂 𝒂𝒏𝒅 𝒃.
Using 𝑙 = 2 𝑖𝑛 (𝑖𝑖), 𝑤𝑒 ℎ𝑎𝑣𝑒
Solution.
2(2) + 𝑚 = 2
Let 𝑝(𝑥) = 𝑎𝑥 3 − 9𝑥 2 + 𝑏𝑥 + 3𝑎
4+𝑚 =2
As
𝑚 =2−4
9|Page
Class 9th Chapter 5 www.notes.pk.com
𝑥 2 − 5𝑥 + 6 = 0 Let
𝑥 2 − 3𝑥 − 2𝑥 + 6 = 0 𝑎0 𝑥 𝑛 + 𝑎1 𝑥 𝑛−1 + 𝑎2 𝑥 𝑛−2 + … + 𝑎𝑛−1 𝑥 +
𝑥(𝑥 − 3) − 2(𝑥 − 3) = 0 𝑎𝑛 , 𝑎𝑛 ≠ 0 …… (i)
(𝑥 − 3)(𝑥 − 2) = 0 Be a polynomial equation of degree 𝑛 with
𝑝
𝑥 − 3 = 0 ,𝑥 − 2 = 0 integral coefficients . If 𝑞 is a rational root of the
By remainder theorem, 𝑝(𝑥) is divided by (𝑥 − 3) equation, then 𝑝 is a factor of the constant term 𝑎𝑛
, then remainder is 0. and 𝑞 is the factor of leading coefficient 𝑎0 .
𝑝(3) = 0
𝑎(3) − 9(3)2 + 𝑏(3) + 3𝑎 = 0
3

27𝑎 − 81 + 3𝑏 + 3𝑎 = 0
EXERCISE# 5.4
Factorize each of the following cubic
30𝑎 + 3𝑏 = 81 − − − (𝑖)
polynomials by factor theorem.
By remainder theorem, 𝑝(𝑥) is divided by (𝑥 − 2)
Q#1) 𝒙𝟑 − 𝟐𝒙𝟐 − 𝒙 + 𝟐
, then remainder is 0.
Solution: Let 𝑃(𝑥) = 𝑥 3 − 2𝑥 2 − 𝑥 + 2…(1)
𝑝(2) = 0
Here, the constant term is 2 and factors of
𝑎(2) − 9(2)2 + 𝑏(2) + 3𝑎 = 0
3
constant terms are ±1, ±2, ±4, …
8𝑎 − 36 + 2𝑏 + 3𝑎 = 0
Therefore, we check ±1, ±2, ±4 for the roots.
11𝑎 + 2𝑏 = 36 − − − (𝑖𝑖)
Now, put 𝑥 = 1 in (1), we have
Multiply equation (i) by 2 and equation (ii) by 3 ,
𝑃(1) = (1)3 − 2(1)2 − (1) + 2
then subtracting eq. (ii) from (i), we have
𝑃(1) = 1 − 2 − 1 + 2
60𝑎 + 6𝑏 = 162 𝑃(1) = 0
±33𝑎 ± 6𝑏 = ±108 Hence, 𝑥 = 1 is the root of 𝑃(𝑥), therefore (𝑥 −
____________________________ 1) is the factor of 𝑃(𝑥).
27𝑎 = 54 Now, put 𝑥 = 2 in (1), we have
𝑎=2 𝑃(2) = (2)3 − 2(2)2 − (2) + 2
Put 𝑎 = 2 𝑖𝑛 𝑒𝑞. (𝑖), 𝑤𝑒 𝑔𝑒𝑡
𝑃(2) = 8 − 8 − 2 + 2
30(2) + 3𝑏 = 81 𝑃(2) = 0
60 + 3𝑏 = 81 Hence, 𝑥 = 2 is the root of 𝑃(𝑥), therefore (𝑥 −
3𝑏 = 81 − 60 2) is the factor of 𝑃(𝑥).
3𝑏 = 21 Now, put 𝑥 = −1 in (1), we have
𝑏=7 𝑃(−1) = (−1)3 − 2(−1)2 − (−1) + 2
Hence 𝑎 = 2 𝑎𝑛𝑑 𝑏 = 7. 𝑃(−1) = −1 − 2 + 1 + 2
𝑃(−1) = 0
Hence, 𝑥 = −1 is the root of 𝑃(𝑥), therefore
(𝑥 − (−1)) = (𝑥 + 1) is the factor of 𝑃(𝑥).
Thus, 𝑃(𝑥) = (𝑥 − 1)(𝑥 − 2)(𝑥 + 1)
Q#2) 𝒙𝟑 − 𝒙𝟐 − 𝟐𝟐𝒙 + 𝟒𝟎
Solution: Let 𝑃(𝑥) = 𝑥 3 − 𝑥 2 − 22𝑥 + 40…(1)
Here, the constant term is 40 and factors of
constant terms are ±1, ±2, ±4, ±5 …
Therefore, we check ±1, ±2, ±4, ±5 for the roots.
Now, put 𝑥 = 1 in (1), we have
𝑃(1) = (1)3 − (1)2 − 22(1) + 40
𝑃(1) = 1 − 1 − 22 + 40
𝑃(1) = 18 ≠ 0
Hence, 𝑥 = 1 is the not the root of 𝑃(𝑥),
Now, put 𝑥 = 2 in (1), we have
𝑃(2) = (2)3 − (2)2 − 22(2) + 40
𝑃(2) = 8 − 4 − 44 + 40
𝑃(2) = 48 − 48 = 0
Rational Root Theorem
10 | P a g e
Class 9th Chapter 5 www.notes.pk.com
Hence, 𝑥 = 2 is the root of 𝑃(𝑥), therefore (𝑥 − 𝑃(2) = (2)3 + (2)2 − 10(2) + 8
2) is the factor of 𝑃(𝑥). 𝑃(2) = 8 + 4 − 20 + 8
Now, put 𝑥 = 4 in (1), we have 𝑃(2) = 20 − 20
𝑃(4) = (4)3 − (4)2 − 22(4) + 40 Hence, 𝑥 = 2 is the root of 𝑃(𝑥), therefore (𝑥 −
𝑃(4) = 64 − 16 − 88 + 40 2) is also a factor of 𝑃(𝑥).
𝑃(4) = 104 − 104 = 0 Now, put 𝑥 = −4 in (1), we have
Hence, 𝑥 = 4 is the root of 𝑃(𝑥), therefore (𝑥 − 𝑃(−4) = (−4)3 + (−4)2 − 10(−4) + 8
1) is the factor of 𝑃(𝑥). 𝑃(−4) = −64 + 16 + 40 + 8
Now, put 𝑥 = −5 in (1), we have 𝑃(−4) = −64 + 64 = 0
𝑃(−5 ) = (−5 )3 − (−5 )2 − 22(−5 ) + 40 Hence, 𝑥 = −4 is the root of 𝑃(𝑥), therefore
𝑃(−5 ) = −125 − 25 + 110 + 40 (𝑥 − (−4)) = (𝑥 + 4) is the factor of 𝑃(𝑥).
𝑃(−5 ) = 150 − 150 = 0 Thus, 𝑃(𝑥) = (𝑥 − 1)(𝑥 − 2)(𝑥 + 4)
Hence, 𝑥 = −5 is the root of 𝑃(𝑥), therefore Q#5) 𝒙𝟑 − 𝟐𝒙𝟐 − 𝟓𝒙 + 𝟔
(𝑥 − (−5 )) = (𝑥 + 5) is the factor of 𝑃(𝑥). Solution: Let 𝑃(𝑥) = 𝑥 3 − 2𝑥 2 − 5𝑥 + 6…(1)
Thus, 𝑃(𝑥) = (𝑥 − 2)(𝑥 − 4)(𝑥 + 5) Here, the constant term is 2 and factors of
Q#3) 𝒙𝟑 − 𝟔𝒙𝟐 + 𝟑𝒙 + 𝟏𝟎 constant terms are ±1, ±2, ±3, …
Solution: Let 𝑃(𝑥) = 𝑥 3 − 6𝑥 2 + 3𝑥 + 10…(1) Therefore, we check ±1, ±2, ±3 for the roots.
Here, the constant term is 10 and factors of Now, put 𝑥 = 1 in (1), we have
constant terms are ±1, ±2, ±4, ±5 … 𝑃(1) = (1)3 − 2(1)2 − 5(1) + 6
Therefore, we check ±1, ±2, ±4, ±5 for the roots. 𝑃(1) = 1 − 2 − 5 + 6
Now, put 𝑥 = −1 in (1), we have 𝑃(1) = 7 − 7 = 0
𝑃(−1) = (−1)3 − 6(−1)2 + 3(−1) + 10 Hence, 𝑥 = 1 is the root of 𝑃(𝑥), therefore (𝑥 −
𝑃(−1) = −1 − 6 − 3 + 10 1) is the factor of 𝑃(𝑥).
𝑃(−1) = 0 Now, put 𝑥 = 3 in (1), we have
Hence, 𝑥 = −1 is the root of 𝑃(𝑥), therefore 𝑃(3) = (3)3 − 2(3)2 − 5(3) + 6
(𝑥— 1) = (𝑥 + 1) is the factor of 𝑃(𝑥). 𝑃(2) = 27 − 2(9) − 15 + 6
Now, put 𝑥 = 2 in (1), we have 𝑃(2) = 27 − 18 − 15 + 6 = 33 − 33 = 0
𝑃(2) = (2)3 − 6(2)2 + 3(2) + 10 Hence, 𝑥 = 3 is the root of 𝑃(𝑥), therefore (𝑥 −
𝑃(2) = 8 − 24 + 6 + 10 3) is the factor of 𝑃(𝑥).
𝑃(2) = 24 − 24 = 0 Now, put 𝑥 = −2 in (1), we have
Hence, 𝑥 = 2 is the root of 𝑃(𝑥), therefore (𝑥 − 𝑃(−2) = (−2)3 − 2(−2)2 − 5(−2) + 6
2) is the factor of 𝑃(𝑥). 𝑃(−2) = −8 − 2(4) + 10 + 6
Now, put 𝑥 = 5 in (1), we have 𝑃(−2) = −8 − 8 + 10 + 6 = −16 + 16 = 0
𝑃(5) = (5)3 − 6(5)2 + 3(5) + 10 Hence, 𝑥 = −2 is the root of 𝑃(𝑥), therefore
𝑃(5) = 125 − 6(25) + 15 + 10 (𝑥 − (−2)) = (𝑥 + 2) is the factor of 𝑃(𝑥).
𝑃(5) = 125 − 150 + 25 = 0 Thus, 𝑃(𝑥) = (𝑥 − 1)(𝑥 − 3)(𝑥 + 2)
Hence, 𝑥 = 5 is the root of 𝑃(𝑥), therefore (𝑥 − Q#6) 𝒙𝟑 + 𝟓𝒙𝟐 − 𝟐𝒙 − 𝟐𝟒
5) is the factor of 𝑃(𝑥). Solution: Let 𝑃(𝑥) = 𝑥 3 + 5𝑥 2 − 2𝑥 − 24…(1)
Thus, 𝑃(𝑥) = (𝑥 + 1)(𝑥 − 2)(𝑥 − 5) Here, the constant term is 24 and factors of
Q#4) 𝒙𝟑 + 𝒙𝟐 − 𝟏𝟎𝒙 + 𝟖 constant terms are ±1, ±2, ±4, …
Solution: Let 𝑃(𝑥) = 𝑥 3 + 𝑥 2 − 10𝑥 + 8…(1) Therefore, we check ±1, ±2, ±4 for the roots.
Here, the constant term is 8 and factors of Now, put 𝑥 = 2 in (1), we have
constant terms are ±1, ±2, ±4, … 𝑃(2) = (2)3 + 5(2)2 − 2(2) − 24
Therefore, we check ±1, ±2, ±4 for the roots. 𝑃(2) = 8 + 5(4) − 4 − 24
Now, put 𝑥 = 1 in (1), we have 𝑃(2) = 8 + 20 − 4 − 24 = 28 − 28 = 0
𝑃(1) = (1)3 + (1)2 − 10(1) + 8 Hence, 𝑥 = 2 is the root of 𝑃(𝑥), therefore (𝑥 −
𝑃(1) = 1 + 1 − 10 + 8 2) is the factor of 𝑃(𝑥).
𝑃(1) = 10 − 10 = 0 Now, put 𝑥 = −3 in (1), we have
Hence, 𝑥 = 1 is the root of 𝑃(𝑥), therefore (𝑥 − 𝑃(−3 ) = (−3 )3 + 5(−3 )2 − 2(−3 ) − 24
1) is the factor of 𝑃(𝑥). 𝑃(−3 ) = −27 + 5(9) + 6 − 24
Now, put 𝑥 = 2 in (1), we have
11 | P a g e
Class 9th Chapter 5 www.notes.pk.com
𝑃(−3 ) = −27 + 45 + 6 − 24 = 51 − 51 = 0 Hence, 𝑥 = 1 is the root of 𝑃(𝑥), therefore (𝑥 −
Hence, 𝑥 = −3 is the root of 𝑃(𝑥), therefore 1) is the factor of 𝑃(𝑥).
(𝑥 + 3) is the factor of 𝑃(𝑥). Now, put 𝑥 = −1 in (1), we have
Now, put 𝑥 = −4 in (1), we have 𝑃(−1) = 2(−1)3 + (−1)2 − 2(−1) − 1
𝑃(−4) = (−4 )3 + 5(−4 )2 − 2(−4 ) − 24 𝑃(−1) = −2 + 1 + 2 − 1
𝑃(−4 ) = −64 + 5(16) + 8 − 24 𝑃(−1) = −3 + 3 = 0
𝑃(−4 ) = −64 + 80 + 8 − 24 = 88 − 88 = 0 Hence, 𝑥 = −1 is the root of 𝑃(𝑥), therefore
Hence, 𝑥 = −4 is the root of 𝑃(𝑥), therefore (𝑥 − (−1)) = (𝑥 + 1) is the factor of 𝑃(𝑥).
(𝑥 − (−4)) = (𝑥 + 4) is the factor of 𝑃(𝑥). Since the leading co-efficient is 2, therefore we
Thus, 𝑃(𝑥) = (𝑥 − 2)(𝑥 + 3)(𝑥 + 4) 1 1
check 𝑥 = − 2 and 𝑥 = 2
Q#7) 𝟑𝒙𝟑 − 𝒙𝟐 − 𝟏𝟐𝒙 + 𝟒 1
First we check at 𝑥 = − 2
Solution: Let 𝑃(𝑥) = 3𝑥 3 − 𝑥 2 − 12𝑥 + 4…(1)
Here, the constant term is 4 and factors of 1 1 3 1 2 1
𝑃 (− ) = 2 (− ) + (− ) − 2 (− ) − 1
constant terms are ±1, ±2, ±4, … 2 2 2 2
Therefore, we check ±1, ±2, ±4 for the roots. 1 1 1
𝑃 (− ) = 2 ( ) − ( ) + 1 − 1
Now, put 𝑥 = 2 in (1), we have 2 8 4
1 1 1
𝑃(2) = 3(2)3 − (2)2 − 12(2) + 4 𝑃 (− ) = − + 1 − 1 = 0
𝑃(2) = 3(8) − 4 − 24 + 4 2 4 4
1
𝑃(2) = 24 − 4 − 24 + 4 = 0 Hence, 𝑥 = − 2 is the root of 𝑃(𝑥), therefore
Hence, 𝑥 = 2 is the root of 𝑃(𝑥), therefore (𝑥 − 1
(𝑥 + 2) = 0 gives that (2𝑥 + 1) is the factor of
2) is the factor of 𝑃(𝑥).
𝑃(𝑥).
Now, put 𝑥 = −2 in (1), we have
Thus, 𝑃(𝑥) = (𝑥 − 1)(𝑥 + 1)(2𝑥 + 1)
𝑃(−2) = 3(−2)3 − (−2)2 − 12(−2) + 4
𝑃(−2) = 3(−8) − 4 + 24 + 4
𝑃(−2) = −24 − 4 + 24 + 4 = 0

Contact or Suggest Us: info@notespk.com


A project of: www.notespk.com
Hence, 𝑥 = −2 is the root of 𝑃(𝑥), therefore
(𝑥 − (−2)) = (𝑥 + 2) is the factor of 𝑃(𝑥).
Since the leading co-efficient is 3, therefore we
1 1
also check at 𝑥 = 3 and 𝑥 = − 3
1
First we check at 𝑥 = 3
1 1 3 1 2 1
𝑃 ( ) = 3 ( ) − ( ) − 12 ( ) + 4
3 3 3 3
1 1 1
𝑃( ) = 3( ) − ( ) − 4 + 4
3 27 9
1 1 1
𝑃( ) = − −4+4 = 0
3 9 9
1
Hence, 𝑥 = 3 is the root of 𝑃(𝑥), therefore
1
(𝑥 − 3) = 0 gives that (3𝑥 − 1) is the factor of
𝑃(𝑥).
Thus, 𝑃(𝑥) = (𝑥 − 2)(𝑥 + 2)(3𝑥 − 1)
Q#8) 𝟐𝒙𝟑 + 𝒙𝟐 − 𝟐𝒙 − 𝟏
Solution: Let 𝑃(𝑥) = 2𝑥 3 + 𝑥 2 − 2𝑥 − 1…(1)
Here, the constant term is 1 and factors of
constant terms are ±1 .
Therefore, we check ±1 for the roots.
Now, put 𝑥 = 1 in (1), we have
𝑃(1) = 2(1)3 + (1)2 − 2(1) − 1
𝑃(1) = 2 + 1 − 2 − 1 Complied by Shumaila Amin
𝑃(1) = 3 − 3 = 0
12 | P a g e
MATHEMATICS 9th Science Group

7/18/2020
Chapter 7.
LINEAR EQUATIONS AND
INEQUALITIES

A project of: www.notespk.com


Contact or Suggest Us: info@notespk.com
Contents
EXERCISE 7.1 ............................................................................ 1
EXERCISE 7.2 ............................................................................ 6
EXERCISE 7.3 ............................................................................ 9
Class 9th Chapter 7 www.notes.pk.com

Radical Equation Multiply by 6(LCM) on both sides


When the variable in an equation occurs under a 𝑥−3 𝑥−2
6×( )−6×( ) = 6 × (−1)
radical, the equation is called a radical equation. 3 2
For example, 2(𝑥 − 3) − 3(𝑥 − 2) = −6
2𝑥 − 6 − 3𝑥 + 6 = −6
√𝑥 − 3 − 7 = 0
−𝑥 = −6
Linear Equation
𝑥=6
A linear equation in one unknown variable x is an
Check:
equation of the form 𝑎𝑥 + 𝑏 = 0, where 𝑎, 𝑏 ∈ 𝑥−3 𝑥−2
𝑅 𝑎𝑛𝑑 𝑎 ≠ 0 − = −1
3 2
A solution to a linear equation is any replacement Put 𝑥 = 6
or substitution for the variable x that makes the (6) − 3 (6) − 2
statement true. Two linear equations are said to be − = −1
3 2
equivalent if they have exactly the same solution. 3 4
For example, 𝑥 + 1 = 0, 2𝑥 + 5 = −1 − = −1
3 2
1 − 2 = −1
EXERCISE 7.1 −1 = −1 (which is true)
Q#1) Solve the following equations. Since 𝑥 = 6 satisfy the given equation, therefore,
𝟐 𝟏
(i). 𝟑 𝒙 − 𝟐 𝒙 = 𝒙 + 𝟔
𝟏 the solution set is {6} i.e. 𝑆. 𝑆 = {6}
𝟏 𝟏 𝟐 𝟓 𝟏 𝟏
2 1 1 (ii). 𝟐 (𝒙 − 𝟔) + 𝟑 = 𝟔 + 𝟑 (𝟐 − 𝟑𝒙)
Solution: As given 3 𝑥 − 2 𝑥 = 𝑥 + 6
Solution:
Multiply by 6(LCM) on both sides 1 1 2 5 1 1
2 1 1 As given 2 (𝑥 − 6) + 3 = 6 + 3 (2 − 3𝑥)
6× 𝑥−6× 𝑥 =6×𝑥+6×
3 2 6 1 1 2 5 1
4𝑥 − 3𝑥 = 6𝑥 + 1 𝑥− + = + −𝑥
2 12 3 6 6
𝑥 = 6𝑥 + 1 Multiply by 12(LCM) on both sides
1 = 𝑥 − 6𝑥 1 1
12 × ( 𝑥) − 12 × ( )
1 = −5𝑥 2 12
1 2 5 1
𝑥=− +12 × ( ) = 12 × ( ) + 12 × ( ) − 12 × (𝑥)
5 3 6 6
Check: 6𝑥 − 1 + 8 = 10 + 2 − 12𝑥
2 1 1 6𝑥 + 7 = 12 − 12𝑥
𝑥− 𝑥=𝑥+
3 2 6 6𝑥 + 12𝑥 = 12 − 7
1
Put 𝑥 = − 5 18𝑥 = 5
2 1 1 1 1 1 5
(− ) − (− ) = (− ) + 𝑥=
3 5 2 5 5 6 18
2 1 1 1 Check:
− + =− + 1 1 2 5 1 1
15 10 5 6 (𝑥 − ) + = + ( − 3𝑥)
Multiply by 30(LCM) on both sides 2 6 3 6 3 2
2 1 1 1 2 5 1
30 × (− ) + 30 × ( ) 𝑥− + = + −𝑥
15 10 2 12 3 6 6
5
1 1 Put 𝑥 = 18
= 30 × (− ) + 30 × ( )
5 6 1 5 1 2 5 1 5
−4 + 3 = −6 + 5 ( )− + = + −( )
2 18 12 3 6 6 18
−1 = −1(which is true) 5 1 2 5 1 5
1 − + = + −
Since 𝑥 = − 5 satisfy the given equation, 36 12 3 6 6 18
1 1 Multiply by 36(LCM) on both sides
therefore, the solution set is {− 5} i.e. 𝑆. 𝑆 = {− 5} 5 1
𝒙−𝟑 𝒙−𝟐 36 × ( ) − 36 × ( )
(ii). − = −𝟏 36 12
𝟑 𝟐
𝑥−3 𝑥−2 2 5 1 5
Solution: As given − = −1 +36 × ( ) = 36 × ( ) + 36 × ( ) − 36 × ( )
3 2 3 6 6 18
1|Page
Class 9th Chapter 7 www.notes.pk.com
5 − 3 + 24 = 30 + 6 − 10 −𝑥 = 63
−3 + 29 = 36 − 10 𝑥 = −63
26 = 26 (which is true) Check:
5
Since 𝑥 = 18 satisfy the given equation, therefore, 5(𝑥 − 3) 𝑥
−𝑥 =1−
5 5 6 9
the solution set is {18} i.e. 𝑆. 𝑆 = {18} Put 𝑥 = −63
𝟏 𝟐
(iv). 𝒙 + 𝟑 = 𝟐 (𝒙 − 𝟑) − 𝟔𝒙 5(−63 − 3) (−63)
− (−63) = 1 −
1 2 6 9
Solution: As given 𝑥 + 3 = 2 (𝑥 − 3) − 6𝑥 5(−66) 63
+ 63 = 1 +
1 4 6 9
𝑥 + = 2𝑥 − − 6𝑥 −55 + 63 = 1 + 7
3 3
Multiply by 3(LCM) on both sides 8 = 8 (which is true)
1 Since 𝑥 = −63 satisfy the given equation,
3 × (𝑥) + 3 × ( )
3 therefore, the solution set is {−63} i.e. 𝑆. 𝑆 =
4 {−63}
= 3 × (2𝑥) − 3 × ( ) − 3 × (6𝑥)
3 𝒙 𝟐𝒙
(vi). 𝟑𝒙−𝟔 = 𝟐 − 𝒙−𝟐, 𝒙 ≠ 𝟐
3𝑥 + 1 = 6𝑥 − 4 − 18𝑥 𝑥 2𝑥
3𝑥 + 1 = −4 − 12𝑥 Solution: As given 3𝑥−6 = 2 − 𝑥−2
3𝑥 + 12 = −4 − 1 𝑥 2(𝑥 − 2) − 2𝑥
15𝑥 = −5 =
3𝑥 − 6 𝑥−2
5 𝑥 2𝑥 − 4 − 2𝑥
𝑥=− =
15 3𝑥 − 6 𝑥−2
1 𝑥 −4
𝑥=− =
3 3𝑥 − 6 𝑥 − 2
Check: 𝑥(𝑥 − 2) = −4(3𝑥 − 6)
1 2 𝑥 2 − 2𝑥 = −12𝑥 + 24
𝑥 + = 2 (𝑥 − ) − 6𝑥
3 3 𝑥 2 − 2𝑥 + 12𝑥 − 24 = 0
1 4 𝑥(𝑥 − 2) + 12(𝑥 − 2) = 0
𝑥 + = 2𝑥 − − 6𝑥
3 3 (𝑥 − 2)(𝑥 + 12) = 0
1
Put 𝑥 = − 3 That is 𝑥 = 2, −12
1 1 1 4 1 Since it is given that 𝑥 ≠ 2, therefore, we ignore
(− ) + = 2 (− ) − − 6 (− ) 𝑥 = 2 and just check 𝑥 = −12 for the solution
3 3 3 3 3
1 1 2 4 set.
− + =− − +2
3 3 3 3 Check:
Multiply by 3(LCM) on both sides 𝑥 2𝑥
−1 + 1 = −2 − 4 + 6 = 2−
3𝑥 − 6 𝑥−2
0 = 0 (which is true) Put 𝑥 = −12
1
Since 𝑥 = − 3 satisfy the given equation, (−12) 2(−12)
= 2−
1 1 3(−12) − 6 (−12) − 2
therefore, the solution set is {− 3} i.e. 𝑆. 𝑆 = {− 3} −12 24
𝟓(𝒙−𝟑) 𝒙 =2+
(v) −𝒙=𝟏−𝟗 −36 − 6 −12 − 2
𝟔 −12 24
5(𝑥−3) 𝑥
Solution: As given −𝑥 =1−9 =2+
6 −42 −14
Multiply by 18 (LCM) on both sides 2 28 − 24
=
5(𝑥 − 3) 7 14
18 × ( ) − 18 × (𝑥) 2 4
6 =
𝑥 7 14
= 18 × (1) − 18 × ( ) 2 2
= 7 (which is true)
9 7
3(5𝑥 − 15) − 18𝑥 = 18 − 2𝑥 Since 𝑥 = −12 satisfy the given equation,
15𝑥 − 45 − 18𝑥 = 18 − 2𝑥 therefore, the solution set is {−12} i.e. 𝑆. 𝑆 =
−45 − 3𝑥 = 18 − 2𝑥 {−12}
−3𝑥 + 2𝑥 = 18 + 45
2|Page
Class 9th Chapter 7 www.notes.pk.com
𝟐𝒙 𝟐 𝟓
(vii). 𝟐𝒙+𝟓 = 𝟑 − 𝟒𝒙+𝟏𝟎, 𝒙 ≠ 𝟐
𝟔 7𝑥 − 1 5𝑥 + 7
=
2𝑥 2 5 3(𝑥 − 1) 6(𝑥 − 1)
Solution: As given 2𝑥+5 = 3 − 4𝑥+10
(7𝑥 − 1)6(𝑥 − 1) = 3(𝑥 − 1)(5𝑥 + 7)
2𝑥 2(4𝑥 + 10) − 15 6(7𝑥 − 1)(𝑥 − 1) − 3(𝑥 − 1)(5𝑥 + 7) = 0
=
2𝑥 + 5 3(4𝑥 + 10) 3(𝑥 − 1)[2(7𝑥 − 1) − (5𝑥 + 7)] = 0
2𝑥 8𝑥 + 20 − 15 3(𝑥 − 1)(14𝑥 − 2 − 5𝑥 − 7) = 0
=
2𝑥 + 5 12𝑥 + 30 3(𝑥 − 1)(9𝑥 − 9) = 0
2𝑥 8𝑥 + 5 3(𝑥 − 1)9(𝑥 − 1) = 0
=
2𝑥 + 5 12𝑥 + 30 27(𝑥 − 1)2 = 0
2𝑥(12𝑥 + 30) = (8𝑥 + 5)(2𝑥 + 5) Which implies that
24𝑥 2 + 60𝑥 = 16𝑥 2 + 40𝑥 + 10𝑥 + 25 (𝑥 − 1) = 0 gives that 𝑥 = 1 which is not
24𝑥 2 + 60𝑥 = 16𝑥 2 + 50𝑥 + 25 possible (given 𝑥 ≠ 1)
24𝑥 2 + 60𝑥 − 16𝑥 2 − 50𝑥 − 25 = 0 therefore, the solution set is {} i.e. 𝑆. 𝑆 = {}
8𝑥 2 + 10𝑥 − 25 = 0 𝟐 𝟏 𝟏
(ix). 𝒙𝟐 −𝟏 − 𝒙+𝟏 = 𝒙+𝟏, 𝒙 ≠ ±𝟏
8𝑥 2 + 20𝑥 − 10𝑥 − 25 = 0
2 1 1
4𝑥(2𝑥 + 5) − 5(2𝑥 + 5) = 0 Solution: As given 𝑥 2 −1 − 𝑥+1 = 𝑥+1
(2𝑥 + 5)(4𝑥 − 5) = 0 2 1 1
5 5 − =
That is 𝑥 = − 2 , 4 (𝑥 + 1)(𝑥 − 1) 𝑥 + 1 𝑥 + 1
5 2 − (𝑥 − 1) 1
Since it is given that 𝑥 ≠ − , therefore, we ignore =
5
2
5
(𝑥 + 1)(𝑥 − 1) 𝑥 + 1
𝑥 = − 2 and just check 𝑥 = 4 for the solution set. 2−𝑥+1 1
=
Check: (𝑥 + 1)(𝑥 − 1) 𝑥 + 1
2𝑥 2 5𝑥 3−𝑥 1
= − =
2𝑥 + 5 3 4𝑥 + 10 (𝑥 + 1)(𝑥 − 1) 𝑥 + 1
5 (3 − 𝑥)(𝑥 + 1) = (𝑥 + 1)(𝑥 − 1)
Put 𝑥 = 4
5 (3 − 𝑥)(𝑥 + 1) − (𝑥 + 1)(𝑥 − 1) = 0
2 (4 ) 2 5 (𝑥 + 1)[(3 − 𝑥) − (𝑥 − 1)] = 0
= −
5 3 4 (5 ) + 10 (𝑥 + 1)(3 − 𝑥 − 𝑥 + 1) = 0
2 (4 ) + 5 4 (𝑥 + 1)(4 − 2𝑥) = 0
5 That is 𝑥 = −1, 2
2 =2− 5
5 3 5 + 10 Since it is given that 𝑥 ≠ ±1, therefore, we ignore
2+5 𝑥 = −1 and just check 𝑥 = 2 for the solution set.
5 Check:
2 2 5 2 1 1
= − − =
5 + 10 3 15 2
𝑥 −1 𝑥+1 𝑥+1
2
5 2 1 Put 𝑥 = 2
= − 2 1 1
15 3 3 2
− =
1 1 (2) − 1 (2) + 1 (2) + 1
= 3 (which is true)
3 2 1 1
5
Since 𝑥 = 4 satisfy the given equation, therefore, − =
4−1 3 3
5 5 2 1 1
the solution set is { } i.e. 𝑆. 𝑆 = { } − =
4 4 3 3 3
𝟐𝒙 𝟏 𝟓 𝟐 1 1
(viii). 𝒙−𝟏 + 𝟑 = 𝟔 + 𝒙−𝟏, 𝒙 ≠ 𝟏 = 3 (which is true)
3
2𝑥 1 5 2
Solution: As given 𝑥−1 + 3 = 6 + 𝑥−1 Since 𝑥 = 2 satisfy the given equation, therefore,
2𝑥 1 5 2 the solution set is {2} i.e. 𝑆. 𝑆 = {2}
+ = + 𝟐 𝟏 𝟏
(x). 𝟑𝒙+𝟔 = 𝟔 − 𝟐𝒙+𝟒, 𝒙 ≠ −𝟐
𝑥−1 3 6 𝑥−1
3(2𝑥) + (𝑥 − 1) 5(𝑥 − 1) + 2(6) 2 1 1
= Solution: As given 3𝑥+6 = 6 − 2𝑥+4
3(𝑥 − 1) 6(𝑥 − 1)
6𝑥 + 𝑥 − 1 5𝑥 − 5 + 12 2 1(2𝑥 + 4) − 6
= =
3(𝑥 − 1) 6(𝑥 − 1) 3𝑥 + 6 6(2𝑥 + 4)

3|Page
Class 9th Chapter 7 www.notes.pk.com
2 2𝑥 + 4 − 6 Since 𝑥 = 0 satisfy the given equation, therefore,
=
3𝑥 + 6 6(2𝑥 + 4) the solution set is {0} i.e. 𝑆. 𝑆 = {0}
2 2𝑥 − 2 𝟑
(ii). √𝟐𝒙 − 𝟒 − 𝟐 = 𝟎
=
3𝑥 + 6 6(2𝑥 + 4) 3
Solution: As given √2𝑥 − 4 − 2 = 0
2 2(𝑥 − 1) 3
= √2𝑥 − 4 = 2
3(𝑥 + 2) 6(2(𝑥 + 2)) Taking cube on both sides
2 (𝑥 − 1) 3 3
= ( √2𝑥 − 4) = (2)3
3(𝑥 + 2) 6(𝑥 + 2)
12(𝑥 + 2) = 3(𝑥 + 2)(𝑥 − 1) 2𝑥 − 4 = 8
12(𝑥 + 2) − 3(𝑥 + 2)(𝑥 − 1) = 0 2𝑥 = 8 + 4
2𝑥 = 12
3(𝑥 + 2)[4 − 𝑥 + 1)] = 0 𝑥=6
3(𝑥 + 2)(5 − 𝑥) = 0 Check:
3
That is 𝑥 = −2, 5 √2𝑥 − 4 − 2 = 0
Since it is given that 𝑥 ≠ −2, therefore, we ignore Put 𝑥 = 6
3
𝑥 = −2 and just check 𝑥 = 5 for the solution set. √2(6) − 4 − 2 = 0
3
Check: √12 − 4 − 2 = 0
2 1 1 3
= − √8 − 2 = 0
3𝑥 + 6 6 2𝑥 + 4 2−2=0
Put 𝑥 = 5
0 = 0 (which is true)
2 1 1
= − Since 𝑥 = 6 satisfy the given equation, therefore,
3(5) + 6 6 2(5) + 4 the solution set is {6} i.e. 𝑆. 𝑆 = {6}
2 1 1
= − (ii). √𝒙 − 𝟑 − 𝟕 = 𝟎
15 + 6 6 10 + 4
2 1 1 Solution: As given √𝑥 − 3 − 7 = 0
= −
21 6 14 √𝑥 − 3 = 7
2 7−3 Taking square on both sides
= 2
21 42 (√𝑥 − 3) = (7)2
2 2
= 𝑥 − 3 = 49
21 42
𝑥 = 49 + 3
2 2 𝑥 = 52
= 21 (which is true)
21 Check:
Since 𝑥 = 5 satisfy the given equation, therefore,
√𝑥 − 3 − 7 = 0
the solution set is {5} i.e. 𝑆. 𝑆 = {5}
Put 𝑥 = 52
Q#2) Solve each equation and check for
√52 − 3 − 7 = 0
extraneous solution if any.
(i). √𝟑𝒙 + 𝟒 = 𝟐 √49 − 7 = 0
7−7=0
Solution: As given √3𝑥 + 4 = 2
0 = 0 (which is true)
On squaring, we get
2 Since 𝑥 = 52 satisfy the given equation, therefore,
(√3𝑥 + 4) = (2)2 the solution set is {52} i.e. 𝑆. 𝑆 = {52}
3𝑥 + 4 = 4 (iii). 𝟐√𝒕 + 𝟒 = 𝟓
3𝑥 = 0 Solution: As given 2√𝑡 + 4 = 5
𝑥=0 Taking saquare on both sides
Check: 2
(2√𝑡 + 4) = (5)2
√3𝑥 + 4 = 2
Put 𝑥 = 0 4(𝑡 + 4) = 25
4𝑡 + 16 = 25
√3(0) + 4 = 2
4𝑡 = 25 − 16
√0 + 4 = 2 9
2 = 2 (which is true) 𝑡=
4
Check:
4|Page
Class 9th Chapter 7 www.notes.pk.com
2√𝑡 + 4 = 5 Taking cube root, we have
Put 𝑡 = 2
9 −8 = −8 (which is true)
Since t= 10 satisfy the given equation, therefore,
9 the solution set is {10} i.e. 𝑆. 𝑆 = {10}
2√ + 4 = 5
4 𝒙+𝟏 𝟓
(viii). √𝟐𝒙+𝟓 = 𝟐, 𝒙 ≠ − 𝟐
9 + 16 𝑥+1
2√ =5 Solution: As given √2𝑥+5 = 2
4
Taking square on both sides
25 2
2√ = 5 𝑥+1
4 (√ ) = (2)2
5 2𝑥 + 5
2( ) = 5
2 𝑥+1
5 = 5 (which is true) =4
2𝑥 + 5
9
Since 𝑡 = 4 satisfy the given equation, therefore, 𝑥 + 1 = 4(2𝑥 + 5)
9 9 𝑥 + 1 = 8𝑥 + 20
the solution set is {4} i.e. 𝑆. 𝑆 = {4}
1 − 20 = 8𝑥 − 𝑥
𝟑 𝟑
(v). √𝟐𝒙 + 𝟑 = √𝒙 − 𝟐 −19 = 7𝑥
3 3
Solution: As given √2𝑥 + 3 = √𝑥 − 2 19
𝑥=−
Taking cube on both sides 7
3 3 3 3 Check:
( √2𝑥 + 3) = ( √𝑥 − 2)
2𝑥 + 3 = 𝑥 − 2 𝑥+1
√ =2
2𝑥 − 𝑥 = −2 − 3 2𝑥 + 5
𝑥 = −5 19
Put 𝑥 = −
Check: 7
3
√2𝑥 + 3 = √𝑥 − 2
3
19
(− 7 ) + 1
Put 𝑥 = −5 √ =2
19
3 3
√2(−5) + 3 = √(−5) − 2 2 (− ) + 5
7
3 3
√−10 + 3 = √−5 − 2 −19 + 7
3 3
√−7 = √−7 √ 7 =2
Taking cube root, we have −38 + 35
−7 = −7 (which is true) 7
Since 𝑥 = −5 satisfy the given equation, −12
therefore, the solution set is {−5} i.e. 𝑆. 𝑆 = {−5} √ 7 =2
𝟑 𝟑 −3
(v). √𝟐 − 𝒕 = √𝟐𝒕 − 𝟐𝟖 7
3 3
Solution: As given √2 − 𝑡 = √2𝑡 − 28
Taking cube on both sides
3 3 3 3 12
( √2 − 𝑡) = ( √2𝑡 − 28) √ =2
3
2 − 𝑡 = 2𝑡 − 28
2 + 28 = 2𝑡 + 𝑡 √4 = 2
30 = 3𝑡 2 = 2 (which is true)
19
𝑡 = 10 Since 𝑥 = − 7 satisfy the given equation,
Check: 19
3 3 therefore, the solution set is {− 7 } i.e. 𝑆. 𝑆 =
√2 − 𝑡 = √2𝑡 − 28 19
Put 𝑡 = 10 {− 7 }
3 3
√2 − 10 = √2(10) − 28
3 3
√−8 = √20 − 28
3 3
√−8 = √−8

5|Page
Class 9th Chapter 7 www.notes.pk.com
1
Absolute Value Since 𝑥 = 3, 3 satisfy the given equation,
The Absolute value of real number ′𝑎′ is denoted 1 1
therefore, the solution set is {3, 3} i.e. 𝑆. 𝑆 = {3, 3}
by |𝑎|, is defined as
𝟏
𝑎 𝑖𝑓 𝑎 ≥ 0 (ii). 𝟐 |𝟑𝒙 + 𝟐| − 𝟒 = 𝟏𝟏
|𝑎| = {
−𝑎 𝑖𝑓 𝑎 < 0 1
Solution: As given 2 |3𝑥 + 2| − 4 = 11
For example, |6| = 6, |−5| = −(−5) = 5
|0| = 0 1
|3𝑥 + 2| = 11 + 4
Some Properties of Absolute value 2
1
If 𝑎, 𝑏 ∈ 𝑅, then |3𝑥 + 2| = 15
2
(i). |𝑎| ≥ 0 |3𝑥 + 2| = 30
(ii). |−𝑎| = |𝑎| By definition, we have
(iii). |𝑎𝑏| = |𝑎||𝑏| 3𝑥 + 2 = 30 or 3𝑥 + 2 = −30
𝑎 |𝑎|
(iv). |𝑏| = |𝑏|, |𝑏| ≠ 0 3𝑥 = 30 − 2 or 3𝑥 = −30 − 2
32
3𝑥 = 28 or 3𝑥 = − 3
EXERCISE 7.2 𝑥=
28
3
or 𝑥 = −
32
3
Q#1) 1. Identify the following statements as Check:
True or False. 1
|3𝑥 + 2| − 4 = 11…(1)
(i) | 𝑥 | = 0 has only one solution. …T… 28
2

(ii) All absolute value equations have two Put 𝑥 = , in (1)


3
solutions. F … 1 28
|3 ( ) + 2| − 4 = 11
(iii) The equation | 𝑥 | = 2 is equivalent to 𝑥 = 2 3
2 𝑜𝑟 𝑥 = – 2. …T…… 1
|28 + 2| − 4 = 11
(iv) The equation | 𝑥 – 4 | = – 4 has no solution. 2
1
…F |30| − 4 = 11
2
(v) The equation | 2𝑥 – 3 | = 5 is equivalent to 1
2𝑥 – 3 = 5 𝑜𝑟 2𝑥 + 3 = 5 . …F … (30) − 4 = 11
2
Q#2) Solve for 𝑥 15 − 4 = 11
(i). |3𝑥 − 5| = 4 11 = 11 (which is true)
Sol: As given |3𝑥 − 5| = 4 32
Put 𝑥 = − 3 , in (1)
By definition, we have 1 32
3𝑥 − 5 = 4 or 3𝑥 − 5 = −4 |3 (− ) + 2| − 4 = 11
2 3
3𝑥 = 4 + 5 or 3𝑥 = −4 + 5 1
3𝑥 = 9 or 3𝑥 = 1 |−32 + 2| − 4 = 11
1
2
𝑥 = 3 or 𝑥 = 3 1
|−30| − 4 = 11
Check: 2
1
|3𝑥 − 5| = 4…(1) (30) − 4 = 11
Put 𝑥 = 3, in (1) 2
15 − 4 = 11
|3(3) − 5| = 4
11 = 11 (which is true)
|9 − 5| = 4 28 32
|4| = 4 Since 𝑥 = ,− satisfy the given equation,
3 3
28 32
4 = 4 (which is true) therefore, the solution set is { 3 , − } i.e. 𝑆. 𝑆 =
1 3
Put 𝑥 = 3, in (1) 28 32
{3 ,− }
3
1
|3 ( ) − 5| = 4 (iii). |𝟐𝒙 + 𝟓| = 𝟏𝟏
3 Solution: As given |2𝑥 + 5| = 11
|1 − 5| = 4
By definition, we have
|−4| = 4
2𝑥 + 5 = 11 or 2𝑥 + 5 = −11
4 = 4 (which is true)
2𝑥 = 11 − 5 or 2𝑥 = −11 − 5
2𝑥 = 6 or 2𝑥 = −16

6|Page
Class 9th Chapter 7 www.notes.pk.com
𝑥 = 3 or 𝑥 = −8 2|𝑥 + 2| = 8
Check: |𝑥 + 2| = 4
|2𝑥 + 5| = 11…(1) By definition, we have
Put 𝑥 = 3, in (1) 𝑥 + 2 = 4 or 𝑥 + 2 = −4
|2(3) + 5| = 11 𝑥 = 4 − 2 or 𝑥 = −4 − 2
|6 + 5| = 11 𝑥 = 2 or 𝑥 = −6
|11| = 11 Check:
11 = 11 (which is true) |𝑥 + 2| − 3 = 5 − |𝑥 + 2|…(1)
Put 𝑥 = −8, in (1) Put 𝑥 = 2, in (1)
|2(−8) + 5| = 11 |2 + 2| − 3 = 5 − |2 + 2|
|−16 + 5| = 11 |4| − 3 = 5 − |4|
|−11| = 11 4−3=5−4
11 = 11 (which is true) 1 = 1 (which is true)
Since 𝑥 = 3, −8 satisfy the given equation, Put 𝑥 = −6, in (1)
therefore, the solution set is {3, −8} i.e. 𝑆. 𝑆 = |−6 + 2| − 3 = 5 − |−6 + 2|
{3, −8} |−4| − 3 = 5 − |−4|
(iii). |𝟑 + 𝟐𝒙| = |𝟔𝒙 − 𝟕| 4−3=5−4
Solution: As given |3 + 2𝑥| = |6𝑥 − 7| 1 = 1 (which is true)
|3 + 2𝑥| Since 𝑥 = 2, −6 satisfy the given equation,
=1
|6𝑥 − 7| therefore, the solution set is {2, −6} i.e. 𝑆. 𝑆 =
3 + 2𝑥 {2, −6}
| |=1 𝟏
6𝑥 − 7 (vi). |𝒙 + 𝟑| + 𝟐𝟏 = 𝟗
𝟐
By definition, we have 1
3+2𝑥 3+2𝑥 Solution: As given 2 |𝑥 + 3| + 21 = 9
= 1 or 6𝑥−7 = −1
6𝑥−7 1
3 + 2𝑥 = 6𝑥 − 7 or 3 + 2𝑥 = −6𝑥 + 7 |𝑥 + 3| = 9 − 21
2
3 + 7 = 6𝑥 − 2𝑥 or 2𝑥 + 6𝑥 = 7 − 3 1
10 = 4𝑥 or 8𝑥 = 4 |𝑥 + 3| = −12
5 1
2
𝑥 = 2 or 𝑥 = 2 |𝑥 + 3| = −24
Check: Which is not possible as modulus value is always
|3 + 2𝑥| = |6𝑥 − 7|…(1) non-negative.
5 3−5𝑥 1 2
Put 𝑥 = 2, in (1) (vii). | |−3=3
4
3−5𝑥 1 2
5 5 Sol: As given | |−3 = 3
|3 + 2 ( )| = |6 ( ) − 7| 4
2 2 3 − 5𝑥 2 1
|3 + 5| = |15 − 7| | |= +
4 3 3
|8| = |8| 3 − 5𝑥 2+1
8 = 8 (which is true) | |=
1
4 3
Put 𝑥 = 2, in (1) 3 − 5𝑥 3
| |=
1 1 4 3
|3 + 2 ( )| = |6 ( ) − 7| 3 − 5𝑥
2 2 | |=1
|3 + 1| = |3 − 7| 4
|4| = |−4| |3 − 5𝑥| = 4
4 = 4 (which is true) By definition, we have
5 1 3 − 5𝑥 = 4 or 3 − 5𝑥 = −4
Since 𝑥 = 2 , 2 satisfy the given equation,
3 − 4 = 5𝑥 or 3 + 4 = 5𝑥
5 1
therefore, the solution set is {2 , 2 } i.e. 𝑆. 𝑆 = −1 = 5𝑥 or 7 = 5𝑥
5 1 1 7
{2 , 2 } 𝑥 = − 5 or 𝑥 = 5
(v). |𝒙 + 𝟐| − 𝟑 = 𝟓 − |𝒙 + 𝟐| Check:
3−5𝑥 1 2
Solution: As given |𝑥 + 2| − 3 = 5 − |𝑥 + 2| | | − 3 = 3…(1)
4
|𝑥 + 2| + |𝑥 + 2| = 5 + 3 1
Put 𝑥 = − 5, in (1)
7|Page
Class 9th Chapter 7 www.notes.pk.com
1 |6| = 6
3 − 5 (− )
| 5 |−1=2 6 = 6 (which is true)
4 3 3 17
Put 𝑥 = 5 , in (1)
3+1 1 2 17
| |− = ( )+5
4 3 3 | 5 |=6
4 1 2 17
| |− = 2−( )
4 3 3 5
1 2 17 + 25
|1| − = 5
3 3 | |=6
10 − 17
1 2
1− = 5
3 3 42
3−1 2
= | 5 |=6
3 3 −7
2 2
= 3(which is true) 5
3 42
7
Put 𝑥 = 5, in (1) | |=6
−7
7 |−6| = 6
3 − 5( )
| 5 |−1=2 6 = 6 (which is true)
4 3 3 17
Since 𝑥 = 1, 5 satisfy the given equation,
3−7 1 2 therefore, the solution set is {1,
17
} i.e. 𝑆. 𝑆 =
| |− = 5
4 3 3 17
−4 1 2 {1, }
| |− = 5
4 3 3
1 2
|−1| − =
3 3
1 2
1− =
3 3
3−1 2
=
3 3
2 2
= 3(which is true)
3
1 7
Since 𝑥 = − 5 , 5 satisfy the given equation,
1 7
therefore, the solution set is {− 5 , 5 } i.e. 𝑆. 𝑆 =
1 7
{− 5 , 5 }
𝒙+𝟓
(viii). |𝟐−𝒙| = 𝟔
𝑥+5
Solution: As given |2−𝑥| = 6
By definition, we have
𝑥+5 𝑥+5
= 6 or 2−𝑥 = −6
2−𝑥
𝑥 + 5 = 12 − 6𝑥 or 𝑥 + 5 = −12 + 6𝑥
𝑥 + 6𝑥 = 12 − 5 or 5 + 12 = 6𝑥 − 𝑥
7𝑥 = 7 or 17 = 5𝑥
17
𝑥 = 1 or 𝑥 = 5
Check:
𝑥+5
|2−𝑥| = 6…(1)
Put 𝑥 = 1, in (1)
1+5
| |=6
2−1
6
| |=6
1
8|Page
Class 9th Chapter 7 www.notes.pk.com
Absolute Value ⇒ (3𝑥 + 2) − 3(2𝑥 + 1) > −9
A linear inequality in one variable 𝑥 is an inequality in ⇒ 3𝑥 + 2 − 6𝑥 − 3 > −9
which the variable 𝑥 occurs only to the first power and ⇒ −3𝑥 − 1 > −9
has the standard form ⇒ −1 + 9 > 3𝑥
𝑎𝑥 + 𝑏 < 0, 𝑎 ≠ 0 ⇒ 8 > 3𝑥
where 𝑎 and 𝑏 are real numbers. We may replace the 8
⇒ >𝑥
symbol < 𝑏𝑦 >, ≤, ≥ also. 3
8
Hence, 𝑆. 𝑆 = {𝑥|𝑥 < 3}
EXERCISE 7.3 (vi). 𝟑(𝟐𝒙 + 𝟏) − 𝟐(𝟐𝒙 + 𝟓) < 𝟓(𝟑𝒙 − 𝟐)
Solution: As given 3(2𝑥 + 1) − 2(2𝑥 + 5) <
Q#1) Solve the following inequalities.
5(3𝑥 − 2)
(i). 𝟑𝒙 + 𝟏 < 𝟓𝒙 − 𝟒
⇒ 6𝑥 + 3 − 4𝑥 − 10 < 15𝑥 − 10
Solution: As given 3𝑥 + 1 < 5𝑥 − 4
⇒ 2𝑥 − 7 < 15𝑥 − 10
⇒ 5 < 2𝑥
5 ⇒ −7 + 10 < 15𝑥 − 2𝑥
⇒ <𝑥 ⇒ 3 < 13𝑥
2 3
5
Hence, 𝑆. 𝑆 = {𝑥|𝑥 > } ⇒ <𝑥
2 13
(ii). 𝟒𝒙 − 𝟏𝟎. 𝟑 ≤ 𝟐𝟏𝒙 − 𝟏. 𝟖 3
Hence, 𝑆. 𝑆 = {𝑥|𝑥 > 13}
Solution: As given 4𝑥 − 10.3 ≤ 21𝑥 − 1.8
(vii). 𝟑(𝒙 − 𝟏) − (𝒙 − 𝟐) > −𝟐(𝒙 + 𝟒)
⇒ −10.3 + 1.8 ≤ 21𝑥 − 4𝑥
Solution: As given 3(𝑥 − 1) − (𝑥 − 2) > −2(𝑥 + 4)
⇒ −8.5 ≤ 17𝑥
⇒ 3𝑥 − 3 − 𝑥 + 2 > −2𝑥 − 8
8.5
⇒ − ≤𝑥 ⇒ 2𝑥 − 1 > −2𝑥 − 8
15
⇒ 𝑥 ≥ −0.5 ⇒ 2𝑥 + 2𝑥 > −8 + 1
Hence, 𝑆. 𝑆 = {𝑥|𝑥 ≥ −0.5} ⇒ 4𝑥 > −7
𝟏 𝟏 7
(iii). 𝟒 − 𝒙 ≥ −𝟕 + 𝒙 ⇒𝑥>−
𝟐 𝟒 4
1 1 7
Solution: As given 4 − 2 𝑥 ≥ −7 + 4 𝑥 Hence, 𝑆. 𝑆 = {𝑥|𝑥 > − 4}
Multiply by 4 𝟐 𝟐 𝟏
(viii). 𝟐 𝟑 + 𝟑 (𝟓𝒙 − 𝟒) > − 𝟑 (𝟖𝒙 + 𝟕)
⇒ 16 − 2𝑥 ≥ −28 + 𝑥 2 2 1
⇒ 16 + 28 ≥ 𝑥 + 2𝑥 Solution: As given 2 3 + 3 (5𝑥 − 4) > − 3 (8𝑥 + 7)
⇒ 44 ≥ 3𝑥 8 2 1
⇒ + (5𝑥 − 4) > − (8𝑥 + 7)
44 3 3 3
⇒ 𝑥≤ Multiply by 3 (LCM)
3
44
Hence, 𝑆. 𝑆 = {𝑥|𝑥 ≤ } 8 2
3 ⇒ 3 × ( ) + 3 × ( (5𝑥 − 4))
𝟏 3 3
(iv). 𝒙 − 𝟐(𝟓 − 𝟐𝒙) ≥ 𝟔𝒙 − 𝟑 𝟐
1
Solution: As given 𝑥 − 2(5 − 2𝑥) ≥ 6𝑥 − 3 2
1 > 3 × (− (8𝑥 + 7))
3
7 ⇒ 8 + 2(5𝑥 − 4) > −(8𝑥 + 7)
𝑥 − 10 + 4𝑥 ≥ 6𝑥 −
2 ⇒ 8 + 10𝑥 − 8 > −8𝑥 − 7)
7 ⇒ 10𝑥 > −8𝑥 − 7
5𝑥 − 10 ≥ 6𝑥 −
2 ⇒ 10𝑥 + 8𝑥 > −7
Multiply by 2 ⇒ 18𝑥 > −7
⇒ 10𝑥 − 20 ≥ 12𝑥 − 7 7
⇒ −20 + 7 ≥ 12𝑥 − 10𝑥 ⇒ 𝑥<−
18
⇒ −13 ≥ 2𝑥 7
Hence, 𝑆. 𝑆 = {𝑥|𝑥 < − }
−13 18
⇒ 𝑥≤ Q#2) Solve the following inequalities.
2
⇒ 𝑥 ≤ −6.5 (i). −𝟒 < 𝟑𝒙 + 𝟓 < 𝟖
Hence, 𝑆. 𝑆 = {𝑥|𝑥 ≤ −6.5} Solution: As given −4 < 3𝑥 + 5 < 8
3𝑥+2 2𝑥+1 ⇒ −4 − 5 < 3𝑥 < 8 − 5
(v). − > −1
9 3
3𝑥+2 2𝑥+1
⇒ −9 < 3𝑥 < 3
Sol: As given 9 − 3 > −1 9 3𝑥 3
⇒ − < <
Multiply by 4 (LCM), we have 3 3 3
3𝑥 + 2 2𝑥 + 1 ⇒ −3 < 𝑥 < 1
⇒9×( )−9×( ) > 9 × (−1) Hence, 𝑆. 𝑆 = {𝑥| − 3 < 𝑥 < 1}
9 3

9|Page
Class 9th Chapter 7 www.notes.pk.com
𝟒−𝟑𝒙 Solution: As given 3𝑥 − 2 < 2𝑥 + 1 < 4𝑥 + 17
(ii). −𝟓 < 𝟐
<𝟏
4−3𝑥 3𝑥 − 2 < 2𝑥 + 1 𝑜𝑟 2𝑥 + 1 < 4𝑥 + 17
Solution: As given −5 < 2
<1 ⇒ 3𝑥 − 2𝑥 < 1 + 2 𝑜𝑟 1 − 17 < 4𝑥 − 2𝑥
Multiply by 2 ⇒ 𝑥 < 3 𝑜𝑟 − 16 < 2𝑥
⇒ −10 < 4 − 3𝑥 < 2 ⇒ 𝑥 < 3 𝑜𝑟 − 8 < 𝑥
⇒ 10 − 4 < 4 − 3𝑥 − 4 < 2 − 4 ⇒ −8 < 𝑥 𝑜𝑟 𝑥 < 3
⇒ −14 < −3𝑥 < −2 ⇒ −8 < 𝑥 < 3
Multiply by −1 (inequality changes) Hence, 𝑆. 𝑆 = {𝑥| − 8 < 𝑥 < 3}
⇒ 14 > 3𝑥 > 2
14 2
⇒ >𝑥>
3 3
14 2
Hence, 𝑆. 𝑆 = {𝑥| 3 > 𝑥 > 3
𝒙−𝟐
(iii). −𝟔 < 𝟒
<𝟔
𝑥−2
Solution: As given −6 < 4
<6
⇒ −24 < 𝑥 − 2 < 24
⇒ −24 + 2 < 𝑥 − 2 + 2 < 24 + 2
⇒ −22 < 𝑥 < 26
Hence, 𝑆. 𝑆 = {𝑥| − 22 < 𝑥 < 26}
𝟕−𝒙
(iv). 𝟑 ≥ 𝟐
≥𝟏
7−𝑥
Solution: As given 3 ≥ 2
≥1
⇒ 6≥ 7−𝑥 ≥2

Contact or Suggest Us: info@notespk.com


A project of: www.notespk.com
⇒ 6 − 7 ≥ −𝑥 ≥ 2 − 7
⇒ −1 ≥ −𝑥 ≥ −5
Multiply by −1
⇒ 1≤𝑥≤5
Hence, 𝑆. 𝑆 = {𝑥|1 ≤ 𝑥 ≤ 5}
(v). 3𝑥 − 10 ≤ 5 < 𝑥 + 3
Sol: As given 3𝑥 − 10 ≤ 5 < 𝑥 + 3
3𝑥 − 10 ≤ 5 𝑜𝑟 5 < 𝑥 + 3
⇒ 3𝑥 ≤ 5 + 10 𝑜𝑟 5−3< 𝑥
⇒ 3𝑥 ≤ 15 𝑜𝑟 2<𝑥
⇒ 𝑥 ≤ 5 𝑜𝑟 2 < 𝑥
⇒ 2 < 𝑥 𝑜𝑟 𝑥 ≤ 5
⇒ 2< 𝑥≤5
Hence, 𝑆. 𝑆 = {𝑥|2 < 𝑥 ≤ 5}
𝒙−𝟒
(vi). −𝟑 < −𝟓
<𝟒
𝑥−4
Solution: As given −3 < −5
<4
Multiply by −5
𝑥−4
⇒ −5 × (−3) > −5 × ( ) < −5 × (4)
−5
⇒ 15 < 𝑥 − 4 < −20
⇒ 15 + 4 < 𝑥 < −20 + 4
⇒ 19 > 𝑥 > −16
Hence, 𝑆. 𝑆 = {𝑥| − 16 < 𝑥 < 19}
(vii). 𝟏 − 𝟐𝒙 < 𝟓 − 𝒙 < 𝟐𝟓 − 𝟔𝒙
Solution : As given 1 − 2𝑥 < 5 − 𝑥 < 25 − 6𝑥
1 − 2𝑥 < 5 − 𝑥 𝑜𝑟 5 − 𝑥 < 25 − 6𝑥
⇒ 1 − 5 < −𝑥 + 2𝑥 𝑜𝑟 − 𝑥 + 6𝑥 < 25 − 5
Complied by Shumaila Amin
⇒ −4 < 𝑥 𝑜𝑟 5𝑥 < 20
⇒ −4 < 𝑥 𝑜𝑟 𝑥<4
⇒ −4 < 𝑥 < 4
Hence, 𝑆. 𝑆 = {𝑥| − 4 < 𝑥 < 4}
(viii). 3𝑥 − 2 < 2𝑥 + 1 < 4𝑥 + 17
10 | P a g e
MATHEMATICS 9th Science Group

7/18/2020
Chapter 9.
INTRODUCTION TO CO-ORDINATES
GEOMETRY

A project of: www.notespk.com


Contact or Suggest Us: info@notespk.com
Contents
EXERCISE 9.1 ................................................................... 1
EXERCISE 9.3 ................................................................... 3
Class 9th Chapter 9 www.notes.pk.com

Coordinate Geometry Put 𝑥1 = −8, 𝑥2 = 6, 𝑦1 = 1 𝑎𝑛𝑑 𝑦2 = 1

The study of geometrical shapes in a plane is called |𝑑| = √(6 − (−8))2 + (1 − 1)2
plane geometry. Coordinate geometry is the study of
|𝑑| = √(6 + 8)2 + (0)2
geometrical shapes in the Cartesian plane (coordinate
plane). |𝑑| = √142
Distance Formula |𝑑| = 14

Let 𝑃(𝑥1 , 𝑦1 ) and 𝑄(𝑥2 , 𝑦2 ) be two points in the (d) 𝐴(−4, √2), 𝐵(−4, −3)
coordinate plane where 𝑑 is the length of the
Sol: As given 𝐴(−4, √2), 𝐵(−4, −3)
line segment 𝑃𝑄. i.e. |𝑃𝑄| = 𝑑 and given as
Using distance formula
|𝑑| = √(𝑥2 − 𝑥1 )2 + (𝑦2 − 𝑦1 )2
|𝑑| = √(𝑥2 − 𝑥1 )2 + (𝑦2 − 𝑦1 )2
EXERCISE 9.1 Put 𝑥1 = −4, 𝑥2 = −4, 𝑦1 = √2 𝑎𝑛𝑑 𝑦2 = −3
Q#1) Find the distance between the following pairs of
points. 2
|𝑑| = √(−4 − (−4))2 + (−3 − √2)
(a) 𝐴(9, 2), 𝐵(7, 2)
2
Sol: As given 𝐴(9, 2), 𝐵(7, 2) |𝑑| = √(−4 + 4)2 + (3 + √2)

Using distance formula 2


|𝑑| = √(3 + √2)
|𝑑| = √(𝑥2 − 𝑥1 )2 + (𝑦2 − 𝑦1 )2
|𝑑| = 3 + √2
Put 𝑥1 = 9, 𝑥2 = 7, 𝑦1 = 2 𝑎𝑛𝑑 𝑦2 = 2
(e) 𝐴(3, −11), 𝐵(3, −4)
|𝑑| = √(7 − 9)2 + (2 − 2)2
Sol: As given 𝐴(3, −11), 𝐵(3, −4)
|𝑑| = √(−2)2 + (0)2
Using distance formula
|𝑑| = √4
|𝑑| = √(𝑥2 − 𝑥1 )2 + (𝑦2 − 𝑦1 )2
|𝑑| = 2
Put 𝑥1 = 3, 𝑥2 = 3, 𝑦1 = −11 𝑎𝑛𝑑 𝑦2 = −4
(b) 𝐴(2, −6), 𝐵(3, −6)
|𝑑| = √(3 − 3))2 + (−4 − (−11))2
Sol: As given 𝐴(2, −6), 𝐵(3, −6)
|𝑑| = √(0)2 + (−4 + 11)2
Using distance formula

|𝑑| = √(𝑥2 − 𝑥1 )2 + (𝑦2 − 𝑦1 )2 |𝑑| = √(7)2

Put 𝑥1 = 2, 𝑥2 = 3, 𝑦1 = −6 𝑎𝑛𝑑 𝑦2 = −6 |𝑑| = 7

(f) 𝐴(0, 0), 𝐵(0, −5)


|𝑑| = √(3 − 2)2 + (2 − 2)2
Sol: As given 𝐴(0, 0), 𝐵(0, −5)
|𝑑| = √(1)2 + (0)2
Using distance formula
|𝑑| = √1
|𝑑| = √(𝑥2 − 𝑥1 )2 + (𝑦2 − 𝑦1 )2
|𝑑| = 1
Put 𝑥1 = 0, 𝑥2 = 0, 𝑦1 = 0 𝑎𝑛𝑑 𝑦2 = −5
(c) 𝐴(−8, 1), 𝐵(6, 1)
|𝑑| = √(0 − 0)2 + (−5 − 0)2
Sol: As given 𝐴(−8, 1), 𝐵(6, 1)
Using distance formula |𝑑| = √(0)2 + (−5)2

|𝑑| = √(𝑥2 − 𝑥1 )2 + (𝑦2 − 𝑦1 )2 |𝑑| = √52

1|Page
Class 9th Chapter 9 www.notes.pk.com
|𝑑| = 5 Using distance formula
Q#2) Let P be the point on x-axis with x-coordinate a |𝑃𝑄| = √(𝑥2 − 𝑥1 )2 + (𝑦2 − 𝑦1 )2
and Q be the point on y-axis with y-coordinate b as
given below. Find the distance between P and Q. |𝑃𝑄| = √(0 − (−2))2 + (−3 − 0)2
(i) 𝑎 = 9, 𝑏 = 7 |𝑃𝑄| = √(2)2 + (−3)2
Sol: As Given 𝑎 = 9, 𝑏 = 7
|𝑃𝑄| = √4 + 9
𝑃(𝑎, 0) = 𝑃(9, 0) and 𝑄(0, 𝑏) = 𝑄(0, 7)
|𝑃𝑄| = √13
Using distance formula
(v) 𝑎 = √2, 𝑏 = 1
|𝑃𝑄| = √(𝑥2 − 𝑥1 )2 + (𝑦2 − 𝑦1 )2
Sol: As Given 𝑎 = √2, 𝑏 = 1
|𝑃𝑄| = √(0 − 9)2 + (7 − 0)2
𝑃(𝑎, 9) = 𝑃(√2, 0) and 𝑄(0, 𝑏) = 𝑄(0, 1)
|𝑃𝑄| = √(−9)2 + (7)2 Using distance formula
|𝑃𝑄| = √81 + 49 |𝑃𝑄| = √(𝑥2 − 𝑥1 )2 + (𝑦2 − 𝑦1 )2
|𝑃𝑄| = √130 2
|𝑃𝑄| = √(0 − √2) + (1 − 0)2
(ii) 𝑎 = 2, 𝑏 = 3
Sol: As Given 𝑎 = 2, 𝑏 = 3 2
|𝑃𝑄| = √(√2) + (1)2
𝑃(𝑎, 0) = 𝑃(2, 0) and 𝑄(0, 𝑏) = 𝑄(0, 3)
|𝑃𝑄| = √2 + 1
Using distance formula
|𝑃𝑄| = √3
|𝑃𝑄| = √(𝑥2 − 𝑥1 )2 + (𝑦2 − 𝑦1 )2
(vi) 𝑎 = −9, 𝑏 = −4
|𝑃𝑄| = √(0 − 2)2 + (3 − 0)2
Sol: As Given 𝑎 = − 9, 𝑏 = −4
|𝑃𝑄| = √(−2)2 + (3)2 𝑃(𝑎, 9) = 𝑃(−9, 0) and 𝑄(0, 𝑏) = 𝑄(0, −4)
|𝑃𝑄| = √4 + 9 Using distance formula
|𝑃𝑄| = √13 |𝑃𝑄| = √(𝑥2 − 𝑥1 )2 + (𝑦2 − 𝑦1 )2
(iii) 𝑎 = −8, 𝑏 = 6
|𝑃𝑄| = √(0 − (−9))2 + (−4 − 0)2
Sol: As Given 𝑎 = −8, 𝑏 = 6
|𝑃𝑄| = √(9)2 + (−4)2
𝑃(𝑎, 9) = 𝑃(−8, 0) and 𝑄(0, 𝑏) = 𝑄(0, 6)
|𝑃𝑄| = √81 + 16
Using distance formula
|𝑃𝑄| = √97
|𝑃𝑄| = √(𝑥2 − 𝑥1 )2 + (𝑦2 − 𝑦1 )2
Collinear or Non-collinear
|𝑃𝑄| = √(0 − (−8))2 + (6 − 0)2
Points in the Plane
|𝑃𝑄| = √(8)2 + (6)2
Two or more than two points which lie on the same
|𝑃𝑄| = √64 + 36 straight line are called collinear points with respect to
that line; otherwise they are called non-collinear.
|𝑃𝑄| = √100 = 10
Let m be a line, then all the points on line m are
(iv) 𝑎 = −2, 𝑏 = −3 collinear.
Sol: As Given 𝑎 = −2, 𝑏 = −3 In the given figure, the points P and Q are collinear
with respect to the line m and the points P and R are
𝑃(𝑎, 9) = 𝑃(−2, 0) and 𝑄(0, 𝑏) = 𝑄(0, −3)
not collinear with respect to it.

2|Page
Class 9th Chapter 9 www.notes.pk.com
A figure formed by four non-collinear points in the
plane is called a parallelogram if
(i) its opposite sides are of equal length

Triangle (ii) its opposite sides are parallel

A closed figure in a plane obtained by joining three


non-collinear points is called a triangle. Let 𝑃(𝑥1 , 𝑦1 ) and 𝑄(𝑥2 , 𝑦2 ) be any two points in the
In the triangle 𝐴𝐵𝐶 the non-collinear points plane and 𝑀(𝑥, 𝑦) be a mid-point of points 𝑃 and 𝑄 on
𝐴, 𝐵 𝑎𝑛𝑑 𝐶 are the three vertices of the triangle 𝐴𝐵𝐶. the line-segment 𝑃𝑄 is given as
The line segments 𝐴𝐵, 𝐵𝐶 𝑎𝑛𝑑 𝐶𝐴 are called sides of 𝑥1 + 𝑥2 𝑦1 + 𝑦2
the triangle. 𝑀𝑖𝑑 − 𝑝𝑜𝑖𝑛𝑡 𝑜𝑓 𝑃𝑄 = 𝑀 ( , )
2 2

EXERCISE 9.3
Q#1) Find the mid-point of the line segment joining
each of the following pairs of points
(a) 𝐴(9, 2), 𝐵(7, 2)
Sol: As given 𝐴(9, 2), 𝐵(7, 2)
Using Mid-point formula
𝑥1 + 𝑥2 𝑦1 + 𝑦2
(i) Equilateral Triangle 𝑀𝑖𝑑 − 𝑝𝑜𝑖𝑛𝑡 𝑜𝑓 𝑃𝑄 = 𝑀 ( , )
2 2
If the lengths of all the three sides of a triangle are Put 𝑥1 = 9, 𝑥2 = 7, 𝑦1 = 2 𝑎𝑛𝑑 𝑦2 = 2
same, then the triangle is called an equilateral triangle
. 9+7 2+2
𝑀𝑖𝑑 − 𝑝𝑜𝑖𝑛𝑡 𝑜𝑓 𝑃𝑄 = 𝑀 ( , )
2 2
(ii) An Isosceles Triangle 16 4
An isosceles triangle 𝑃𝑄𝑅 is a triangle which has two 𝑀𝑖𝑑 − 𝑝𝑜𝑖𝑛𝑡 𝑜𝑓 𝑃𝑄 = 𝑀 ( , )
2 2
of its sides with equal length while the third side has a
different length. 𝑀𝑖𝑑 − 𝑝𝑜𝑖𝑛𝑡 𝑜𝑓 𝑃𝑄 = 𝑀(8, 2)

(iii) Right Angle Triangle (b) 𝐴(2, −6), 𝐵(3, −6)


A triangle in which one of the angles has measure Sol: As given 𝐴(2, −6), 𝐵(3, −6)
equal to 900 is called a right angle triangle.
Using Mid-point formula
(iv) Scalene Triangle 𝑥1 + 𝑥2 𝑦1 + 𝑦2
A triangle is called a scalene triangle if measures of all 𝑀𝑖𝑑 − 𝑝𝑜𝑖𝑛𝑡 𝑜𝑓 𝑃𝑄 = 𝑀 ( , )
2 2
the three sides are different.
Put 𝑥1 = 2, 𝑥2 = 3, 𝑦1 = −6 𝑎𝑛𝑑 𝑦2 = −6
Square 2 + 3 −6 − 6
𝑀𝑖𝑑 − 𝑝𝑜𝑖𝑛𝑡 𝑜𝑓 𝑃𝑄 = 𝑀 ( , )
A square is a closed figure in the plane formed by four 2 2
non-collinear points such that lengths of all sides are 5 −12
equal and measure of each angle is 90𝑜 . 𝑀𝑖𝑑 − 𝑝𝑜𝑖𝑛𝑡 𝑜𝑓 𝑃𝑄 = 𝑀 ( , )
2 2
Rectangle 𝑀𝑖𝑑 − 𝑝𝑜𝑖𝑛𝑡 𝑜𝑓 𝑃𝑄 = 𝑀(2.5, −6)

A figure formed in the plane by four non-collinear (c) 𝐴(−8, 1), 𝐵(6, 1)
points is called a rectangle if,
Sol: As given 𝐴(−8, 1), 𝐵(6, 1)
(i) Its opposite sides are equal in length;
Using Mid-point formula
(ii) The angle at each vertex is of measure 90𝑜 . 𝑥1 + 𝑥2 𝑦1 + 𝑦2
𝑀𝑖𝑑 − 𝑝𝑜𝑖𝑛𝑡 𝑜𝑓 𝑃𝑄 = 𝑀 ( , )
Parallelogram 2 2

3|Page
Class 9th Chapter 9 www.notes.pk.com
Put 𝑥1 = −8, 𝑥2 = 6, 𝑦1 = 1 𝑎𝑛𝑑 𝑦2 = 1 REVIEW EXERCISE
−8 + 6 1 + 1
𝑀𝑖𝑑 − 𝑝𝑜𝑖𝑛𝑡 𝑜𝑓 𝑃𝑄 = 𝑀 ( , ) Q#2) Answer the following, which is true and which is
2 2
false.
−2 2
𝑀𝑖𝑑 − 𝑝𝑜𝑖𝑛𝑡 𝑜𝑓 𝑃𝑄 = 𝑀 ( , ) (i) A line has two end points…F …..
2 2
𝑀𝑖𝑑 − 𝑝𝑜𝑖𝑛𝑡 𝑜𝑓 𝑃𝑄 = 𝑀(−1, 1) (ii) A line segment has one end point…F …..

(d) 𝐴(−4, 9), 𝐵(−4, −3) (iii) A triangle is formed by three collinear points. …F
(iv) Each side of a triangle has two collinear
Sol: As given 𝐴(−4, √2), 𝐵(−4, −3)
vertices…T…
Using Mid-point formula
(v) The end points of each side of a rectangle are
𝑥1 + 𝑥2 𝑦1 + 𝑦2 collinear…T ….
𝑀𝑖𝑑 − 𝑝𝑜𝑖𝑛𝑡 𝑜𝑓 𝑃𝑄 = 𝑀 ( , )
2 2
(vi) All the points that lie on the x-axis are
Put 𝑥1 = −4, 𝑥2 = −4, 𝑦1 = 9 𝑎𝑛𝑑 𝑦2 = −3 collinear…T …
−4 − 4 9 − 3 (vii) Origin is the only point collinear with the points
𝑀𝑖𝑑 − 𝑝𝑜𝑖𝑛𝑡 𝑜𝑓 𝑃𝑄 = 𝑀 ( , )
2 2 of both the axes separately. …T..
−8 6
𝑀𝑖𝑑 − 𝑝𝑜𝑖𝑛𝑡 𝑜𝑓 𝑃𝑄 = 𝑀 ( , )
2 2
𝑀𝑖𝑑 − 𝑝𝑜𝑖𝑛𝑡 𝑜𝑓 𝑃𝑄 = 𝑀(−4, 3)
(e) 𝐴(3, −11), 𝐵(3, −4)
Sol: As given 𝐴(3, −11), 𝐵(3, −4)
Using Mid-point formula
𝑥1 + 𝑥2 𝑦1 + 𝑦2
𝑀𝑖𝑑 − 𝑝𝑜𝑖𝑛𝑡 𝑜𝑓 𝑃𝑄 = 𝑀 ( , )
2 2
Put 𝑥1 = 3, 𝑥2 = 3, 𝑦1 = −11 𝑎𝑛𝑑 𝑦2 = −4
3 + 3 −11 − 4
𝑀𝑖𝑑 − 𝑝𝑜𝑖𝑛𝑡 𝑜𝑓 𝑃𝑄 = 𝑀 ( , )
2 2
6 −15
𝑀𝑖𝑑 − 𝑝𝑜𝑖𝑛𝑡 𝑜𝑓 𝑃𝑄 = 𝑀 ( , )
2 2
𝑀𝑖𝑑 − 𝑝𝑜𝑖𝑛𝑡 𝑜𝑓 𝑃𝑄 = 𝑀(3, −7.5)
(f) 𝐴(0, 0), 𝐵(0, −5)
Sol: As given 𝐴(0, 0), 𝐵(0, −5)
Using Mid-point formula
𝑥1 + 𝑥2 𝑦1 + 𝑦2
𝑀𝑖𝑑 − 𝑝𝑜𝑖𝑛𝑡 𝑜𝑓 𝑃𝑄 = 𝑀 ( , )
2 2
Put 𝑥1 = 0, 𝑥2 = 0, 𝑦1 = 0 𝑎𝑛𝑑 𝑦2 = −5
0+0 0−5
𝑀𝑖𝑑 − 𝑝𝑜𝑖𝑛𝑡 𝑜𝑓 𝑃𝑄 = 𝑀 ( , )
2 2
0 −5
𝑀𝑖𝑑 − 𝑝𝑜𝑖𝑛𝑡 𝑜𝑓 𝑃𝑄 = 𝑀 ( , )
2 2
𝑀𝑖𝑑 − 𝑝𝑜𝑖𝑛𝑡 𝑜𝑓 𝑃𝑄 = 𝑀(0, 2.5)

4|Page

You might also like